You are on page 1of 192

CBSE Term II

2022
Mathematics
Basic
Class X
CBSE Term II
2022
Mathematics
Basic
Class X
Complete Case Short/Long 3
Theory Covering Based Answer Practice Papers
NCERT Questions Questions with Explanations

Authors
Vishal Kumar Mehta
Alok Sharma

ARIHANT PRAKASHAN (School Division Series)


CBSE Term II
2022

ARIHANT PRAKASHAN (School Division Series)

© Publisher
No part of this publication may be re-produced, stored in a retrieval system or by any
means, electronic, mechanical, photocopying, recording, scanning, web or otherwise
without the written permission of the publisher. Arihant has obtained all the information in
this book from the sources believed to be reliable and true. However, Arihant or its editors
or authors or illustrators don’t take any responsibility for the absolute accuracy of any
information published and the damage or loss suffered thereupon.
All disputes subject to Meerut (UP) jurisdiction only.

Administrative & Production Offices


Regd. Office
‘Ramchhaya’ 4577/15, Agarwal Road, Darya Ganj, New Delhi -110002
Tele: 011- 47630600, 43518550

Head Office
Kalindi, TP Nagar, Meerut (UP) - 250002, Tel: 0121-7156203, 7156204
Sales & Support Offices
Agra, Ahmedabad, Bengaluru, Bareilly, Chennai, Delhi, Guwahati,
Hyderabad, Jaipur, Jhansi, Kolkata, Lucknow, Nagpur & Pune.

ISBN : 978-93-25796-61-4
PO No : TXT-XX-XXXXXXX-X-XX
Published by Arihant Publications (India) Ltd.
For further information about the books published by Arihant, log on to
www.arihantbooks.com or e-mail at info@arihantbooks.com

Follow us on
CBSE Term II
2022

Contents
CHAPTER
Quadratic Equations -
CHAPTER
Arithmetic Progressions -
CHAPTER
Circles -
CHAPTER
Constructions -
CHAPTER
Applications of Trigonometry -

CHAPTER
Surface Areas and Volumes -

CHAPTER
Statistics -

Practice Papers - -

Watch Free Learning Videos


Subscribe arihant Channel

þ Video Solutions of CBSE Sample Papers


þ Chapterwise Important MCQs
þ CBSE Updates
CBSE Term II
2022

Syllabus
CBSE Term II Class XII
One Paper Max Marks:
No. Units Marks
I. Algebra Cont.
II. Geometry Cont.
III. Trigonometry Cont.
IV. Mensuration Cont.
V. Statistics Probability Cont.

Total
Internal Assessment
Total

UNIT-I ALGEBRA
. Quadratic Equations Periods
Standard form of a quadratic equation ax + bx + c = , a ≠ . Solutions of quadratic equations
only real roots by factorisation, and by using quadratic formula. Relationship between
discriminant and nature of roots. Situational problems based on quadratic equations related to
day to day activities problems on equations reducible to quadratic equations are excluded

. Arithmetic Progressions
Motivation for studying Arithmetic Progression Derivation of the nth term and sum of the first n
terms of AP and their application in solving daily life problems. Applications based on sum to n
terms of an AP. are excluded

UNIT-II GEOMETRY
. Circles
Tangent to a circle at point of contact
. Prove The tangent at any point of a circle is perpendicular to the radius through the
point of contact.
. Prove The lengths of tangents drawn from an external point to a circle are equal.
CBSE Term II
2022

. Constructions
. Division of a line segment in a given ratio internally .
. Tangents to a circle from a point outside it.

UNIT-III TRIGONOMETRY
. Some Applications of Trigonometry
HEIGHTS AND DISTANCES-Angle of elevation, Angle of Depression.
Simple problems on heights and distances. Problems should not involve more than two right
triangles. Angles of elevation depression should be only , , .

UNIT-IV MENSURATION
. Surface Areas and Volumes
. Surface areas and volumes of combinations of any two of the following: cubes, cuboids,
spheres, hemispheres and right circular cylinders cones.
. Problems involving converting one type of metallic solid into another and other mixed
problems. Problems with combination of not more than two different solids be taken .

UNIT-V STATISTICS PROBABILITY


. Statistics
Mean, median and mode of grouped data bimodal situation to be avoided . Mean by Direct
Method and Assumed Mean Method only

Internal Assessment Marks Total Marks

Periodic Test 3
10 Marks
Multiple Assessments 2
for the
Portfolio 2
Term
Student Enrichment Activities-practical work 3
CBSE Circular
Acad - 51/2021, 05 July 2021

Exam Scheme Term I & II

dsUnh;
z ek/;fed f'k{kk cksMZ
(f'k{kk ea=ky;] Hkkjr ljdkj ds v/khu ,d Lok;r laxBu)

Hkkjr CENTRAL BOARD OF SECONDARY EDUCATION


(An Autonomous Organisation under the Ministry of Education, Govt. of India)
dsUnh;
z ek/;fed f'k{kk cksMZ
(f'k{kk ea=ky;] Hkkjr ljdkj ds v/khu ,d Lok;r laxBu)
Hkkjr
CENTRAL BOARD OF SECONDARY EDUCATION
(An Autonomous Organisation under the Ministry of Education, Govt. of India)
dsUnh;
z ek/;fed f'k{kk cksMZ
(f'k{kk ea=ky;] Hkkjr ljdkj ds v/khu ,d Lok;r laxBu)

Hkkjr CENTRAL BOARD OF SECONDARY EDUCATION


(An Autonomous Organisation under the Ministry of Education, Govt. of India)

To cover this
situation, we
have given
both MCQs and
Subjective
Questions in
each Chapter.
dsUnh;
z ek/;fed f'k{kk cksMZ
(f'k{kk ea=ky;] Hkkjr ljdkj ds v/khu ,d Lok;r laxBu)

Hkkjr CENTRAL BOARD OF SECONDARY EDUCATION


(An Autonomous Organisation under the Ministry of Education, Govt. of India)
CBSE Term II Mathematics X (Basic) 1

CHAPTER 01

Quadratic
Equations
In this Chapter...
l Quadratic Equation and its Solutions
l Solution of a Quadratic Equation by Factorisation
l Solution of a Quadratic Equation by Quadratic Formula
l Relationship between Discriminant and Nature of Roots

An equation of the form ax 2 + bx + c = 0 is called quadratic Solutions or Roots of a Quadratic Equation


equation in variable x, where a, b and c are real numbers and All the values of variable which satisfy the given quadratic
a ≠ 0. equation, are called roots or zeroes or solutions of given
quadratic equation.
e.g. 2 x 2 + x − 100 = 0, − x 2 + 1 + 300 x = 0,
In other words, a real number α is said to be a root or zero or
4 x − 3 x 2 + 7 = 0, 4 x 2 − 25 = 0 are quadratic equations. solution of a quadratic equation ax 2 + bx + c = 0, a ≠ 0,
The form ax 2 + bx + c = 0, a ≠ 0 is called the standard form of if a(α ) 2 + b(α ) + c = 0.
a quadratic equation. Any quadratic equation can have atmost two roots.
To express a quadratic equation in its standard form, write Method to Check Whether the Given Value is a
the terms of given equation in the descending order of their Solution of the Given Quadratic Equation
degrees.
Let p( x) = 0 be the given quadratic equation and x = α be the
e.g. 3 x 2 + x + 2 = 0 and x 2 − 2 x + 6 = 0, are in standard form given value of x.
whereas, x 2 − 3 + 4 x = 0 and x + x 2 + 8 = 0 are not in their To check whether x = α is a solution of the given equation or
standard form. not, use the following steps
Method to Check Whether a Given Equation is Case I Write the given equation in the form, p( x) = 0.
Quadratic or Not Case II Now, put x = α in p( x). If p(α ) = 0, then x = α is the
To check whether a given equation is quadratic or not, first solution of given equation, otherwise not.
write the given equation in its simplest form and then Method to Determine An Unknown Constant in a
compare the equation with the standard form of a quadratic
equation, Quadratic Equation when its Solution or Root is Given

i.e. ax 2 + bx + c = 0, a ≠ 0. I. Sometimes, given quadratic equation involves one


unknown constant and its solution or root is given.
If the given equation follows the form of quadratic equation Then, to find the value of unknown constant, we put
(ax 2 + bx + c = 0, a ≠ 0 ), then it is a quadratic equation the value of root or solution in given quadratic equation
otherwise not. and simplify it to get the required unknown constant.
2 CBSE Term II Mathematics X (Basic)

II. Sometimes, quadratic equation involves two unknown So, the quadratic equation has no real roots or
constants and its both roots are given. Then, to find imaginary roots or we can say that roots of
unknowns we put both roots one-by-one in the quadratic quadratic equation does not exist. This can be
equation and get two linear equations in two unknowns. On
explained using the flow chart.
solving these equations, we get the required values of
unknown constants. Quadratic equation
ax2+bx+c=0, a ≠ 0
Solution of a Quadratic Equation
by Factorisation Find discriminant, D
To find the solution of a quadratic equation by factorisation
method, we use the following steps.
Step I Write the given equation in standard form D=0 D<0
D>0
⇒ Roots are real ⇒ Roots are
i.e. ax 2 + bx + c = 0 (if not given in standard form) and find ⇒ Roots are
and equal imaginary or
real and distinct
the value of a, b and c. real roots
does not exist
Step II Find the product of a and c and write it as a sum of its two
factor such that sum is equal to b. i.e. write ac = p × q and
p + q = b where, p and q are factors of ac. Method to Determine The Value of Unknown
when Nature of Roots is Given
Step III Put the value of b obtained from step II in given equation
and write it LHS as product of two linear factors. If nature of roots of a quadratic equation is given
Step IV Now, equate each factor equal to zero and get desired and quadratic equation involves an unknown.
roots of given quadratic equation. Then to find the value of unknown, first we find the
value of discriminant in terms of unknown. After that
Solution of a Quadratic Equation by use the given condition i.e. D> 0 or D = 0 or D< 0 and
Quadratic Formula simplify it.
In a quadratic equation ax 2 + bx + c = 0, a ≠ 0, if b 2 − 4ac ≥ 0, then Some Important Points
the roots of the quadratic equation are given by (i) Three consecutive numbers are x, ( x +1) and
(x + 2), respectively.
− b ± b 2 − 4ac −b ± D
x= or x = (ii) Three consecutive even and odd numbers are
2a 2a
2 x, ( 2 x + 2 ), ( 2 x + 4 ) and
where, D = b 2 − 4ac is known as discriminant. This result is known ( 2 x + 1), ( 2 x + 3 ), ( 2 x + 5 ), respectively.
as quadratic formula or Sridharacharya formula. (iii) Pythagoras theorem,
Relationship between Discriminant and (Hypotenuse) 2 = (Perpendicular) 2 + (Height) 2
Nature of Roots (iv) Area of triangle =
1
× Base × Height
The nature of roots depends upon the value of the discriminant D, 2
whereas, D can be zero, positive or negative, so three cases may (v) Area of right angled triangle
arise. 1
= × Base × Perpendicular
Case I When D = 0 i.e. b 2 − 4ac = 0. 2
−b ± 0 b b (vi) Area of rectangle = Length × Breadth
If D = b 2 − 4ac = 0, then x = ⇒x=− ,−
2a 2a 2a (vii) Perimeter of rectangle = 2 × (Length + Breadth)
So, the quadratic equation has two equal real roots or Distance
(viii) Speed =
repeated roots or coincident roots. Time
Case II When D> 0 i.e. b 2 − 4ac > 0. (ix) Two-digit number = 10x + y, where x and y are
−b + D −b − D the digits of ten’s place and unit place,
If D = b 2 − 4ac > 0, then x = and
2a 2a respectively.
So, the quadratic equation has two distinct real roots. On reversing the digits, new number = 10y + x
Case III When D< 0 i.e. b 2 − 4ac < 0. (x) If speed of stream be x km/h and speed of boat in
still water be y km/h. Then speed of boat in
If D = b 2 − 4ac < 0, then D can not be evaluated as
upstream = ( y − x) km/h and speed of boat in
square root of negative value is not defined. downstream = ( y + x) km/h.
CBSE Term II Mathematics X (Basic) 3

Solved Examples
1 2
= + −1
Example 1. Check whether the following equations are 3 3
quadratic or not. 1+ 2 −3
= =0
3 3
(i) x + = x 2 (ii) 2 x 2 − 5 x = x 2 − 2 x + 3 1
x So, x = is a root of the given equation.
1 3
(iii) x − 2 = 5
2
(iv) x 2 − 3 x − x + 4 = 0 1
x (iii) On putting x = − in Eq. (i), we get
2
Sol. 3
(i) Given that, x + = x2  1  1
2
 1
x p  −  = 3 −  + 2  −  − 1
 2  2  2
⇒ x2 + 3 = x3
3
⇒ x3 − x2 − 3 = 0 = −1 −1
4
Which is not of the form ax 2 + bx + c, a ≠ 0. 3 3 − 8 −5
Thus, the equation is not a quadratic equation. = −2= = ≠0
4 4 4
(ii) Given that, 2 x 2 − 5x = x 2 − 2 x + 3 1
So, x = − is not a root of the given equation.
⇒ 2 x 2 − x 2 − 5x + 2 x − 3 = 0 2
⇒ x 2 − 3x − 3 = 0 (iv) On putting x = 2 in Eq. (i), we get
Which is of the form ax 2 + bx + c, a ≠ 0. p( 2 ) = 3( 2 )2 + 2( 2 ) − 1
Thus, the equation is a quadratic equation. = 12 + 4 − 1 = 15 ≠ 0
1 So, x = 2 is not a root of the given equation.
(iii) Given that, x 2 − 2 = 5
x Example 3. In each of the following equations, find the
⇒ x 4 − 1 = 5x 2 value of unknown constant(s) for which the given
⇒ x − 5x − 1 = 0
4 2 value(s) is (are) solution of the equations.
Which is not of the form ax 2 + bx + c, a ≠ 0. (i) x 2 − k 2 = 0; x = 0.3
Thus, the equation is not a quadratic equation. −1
(iv) Given that, x 2 − 3x − x + 4 = 0 (ii) 3 x 2 + 2ax − 3 = 0; x =
2
Which is not of the form ax 2 + bx + c, a ≠ 0. Sol. (i) We have, x 2 − k2 = 0, here k is unknown.
Thus, the equation is not a quadratic equation. Since, x = 0.3 is a solution of given equation, so it will
Example 2. Which of the following are the roots of satisfy the given equation,
3x 2 + 2x − 1 = 0 ? On putting x = 0.3 in the given equation, we get
1 (0.3)2 − k2 = 0
(i) x = − 1 (ii) x = ⇒ k2 = (0.3)2
3
1 ⇒ k = ± 0.3
(iii) x = − (iv) x = 2
2 (ii) We have, 3x 2 + 2ax − 3 = 0, here a is unknown.
Sol. Given equation is of the form p( x ) =0, where 1
Since, x = − is a solution of given equation, so it will
p( x ) = 3x + 2 x − 1
2
…(i) 2
satisfy the given equation.
(i) On putting x = − 1 in Eq. (i), we get
1
p( − 1) = 3( − 1)2 + 2( − 1) − 1 On putting x = − in the given equation, we get
2
= 3 − 2 −1 = 0 2
 1  1
So, x = − 1 is a root of the given quadratic equation. 3 −  + 2a  −  − 3 = 0
 2  2
1
(ii) On putting x = in Eq. (i), we get 3
3 ⇒ −a − 3=0
2 4
 
1  1  1
p   = 3  + 2   − 1 3
 3  3  3 ⇒ a= −3
4
4 CBSE Term II Mathematics X (Basic)

3 − 12 9 x + 5 − ( x − 3) 1
⇒ a= =− ⇒ =
4 4 ( x − 3)( x + 5) 6
Example 4. Find the roots of the quadratic equation x+ 5−x+ 3 1
⇒ = ⇒ 8 × 6 = ( x − 3) ( x + 5)
5 ( x − 3) ( x + 5) 6
2x 2 + x − 2 = 0 by factorisation method.
3 ⇒ 48 = x 2 + 2 x − 15
5 ⇒ x 2 + 2 x − 63 = 0
Sol. Given equation is 2 x 2 + x − 2 =0
3 ⇒ x 2 + 9x − 7 x − 63 = 0 [by splitting the middle term]
On multiplying by 3 both sides, we get
⇒ x( x + 9) − 7( x + 9) = 0
6x 2 + 5x − 6 = 0
⇒ ( x − 7 ) ( x + 9) = 0
⇒ 6 x 2 + (9 x − 4 x ) − 6 = 0 ⇒ x = 7 and x = − 9
[by splitting the middle term] ∴Sum of roots = 7 + ( − 9) = − 2
⇒ 6x 2 + 9x − 4x − 6 = 0 Example 7. Using the quadratic formula, solve the
⇒ 3x ( 2 x + 3) − 2 ( 2 x + 3) = 0 quadratic equation.
⇒ ( 2 x + 3) ( 3x − 2 ) = 0 x 2 + 2 2 x − 6 = 0.
Now, 2x + 3 = 0 Sol. Given equation is x 2 + 2 2 x − 6 = 0 .
3
⇒ x=− On comparing with ax 2 + bx + c = 0, we get
2
and 3x − 2 = 0 a = 1, b = 2 2 and c = − 6
2 −b± b 2 − 4ac
⇒ x= By quadratic formula, x =
3 2a
5
Hence, the roots of the equation 2 x 2 + x − 2 = 0 are − ( 2 2 ) ± ( 2 2 )2 − 4 (1) ( −6)
3 =
−3 2 2 (1 )
and .
2 3 − 2 2 ± 8 + 24
=
Example 5. Solve the quadratic equation by 2
factorisation method. −2 2 ± 32 − 2 2 ± 4 2
= =
3 2x 2 − 5x − 2 = 0 2 2
−2 2 +4 2 −2 2 −4 2
Sol. Given equation is 3 2 x 2 − 5x − 2 = 0 = ,
2 2
3 2 x 2 − (6 x − x ) − 2 = 0
= 2, − 3 2
[by splitting the middle term]
So, 2 and − 3 2 are the roots of the given equation.
3 2 x − 6x + x − 2 = 0
2

3 2 x2 − 3 2 ⋅ 2 x + x − 2 = 0
Example 8. Find discriminant of the quadratic equation
3 x 2 + 4 x − 5 = 0.
⇒ 3 2 x (x − 2) + 1 (x − 2 ) = 0
Sol. Comparing the given quadratic equation
⇒ (x − 2) (3 2x + 1) = 0
3x 2 + 4x − 5 = 0
Now, x − 2 =0 ⇒ x = 2
with standard quadratic equation ax 2 + bx + c = 0, we get
and 3 2 x +1=0
a = 3, b = 4 and c = − 5
− 2
1 ∴ Discriminant (D) = b 2 − 4ac
⇒ x=− =
3 2 6
= ( 4)2 − 4 × ( 3) × ( − 5) = 16 + 60 = 76
Hence, the roots of the equation 3 2 x 2 − 5x − 2 = 0 are
2
Example 9. Check whether the quadratic equation has
− and 2 . real roots. If real roots exist, find them
6
8 x 2 + 2x − 3 = 0
Example 6. Find the sum of the roots of the equation,
Sol. Given equation is 8x 2 + 2 x − 3 = 0.
 1 1 1
 x − 3 − x + 5 = 6 . On comparing with ax 2 + bx + c = 0, we get
 
 1 a = 8, b = 2 and c = − 3
1  1
Sol. Given  − =
 x − 3 x + 5  6 ∴Discriminant, D = b 2 − 4ac
CBSE Term II Mathematics X (Basic) 5

= ( 2 ) 2 − 4 ( 8) ( − 3) On comparing with ax 2 + bx + c = 0, we get


= 4 + 96 = 100 > 0 a = 2 , b = 1 and c = − 1
Therefore, the equation 8x 2 + 2 x − 3 = 0 has two distinct ∴ Discriminant,
real roots as the discriminant greater than zero. D = b 2 − 4ac = (1 )2 − 4 ( 2 ) ( − 1 )
− b ± D − 2 ± 100 − 2 ± 10 = 1 + 8 = 9 > 0 i.e. D > 0
Thus roots, x = = =
2a 16 16 Hence, the equation 2 x 2 + x − 1 = 0 has two distinct
− 2 + 10 − 2 − 10 real roots.
= ,
16 16 9
(iii) Given equation is 2 x 2 − 6x + = 0.
8 12 1 3 2
= ,− = ,−
16 16 2 4 On comparing with ax 2 + bx + c = 0, we get
Example 10. Find the nature of roots of the quadratic a = 2 , b = − 6 and c =
9
equation 3x 2 − 4 3x + 4 = 0. 2
∴ Discriminant, D = b 2 − 4ac
If the roots are real, find them.
 9
Sol. Given quadratic equation is = ( − 6 )2 − 4 ( 2 )  
 2
3x 2 − 4 3x + 4 = 0
Compare with standard quadratic equation = 36 − 36 = 0
i.e. D = 0
ax 2 + bx + c = 0, we get
9
a = 3, b = −4 3 and c = 4 Hence, the equation 2 x 2 − 6x + = 0 has equal and
2
Now, discriminant = b 2 − 4ac real roots.
= ( −4 3 ) − 4 × 3 × 4
2
Example 12. The quadratic equation x 2 − 4 x + k = 0 has
= 48 − 48 = 0 distinct real roots, if k = 4. Why or why not?
Hence, roots are real and equal. Sol. Given quadratic equation is x 2 − 4x + k = 0
By using Sridharacharya formula, Compare with standard equation ax 2 + bx + c = 0, we get
−b ± D a = 1, b = − 4 and c = k
x=
2a The condition for distinct real root is b 2 − 4ac > 0
− ( −4 3 ) ± 0
= ⇒ ( − 4) 2 − 4 × 1 × k > 0
2×3
4 3 2 3 ⇒ 16 − 4k > 0
= = ⇒ 16 > 4k
2×3 3
16
2 3 2 3 ⇒ k< ⇒ k<4
Hence, roots of given quadratic equation are and . 4
3 3
Example 13. Find the value of k, for which the
Example 11. State whether the following quadratic quadratic equation ( k + 4 )x 2 + ( k + 1)x + 1 = 0 has
equations have two distinct real roots. Justify your
answer. equal roots.
Sol. Given, quadratic equation is
(i) x 2 − 3 x + 4 = 0
( k + 4 )x 2 + ( k + 1 )x + 1 = 0
(ii) 2 x + x − 1 = 0
2
Compare with ax 2 + bx + c = 0, we get
9
(iii) 2 x 2 − 6 x + = 0 a = k + 4, b = k + 1 and c = 1
2
Condition for equal roots, b 2 − 4ac = 0
Sol. (i) Given equation is x 2 − 3x + 4 = 0.
∴ ( k + 1)2 − 4 × ( k + 4)(1) = 0
On comparing with ax 2 + bx + c = 0, we get
⇒ k2 + 12 + 2 k − 4k − 16 = 0
a = 1, b = − 3 and c = 4
[Q(a + b )2 = a 2 + b 2 + 2ab ]
∴ Discriminant,
D = b 2 − 4ac = ( −3)2 − 4 (1 ) ( 4) ⇒ k2 − 2 k − 15 = 0
= 9 − 16 = − 7 < 0 ⇒ k − ( 5 − 3)k − 15 = 0
2

i.e. D <0 [by splitting middle term]


Hence, the equation x 2 − 3x + 4 = 0 has no real root . ⇒ k2 − 5k + 3k − 15 = 0
(ii) Given equation is 2 x 2 + x − 1 = 0 ⇒ k( k − 5) + 3( k − 5) = 0
6 CBSE Term II Mathematics X (Basic)

⇒ ( k + 3) ( k − 5) = 0 would have been 11 more than five times her actual


⇒ k = − 3, 5 age, what is her age now?
Example 14. The denominator of a fraction is 3 more Sol. Let the actual age of Zeba = x yr
than its numerator. The sum of the fraction and its Her age when she was 5 yr younger = ( x – 5) yr
29 Now, by given condition,
reciprocal is . Find the fraction.
10 Square of her age = 11 more than five times her actual age
Sol. Let numerator = x ( x – 5)2 = 5 × actual age + 11
Then denominator = x + 3 ⇒ ( x – 5)2 = 5x + 11
x
∴ The fraction is the form of ⇒ x 2 + 25 – 10x = 5x + 11
x+3
⇒ x 2 – 15x + 14 = 0
According to the question,
x x + 3 29 ⇒ x 2 – 14x – x + 14 = 0 [by splitting the middle term]
+ =
x+3 x 10 ⇒ x ( x – 14) – 1 ( x – 14) = 0
x + ( x + 3)
2 2
29 ⇒ ( x – 1) ( x – 14) = 0
⇒ =
x ( x + 3) 10 ⇒ x = 14
⇒ 10 ( x 2 + x 2 + 9 + 6x ) = 29 ( x 2 + 3x ) [here, x ≠ 1 because her age is x – 5. So, x – 5 = 1 – 5 = – 4
i.e. age cannot be negative]
⇒ 20x 2 + 60x + 90 = 29x 2 + 87 x Hence, required Zeba’s age now is 14 yr.
⇒ 9x 2 + 27 x − 90 = 0
Example 17. A two-digit number is such that the
⇒ x 2 + 3x − 10 = 0 [divide by 9] product of its digit is 35. When 18 is added to the
⇒ x + 5x − 2 x − 10 = 0
2
[by splitting middle term] number the digits interchange their places. Find
the number.
⇒ x ( x + 5) − 2 ( x + 5) = 0
Sol. Let the ten’s digit number be x.
⇒ ( x + 5) ( x − 2 ) = 0
According to the question,
⇒ x + 5 = 0 and x − 2 = 0
Product of the digits = 35
⇒ x = − 5 and x = 2
i.e. Ten’s digits × Unit digit = 35
Example 15. Find a natural number whose square 35
⇒ Units digit =
diminished by 84 is equal to thrice of 8 more than x
the given number. 35
∴ Two digit number = 10x +
Sol. Let n be a required natural number. x
Square of a natural number diminished by 84 = n 2 – 84 Also it is given that if 18 is added to the number, the digits
And thrice of 8 more than the natural number = 3 ( n + 8) gets interchange.
Now, by given condition, 35 35
∴ 10x + + 18 = 10 × +x
n 2 – 84 = 3 ( n + 8) x x
10x 2 + 35 + 18x 350 + x 2
⇒ n 2 – 84 = 3n + 24 ⇒ =
x x
⇒ n 2 – 3n – 108 = 0
⇒ 9x 2 + 18x − 315 = 0
⇒ n 2 – 12 n + 9n – 108 = 0
⇒ x 2 + 2 x − 35 = 0 [divide by 9]
[by splitting the middle term]
⇒ n ( n – 12 ) + 9 ( n – 12 ) = 0 ⇒ x + 7 x − 5x − 35 = 0
2

⇒ ( n – 12 ) ( n + 9) = 0 ⇒ x (x + 7) − 5 (x + 7) = 0
⇒ n = 12 ⇒ ( x − 5) ( x + 7 ) = 0
[Q n ≠ – 9 because n is a natural number] ⇒ x = 5, − 7
Hence, the required natural number is 12. But a digit can never be negative.
So, x = − 7 is rejected.
Example 16. If Zeba were younger by 5 yr than what
∴ The required number is
she really is, then the square of her age (in years)
10 × 5 + 5 = 50 + 5
= 55
CBSE Term II Mathematics X (Basic) 7

Chapter
Practice
PART 1
Objective Questions
l
Multiple Choice Questions 8. The roots of the quadratic equation x 2 + 4 x + 5 = 0
1. Which of the following is a quadratic equation? are [CBSE 2020 (Basic)]
[NCERT Exemplar] (a) real (b) real and distinct
(a) x 2 + 2 x + 1 = ( 4 − x )2 + 3 (c) not real (d) real and equal

(b) − 2 x 2 = ( 5 − x )  2 x − 
2 9. If b = 0 , c < 0, then the roots of x 2 + bx + c = 0 are
 5 numerically equal and opposite in sign.
3 [NCERT Exemplar]
(c) ( k + 1 ) x 2 + x = 7, where k = −1
2 (a) True (b) False
(d) x 3 − x 2 = ( x − 1)3 (c) Can’t determined (d) None of these
2. Which of the following is not a quadratic
[NCERT equation?
Exemplar] 10. The value of the roots of quadratic equation
(a) 2 ( x – 1 ) = 4x – 2 x + 1
2 2 x 2 − x − 6 = 0 are ...... . [CBSE 2020 (Basic)]
(b) 2 x – x 2 = x 2 + 5 (a) 3 and − 2 (b) 3 and 2
(c) ( 2 x + 3 )2 = 3x 2 – 5x (c) − 3 and − 2 (d) − 3 and 2
(d) ( x 2 + 2 x )2 = x 4 + 3 + 4x 2 11. Which constant must be added and subtracted to
3
3. If a number x is added to twice its square, then the solve the quadratic equation 9 x 2 + x − 2 = 0.
resultant is 21. Then the quadratic representation of 4
[NCERT Exemplar]
this statement is 1 1 1 9
(a) 2x − x + 21 = 0
2
(b) 2x + x − 21 = 0
2 (a) (b) (c) (d)
8 64 4 64
(c) 2x − x − 20 = 0
2
(d) None of these
12. Solve 12x 2 + 5x − 3 = 0.
4. Which of the following equations has 2 as a root? 1 4
(a) , (b)
1 3
, (c) −
1 3
,
1
(d) , −
3
(a) x − 4x + 5 = 0
2
(b) x + 3x − 12 = 0
2 3 3 2 4 3 4 3 4
(c) 2 x 2 − 7 x + 6 = 0 (d) 3x 2 − 6x − 2 = 0 13. The discriminant of the quadratic equation
5. If one zero of a quadratic polynomial ( kx + 3x + k ) 2 2x 2 − 4 x + 3 = 0 is [CBSE 2020 (Basic)]
is 2, then the value of k is [CBSE 2020 (Basic)] (a) −8 (b) 10 (c) 8 (d) 2 2
5 5 6 6 14. If the discriminant of the equation
(a) (b) − (c) (d) −
6 6 5 5 6x 2 − bx + 2 = 0 is 1, then the value of b is [CBSE 2012]
6. Which of the following equation has root as 3? (a) 7 (b) − 7
(a) x 2 – 5x + 6 = 0 (b) – x 2 + 3x – 3 = 0 (c) Both (a) and (b) (d) None of these
(c) 2 x 2 –
3
x +1=0 (d) 3x 2 – 3x + 3 = 0 15. If quadratic equation 3x 2 − 4 x + k = 0 has equal
2
roots, then the value of k is ……… .
7. 0.2 is a root of the equation x 2 − 0.4 = 0? [CBSE 2020 (Basic)]
[NCERT Exemplar] 1 4
(a) (b)
(a) True (b) False 3 3
(c) Can’t determined (d) None of these 3
(c) (d) None of these
2
8 CBSE Term II Mathematics X (Basic)

16. The roots of the equation, x 2 + bx + c = 0 are equal, (i) What will be the distance covered by Ajay’s car in
if ……… [CBSE 2020 (Basic)] two hours?
(a) D > 0 (b) D < 0 (a) 2 ( x + 5) km (b) ( x – 5) km
(c) D = 0 (d) None of these (c) 2 ( x + 10) km (d) ( 2 x + 5) km

17. If the discriminant of the equation (ii) Which of the following quadratic equation
describe the speed of Raj’s car?
kx 2 − 3 2x + 4 2 = 0 is 14, then the value of k is (a) x 2 − 5x − 500 = 0 (b) x 2 + 4x − 400 = 0
1 1 1
(a) 2 (b) (c) (d) (c) x 2 + 5x − 500 = 0 (d) x 2 − 4x + 400 = 0
3 2 2 4 2
(iii) What is the speed of Raj’s car?
18. Which of the following equations has two distinct (a) 20 km/h (b) 15 km/h
real roots? [NCERT Exemplar] (c) 25 km/h (d) 10 km/h
9
(a) 2 x 2 − 3 2 x + =0 (b) x 2 + x − 5 = 0 (iv) How much time took Ajay to travel 400 km?
4
(a) 20 h (b) 40 h (c) 25 h (d) 16 h
(c) x 2 + 3x + 2 2 = 0 (d) 5x 2 − 3x + 1 = 0
(v) How much time took Raj to travel 400 km?
19. Which of the following equations has no real roots? (a) 15 h (b) 20 h (c) 18 h (d) 22 h
(a) x 2 − 4x + 3 2 = 0 (b) x 2 + 4x − 3 2 = 0 25. The speed of a motor boat is 20 km/h. For covering
(c) x 2 − 4x − 3 2 = 0 (d) 3x 2 + 4 3x + 4 = 0 the distance of 15 km the boat took 1 h more for
upstream than downstream. [CBSE Question Bank]
20. ( x 2 + 1)2 − x 2 = 0 has [NCERT Exemplar]
(a) four real roots (b) two real roots
(c) no real roots (d) one real root
21. A two-digit number is such that the product of its
digits is 14. If 45 is added to the number, the digits
interchange their places. Find the number.
[CBSE 2020 (Basic)] Downstream (a) Upstream (b)
(a) 21 (b) 27 (c) 24 (d) 20
22. The difference of squares of two numbers is 204.
The square of the smaller number is 4 less than
10 times the larger number. Find the two2020
[CBSE numbers.
(Basic)]
(a) 20, 14 (b) 16, 14
(c) 18, 12 (d) None of these
Direction of boat Direction of boat
23. One year ago, a man was 8 times as old as his son. Direction of stream Direction of stream
Now, his age is equal to the square of his son’s age.
Present age of man is (i) Let speed of the stream be x km/h, then speed of
(a) 49 yr (b) 37 yr (c) 59 yr (d) 39 yr the motorboat in upstream will be
(a) 20 km/h (b) ( 20 + x ) km/h
l
Case Based MCQs (c) ( 20 − x ) km/h (d) 2 km/h
24. Raj and Ajay are very close friends. Both the (ii) What is the relation between speed, distance and
families decide to go to Ranikhet by their own cars. time?
Distance
Raj’s car travels at a speed of x km/h while Ajay’s (a) Speed =
Time
car travels 5 km/h faster than Raj’s car. Raj took 4 h
Speed
more than Ajay to complete the journey of 400 km. (b) Distance =
[CBSE Question Bank] Time
(c) Time = Speed × Distance
(d) Speed = Distance × Time
(iii) Which is the correct quadratic equation for the
speed of the current ?
(a) x 2 + 30x − 200 = 0 (b) x 2 + 20x − 400 = 0
(c) x 2 + 30x − 400 = 0 (d) x 2 − 20x − 400 = 0
(iv) What is the speed of current ?
(a) 20 km/h (b) 10 km/h (c) 15 km/h (d) 25 km/h
CBSE Term II Mathematics X (Basic) 9

(v) How much time boat took in downstream?


(a) 90 min (b) 15 min PART 2
(c) 30 min (d) 45 min
26. By quadratic formula, the roots of the quadratic Subjective Questions
equation ax 2 + bx + c = 0, a ≠ 0 are given by
l
Short Answer Type Questions
−b ± b 2 − 4 ac −b ± D
x= or x = 1. Check whether the following are quadratic
2a 2a
equations or not.
where, D = b − 4 ac is called discriminant.
2
(i) ( x − 1) ( x + 2 ) = ( x − 3 ) ( x + 1)
(i) The roots of the quadratic equation
(ii) ( x + 2 ) 2 = 4 ( x + 3 )
8 x 2 − 22 x − 21 = 0 are
7 3 7 3 1
(a) − ,− (b) , 2. If x = is root of the equation
2 4 2 4 3
7 3 7 3
(c) , − (d) − , Px 2 + ( 3 − 2 ) x − 1 = 0, then find the value of
2 4 2 4
(ii) The discriminant of x 2 + x + 7 = 0 is P 2 + 1. [NCERT Exemplar]
(a) 27 (b) − 27 3. In each of the following equations, determine the
(c) 27 (d) −27 value of k for which the given value is a solution of
(iii) Roots of 4 x 2 − 2 x = 3 are the equation.
(a) Real and distinct (b) Real and equal (i) kx 2 + 2 x − 3 = 0, x = 2
(c) Imaginary (d) More than two real roots
(ii) x 2 + 2ax − k = 0, x = − a
(iv) The value of k for which
4 x 2 + kx + 9 = 0 has real and equal roots is 4. Find the value of k in the following equations
(a) 12 (b) − 12 (i) x 2 − 2 kx − 6 = 0, when x = 3
(c) Both (a) and (b) (d) None of these 5 1
(ii) x 2 − kx − = 0, when x =
(v) The least positive value of k for which 4 2
x 2 + kx + 16 = 0 has real roots, is −1 1
(a)18 (b) 4 (c) 2 (d) 8 5. Determine whether x = , x = are the solutions
2 3
27. Seven years ago, Varun’s age was five times the of the given equation 6 x 2 − x − 2 = 0, or not.
square of Swati’s age. Three years hence, Swati’s
age will be two-fifth of Varun’s age. 6. Solve the quadratic equation by factorisation
method.
(i) If seven years ago, Swati’s age be
x yr, then Varun’s age is 4 3x 2 + 5x − 2 3 = 0
(a) ( 5x − 7 )2 yr (b) 5x 2 yr 7. Solve for
(c) ( 5x 2 + 7 ) yr (d) ( 5x 2 − 7 ) yr 16 15
x: −1 = ; x ≠ 0 , − 1.
(ii) After three years, Swati’s age is x x +1
(a) ( x + 3) yr (b) ( x − 3) yr
8. Find the roots of the equation
(c) ( x + 7 ) yr (d) ( x + 10) yr
ax 2 + a = a 2 x + x. [CBSE 2012]
(iii) The quadratic equation related to the given
problem is 9. Solve for x , 6 x + 7 − ( 2x − 7 ) = 0 [CBSE 2016]
(a) 2 x 2 − x − 6 = 0 (b) 5x 2 − x + 6 = 0
10. Find the numerical difference of the
(c) 3x 2 − 2 x + 5 = 0 (d) 7 x 2 − 3x + 1 = 0
roots of equation x 2 − 7 x − 18 = 0. [CBSE 2015]
(iv) Present age of Varun’s is
(a) 27 yr (b) 20 yr
11. Using the quadratic formula, solve the quadratic
(c) 30 yr (d) 37 yr
equation.
(v) If Swati’s present age 10 yr, then present age of 3x 2 + 11x + 6 3 = 0
Varun’s is 12. If the discriminant of the equation 5x 2 − sx + 4 = 0
(a) 40 yr (b) 47 yr
is 1, then find the value of s.
(c) 45 yr (d) 52 yr
10 CBSE Term II Mathematics X (Basic)

13. Find the values of p for which the quadratic equation 27. The sum of the reciprocals of Anjali’s age 3 yr ago
x 2 − 2px + 1 = 0 has no real roots. [CBSE 2020 (Basic)] 1
and 5 yr from now is . Find the present age of
3
14. Find the value of k for which the quadratic equation Anjali.
2x 2 − kx + k = 0 has equal roots. [NCERT Exemplar]
28. The difference of the squares of two numbers is
15. Find the values of k for which the equation 180. The square of the smaller number is 8 times
9 x 2 + 3kx + 4 = 0 has real roots. the larger number. Find the two numbers.
[CBSE 2020 (Basic)]
16. The sum of the reciprocals of the ages of a child 29. ‘‘John and Janvi together have 45 marbles. Both of
1
3 yr ago and 5 yr hence from now is . Find his them lost 5 marbles each and the product of the
3 number of marbles they now have, is 124. Find out
present age. [CBSE 2020 (Basic)] how many marbles they had to start with?’’
17. The sum of two numbers is 11 and the sum of their 30. A person on tour has ` 4200 for his expenses. If he
11 extends his tour for 3 days, he has to cut down his
reciprocals is . Find the numbers.
28 [CBSE 2020 (Basic)] daily expenses by ` 70. Find the original duration of
the tour. [CBSE 2020 (Basic)]
18. Sum of the areas of two squares is 452 m 2 . If the
difference of their perimeters is 8 m, find the sides 31. The difference of two natural numbers is 5 and the
1
of the two squares. [CBSE 2020 (Basic)] difference of their reciprocals is .
10
l
Long Answer Type Questions Find the numbers. [CBSE 2020 (Basic)]

19. If x = 2 and x = 3 are roots of the equation 32. A train travels 360 km at a uniform speed. If the
speed had been 5 km/h more, it would have taken
3x 2 − 2ax + 2b = 0, then find the values of a and b. 1 h less for the same journey. Find the original
20. Find the nature of roots of the following quadratic speed of the train. [CBSE 2020 (Basic)]
equations. If the real roots exist, then also find the 7
roots. 33. Two water taps together can fill a tank in 1 h.
8
(i) 4 x 2 + 12 x + 9 = 0 (ii) 3 x 2 + 5 x − 7 = 0 The tap with longer diameter takes 2 h less than
21. Find the value of k for which the given equation has the tap with smaller one to fill the tank separately.
Find the time in which each tap can fill the tank
equal roots.
separately. [CBSE 2019]
( k − 12) x 2 + 2( k − 12) x + 2 = 0
22. If x = − 2 is a root of the equation 3x 2 + 7 x + p = 0.
l
Case Based Questions
Find the values of k, so that the roots of the equation 34. In the centre of a rectangular lawn of dimensions
x 2 + k (4 x + k − 1) + p = 0 are equal. 50 m × 40 m, a rectangular pond has to be
[CBSE 2015]
constructed, so that the area of the grass
23. Find two consecutive odd natural numbers, sum of surrounding the pond would be 1184 m 2
whose squares is 130. [CBSE 2013]
24. A piece of cloth costs ` 200. If the piece was 5 m
longer and each metre of cloth costs ` 2 less, the
cost of the piece would have remained unchanged.
How long is the piece and what is the original rate
per metre? [CBSE 2015]
25. A two-digit number is such that the product of its
digits is 14. If 45 is added to the number, the digits (i) If the distance between pond and lawn is x m. Find
the length and breadth of rectangular pond.
interchange their places. Find the number.
[CBSE 2020 (Basic)] (ii) Find the quadratic equation related to the given
problem.
26. If the price of a book is reduced by ` 5, a person can
buy 4 more books for ` 600. Find the original price (iii) Find the length and breadth of the pond.
of the book. [CBSE 2020 (Basic)]
CBSE Term II Mathematics X (Basic) 11

SOLUTIONS
Objective Questions which is not of the form ax 2 + bx + c = 0, a ≠ 0.
1. (d) (a) Given that, Thus, the equation is not quadratic.
x 2 + 2 x + 1 = ( 4 − x )2 + 3 This is a cubic equation.
⇒ x 2 + 2 x + 1 = 16 + x 2 − 8x + 3 3. (b) Let the number be x.
Then according to the given condition,
⇒ 10x − 18 = 0
which is not of the form ax 2 + bx + c = 0, a ≠ 0. 2 x 2 + x = 21
Thus, the equation is not a quadratic equation. ⇒ 2 x 2 + x − 21 = 0
4. (c) (a) Substituting x = 2 in x 2 − 4x + 5 , we get
(b) Given that, − 2 x 2 = ( 5 − x )  2 x − 
2
 5 ( 2 )2 − 4 ( 2 ) + 5 = 4 − 8 + 5 = 1 ≠ 0.

⇒ − 2 x 2 = 10x − 2 x 2 − 2 +
2x So, x = 2 is not a root of x 2 − 4x + 5 = 0.
5 (b) Substituting x = 2 in x 2 + 3x − 12, we get
⇒ 50x + 2 x − 10 = 0 ⇒ 52 x − 10 = 0
( 2 )2 + 3( 2 ) − 12
which is also not a quadratic equation.
3 = 4 + 6 − 12 = −2 ≠ 0
(c) Given that, x 2 ( k + 1) + x = 7
2 So, x = 2 is not a root of x 2 + 3x − 12 = 0.
Given, k = −1 (c) Substituting x = 2 in 2 x 2 − 7 x + 6, we get

3
x ( − 1 + 1) + x = 7
2 2 ( 2 )2 − 7 ( 2 ) + 6 = 2 ( 4) − 14 + 6
2 = 8 − 14 + 6 = 14 − 14 = 0
⇒ 3x − 14 = 0 So, x = 2 is root of the equation 2 x 2 − 7 x + 6 = 0.
which is also not a quadratic equation.
(d) Substituting x = 2 in 3x 2 − 6x − 2 , we get
(d) Given that, x 3 − x 2 = ( x − 1 )3
3( 2 )2 − 6( 2 ) − 2 = 12 − 12 − 2 = −2 ≠ 0
⇒ x 3 − x 2 = x 3 − 3x 2 (1 ) + 3x (1 )2 − (1 )3
So, x = 2 is not a root of 3x 2 − 6x − 2 = 0.
[Q (a − b )3 = a 3 − b 3 + 3ab 2 − 3a 2 b]
5. (d) Let P ( x ) = kx 2 + 3x + k
⇒ x 3 − x 2 = x 3 − 3x 2 + 3x − 1
Since, 2 is a zero of the quadratic polynomial
⇒ − x + 3x 2 − 3x + 1 = 0 ⇒ 2 x 2 − 3x + 1 = 0
2
P ( x ) = kx 2 + 3x + k
which represents a quadratic equation because it has
the quadratic form ax 2 + bx + c = 0, a ≠ 0. Therefore, P( 2 ) = 0
or 0 = k( 2 )2 + 3 × 2 + k
2. (d) (a) Given that, 2 ( x – 1)2 = 4x 2 – 2 x + 1
or 4k + 6 + k = 0
⇒ 2 ( x 2 + 1 – 2 x ) = 4x 2 – 2 x + 1
or 5k = − 6
⇒ 2 x 2 + 2 – 4x = 4x 2 – 2 x + 1
−6
⇒ 2 x2 + 2 x – 1 = 0 or k=
5
which represents a quadratic equation because it has 6. (a) (a) Given that, x 2 – 5x + 6 = 0
the quadratic form ax 2 + bx + c = 0, a ≠ 0. Put x = 3, we get
(b) Given that, 2 x – x 2 = x 2 + 5 ( 3)2 − 5( 3) + 6 = 9 − 15 + 6 = 0
⇒ 2 x2 – 2 x + 5 = 0 Hence, x = 3 is a root of the equation.
which also represents a quadratic equation because it (b) – x 2 + 3x – 3 = 0
has the quadratic form ax 2 + bx + c = 0, a ≠ 0. Put x = 3, we get
(c) Given that, ( 2 ⋅ x + 3 ) = 3x – 5x
2 2 −( 3)2 + 3( 3) − 3 = −9 + 9 − 3 = −3 ≠ 0
⇒ 2 ⋅ x + 3 + 2 6 ⋅ x = 3x 2 – 5x
2 Hence, x = 3 is not a root of the equation.
3
⇒ x – (5 + 2 6) x – 3 = 0
2
(c) 2 x 2 – x +1=0
2
which also represents a quadratic equation because it
Put x = 3, we get
has the quadratic form ax 2 + bx + c = 0, a ≠ 0. 3( 3)
2 ( 3) 2 − +1
(d) Given that, ( x 2 + 2 x )2 = x 4 + 3 + 4x 2 2 ( 3)
⇒ x 4 + 4x 2 + 4x 3 = x 4 + 3 + 4x 2 ⇒9 2 −
9
+1 =
9
+1 ≠ 0
⇒ 4x 3 – 3 = 0 2 2
Hence, x = 3 is not a root of the equation.
12 CBSE Term II Mathematics X (Basic)

(d) 3x 2 – 3x + 3 = 0 12. (d) Given quadratic equation is12 x 2 + 5x − 3 = 0.


Put x = 3, we get On comparing the given equation with
3( 3)2 − 3( 3) + 3 = 27 − 9 + 3 = 21 ≠ 0. ax 2 + bx + c = 0, we get a = 12 , b = 5 and c = −3
Hence, x = 3 is not a root of the equation. On substituting the values of a = 12 , b = 5 and c = −3 in
7. (b) False, since 0.2 does not satisfy the quadratic formula,
equation i.e. (0.2 )2 − 0.4 = 004
. − 0. 4 ≠ 0. − b ± b 2 − 4ac
x= , we get
8. (c) Given, x + 4 x + 5 = 0
2 2a

On comparing with ax 2 + bx + c = 0, we get −5 ± ( 5)2 − 4 × 12 × ( −3)


x=
2 × 12
a = 1, b = 4, c = 5
−5 ± 25 + 144
Now, discriminant ⇒ x=
24
D = b 2 − 4ac
− 5 ± 169
∴ D = ( 4)2 − 4(1)( 5) =
24
16 − 20 = − 4 < 0 − 5 ± 13
=
Q D <0 24
−5 + 13 8 1
∴The roots of given quadratic equation are not real. Now, x= = = [taking +ve sign]
24 24 3
9. (a) Given that, b = 0 and c < 0 and quadratic equation
−5 − 13 18 3
Or x= = − =− [taking −ve sign]
x 2 + bx + c = 0 ...(i) 24 24 4
Put b = 0 in Eq. (i), we get 1 3
Hence, the roots of the given equation are and − .
x2 + 0 + c = 0 3 4
13. (a) Given, 2 x 2 − 4x + 3 = 0
here, c > 0
⇒ x2 = – c ∴ − c > 0 
  We know that, D = b 2 − 4ac
∴ x=± −c ∴ D = ( −4)2 − 4( 2 )( 3)
So, the roots of x 2 + bx + c = 0 are numerically equal and = 16 − 24
opposite in sign. ⇒ D = −8
10. (a) Given equation is x 2 − x − 6 = 0 14. (c) Given, 6x 2 − bx + 2 = 0
⇒ x 2 − 3x + 2 x − 6 = 0 On comparing with Ax 2 + Bx + C = 0, we get
A = 6, B = − b and C = 2
⇒ x( x − 3) + 2( x − 3) = 0
We know that,
⇒ ( x − 3) ( x + 2 ) = 0
Discriminant, D = B2 − 4AC
⇒ x = 3, − 2
⇒ 1 = ( − b )2 − 4 × 6 × 2 [given, D = 1]
Hence, roots are 3 and −2.
3 ⇒ 1 = b − 48
2
11. (b) Given equation is 9x 2 + x − 2 = 0.
4 ⇒ b 2 = 49
1 ⇒ b=±7
( 3x ) + ( 3x ) − 2 = 0
2 [taking square root on both sides]
4
Hence, the required value of b is −7 or 7.
On putting 3x = y, we have
1 15. (b) We have, 3x 2 − 4x + k = 0
y2 + y − 2 = 0
4 ∴ For equal roots, D =0
2 2
1  1  1 ⇒ D = b 2 − 4ac
y2 + y +   −   − 2 = 0
4  8  8 ⇒ b 2 − 4ac = 0
2
 1 1 ⇒ ( −4)2 − 4( 3) × k = 0 [Q a = 3, b = −4, c = k]
y +  = + 2
 8 64
⇒ 16 −12 k = 0
[Q (a + b 2 ) = a 2 + b 2 + 2ab] 16 4
2 ⇒ k= =
 1 1 + 64⋅ 2 12 3
y +  =
 8 64 4
Thus, the value of k is .
1 3
Thus, must be added and subtracted to solve the given
64 16. (c) If discriminant, D = b 2 − 4ac = 0, then roots are equal and
equation. real.
CBSE Term II Mathematics X (Basic) 13

17. (d) Given quadratic equation is Then, D = b 2 − 4ac = ( 4)2 − 4(1)( −3 2 ) = 16 + 12 2 > 0
kx − 3 2 x + 4 2 = 0
2
Hence, the equation has real roots.
On comparing with ax + bx + c = 0, we get
2 (c) Given equation is x 2 − 4x − 3 2 = 0
a = k, b = − 3 2 and c = 4 2 On comparing the equation with ax 2 + bx + c = 0,
we get
Q Discriminant, D = b 2 − 4ac a = 1, b = − 4 and c = – 3 2
⇒ 14 = ( − 3 2 )2 − 4 × k × 4 2 Then, D = b 2 − 4ac = ( −4)2 − 4(1)( −3 2 )
⇒ 14 = 18 − 16 2 k = 16 + 12 2 > 0
⇒ 16 2 k = 4 Hence, the equation has real roots.
1 (d) Given equation is 3x 2 + 4 3x + 4 = 0.
⇒ k= On comparing the equation withax 2 + bx + c = 0,we get
4 2
18. (b) (a) Given equation is 2 x 2 − 3 2 x + 9 / 4 = 0, we get a = 3, b = 4 3 and c = 4
On comparing with ax 2 + bx + c = 0 Then, D = b 2 − 4ac = ( 4 3 )2 − 4( 3)( 4)
a = 2, b = −3 2 and c = 9/4 = 48 − 48 = 0
Now, D = b 2 − 4ac Thus, the equation has real roots.
= ( −3 2 )2 − 4( 2 )(9 / 4) = 18 − 18 = 0 Hence, x 2 − 4x + 3 2 = 0 has no real roots.
Thus, the equation has real and equal roots. 20. (c) Given equation is ( x 2 + 1 )2 − x 2 = 0
(b) The given equation is x 2 + x − 5 = 0 ⇒ x4 + 1 + 2 x2 − x2 = 0
On comparing with ax + bx + c = 0, we get
2
[Q(a + b )2 = a 2 + b 2 + 2ab]
a = 1, b = 1 and c = − 5 ⇒ x4 + x2 + 1 = 0
The discriminant of x 2 + x − 5 = 0 is
Let x2 = y
D = b 2 − 4ac = (1)2 − 4 (1) ( −5)
∴ ( x 2 )2 + x 2 + 1 = 0
= 1 + 20 = 21
⇒ b 2 − 4ac > 0 ⇒ y2 + y + 1 = 0

So, x 2 + x − 5 = 0 has two distinct real roots. On comparing with ay 2 + by + c = 0, we get

(c) Given equation is x 2 + 3x + 2 2 = 0 a = 1, b = 1 and c = 1


On comparing with ax 2 + bx + c = 0, we get Discriminant, D = b 2 − 4ac
a = 1, b = 3 and c = 2 2 = (1 )2 − 4 (1 ) (1 )
Now, D = b 2 − 4ac = ( 3)2 − 4(1)( 2 2 ) =1 − 4 = − 3
=9−8 2 <0 Since, D <0
∴ Roots of the equation are not real. ∴ y 2 + y + 1 = 0 i.e. x 4 + x 2 + 1 = 0
(d) Given equation is, 5x 2 − 3x + 1 = 0 or ( x 2 + 1)2 − x 2 = 0 has no real roots.
On comparing with ax 2 + bx + c = 0, we get
21. (b) Let the ten’s digit of the number be x.
a = 5, b = − 3, c = 1
According to the question,
Now, D = b 2 − 4ac = ( −3)2 − 4( 5)(1) = 9 − 20 <0
Product of the digits = 12
Hence, roots of the equation are not real.
i.e. Ten’s digit × Unit’s digit = 14
19. (a) (a) The given equation is x 2 − 4x + 3 2 = 0.
14
On comparing with ax 2 + bx + c = 0, we get ⇒ Unit’s digit =
x
a = 1, b = − 4 and c = 3 2 14
∴ Two-digit number = 10x +
The discriminant of x 2 − 4x + 3 2 = 0 is x
D = b 2 − 4ac Also, it is give that if 45 is added to the number, the digits
get interchange.
= ( − 4)2 − 4 (1) ( 3 2 )
14 14
= 16 − 12 2 = 16 − 12 × (1 .41) 10x + + 45 = 10 × + x
x x
= 16 − 16.92 = − 092
.
⇒ 10x 2 + 14 + 45x = 140 + x 2
⇒ b 2 − 4ac < 0
⇒ 9x 2 + 45x − 126 = 0
(b) The given equation is x 2 + 4x − 3 2 = 0
⇒ x 2 + 5x − 14 = 0
On comparing the equation withax 2 + bx + c = 0, we get
a = 1, b = 4 and c = −3 2 ⇒ x + 7 x − 2 x − 14 = 0
2
14 CBSE Term II Mathematics X (Basic)

⇒ x( x + 7 ) − 2 ( x + 7 ) = 0 24. (i) (a) Given, Raj’s car travel at a speed of x km/h. Then Ajay’s
⇒ ( x + 7 )( x − 2 ) = 0 car travels a distance in one hour is( x + 5) km. Therefore,
Ajay’s car travels a distance in two hours is 2( x + 5) km.
⇒ x = −7 , 2 Distance
But a digit can never be negative, so x = 2. (ii) (c)Q Time =
Speed
14
Hence, the required two-digit number = 10 × 2 + Time taken by Ajay and Raj to complete the 400 km
2 journey
= 20 + 7 = 27 400 400
t1 = and t 2 =
22. (a) Let the two number be x and y and x > y. x+5 x
According to the question, According to the question,
x − y = 204
2 2
…(i) t 2 = t1 + 4
400 400
y = 10x − 4
2
…(ii) ∴ = +4
x x+5
From Eq. (i), y = x − 204
2 2
…(iii) 100 100
⇒ = + 1 (divide by 4)
From Eqs. (iii) and (ii), x x+5
x 2 − 204 = 10x − 4 ⇒ 100( x + 5) = 100x + x( x + 5)
⇒ x 2 − 10x − 200 = 0 ⇒ 100x + 500 = 100x + x 2 + 5x

10 ± (10)2 − 4(1) ( − 200) ⇒ x 2 + 5x − 500 = 0


⇒ x= (iii) (a) Consider the quadratic equation x 2 + 5x − 500 = 0
2
10 ± 900 10 ± 30 On comparing with ax 2 + bx + c = 0, we get
⇒ x= =
2 2 a = 1, b = 5 and c = − 500
10 + 30 10 − 30 − b ± b 2 − 4ac
⇒ x= or Q x=
2 2 2a
⇒ x = 20 or − 10
− 5 ± ( 5)2 − 4 × (1)( − 500)
Now, from Eq. (ii), =
2 ×1
y 2 = ( 20)2 − 204 = 400 − 204 = 196
− 5 ± 25 + 2000 − 5 ± 2025
⇒ y = 14 = =
2 2
or y 2 = ( − 10)2 − 204 = 100 − 204 −5 ± 45 −50 40
= = , = − 25, 20
2 2 2
= − 104 (not possible)
Since, speed cannot be negative, so we consider only,
∴ The two numbers are x = 20 and y = 14 x = 20.
23. (a) Let present age of his son = x yr Hence, speed of Raj’s car is 20 km/h.
(iv) (d) To travel 400 km, time taken by Ajay
One year ago, his son’s age = ( x − 1) yr 400 400 400
One year ago, man’s age = 8( x − 1) yr t1 = = = = 16 h
( x + 5) 20 + 5 25
= ( 8x − 8) yr (v) (b) To travel 400 km, time taken by Raj,
Present age of man = ( 8x − 8 + 1) yr 400 400
t2 = = = 20 h
= ( 8x − 7 ) yr x 20
According to the question, 25. (i) (c) Since, the speed of stream be x km/h and speed of
8x − 7 = x 2 ⇒ x 2 − 8x + 7 = 0 motorboat is 20 km/h. Therefore, the speed of
motorboat in upstream will be ( 20 − x ) km/h.
which is the required quadratic equation.
(ii) (a) The relation between speed, distance and time is
Now, x 2 − 7 x − x + 7 = 0 [by factorisation] Distance
Speed =
⇒ x( x − 7 ) − 1( x − 7 ) = 0 Time
⇒ ( x − 7 )( x − 1) = 0 (iii) (c)Q Time =
Distance
⇒ x − 7 = 0 or x − 1 = 0 Speed
⇒ x = 7 or x = 1 Here, distance = 15 km/h
Speed of motorboat in downstream = ( 20 + x ) km/h
But x = 1 is not possible because if x = 1, then present age of and speed of motorboat in upstream = ( 20 − x ) km/h
the son and father are same. Time taken by motorboat in downstream and upstream
So, x = 7. 15 15
are t1 = h and t 2 = h.
Hence, present age of his son = 7 yr 20 + x 20 − x
and present age of man = 8 × 7 − 7 = 49 yr.
CBSE Term II Mathematics X (Basic) 15

According to the question, (iv) (c) Given equation is 4x 2 + kx + 9 = 0.


t 2 = 1 + t1 On comparing with ax 2 + bx + c = 0, we get
15 15 15 15
∴ =1 + ⇒ − =1 a = 4, b = k and c = 9
20 − x 20 + x 20 − x 20 + x
Now, D = b 2 − 4ac
⇒ 15 ( 20 + x − 20 + x ) = ( 20 + x )( 20 − x )
= k2 − 4 × 4 × 9 = k2 − 144
⇒ 15( 2 x ) = 400 − x 2
Since, roots of given equation are real and equal.
⇒ x 2 + 30x − 400 = 0
∴ D =0
(iv) (b) Consider quadratic equation,
⇒ k2 − 144 = 0 ⇒ k2 = 144
x 2 + 30x − 400 = 0
⇒ k = ± 12
⇒ x 2 + ( 40 − 10)x − 400 = 0
(v) (d) Given equation is x 2 + kx + 16 = 0
⇒ x 2 + 40x − 10x − 400 = 0
On comparing with ax 2 + bx + c = 0, we get
⇒ x( x + 40) − 10 ( x + 40) = 0
a = 1, b = k and c = 16
⇒ ( x − 10) ( x + 40) = 0
Now, D = b 2 − 4ac
⇒ x = 10, − 40
= k2 − 4 × 1 × 16 = k2 − 64
Since, speed cannot be negative, so we consider only
positive value. Since, roots of given equation are real.
∴ x = 10 ∴ D ≥0
Hence, speed of current is 10 km/h. ⇒ k2 − 64 ≥ 0
(v) (c) The time taken by motorboat in downstream ⇒ k2 ≥ 64
15 15 15 1 ⇒ k ≥ 8 and k ≤ −8
t1 = = = = h = 30 min
20 + x 20 + 10 30 2 Hence, positive least value of k is 8.
26. (i) (c) Given quadratic equation is 27. (i) (b) Seven years ago,
8x − 22 x − 21 = 0
2 Swati’s age = x yr
On comparing the given equation with Varun’s age = 5x 2 yr
ax 2 + bx + c = 0, we get (ii) (d) Swati’s present age = ( x + 7 ) yr
and Varun’s present age = ( 5x 2 + 7 ) yr
a = 8, b = − 22 and c = − 21
By quadratic formula, After three years, we have
Swati’s age = ( x + 7 + 3) = ( x + 10) yr
− ( − 22 ) ± ( − 22 )2 − 4 × 8 × ( − 21)
x= Varun’s age = ( 5x 2 + 7 + 3) = ( 5x 2 + 10)yr
2×8
(iii) (a) According to the question,
22 ± 484 + 672 22 ± 1156
= = 2
x + 10 = ( 5x 2 + 10)
16 16 5
22 ± 34 11 ± 17
⇒ x= = ⇒ 2 x2 − x − 6 = 0
16 8
11 + 17 28 7 (iv) (a) Now, 2 x2 − x − 6 = 0
Now, x= = = [taking + ve sign] ⇒ 2 x 2 − 4x + 3x − 6 = 0
8 8 2
11 − 17 6 3 ⇒ 2x (x − 2) + 3 (x − 2) = 0
or x= =− =− [taking − ve sign]
8 8 4 ⇒ ( 2 x + 3) ( x − 2 ) = 0
7 3 3
Hence, the roots of the given equation are and − . ⇒ x=− ,2
2 4 2
(ii) (b) Given quadratic equation is x 2 + x + 7 = 0. ∴ x=2 [Q age can’t negative]
On comparing with ax 2 + bx + c, we get ⇒ Present age of Varun’s = ( 5x 2 + 7 ) yr
a = 1, b = 1 and c = 7 = ( 5 × 4 + 7 ) yr
Now, discriminant (D) = b 2 − 4ac = 12 − 4 × 1 × 7 = ( 20 + 7 ) yr
= 1 − 28 = − 27 = 27 yr
(iii) (a) Given equation is 4x 2 − 2 x − 3 = 0 (v) (d) Here, Swati’s present age = 10 yr
⇒ x + 7 = 10
On comparing with ax 2 + bx + c = 0, we get
⇒ x=3
a = 4, b = − 2 and c = − 3
So, Varun’s present age = ( 5x 2 + 7 ) yr
∴ Discriminant (D) = b 2 − 4ac
= 5 ( 3) 2 + 7
= ( −2 )2 − 4 × 4 × ( −3) = 4 + 48 = 52 > 0
= 45 + 7 = 52 yr
So, 4x 2 − 2 x = 3 has two distinct real roots.
16 CBSE Term II Mathematics X (Basic)

Subjective Questions On putting x = − a in the given equation, we get


1. (i) Given, ( x − 1)( x + 2 ) = ( x − 3)( x + 1) ...(i) ( −a )2 + 2a( −a ) − k = 0
LHS = ( x − 1) ( x + 2 ) = x 2 + 2 x − x − 2 ⇒ a 2 − 2a 2 − k = 0
= x2 + x − 2 ⇒ − a2 − k = 0
RHS = ( x − 3) ( x + 1) = x 2 + x − 3x − 3 ⇒ k = −a2
= x2 − 2 x − 3 4. (i) Given quadratic equation is
On substituting these values in Eq. (i), we get x 2 − 2 kx − 6 = 0 …(i)
x2 + x − 2 = x2 − 2 x − 3 Since, x = 3 is one of the root of the given quadratic
equation. Then, it satisfies the given equation.
⇒ x − x + x + 2x − 2 + 3 = 0
2 2
So, put x = 3 in Eq. (i), we get
⇒ 3x + 1 = 0
It is not of the form ax 2 + bx + c = 0,a ≠ 0. ( 3)2 − 2 k( 3) − 6 = 0
As a = 0 and it is an equation of degree 1. ⇒ 9 − 6k − 6 = 0
Hence, the given equation does not represent a ⇒ 6k = 3
quadratic equation. 1
⇒ k=
(ii) Given, ( x + 2 )2 = 4( x + 3) ...(ii) 2
x 2 + 4 + 4x = 4x + 12 5
(ii) Given quadratic equation is x 2 − kx − =0
4
[Q(a + b ) = a + b + 2ab]
2 2 2
1
Put x = , we get
⇒ x + 4x − 4x + 4 − 12 = 0
2
2
⇒ x 2 − 8 = 0 or x 2 + 0x − 8 = 0 1
2
1 5
It is of the form ax 2 + bx + c = 0,a ≠ 0.   − k  − = 0
 2  2 4
Hence, given equation represents a quadratic equation. 1 k 5
⇒ − − =0
2. Given equation is 4 2 4
Px 2 + ( 3 − 2 ) x − 1 = 0 1 − 2k − 5
⇒ =0
1 4
and x = is a root of the equation.
3 ⇒ 2k = − 4
2 ⇒ k=−2
 1  1
∴ P  + ( 3 − 2) −1 = 0 5. Given equation is in the form p( x ) = 0, where
 3 3
p( x ) = 6x 2 − x − 2 ...(i)
P 3− 2 − 3
⇒ + =0 −1
3 3 On putting x = in Eq. (i), we get
2
P 2
⇒ − =0  −1  −1  −1
2
3 3 p   = 6  −   − 2
 2  2  2
2
⇒ P= ×3 6 1 6 + 2 − 8 8−8
3 = + −2= =
4 2 4 4
⇒ P= 2 × 3= 6
 −1
⇒ p  =0
∴ P + 1 = ( 6 )2 + 1
2
 2
=6 +1=7 −1
So, x = is a solution of the given equation.
3. (i) We have, 2
kx 2 + 2 x − 3 = 0, here k is unknown. 1
Now, on putting x = in Eq. (i), we get
Since, x = 2 is a solution of given equation, so it will 3
satisfy the given equation. 2
 1  1  1
p  = 6  −   − 2
On putting x = 2 in the given equation, we get  3  3  3
k( 2 )2 + 2( 2 ) − 3 = 0 1 1 6 1
=6× − −2= − − 2
⇒ 4k + 4 − 3 = 0 9 3 9 3
⇒ 4k + 1 = 0 6 − 3 − 18 −15
= = ≠0
−1 9 9
⇒ k=
4  1
⇒ p  ≠ 0
(ii) We have, x 2 + 2ax − k = 0, here k is unknown.  3
Since, x = − a is a solution of given equation, so it will 1
satisfy the given equation. So, x = is not a solution of the given equation.
3
CBSE Term II Mathematics X (Basic) 17

6. Given, 4 3x 2 + 5x − 2 3 = 0. ⇒ ( x + 2 ) ( x − 9) = 0
On comparing with standard form of quadratic equation ⇒ x = − 2, 9
i.e. ax 2 + bx + c = 0, we get So, the roots of given equation are − 2 and 9.
a = 4 3, b = 5 and c = −2 3 ∴ Required numerical difference of the roots
Here, ac = 4 3 × ( −2 3 ) = −24 = 9 − ( − 2 ) = 11
Then, factors of ac are 8 and −3. 11. The given equation is 3x 2 + 11x + 6 3 = 0.
∴ 4 3 x 2 + ( 8 − 3 )x − 2 3 = 0 On comparing with ax 2 + bx + c = 0, we get
⇒ 4 3x + 8x − 3x − 2 3 = 0
2 a = 3, b = 11 and c = 6 3
⇒ 4x( 3x + 2 ) − 3 ( 3x + 2 ) = 0 On substituting the values of a, b and c in the quadratic
formula,
⇒ ( 4x − 3 ) ( 3x + 2 ) = 0
− b ± b 2 − 4ac
⇒ 4x − 3 = 0 x=
2a
and 3x + 2 = 0
− 11 ± (11)2 − 4( 3 )(6 3 )
⇒ x=
3 ⇒ x=
4 2( 3 )
−2 − 11 ± 121 − 72
or x= =
3 2 3
Hence, roots of equation 4 3x + 5x − 2 3 = 0 are
2 3 −11 ± 49
=
4 2 3
−2 −11 ± 7
and . =
3 2 3
16
−1 =
15 −11 + 7 −4 −2
7. Given, ⇒ x= = = [taking +ve sign]
x x +1 2 3 2 3 3
16 15 16( x + 1) − 15x −11 − 7 −18 −9
⇒ − =1 ⇒ =1 and x= = = [taking −ve sign]
x x +1 x ( x + 1) 2 3 2 3 3
⇒ 16x + 16 − 15x = x 2 + x −2 −9 −2 3
Hence, and (or and −3 3) are the required
⇒ x 2 = 16 3 3 3
⇒ x2 = ± 4 solutions of the given equation.
Hence, the roots are 4 and −4. 12. Given equation is 5x 2 − sx + 4 = 0
8. Given that, ax 2 + a = a 2 x + x On comparing with ax 2 + bx + c = 0, we get
⇒ ax 2 − a 2 x − x + a = 0 a = 5, b = − s and c = 4
⇒ ax ( x − a ) − 1 ( x − a ) = 0 ∴ Discriminant (D) = b 2 − 4ac
⇒ (ax − 1) ( x − a ) = 0 = ( − s)2 − 4 × 5 × 4 = s 2 − 80
1 D =1
⇒ x = ,a Given,
a ⇒ s 2 − 80 = 1
9. Given that, 6x + 7 − ( 2 x − 7 ) = 0
⇒ s 2 = 81
⇒ 6x + 7 = 2 x − 7
⇒ s=±9
On squaring both sides, we get 13. Given, quadratic equation is x 2 − 2 px + 1 = 0
6x + 7 = ( 2 x − 7 ) 2 On comparing with ax 2 + bx + c = 0, we get
6x + 7 = 4x + 49 − 28x
2
a = 1, b = − 2 p and c = 1
⇒ 4x 2 − 34x + 42 = 0
Now, D = b − 4ac = ( −2 p)2 − 4(1)(1) = 4p 2 − 4
2
⇒ 2 x 2 − 17 x + 21 = 0 [divide by 2]
Since, roots of given equation are not real.
⇒ 2 x + 14x + 3x + 21 = 0
2
∴ D <0
⇒ 2 x ( x + 7 ) + 3( x + 7 ) = 0 ⇒ 4p 2 − 4 < 0
⇒ ( 2 x + 3) ( x + 7 ) = 0
3 ⇒ 4p 2 < 4
⇒ x = − ,−7 ⇒ p2 < 1
2
10. Given equation is x 2 − 7 x − 18 = 0 ⇒ −1 < p < 1 [Q a 2 < b 2 ⇒ − b < a < b]
⇒ x − 9x + 2 x − 18 = 0
2
Hence, for all the values of p in the interval ( −1, 1), given
⇒ x ( x − 9) + 2 ( x − 9) = 0 quadratic equation has no real roots.
18 CBSE Term II Mathematics X (Basic)

14. Given equation is 2 x 2 − kx + k = 0 17. Let one number be x.


On comparing with ax 2 + bx + c = 0, we get Then, another number = (11 − x )
a = 2, b = − k and c = k [Q sum of two numbers = 11, given]
∴ D = b 2 − 4ac According to the question,
1 1 11
= ( − k)2 − 4 × 2 × k + =
x (11 − x ) 28
= k2 − 8k 11 − x + x 11
⇒ =
Since, the given equation has equal roots. x(11 − x ) 28
∴ D =0 ⇒ x(11 − x ) = 28
⇒ k2 − 8k = 0 ⇒ x 2 − 11x + 28 = 0
⇒ k ( k − 8) = 0 ⇒ x 2 − (7 + 4)x + 28 = 0
⇒ k = 0, 8 ⇒ x 2 − 7 x − 4x + 28 = 0
15. Given quadratic equation is ⇒ x( x − 7 ) − 4( x − 7 ) = 0
9x 2 + 3kx + 4 = 0 ⇒ ( x − 7 )( x − 4) = 0
On comparing with ax 2 + bx + c = 0, we get ⇒ x = 4 or x = 7
a = 9, b = 3k and c = 4 When x = 4, then11 − x = 11 − 4 = 7
Now, D = b 2 − 4ac = ( 3k)2 − 4(9) ( 4) When x = 7, then11 − x = 11 − 7 = 4
= 9k2 − 144 Hence, the numbers are 4 and 7.
Since, roots of given equation are real. 18. Let the side of first square is x m and the side of second
square is y m.
∴ D ≥ 0 ⇒ 9k2 − 144 ≥ 0
∴ According to the question,
⇒ 9( k2 − 16) ≥ 0
x 2 + y 2 = 452 …(i)
∴ k − 16 ≥ 0
2
[Q9 ≠ 0]
4x − 4y = 8
⇒ k2 − ( 4)2 ≥ 0 or x−y=2 …(ii)
⇒ ( k − 4) ( k + 4) ≥ 0 [Qa 2 − b 2 = (a − b )(a + b )] From Eqs. (i) and (ii), we get
⇒ k ≤ − 4 or k ≥ 4 x 2 + ( x − 2 )2 = 452
16. Let present age of child be x yr. ⇒ x 2 + x 2 − 4x + 4 = 452
∴ Child’s age 3 yr ago = ( x − 3) yr ⇒ 2 x 2 − 4x − 448 = 0
and child’s age 5 yr from now = ( x + 5) yr ⇒ x 2 − 2 x − 224 = 0
According to the question, −( −2 ) ± ( − 2 )2 − 4(1) ( − 224)
1 1 1 ∴ x=
+ = 2
x −3 x + 5 3
2 ± 4 + 896 2 ± 900 2 ± 30
x + 5 + x −3 1 = = =
⇒ = 2 2 2
( x − 3)( x + 5) 3
Take + ve sign
2x + 2 1 2 + 30 32
⇒ = x= = = 16
x 2 − 3x + 5x − 15 3 2 2
⇒ 3( 2 x + 2 ) = x 2 + 2 x − 15 ∴ y = x − 2 = 16 − 2 = 14
⇒ 6x + 6 = x 2 + 2 x − 15 ∴ Side of the two squares are 16 m and 14 m.
⇒ x 2 + 2 x − 15 − 6x − 6 = 0 ⇒ x 2 − 4x − 21 = 0 19. Given, 3x 2 − 2ax + 2 b = 0 ...(i)
which is the required quadratic equation. Here, a and b are unknown constants. Since, x = 2 and x = 3
are the solutions of given equation, so it will satisfy the given
Now, by factorisation method, we get equation.
x 2 − 7 x + 3x − 21 = 0
On putting x = 2 and x = 3 one-by-one,
⇒ x( x − 7 ) + 3( x − 7 ) = 0
in Eq. (i), we get
⇒ ( x − 7 )( x + 3) = 0
⇒ x − 7 = 0 or x + 3 = 0 3( 2 )2 − 2a × ( 2 ) + 2 b = 0
⇒ x = 7 or x = −3 ⇒ 3 × 4 − 4a + 2 b = 0
But x = − 3 is not possible because age cannot be negative. ⇒ 12 − 4a + 2 b = 0
∴ x =7 ⇒ − 2( 2a − b − 6) = 0
Hence, child’s present age is 7 yr. ⇒ 2a − b = 6 [Q − 2 ≠ 0] ...(ii)
CBSE Term II Mathematics X (Basic) 19

and 3( 3)2 − 2a × 3 + 2 b = 0 21. Given quadratic equation is


⇒ 27 − 6a + 2 b = 0 ( k − 12 ) x 2 + 2( k − 12 ) x + 2 = 0
⇒ 6a − 2 b = 27 … (iii) On comparing with ax 2 + bx + c = 0 , we get
On multiplying Eq. (ii) by 2 and then subtract it from
a = k − 12 , b = 2( k − 12 ) and c = 2
Eq. (iii), we get
Now, D = b 2 − 4ac
6a − 2 b − 4a + 2 b = 27 − 12
15 = [2( k − 12 )]2 − 4( k − 12 ) ( 2 )
⇒ 2a = 15 ⇒ a =
2 = 4( k − 12 )2 − 8( k − 12 )
15
On substituting a = in Eq. (ii), we get = ( k − 12 ) [4( k − 12 ) − 8]
2
15 = ( k − 12 ) ( 4k − 48 − 8)
2× − b =6
2 = ( k − 12 ) ( 4k − 56)
⇒ 15 − b = 6 Since, roots of given equation are equal.
⇒ b = 15 − 6 = 9 ∴ D =0
⇒ b =9 ⇒ ( k − 12 ) ( 4k − 56) = 0
Hence, the required values of a and b are 15/2 and 9,
⇒ k − 12 = 0 or 4k − 56 = 0
respectively.
56
⇒ k = 12 or k =
20. (i) Given quadratic equation is 4
4x 2 + 12 x + 9 = 0 ⇒ k =12 or k = 14
On comparing with ax + bx + c = 0, we get
2
But k = 12 does not satisfy the given equation because if
a = 4, b = 12 and c = 9 k = 12, then coefficients of x 2 and x become zero.
Now, D = b 2 − 4ac = (12 )2 − 4( 4) (9) Hence, required value of k is 14.
= 144 − 144 = 0 22. Given equation is 3x 2 + 7 x + p = 0
Since, D = 0, so given quadratic equation has two equal Since, x = − 2 is a root of the given equation, so it will satisfy
and real roots which are given by the given equation.
− b ± D −12 ± 0 On putting x = − 2 in the given equation, we get
x= =
2a 2( 4) 3 ( − 2 )2 + 7 ( − 2 ) + p = 0
−12 + 0 −12 − 0 ⇒ 12 − 14 + p = 0
⇒ x= or x =
8 8 ⇒ − 2 + p =0
3 3
⇒ x = − or x = − ⇒ p=2
2 2
On putting p = 2 in x 2 + k ( 4x + k − 1) + p = 0, we get
−3 3
Hence, the roots are and − . x 2 + k ( 4x + k − 1) + 2 = 0
2 2
(ii) Given quadratic equation is ⇒ x 2 + 4kx + ( k2 − k + 2 ) = 0
3x 2 + 5x − 7 = 0 On comparing with ax 2 + bx + c = 0, we get
On comparing with ax 2 + bx + c = 0, we get a = 1, b = 4k and c = k2 − k + 2

a = 3, b = 5 and c = − 7 ∴ D = b 2 − 4ac
Now, D = b 2 − 4ac = ( 5)2 − 4( 3) ( −7 ) = 25 + 84 = 109 = ( 4k)2 − 4 × 1 × ( k2 − k + 2 )
Since, D > 0, so given quadratic equation has two = 16k2 − 4k2 + 4k − 8
distinct real roots which are given by = 12 k2 + 4k − 8
−b ± D
x= Since, roots are equal.
2a
∴ D =0
−5 ± 109
= ⇒ 12 k2 + 4k − 8 = 0 [divide by 4]
2( 3)
⇒ 3k + k − 2 = 0
2
−5 + 109
⇒ x= [taking +ve sign] ⇒ 3k + 3k − 2 k − 2 = 0
2
6
−5 − 109 ⇒ 3k ( k + 1) − 2 ( k + 1) = 0
or x= [taking −ve sign]
6 ⇒ ( 3k − 2 ) ( k + 1) = 0
−5 + 109 −5 − 109 2
Hence, the roots are and . ⇒ k = , −1
6 6 3
20 CBSE Term II Mathematics X (Basic)

23. Let two consecutive odd natural numbers are x and x + 2. ⇒ 9x 2 + 45x − 126 = 0
Then according to the given condition, ⇒ x 2 + 5x − 14 = 0
x 2 + ( x + 2 )2 = 130 ⇒ x + 7 x − 2 x − 14 = 0
2

⇒ x 2 + x 2 + 4x + 4 = 130 ⇒ x( x + 7 ) − 2 ( x + 7 ) = 0
⇒ 2 x 2 + 4x − 126 = 0 ⇒ ( x + 7 )( x − 2 ) = 0
⇒ x = −7 , 2
⇒ x 2 + 2 x − 63 = 0 [divide by 2]
But a digit can never be negative, so x = 2.
⇒ x + 9x − 7 x − 63 = 0
2
14
⇒ x ( x + 9) − 7 ( x + 9) = 0 Hence, the required two-digit number = 10 × 2 +
2
⇒ ( x − 7 ) ( x + 9) = 0
= 20 + 7 = 27
⇒ x = 7, − 9
26. Let the original price of the book be ` x.
Since, natural number cannot be negative.
600
So, we neglect x = − 9. Number of books bought at original price for `600 = .
x
Thus, x = 7 and x + 2 = 7 + 2 = 9
If the price of a book is reduced by ` 5, then the new price of
Hence, two consecutive odd numbers are 7 and 9.
the book is `( x − 5).
24. Let the length of piece be x m. 600
200 Number of books bought at reduced price for `600 = .
Then, rate = ` per m ( x − 5)
x
According to the given condition ,
Now, new length = ( x + 5) m
600 600 600x − 600x + 3000
Since, the cost remains same. − =4 ⇒ =4
200 x −5 x x( x − 5)
∴ New rate = ` per m 3000
x+5 ⇒ = 4 ⇒ x 2 − 5x = 750
According to the given condition, x 2 − 5x
200 200 ⇒ x 2 − 5x − 750 = 0 ⇒ x 2 − 30x + 25x − 750 = 0
= −2
x+5 x ⇒ x( x − 30) + 25( x − 30) = 0
200 100 − x  ⇒ x − 30 = 0 or x + 25 = 0 ⇒ x = 30 or x = − 25
⇒ =2 
x+5  x  ∴ x = 30 [price cannot be negative]
⇒ 100x = ( x + 5) (100 − x ) Hence, the original price of the book is ` 30.
⇒ 100x = 100x − x 2 + 500 − 5x 27. Let present age of Anjali be x yr.
⇒ x 2 + 5x − 500 = 0 ∴ Anjali’s age 3 yr ago = ( x − 3) yr
⇒ x + 25x − 20x − 500 = 0
2 and Anjali’s age 5 yr from now = ( x + 5) yr
⇒ x ( x + 25) − 20 ( x + 25) = 0 According to the question,
⇒ ( x − 20) ( x + 25) = 0 1 1 1
+ =
⇒ x = 20, − 25 x−3 x+ 5 3
x+ 5+ x−3 1
Since, length of piece cannot be negative, so neglect x = − 25. ⇒ =
( x − 3)( x + 5) 3
Thus, x = 20
2x + 2 1
200 200 ⇒ =
Now, rate = = = `10 x 2 − 3x + 5x − 15 3
x 20
⇒ 3( 2 x + 2 ) = x 2 + 2 x − 15
Hence, length of piece is ` 20 m and rate per metre is ` 10.
25. Let the ten’s digit of the number be x. ⇒ 6x + 6 = x 2 + 2 x − 15
According to the question, ⇒ x 2 + 2 x − 15 − 6x − 6 = 0
Product of the digits = 12 ⇒ x 2 − 4x − 21 = 0,
i.e. Ten’s digit × Unit’s digit = 14 which is the required quadratic equation.
14 Now, by factorisation method, we get
⇒ Unit’s digit =
x x 2 − 7 x + 3x − 21 = 0
14 ⇒ x( x − 7 ) + 3( x − 7 ) = 0
∴ Two-digit number = 10x +
x ⇒ ( x − 7 )( x + 3) = 0
Also, it is give that if 45 is added to the number, the digits ⇒ x − 7 = 0 or x + 3 = 0
get interchange. ⇒ x = 7 or x = − 3
14 14 But x = − 3 is not possible because age cannot be negative.
10x + + 45 = 10 × + x
x x ∴ x =7
⇒ 10x 2 + 14 + 45x = 140 + x 2 Hence, Anjali’s present age is 7 yr.
CBSE Term II Mathematics X (Basic) 21

28. Let the required numbers be x and y, where x > y. According to the question,
Given, difference of squares of two numbers = 180 4200 4200
− = 70
We have, x 2 − y 2 =180 …(i) x x+3
and also it is given that the square of smaller number 1 1 
⇒ 4200  − = 70
= 8 × larger number  x x + 3 
We have, y 2 = 8x …(ii) x + 3 − x 70
⇒  x ( x + 3)  = 4200
From Eqs. (i) and (ii), we get  
x 2 −180 = 8x 3 × 420
⇒ = x( x + 3) ⇒ 180 = x 2 + 3x
⇒ x 2 − 8x − 180 = 0 7
⇒ x − 18x + 10x − 180 = 0
2
[by factorisation] ⇒ x 2 + 3x − 180 = 0
⇒ x( x − 18) + 10( x + 10) = 0 ⇒ x + 15x − 12 x − 180 = 0 ⇒ x ( x + 15) − 12( x + 15) = 0
2

⇒ x − 18 = 0 or x + 10 = 0 ⇒ ( x − 12 )( x + 15) = 0 ⇒ x =12 and x = −15


⇒ x =18 or x = −10
Since, x cannot be negative so, x =12
Now, if x =18, then square of smaller number
Thus, the original duration of the tour is 12 days.
8 × 18 = 144 [from Eq. (ii)]
31. Let the natural numbers be x and y, where x > y.
⇒ Smaller number = ± 12
x−y = 5
⇒ Smaller number −12 or −12
x=y+5 …(i)
and if x = −18, which is not possible as square of a number 1 1 1
cannot be negative. and − =
y x 10
Hence, the required numbers are 18 and 12 or 18 and −12 . 1 1 1
29. Given, John and Janvi together have 45 marbles. ⇒ − =
y y + 5 10
Let John has x marbles. [put the value of x from Eq. (i)]
Then, number of marbles Janvi has = 45 − x y + 5−y 1
Q Both of them lost 5 marbles each. ⇒ =
y ( y + 5) 10
∴ The number of marbles John has = x − 5
⇒ 10 × 5 = y 2 + 5y
and the number of marbles Janvi has = 45 − x − 5 = 40 − x
Now, product of the number of marbles = 124 ⇒ y 2 + 5y − 50 = 0
∴ ( x − 5) ( 40 − x ) = 124 ⇒ y + 10y − 5y − 50 = 0 ⇒ y ( y + 10) − 5( y + 10) = 0
2

⇒ 40x − x 2 − 200 + 5x = 124 ⇒ ( y − 5)( y + 10) = 0 ⇒ y = 5 or y = −10


⇒ − x 2 + 45x − 200 − 124 = 0 So, clearly y = −10 is not a natural number
⇒ − x 2 + 45x − 324 = 0 y = 5 and x = y + 5 [from Eq. (i)]
⇒ x 2 − 45x + 324 = 0 [multiplying by ( −1)] y = 5 or x = 5 + 5 = 10
Hence, the natural number 5 and 10.
which is the required quadratic equation.
Now, by factorisation method, we get 32. Let the speed of train be x km/h.
Normal Speed
x 2 − 36x − 9x + 324 = 0
Distance = 360 km
⇒ x( x − 36) − 9( x − 36) = 0 Speed = x km/hr
⇒ ( x − 36)( x − 9) = 0 Distance
⇒ x − 36 = 0 or x − 9 = 0 ∴ Speed =
Time
⇒ x = 36 or x = 9 360
x=
when John has 36 marbles, then Time
Janvi has = 45 − 36 = 9 marbles. 360
Time =
when John has 9 marbles, then x
Janvi has = 45 − 9 = 36 marbles. Speed 5 km/h more
30. Let the original duration of tour be x days. Distance = 360 km
Amount with the person is ` 4200. Speed = ( x + 5) km/h
360
Daily expenses = `
4200 Time = −1
x x
Distance
Given, tour extended for 3 days. ∴ Speed =
Time
Hence, total number of days = ( x + 3) days. 360
4200 x +5=
Daily expenses =  360 
x+3  − 1
 x 
22 CBSE Term II Mathematics X (Basic)

 360  Time taken by smaller tap = 5 h


( x + 5 ) − 1 = 360 …(i)
 x  Time taken by larger tap = x − 2 = 5 − 2 = 3 h
From Eq. (i), 3
When, x =
 360  4
( x + 5 ) − 1 = 360
 x  Time taken by smaller tap = h
3
4
 360 − x 
⇒ ( x + 5 )  = 360 Time taken by larger tap = x − 2
 x 
3 −5
⇒ ( x + 5)( 360 − x ) = 360x = − 2 = , which is not solution.
4 4
⇒ x( 360 − x ) + 5( 360 − x ) = 360x
Hence, time taken by smaller tap = 5 h and time taken by
⇒ 360x − x 2 + 5( 360) − 5x = 360x larger tap = 3 h.
⇒ 360x − x 2 + 1800 − 5x − 360x = 0 34. (i) Given that a rectangular pond has to be constructed in the
centre of a rectangular lawn of dimensions 50 m × 40 m.
⇒ − x 2 − 5x + 1800 = 0
x
⇒ x 2 + 5x − 1800 = 0
x 40 m
⇒ x 2 + 45x − 40x − 1800 = 0 x
⇒ x( x + 45) − 40( x + 45) = 0
⇒ ( x + 45)( x − 40) = 0 x
⇒ x + 45 = 0 ⇒ x = − 45 50 m
⇒ x − 40 = 0 ⇒ x = 40 Now, length of rectangular lawn
Q x is speed of train. It cannot be negative. ( l1 ) = 50 m
∴ x = 40 is the solution. and breadth of rectangular lawn
So, speed of train x = 40 km/h. ( b1 ) = 40 m
33. Let the time taken by smaller tap to fill tank completely = x h ∴ Length of rectangular pond
1 ( l2 ) = 50 – ( x + x ) = 50 – 2 x
So, volume of tank filled by smaller tap in 1 h =
x and breadth of rectangular pond
1 ( b 2 ) = 40 – ( x + x ) = 40 – 2 x
Volume of tank filled by larger tap in 1 h =
x−2 (ii) Given, area of the grass surrounding the pond = 1184 m2
7 15 ∴ Area of rectangular lawn – Area of rectangular pond
Now, time taken by both taps to fill = 1 = h
8 8 = Area of grass surrounding the pond
15 1 15 15 l1 × b1 – l2 × b 2 = 1184
Tank filled by smaller tap in h = × =
8 x 8 8x [Q area of rectangle = length × breadth]
15 1 15 15 ⇒ 50 × 40 − ( 50 − 2 x ) ( 40 − 2 x ) = 1184
Tank filled by larger tap in h = × =
8 x − 2 8 8( x − 2 ) ⇒ 2000 − ( 2000 − 80x − 100x + 4x 2 ) = 1184
15 15 15 1 1  ⇒ 80x + 100x – 4x 2 = 1184
Therefore, + =1 ⇒ + =1
8x 8( x − 2 ) 8  x x − 2  ⇒ 4x 2 − 180x + 1184 = 0
2( x −1) 8 ⇒ x 2 – 45x + 296 = 0 [divide by 4]
⇒ = ⇒ 15( x − 1) = 4( x 2 − 2 x )
x 2 − 2 x 15 (iii) Now, x − 4.5x + 296 = 0 ⇒ x – 37 x – 8x + 296 = 0
2 2

⇒ 15x − 15 = 4x 2 − 8x [by splitting the middle term]


⇒ 23x = 4x 2 + 15 ⇒ x ( x – 37 ) – 8 ( x – 37 ) = 0
⇒ ( x – 37 ) ( x – 8) = 0
⇒ 4x 2 − 23x + 15 = 0
∴ x=8
By using quadratic formula [at x = 37, length and breadth of pond are –24 and –34,
− ( −23) ± ( −23)2 − 4 ⋅ 4 ⋅15 23 ± 529 − 240 respectively but length and breadth cannot
x= ⇒ x= be negative. So, x = 37 cannot be possible]
2⋅4 8
23 ± 289 23 ± 17 ∴ Length of pond = 50 – 2 x = 50 – 2 ( 8)
⇒ x= ⇒ x= = 50 – 16 = 34 m
8 8
23 + 17 40 and breadth of pond = 40 – 2 x
Taking positive sign, x = = =5 = 40 – 2( 8) = 40 – 16 = 24 m
8 8
23 − 17 6 3 Hence, required length and breadth of pond are 34 m
Taking negative sign, x = = = and 24 m, respectively.
8 8 4
When, x = 5
Chapter Test
Multiple Choice Questions (ii) 15 y 2 − 41y − 14 = 0
1. Is − 8 a solution of the equation 3 x 2 + 8 x + 2 = 0? 3
(a) ,
2
(b)
3
,
7
[NCERT Exemplar] 7 3 5 2
2 7 2 7
(a) Yes (c) , (d) ,
(b) No 3 5 5 3
(c) Cannot be determined 1
(iii) 21x 2 − 2x + =0
(d) None of the above 21
2. Solve the quadratic equation 1 1
(a) 21, 3 (b) ,
21 21
x − 14 x + 24 = 0
2
[CBSE 2013] 1 1 3
(c) − 21, (d) ,
(a) 2, 12 (b) 3, 8 21 21 21
(c) 8, 3 (d) None of these
(iv) 6 x 2 − 31x + 40 = 0
3. The roots of a equation 2x + 5 2x + 5 = 0 are
2
(a)
5 8
, (b) −
5 8
,−
2 3 2 3
−5 2 ± 5 −5 2 ± 10
(a) (b) 5 3 2 5
2 4 (c) − , (d) ,
2 8 3 8
−5 2 ± 10
(c)
5
(d) None of these (v) 3 x 2 + 2 5x − 5 = 0
5
4. The quadratic equation 7 y 2 − 4 y + 5 = 0 has (a) 2 5 , − 5 (b) 5 , −
3
(a) Real and distinct
5
(b) Real and equal (c) − 5 , 3 5 (d) − 5 ,
(c) Imaginary 3
(d) More than 2 real roots Short Answer Type Questions
5. If a number is added to twice its square, then 7. Find the roots of the equation x 2 + 182 = 27 x
the resultant is 21. The quadratic representation
of this situation is [CBSE 2014, 15]
8. Find the roots of the quadratic equation
(a) 2 x2 + x − 21 = 0 a2b 2x 2 + b 2x − a2x − 1 = 0
(b) 2 x2 + x + 21 = 0 9. If the roots of the equation x 2 + 2cx + ab = 0 are
(c) 2 x2 − x + 21 = 0
real and unequal, then prove that the equation
(d) 2 x2 − x − 21 = 0 x 2 − 2(a + b) x + a2 + b 2 + 2c2 = 0 has no real roots.
Case Study MCQs Long Answer Type Questions
6. Sohan is preparing for UPSC exam. For this, he 10. If x = − 5 is a root of the quadratic equation
has to practice the chapter of quadratic
equations. So, he started with factorisation 2x 2 + px − 15 = 0 and the quadratic equation
method. p(x 2 + x) + k = 0 has equal roots, then find the
Let two roots of ax 2 + bx + c be p and q. value of k.

∴ ac = p × q and p + q = b 11. A rectangular park is to be designed whose


breadth is 3 m less than its length. Its area is to
Now, factorize each of the following quadratic
be 4 sq m more than the area of a park that has
equations and find the roots.
already been made in the shape of an isosceles
(i) x 2 − 10x + 21 = 0 triangle with its base as the breadth of the
(a) 9, 3 (b) 21, 1 rectangular park and of altitude 12 m. Find its
(c) 3, 7 (d) 3, 9 length and breadth of the rectangular park.

Answers
1. (b) 2. (a) 3. (b) 4. (c) 5. (a) 6. (i) (c) (ii) (d) (iii) (b) (iv) (a) (v) (d) For Detailed Solutions
1 1
7. 13 ,14 8. x = 2 , − 2 10. 7/4 Scan the code
b a
11. Length = 7 m and Breadth = 4 m
24 CBSE Term II Mathematics X (Basic)

CHAPTER 02

Arithmetic
Progressions
In this Chapter...
l Arithmetic Progression
l nth Term of an AP
l Sum of n-Term of an AP
l Arithmetic Mean

Sequence Some numbers arranged in definite order, In general, a, a + d, a + 2d, a + 3d, … represent an arithmetic
according to a definite rule are said to form a sequence. progression, where a is the first term and d is the common
Progression Sequences which follow a definite pattern are difference. This is called general form of an AP.
called progressions. If number of terms in an AP is finite, then it is called a finite
AP, otherwise it is called an infinite AP and such AP ’s do not
Arithmetic Progression have a last term.
An Arithmetic Progression (AP) is a list of numbers in which Method to Check an AP
each term is obtained by adding a fixed number to the When a List of Numbers is Given
preceding term except the first term.
Sometimes, a list of numbers or sequence is given and we
This fixed number is called the common difference (d ) of the have to check that this sequence is an AP or not. For this, we
AP. It can be positive, negative or zero. find the differences of consecutive terms. If these differences
In other words, a list of numbers a 1 , a 2 , a 3 , ..., a n is called are same, then given list of numbers or sequence is an AP,
an arithmetic progression (AP), if there exists a constant otherwise not.
number d (called common difference) such that
Method to Write an AP When First Term and
a 2 − a1 = d Common Difference are Given
a3 −a2 = d
To write an AP, the minimum information required to know
a4 −a3 = d
the first term a and the common difference d of the
M arithmetic progression. Then, we put the values of a and d in
a n − a n − 1 = d and so on. a, a + d, a + 2d, a + 3d, … to get the required AP.
Each of the number in this list is called a term.
CBSE Term II Mathematics X (Basic) 25

nth Term of an AP Sum of First n-Terms of an AP


If the first term of an AP is ‘a’ and its common difference is If first term of an AP is ‘a’ and its common difference is ‘d’,
‘d’, then its nth term is given by the formula then the sum of its first n terms Sn , is given by the formula
a n = a + ( n − 1)d n
Sn = [ 2a + ( n − 1) d ]
The nth term of an AP is also called its general term. 2
In an AP, nth term is known as last term of an AP and it is n
or Sn = [a + a n ]
denoted by l, which is given by the formula 2
l = a + ( n − 1)d where, a n = nth term of an AP.
(i) If l is the last term of an AP having n terms, then sum of
nth Term from the End of an AP
all the terms is given by this formula
Let ‘a’ be the first term, ‘d’ be the common difference and ‘l’ n
be the last term of an AP, then nth term from the end can be Sn = [a + l ]
found by the formula 2
(ii) If Sn and Sn−1 are the sums of first n and ( n −1) terms of an
nth term from the end = l − ( n − 1)d
AP respectively, then its nth term a n is given by
Selection of Terms in an AP
a n = Sn − Sn − 1
Number of terms Terms Common difference
3 a − d, a , a + d d
Arithmetic Mean
If a, b and c are in AP, then b is known as arithmetic mean of
4 a − 3 d ,a − d , a + d , a + 3 d 2d a+c
a and c, i.e. b = .
5 a − 2d ,a − d , a , a + d ,a + 2d d 2
26 CBSE Term II Mathematics X (Basic)

Solved Examples
Example 1. Examine that the sequence 13, 10, 7, 4,... is Example 5. How many terms are there in the sequence
an AP. 3, 6, 9, 12, ..., 111?
Sol. Given, sequence is 13, 10, 7, 4, …… Sol. Given, sequence is 3, 6, 9, 12, ..., 111.
Here, a1 = 13, a 2 = 10, a 3 = 7, a 4 = 4, ……… Here, 6 − 3 = 9 − 6 = 12 − 9 ... = 3
Here, we have a 2 − a1 = 10 − 13 = −3,
So, it is an AP with first term, a = 3 and common difference,
a 3 − a 2 = 7 − 10 = −3, d = 3. Let there be n terms in the given sequence.
a 4 − a 3 = 4 − 7 = −3 and so on.
Then, nth term = 111
Since, difference of any two consecutive terms is same.
So, the given sequence is an AP. ⇒ a + ( n − 1)d = 111 [Qan = (a + ( n − 1)d)]

Example 2. Find the common difference of the following ⇒ 3 + ( n − 1) × 3 = 111


AP’s . ⇒ 3(1 + n − 1) = 111
(i) 3, − 2 , − 7 , − 12 , ... 111
(ii) 11, 11, 11, 11, ... ⇒ n= ⇒ n = 37
3
1 1 1 1
(iii) 5 , 9 , 13 , 17 …… Hence, the given sequence contains 37 terms.
2 2 2 2
Sol. (i) Given, AP is 3, − 2 , − 7 , − 12 ,... Example 6. Which term of the AP: 21, 18 , 15, . . . is −81 ?
Here, a1 = 3, a 2 = −2 , a 3 = −7, a 4 = −12 and so on. Sol. Given, AP is 21, 18, 15,... .
∴ Common difference (d) = a 2 − a1 = −2 − 3 = −5 Here, a = 21 and d = 18 − 21 = −3
(ii) Given, AP is 11, 11, 11, 11, ... Let nth term of given AP be − 81
Here, a1 = 11, a 2 = 11, a 3 = 11, a 4 = 11 and so on. Then, an = −81
∴Common difference (d) = a 2 − a1 = 11 − 11 = 0 ⇒ a + ( n − 1)d = −81 [Qan = a + ( n − 1)d ]
1 1 1 1 On putting the values of a and d, we get
(iii) Given, AP is 5 , 9 ,13 ,17 ,……
2 2 2 2 21 + ( n − 1)( −3) = −81 ⇒ 21 − 3n + 3 = −81
1 1 1 1 ⇒ 24 − 3n = −81 ⇒ −3n = −81 − 24 = −105
Here, a1 = 5 , a 2 = 9 , a 3 = 13 , a 4 = 17 and so on.
2 2 2 2 − 105
1 1 ⇒ n= = 35
∴ Common difference (d) = a 2 − a1 = 9 − 5 −3
2 2 Hence, 35th term of given AP is − 81.
19 11 8
= − = =4 Example 7. How many numbers of two digits are
2 2 2
divisible by 7?
Example 3. Write an AP having 4 as the first term and Sol. Two-digits numbers are10, 11, 12 , 13, 14, 15,..., 97 , 98, 99
− 3 as the common difference. in which only 14, 21,28,..., 98 are divisible by 7.
Sol. Given, first term (a ) = 4 and common difference (d) = − 3 Here, 21 − 14 = 28 − 21... = 7.
On putting the values of a and d in general form So, this list of numbers forms an AP, whose first term (a ) = 14,
a ,a + d,a + 2d,a + 3d,... , we get common difference (d) = 7.
4, 4 − 3, 4 + 2( −3), 4 + 3( −3), ... Let there are n terms in the above sequence, then an = 98
4, 1, 4 – 6, 4 – 9, … or 4, 1, − 2 , − 5,... ⇒ a + ( n − 1)d = 98 [Qan = a + ( n − 1)d]
Which is the required AP. ⇒ 14 + ( n − 1)7 = 98 ⇒14 + 7 n − 7 = 98
91
⇒ 7 n = 91 ⇒ n = = 13
Example 4. Find the 20th term of the sequence 7
7, 3, −1 , −5 . . . Hence, 13 numbers of two digits are divisible by 7.
Sol. Given, sequence is 7 , 3, − 1, − 5,... .
Example 8. Determine the 10 th term from the end of
Here, 3 − 7 = − 4, −1 − 3 = − 4, −5 + 1 = − 4 and so on.
the AP : 4, 9, 14, ..., 254.
So, given sequence is an AP, in which a = 7 and d = − 4.
Sol. Given, AP is 4, 9, 14,..., 254.
Since, nth term, an = a + ( n − 1)d
Here, l = last term = 254
On putting n = 20, we get
d = common difference = 9 − 4 = 5
a 20 = a + ( 20 − 1)d = 7 + 19 ( −4) [Q a = 7 , d = −4]
∴10th term from the end = l − (10 − 1)d = l − 9d
= 7 − 19 × 4 = 7 − 76 = − 69
= 254 − 9 × 5 = 254 − 45 = 209
Hence, 20th term of given sequence is −69.
CBSE Term II Mathematics X (Basic) 27

Alternate Method Sol. Given, initial money P = ` 2000


On reversing the given AP, new AP is 254, ..., 14, 9, 4. Rate of interest, R = 7% per year; Time, T = 1, 2, 3, 4,...
Here, first term (a ) = 254 and We know that, simple interest is given by the following formula
common difference (d) = 4 − 9 = −5 PRT
SI =
Now, 10th term of new AP = a10 100
2000 × 7 × 1
= 254 + (10 − 1)( −5) ∴SI at the end of 1st year = = `140
100
= 254 − 9 × 5 = 209
2000 × 7 × 2
Hence, 10th term from the end of given AP is 209. SI at the end of 2nd year = = ` 280
100
Example 9. Determine the general term of an AP 2000 × 7 × 3
SI at the end of 3rd year = = ` 420
whose 7th term is −1 and 16 th term is 17. 100
Sol. Let a be the first term and d be the common difference of Thus, required list of numbers is 140, 280, 420, ... .
the AP, whose 7th term is −1 and 16th term is 17. Here, 280 − 140 = 420 − 280 K = 140
Since, a 7 = − 1 and a16 = 17 So, above list of numbers forms an AP, whose first term
∴ We have, a + (7 − 1)d = − 1 ⇒ a + 6d = − 1 ...(i) (a ) = 140 and common difference (d) = 140.
and a + (16 − 1)d = 17 ⇒ a + 15d = 17 ...(ii) Now, SI at the end of 20th year will be equal to 20th term of
[Qan = a + ( n − 1)d] the above AP.
On subtracting Eq. (i) from Eq. (ii), we get Q a 20 = a + ( 20 − 1) d = 140 + 19 × 140 = 140 + 2660 = 2800
a + 15d − a − 6d = 17 + 1 Hence, the interest at the end of 20th year will be ` 2800.
⇒ 9d = 18 ⇒ d = 2 Example 12. Each year, a tree grow 5 cm less than the
On substituting d = 2 in Eq. (i), we get preceding year. If it grew by 1m in the first year,
a + 6 × 2 = −1 then in how many years will it have ceased
⇒ a + 12 = − 1 growing? [CBSE 2015]
⇒ a = −13 Sol. Given that, tree grow 5 cm or 0.05 m less than preceding
Hence, general term, year.
an = a + ( n − 1)d ∴The following sequence can be formed.
= − 13 + ( n − 1)2 [Qa = −13 and d = 2] 1, (1 − 005
. ), (1 − 2 × 005. ), ...,0
= − 13 + 2 n − 2 = 2 n − 15 i.e. 1, 0.95, 0.90, ... ,0 which is an AP.
Example 10. Find four numbers in AP whose sum is 20 Here, a = 1, d = 095
. − 1 = − 005 . and l = 0
and the sum of whose squares is 120. Let l = an = a + ( n − 1)d
Sol. Let the numbers be a − 3d, a − d, a + d and a + 3d. Then, 0 = 1 + ( n − 1) ( − 005
. )
Then, according to the given condition, we have ⇒ ( n − 1) (005
. ) =1
(a − 3d) + (a − d) + (a + d) + (a + 3d) = 20 …(i) 1
⇒ n −1 =
and (a − 3d)2 + (a − d)2 + (a + d)2 + (a + 3d)2 = 120 …(ii) 005
.
1
From Eq. (i), we get ⇒ n − 1 = × 100
5
4a = 20 ⇒ a = 5
⇒ n − 1 = 20
From Eq. (ii), we get
⇒ n = 21
a 2 + 9d2 − 6da + a 2 + d2 − 2ad + a 2 + d2 + 2ad
Hence, in 21 yr, tree will have ceased growing.
+ a 2 + 9d2 + 6ad = 120
Example 13. The eighth term of an AP is half its second
⇒ 4a 2 + 20d2 = 120
term and the eleventh term exceeds one-third of its
⇒ a 2 + 5d2 = 30 fourth term by 1. Find the 15th term.
⇒ 25 + 5d2 = 30 [Qa = 5] [NCERT Exemplar]
⇒ 5d = 5 ⇒ d = 1 ⇒ d = ±1
2 2
Sol. Let a and d be the first term and common difference of
If d = 1, then the numbers are 2, 4, 6, 8 and if d = −1, then the an AP. Then,
numbers are 8, 6, 4, 2. 1 1
a 8 = a 2 and a11 = a 4 + 1
Hence, the numbers are 2, 4, 6, 8 or 8, 6, 4, 2. 2 3
1
Example 11. A sum of ` 2000 is invested at 7% simple ⇒ a + ( 8 − 1) d = [a + ( 2 − 1) d]
2
interest per year. Calculate the interest at the end 1
of each year. Do these interest form an AP? If so, and a + (11 − 1) d = [(a + ( 4 − 1) d) + 1]
3
then find the interest at the end of 20th year 1
making use of this fact. ⇒ a + 7 d = (a + d )
2
28 CBSE Term II Mathematics X (Basic)

1 Sol. Let the first term of this AP be a and common difference be d.


and a + 10d = [(a + 3d) + 1]
3 Given, sum of first 10 terms, (S 10 ) = 140
⇒ 2a + 14d − a − d = 0 10 140
⇒ [2a + (10 − 1)d] = 140 ⇒ 2a + 9d =
and 3a + 30d = a + 3d + 1 2 5
⇒ a + 13d = 0 …(i)  n 
and 2a + 27d − 1 = 0 …(ii) Q Sn = 2 [2a + ( n − 1)d]
On solving Eqs. (i) and (ii), we get ⇒ 2a + 9d = 28 ...(i)
a = − 13, d = 1 Also, given sum of first 16 terms, (S 16 ) = 320
∴ a15 = a + (15 − 1) (1) 16
⇒ [2a + (16 − 1)d] = 320
= − 13 + 14 = 1 2
320
⇒ 2a + 15d = ⇒ 2a + 15d = 40 ...(ii)
Example 14. The fourth term of an AP is 11. The sum 8
of the fifth and seventh terms of the AP is 24. Find On subtracting Eq. (i) from Eq. (ii), we get
its common difference. [CBSE 2015] 6d = 12 ⇒ d = 2
Sol. Let a be the first term and d be the common difference. On putting d = 2 in Eq. (i), we get
Then, 10
2a + 9( 2 ) = 28 ⇒ 2a = 28 − 18 ⇒ a = =5
a 4 = 11 ⇒ a + ( 4 − 1) d = 11 2
⇒ a + 3d = 11 …(i) Thus, a = 5 and d = 2.
m
Also, given a5 + a 7 = 24 Hence, sum of first m terms, (Sm ) = [2a + ( m − 1)d]
2
⇒ a + ( 5 − 1) d + a + (7 − 1) d = 24 m
⇒ a + 4d + a + 6d = 24 = [2( 5) + ( m − 1)2] = m[5 + ( m − 1)]
2
⇒ 2a + 10d = 24 = m ( 5 + m − 1) = m ( m + 4) = m 2 + 4m
⇒ a + 5d = 12 [divide by 2] …(ii)
On subtracting Eq. (i) from Eq. (ii), we get Example 18. Find the sum of all three-digit natural
1 numbers, which are multiples of 11. [CBSE 2009]
2d = 1 ⇒ d =
2 Sol. All three-digit natural numbers, multiples of 11 are 110, 121,
1 132, …, 990.
Hence, common difference is .
2 Here, common difference,121 − 110 = 132 − 121 = ... = 11.
So, it is an AP with first term, a = 110, common difference,
Example 15. Find the sum of the first 22 terms of the d = 11 and last term, l = 990.
AP : 8, 3, − 2, ...
Let l = an = a + ( n − 1)d
Sol. Given, AP is 8, 3, − 2, ...
∴ 990 = 110 + ( n − 1) × 11
Here, first term, (a ) = 8
⇒ 990 = 110 + 11n − 11
Common difference, (d) = 3 − 8 = − 5 and n = 22 ⇒ 11n = 891 ⇒ n = 81
n
Q Sum of first n terms, (Sn ) = [2a + ( n − 1) d] n
2 Q Sn = [a + l]
2
22
∴ Sum of first 22 terms, (S22 ) = [2 × 8 + ( 22 − 1) × ( − 5)] 81
2 ∴ S81 = [110 + 990]
2
= 11 [16 + 21 × ( − 5)] 81
= 11 [16 − 105] = × 1100 = 81 × 550 = 44550
2
= 11 ( − 89) = − 979
Hence, sum of first 22 terms of an AP is − 979.
Example 19. If S n , the sum of first n terms of an AP is
given by S n = 3n 2 − 4 n, find the nth term.[CBSE 2019]
Example 16. Find the sum of first 24 terms of an AP,
whose nth term is given by a n = 3 + 2n. Sol. Given, Sn = 3n 2 − 4n …(i)
Sol. Given, nth term of an AP, an = 3 + 2 n On replacing n by ( n − 1) in Eq. (i), we get
Sn − 1 = 3( n − 1)2 − 4( n − 1)
Clearly, sum of first 24 terms, (S24 )
24 nth term of the AP an = Sn − Sn − 1
= (a + a 24 ) = 12( 5 + 51)
2 ∴ an = ( 3n 2 − 4n) − [3( n − 1)2 − 4( n − 1)]
[Qa1 = 3 + 2 = 5 and a 24 = 3 + 2 × 24 = 3 + 48 = 51] ⇒ an = 3[n 2 − ( n − 1)2 ] − 4[n − ( n − 1)]
= 12 × 56 = 672 . ⇒ an = 3[n 2 − n 2 + 2 n − 1] − 4[n − n + 1]
Example 17. If the sum of first 10 terms of an AP is 140 ⇒ a n = 3 ( 2 n − 1) − 4
and the sum of first 16 terms is 320, then find the ⇒ a n = 6n − 3 − 4 ⇒ a n = 6n − 7
sum of first m terms. Thus, the nth term of the AP = 6n − 7.
CBSE Term II Mathematics X (Basic) 29

Chapter
Practice
PART 1
Objective Questions
l
Multiple Choice Questions 8. The nth term of the AP (1 + 3 ), (1 + 2 3 ), (1 + 3 3 ),
1. Which of the following form of an AP? … is [CBSE 2020 (Basic)]
[NCERT Exemplar] (a)1 + n 3 (b) n + 3
(a) − 1, − 1 , − 1 , − 1 , ... (b) 0, 2 , 0, 2, … (c) n(1 + 3) (d) n 3
1 1 1
(c) 1 , 1 , 2 , 2 , 3, 3, K (d) , , , K
2 3 4 9. If an AP have 8 as the first term and −5 as the
common difference and its first three terms are
2. Which of the following is not an AP? 8, A , B, then ( A + B ) is equal to
(a) −1.2 , 0.8, 2.8, ... (a) 0 (b) −1
(b) 3, 3 + 2 , 3 + 2 2 , 3 + 3 2 , … (c)1 (d) 2
4 7 9 12
(c) , , , , .... 10. In an AP, if d = −4, n = 7 and a n = 4, then a is equal
3 3 3 3
−1 −2 −3 to
(d) , , , .... (a) 6 (b) 7
5 5 5
(c) 20 (d) 28
5
3. If − , a, 2 are consecutive terms in an Arithmetic 11. Write the 31st term of the AP −50 , − 47 , − 44, … .
7
[CBSE 2020 (Basic)]
Progression, then the value of ‘a’ is
9 9 (a) 30 (b) 50
(a) (b) (c) 40 (d) 35
7 14
19 19 12. The 21st term of an AP whose first two terms are
(c) (d)
7 14 − 3 and 4, is [NCERT Exemplar]
(a) 17 (b) 137
4. The common difference of an AP, whose nth term is
(c) 143 (d) − 143
a n = ( 3n + 7 ), is
(a) 3 (b) 7 13. Write the 26th term of the AP 7, 4, 1, −2, ………
(c) 10 (d) 6 [CBSE 2020 (Basic)]
(a) −60 (b) −68
5. Find the value of x, so that − 6 , x , 8 are AP. (c) 50 (d) 60
[CBSE 2020 (Basic)]
14. Which term of an AP : 21, 42, 63, 84, ... is 210?
(a) 1 (b) 3 [NCERT Exemplar]
(c) −1 (d) 5
(a) 9th (b) 10th
6. The value of p for which ( 2p + 1), 10 and ( 5p + 5) are (c) 11th (d) 12th
three consecutive terms of an AP is 15. If the common difference of an AP is 5, then what
(a) − 1 (b) − 2 is a 18 − a 13 ?
(c) 1 (d) 2 (a) 5 (b) 20
7. The nth term of the AP 2 , 2 2 , 3 2 , … is (c) 25 (d) 30
[CBSE 2020 (Basic)] 16. What is the common difference of an AP in which
(a) n 2 (b) 2n a 18 − a 14 = 32 ? [NCERT Exemplar]
(c) 2 n (d) ( n − 1) 2 (a) 8 (b) − 8
(c) − 4 (d) 4
30 CBSE Term II Mathematics X (Basic)

17. Two APs have the same common difference. l


Case Based MCQs
The first term of one of these is −1 and that of
the other is − 8. The difference between their 4th 27. In a flower bed, there are 43 rose plants in the first
terms is [NCERT Exemplar] row, 41 in the second, 39 in the third and so on.
(a) − 1 (i) If there are 11 rose plants in the last row, then
(b) − 8 number of rose required are
(c) 7 (a) 16 (b) 15
(d) − 9 (c) 17 (d) 10
18. If 7 times the 7th term of an AP is equal to 11 times
its 11th term, then its 18th term will be (ii) Difference of rose plants in 7th row and 13th row is
(a) 7 (a) 11 (b) 12
(b) 11 (c) 13 (d) 14
(c) 18 (iii) If there are x rose plants in 15 rose, then x is
(d) 0 equal to
19. The 4th term from the end of an AP (a) 10 (b) 12
− 11 , − 8 , − 5, . . . , 49 is (c) 13 (d) 15
(a) 37 (b) 40 (iv) The rose plants in 6th row is
(c) 43 (d) 58 (a) 35 (b) 37
20. Which term of the AP 5, 15, 25, ... will be 130 more (c) 33 (d) 31
than its 31st term? (v) The total number of rose plants in 5th and
(a) 42 (b) 44 8th row is
(c) 46 (d) 48 (a) 64 (b) 54
(c) 46 (d) 45
21. The second term from the end of the AP
5, 8, 11, ..., 47 is [CBSE 2020 (Basic)] 28. The sum of the first five terms of an AP and the
(a) 50 (b) 45 sum of the first seven terms of the same AP is 167.
(c) 44 (d) 41 The sum of the first ten terms of this AP is 235.
22. If S n denotes the sum of first n terms of an AP, then (i) Let first term and common difference of an AP be a
and d, respectively. Then pair of linear equations
S 2 − S1 = _________ . [CBSE 2020 (Basic)] for given problem is
(a) first term (a)13a + 31d = 167, 2a + 9d = 47
(b) second term (b)13a + 31d = 169, 2a + 9d = 45
(c) third term (c)12a + 31d = 167 , 2a + 9d = 47
(d) None of the above (d)12a + 31d = 169, 2a + 9d = 45
23. The sum of first 16 terms of the AP 10, 6, 2, ... is (ii) Common difference of given AP is
[NCERT Exemplar] (a) 5 (b) 7
(a) − 320 (c) 9 (d) 11
(b) 320
(iii) First term of given AP is
(c) − 352
(d) − 400 (a) 3 (b) 4
(c) 2 (d) 1
24. If the first term of an AP is − 5 and the common
(iv) Fourth term of the AP is
difference is 2, then the sum of the first 6 terms is (a) 15 (b) 16
(a) 0 (b) 5 (c) 17 (d) 18
(c) 6 (d) 15
(v) Sum of first twenty terms is
25. In an AP, if a = 1, a n = 20 and S n = 399, then n is (a) 970 (b) 990
equal to (c) 950 (d) 980
(a) 19 29. India is competitive manufacturing location due to
(b) 21
the low cost of manpower and strong technical and
(c) 38
(d) 42
engineering capabilities contributing to higher
quality production runs. The production of TV sets
26. The sum of first five multiples of 3 is [NCERT Exemplar] in a factory increases uniformly by a fixed number
(a) 45 (b) 55 every year. It produced 16000 sets in 6th year and
(c) 65 (d) 75 22600 in 9th year.
CBSE Term II Mathematics X (Basic) 31

(ii) What is the minimum number of days he needs to


practice till his goal is achieved?
(a) 10 (b) 12
(c) 11 (d) 9
(iii) Which of the following term is not in the AP of the
above given situation?
(a) 41 (b) 30
(c) 37 (d) 39
(iv) If nth term of an AP is given by a n = 2 n + 3, then
common difference of an AP is
Based on the above information, answer the (a) 2 (b) 3
following questions: (c) 5 (d) 1

(i) Find the production during first year. (v) The value of x, for which 2 x, x + 10, 3 x + 2 are three
(a) 4000 sets (b) 5000 sets consecutive terms of an AP, is
(c) 6000 sets (d) 7000 sets (a) 6 (b) − 6
(ii) Find the production during 8th year. (c) 18 (d) − 18
(a) 48000 sets (b) 20400 sets 31. Your elder brother wants to buy a car and plans to
(c) 43000 sets (d) None of these take loan from a bank for his car. He repays his total
(iii) Find the production during first 3 years. loan of ` 118000 by paying every month starting
(a) 20000 sets (b) 25000 sets with the first installment of ` 1000. If he increases
(c) 31000 sets (d) 21600 sets the installment by ` 100 every month, answer the
(iv) In which year, the production is ` 29200. following :
(a) 11 (b) 12
(c) 10 (d) 8
(v) Find the difference of the production during
7th year and 4th year.
(a) 5500 (b) 6700
(c) 5400 (d) 6600
30. Your friend Veer wants to participate in a 200 m
race. He can currently run that distance in 51
seconds and with each day of practice it takes him
2 seconds less. He wants to do in 31 seconds.
[CBSE Question Bank]
(i) The amount paid by him in 30th installment is
(a) 3900 (b) 3500
(c) 3700 (d) 3600
(ii) The amount paid by him in the 30 installments is
(a) 37000 (b) 73500
(c) 75300 (d) 75000
(iii) What amount does he still have to pay after 30th
installment?
(a) 45500 (b) 49000
(c) 44500 (d) 54000
(iv) If total installments are 40, then amount paid in
[CBSE Question Bank] the last installment?
(a) 4900 (b) 3900
(i) Which of the following terms are in AP for the
(c) 5900 (d) 9400
given situation?
(a) 51, 53, 55…. (v) The ratio of the 1st installment to the last
(b) 51, 49, 47…. installment is
(c) − 51, − 53, − 55…. (a) 1 : 49 (b) 10 : 49
(d) 51, 55, 59… (c) 10 : 39 (d) 39 : 10
32 CBSE Term II Mathematics X (Basic)

15. Split 207 into three parts such that these are in AP
PART 2 and the product of the two smaller parts is 4623.
[NCERT Exemplar]
Subjective Questions 16. Find the 12th term from the end of the AP
− 2, − 4 , − 6 , . . . , − 100 . [NCERT Exemplar]
l
Short Answer Type Questions
17. How many numbers lie between 10 and 300, which
1. Justify whether it is true to say that divided by 4 leave a remainder 3?
−3 5
−1, , − 2, , . . . forms an AP as a 2 − a 1 = a 3 − a 2 . 18. If m times the mth term of an AP is equal to n
2 2 times its nth term, show that the ( m + n)th term of
[NCERT Exemplar]
the AP is zero.
2. If 3y − 1, 3y + 5 and 5y + 1 are three consecutive 19. Find the sum of first 20 terms of the following AP
terms of an AP, then find the value of y. sequence 1, 4, 7, 10, ……
[CBSE 2020 (Basic)]
20. Which term of the AP : 120, 116, 112, … is first
3. The angles of a triangle are in AP. The greatest angle negative term? [CBSE 2012]
is twice the least. Find all the angles of the triangle.
[NCERT Exemplar] 21. How many terms of AP 18, 16, 14, ... should be taken,
so that their sum is zero? [CBSE 2013]
4. The taxi fare after each km, when the fare is ` 15 for
the first kilometre and ` 8 for each additional 22. Find the sum of first 8 multiples of 3. [CBSE 2018]
kilometre, does not form an AP as the total fare 23. Subha Rao started work in 1995 at an annual salary
(in `) after each kilometre is 15, 8, 8, 8, … . Is the of ` 5000 and received an increment of ` 200 each
statement true? Give reasons. year. In which year did his income reach ` 7000?
5. If 3k − 2, 4 k − 6 and k + 2 are three consecutive [NCERT Exemplar]
terms of AP, then find the value of k. 24. Ramkali saves ` 5 in the first week of a year and
[CBSE 2020 (Basic)] then increased her weekly savings by ` 1.75. If in
6. Show that ( a − b )2 , ( a 2 + b 2 ) and ( a + b )2 are in AP. the nth week, her weekly saving becomes ` 20.75.
Find n.
7. Find the 11th term of the AP −27 , − 22, − 17 , − 12, … .
1 + 3 + 5 + Kupto n terms
[CBSE 2020 (Basic)] 25. If = 9, then find the
2 + 5 + 8 + . . . upto 8 terms
8. For the AP −3, − 7 , − 11 , K can we find directly
value of n.
a 30 − a 20 without actually finding a 30 and a 20 ?
Give reason for your answer. 26. In an AP, if S n = 3n 2 + 5n and a k =164, then find
the value of k.
9. Is 0 a term of the AP 31, 28, 25,…? Justify your
answer. 27. In an AP it is given that common difference is 5
and sum of its first ten terms is 75. Find the first
10. If four numbers are in AP such that their sum is
term of the AP. [CBSE 2020 (Basic)]
50 and the greatest number is 4 times the least, then
find the numbers. 28. If the sum of the first 14 terms of an AP is 1050 and
11. Find the 20th term of the AP whose 7th term is 24 its first term is 10, then find the 21st term of the AP.
[CBSE 2020 (Basic)]
less than the 11th term, first term being 12.
[NCERT Exemplar] 29. The sum of the first n terms of an AP whose first
12. If the 9th term of an AP is zero, then prove that its term is 8 and the common difference is 20, is equal
29th term is twice its 19th term. to the sum of first 2n terms of another AP whose
first term is −30 and the common difference is 8.
13. The 16th term of an AP is 1 more than twice its 8th Find the value of n. [NCERT Exemplar]
term. If the 12th term of an AP is 47, then find its
nth term. 30. Find the sum of 10 terms of an AP.
2 , 8 , 18 , 32, …,
14. Find the 19th term of the following sequence.
 n 2 , where n is even 31. Find the sum of all odd numbers between 100
tn =  2 and 300. [CBSE 2020 (Basic)]
n − 1, where n is odd [CBSE 2015]
CBSE Term II Mathematics X (Basic) 33

(ii) 4 −  + 4 −  + 4 −  + ... upto n terms.


32. How many two-digit numbers are divisible by 6 ? 1 2 3
[CBSE 2020 (Basic)]  n  n  n
33. If S n denotes the sum of first n terms of an AP, then [NCERT Exemplar]
prove that S12 = 3 (S 8 − S 4 ). [NCERT Exemplar] 48. Find the sum of the two middle most terms of an
4 2 1
34. Find the sum of last ten terms of the AP 8, 10, AP − , − 1, − , . . . , 4 .
12,..., 126. 3 3 3
35. How many terms of AP; 3, 5, 7, 9 ... must be taken 49. Find the sum of first 17 terms of an AP whose 4th
to get the sum 120? [CBSE 2020 (Basic)] and 9th terms are −15 and − 30, respectively.
36. Find the sum of first seven numbers which are 50. The sum of first n terms of three APs are S1 , S 2 and
multiples of 2 as well as of 9. [NCERT Exemplar] S 3 . The first term of each AP is unity and their
37. For an AP, it is given that the first term ( a ) = 5, common differences are 1, 2 and 3, respectively.
Prove that
common difference ( d ) = 3, and the nth term
( a n ) = 50. Find n and sum of first n terms (S n ) of S1 + S 3 = 2S 2 .
the AP. 51. If the sum of first four terms of an AP is 40 and that
38. How many terms of the AP : 24, 21, 18 …… must of first 14 terms is 280. Find the sum of its first n
be taken so that their sum is 78? [CBSE 2020 (Basic)] terms. [CBSE 2019]

39. If the sum of first 14 terms of an Arithmetic 52. The ratio of the 11th term to the 18th term of an AP is
Progression is 1050 and its fourth term is 40, find 2 : 3. Find the ratio of the 5th term to the 21st term
its 20th term. and also the ratio of the sum of the first five terms to
the sum of the first 21 terms. [NCERT Exemplar]
l
Long Answer Type Questions 53. If the sum of first 7 terms of an AP is 49 and that of
40. If the nth terms of the two AP’s 9, 7, 5, ... and 24, 17 terms is 289, then find the sum of first n terms.
[CBSE 2020 (Basic)]
21, 18, ... are the same, then find the value of n.
Also, that term. [NCERT Exemplar] 54. Show that the sum of an AP whose first term is a,
41. The 26th, 11th and the last terms of an AP are, 0, 3 the second term b and the last term c, is equal to
1 ( a + c )( b + c − 2a )
and − , respectively. Find the common difference . [NCERT Exemplar]
5 2 (b − a )
and the number of terms. 1 2
55. How many terms of the AP 20, 19 , 18 , ... must
42. The 4th term of an AP is zero. Prove that the 25th 3 3
term of the AP is three times its 11th term. be taken, so that their sum is 300?
43. The 17th term of an AP is 5 more than twice its
8th term. If 11th term of AP is 43, then find its
l
Case Base Questions
nth term. [CBSE 2020 (Basic)] 56. Kanika was given her pocket money on Jan 1st,
44. If the nth terms of two AP’s 23, 25, 27, ... and 2008. She puts ` 1 on day 1, ` 2 on day 2, ` 3 on day
5, 8, 11, 14, ... are equal, then find the value of n. 3 and continued doing so till the end of the month.
[CBSE 2020 (Basic)] From this money into her piggy bank, she also
45. The sum of first 6 terms of an AP is 42. The ratio of spent ` 204 of her pocket money and found that at
its 10th term to 30th term is 1 : 3. Find the first and the end of the month she still had ` 100 with her.
[NCERT Exemplar]
the 13th term of the AP. [CBSE 2020 (Basic)]
(i) How much Kanika take till the end of the month
46. Solve the equation:
from pocket money?
1 + 5 + 9 + 13 + K + x = 1326
(ii) How much was pocket money for the month?
[CBSE 2020 (Basic)]
(iii) What is the amount saved by Kanika, till
47. Find the sum
January 13th, 2008?
(i) 1 + ( − 2 ) + ( − 5 ) + ( − 8 ) + ... + ( − 236 )
34 CBSE Term II Mathematics X (Basic)

SOLUTIONS
Objective Questions −1 −2 −3
(d) We have, , , , ....
1. (a) (a) Here, t1 = − 1, t 2 = − 1, t 3 = − 1 and t 4 = − 1 5 5 5
1 2 −3
Now, t 2 − t1 = − 1 + 1 = 0 Here, a1 = − , a 2 = − , a 3 =
t3 − t2 = − 1 + 1 = 0 5 5 5
2  −1 1
t4 − t3 = − 1 + 1 = 0 Now, a 2 − a1 = − −   = −
5  5 5
Clearly, the difference of successive terms is same,
−3  2  −3 2 1
therefore given list of numbers forms an AP. a3 − a2 = − −  = + =−
(b) Here, t1 = 0, t 2 = 2 , t 3 = 0 and t 4 = 2 5  5 5 5 5
Now, t 2 − t1 = 2 − 0 = 2 Thus, given series is an AP.
t3 − t2 = 0 − 2 = − 2 Hence, in the given options, option (c) is not an AP.
5
t4 − t3 = 2 − 0 = 2 3. (b) Given, − , a , 2 are consecutive terms in AP.
7
Clearly, the difference of successive terms is not same,
 5
therefore given list of numbers does not form an AP. ∴ a − −  = 2 −a [Q In AP, a 2 − a1 = a 3 − a 2 ]
 7
(c) Here, t1 = 1, t 2 = 1, t 3 = 2 and t 4 = 2
5 9 9
Now, t 2 − t1 = 1 − 1 = 0 ⇒ 2a = 2 − ⇒ 2a = ⇒ a =
7 7 14
t3 − t2 = 2 − 1 = 1
4. (a) Given, nth term of an AP is
t4 − t2 = 2 − 2 = 0
a n = ( 3n + 7 )
Clearly, the difference of successive terms is not same,
therefore given list of numbers does not form an AP. ∴ The common difference of an AP = an − an − 1
1 1 1 = ( 3n + 7 ) − [3( n − 1) + 7]
(d) , , , …
2 3 4 = 3n + 7 − ( 3n + 4) = 7 − 4 = 3
1 1
Here, t1 = , t 2 = and t 3 =
1 5. (a) Given, AP is −6, x , 8.
2 3 4 Here, a1 = − 6, a 2 = x and a 3 = 8
1 1 2−3 1 ∴ Common difference (d) = a 2 − a1 = a 3 − a 2
Now, t 2 − t1 = − = =−
3 2 6 6 ⇒ x − ( − 6) = 8 − x
1 1 3−4 1 ⇒ x + 6 = 8 −x
t3 − t2 = − = =−
4 3 12 12 ⇒ 2x = 2
Clearly, the difference of successive terms is not same, ∴ x =1
therefore given list of numbers does not form an AP.
6. (d) Let a1 = 2 p + 1, a 2 = 10 and a 3 = 5p + 5.
2. (c) The condition for given series is not AP is the common
difference of two consecutive terms is not constant. Given that three consecutive terms are in AP.
∴ a 2 − a1 = a 3 − a 2
(a) We have, − 1.2 , 0.8, 2.8, ...
⇒ 10 − ( 2 p + 1) = 5p + 5 − 10
Here, a1 = − 1.2, a 2 = 0.8, a 3 = 2.8
⇒ 10 + 10 = 5p + 5 + 2 p + 1
Now, a 2 − a1 = 0.8 − ( −1.2 ) = 2.0
⇒ 20 = 7 p + 6
and a 3 − a 2 = 2.8 − 0.8 = 2
⇒ 7 p = 20 − 6
Thus, given series is an AP.
⇒ 7 p = 14
(b) We have, 3, 3 + 2 , 3 + 2 2 , 3 + 3 2 , … 14
⇒ p= =2
Here, a1 = 3, a 2 = 3 + 2, a3 = 3 + 2 2 7
Now, a 2 − a1 = 3 + 2 −3= 2 7. (a) Given, AP 2, 2 2 , 3 2 ....
and a 3 − a 2 = 3 + 2 2 − ( 3 + 2) = 2 a1 = 2 , a 2 = 2 2
Thus, given series is an AP. ∴ Common difference
4 7 9 12 d = a 2 − a1 = 2 2 − 2 = 2
(c) We have, , , , ....
3 3 3 3 nth term of an AP is given by
4 7 9 Tn = a + ( n − 1)d = 2 + ( n − 1) 2
Here, a1 = , a 2 = , a 3 =
3 3 3 = 2 + 2n − 2
7 4 3 Tn = 2 n
Now, a 2 − a1 = − = = 1
3 3 3
9 7 2 8. (a) Given, AP (1 + 3 ), (1 + 2 3 ), (1 + 3 3 ) …
a3 − a2 = − = a1 = 1 + 3 a 2 = 1 + 2 3
3 3 3
Thus, given series is not an AP, as common difference is ∴ Common difference d = a 2 − a1
not constant. = (1 + 2 3 ) − (1 + 3) = 3
CBSE Term II Mathematics X (Basic) 35

nth term of an AP is given by 17. (c) Let the common difference of two APs are d1 and d2 ,
Tn = a + ( n − 1)d respectively.
Tn = 1 + 3 + ( n − 1) 3 By condition, d1 = d2 = d ...(i)
Let the first term of first AP (a1 ) = − 1
=1 + 3+ 3n − 3
and the first term of second AP (a 2 ) = − 8
Tn = 1 + 3n We know that, the nth term of an AP, Tn = a + ( n − 1 ) d
9. (c) Given, first term (a ) = 8, common difference (d) = − 5 ∴ 4th term of first AP, T4 = a1 + ( 4 − 1) d = − 1 + 3 d
On putting the values of a and d in general form, and 4th term of second AP, T4 ′ = a 2 + ( 4 − 1 ) d = − 8 + 3d
a , a + d, a + 2d, a + 3d, ..., we get
Now, the difference between their 4th terms is
8, 8 − 5, 8 + 2( −5), 8 + 3( −5), ... or 8, 3, −2 , − 7 , ...
|T4 − T4 ′| = ( − 1 + 3d ) − ( − 8 + 3d )
On comparing with given terms 8, A ,B, ... , we get = − 1 + 3d + 8 − 3d = 7
A = 3, B = − 2 Hence, the required difference is 7.
∴ A + B = 3 + ( −2 ) = 3 − 2 = 1 18. (d) According to the question,
10. (d) In an AP, an = a + ( n − 1 ) d 7a 7 = 11a11
⇒ 4 = a + ( 7 − 1 ) ( − 4) [by given condition] ⇒ 7 [a + ( 7 − 1 ) d] = 11 [ a + (11 − 1 ) d]
⇒ 4 = a + 6 ( − 4) ⇒ 4 + 24 = a [Q an = a + ( n − 1 ) d]
∴ a = 28 ⇒ 7 (a + 6 d) = 11 (a + 10 d)
11. (c) a = − 50 ⇒ 7a + 42 d = 11a + 110 d
d = − 47 − ( −50) = 3 ⇒ 4a + 68 d = 0
a 31 = a + 30d = − 50 + 30 × 3 = − 50 + 90 = 40 ⇒ 4 (a + 17 d ) = 0
12. (b) Given, first two terms of an AP are a = − 3 and a + d = 4. ⇒ a + 17d = 0 …(i)
⇒ −3 + d = 4
∴ 18th term of an AP, a18 = a + (18 − 1 ) d
Common difference, d = 7
= a + 17d = 0 [from Eq. (i)]
∴ a 21 = a + ( 21 − 1 ) d [Q an = a + ( n − 1 ) d]
19. (b) We know that, the n th term of an AP from the end is
= − 3 + ( 20 ) 7
a n = l − ( n − 1) d …(i)
= − 3 + 140 = 137
Here, l = Last term and l = 49 [given]
13. (b) AP 7, 4, 1, −2, ....
Common difference, d = − 8 − ( − 11 )
a = 7, d = 4 − 7 = − 3, n = 26
= − 8 + 11 = 3
an = a + ( n − 1)d
From Eq. (i), a 4 = 49 − ( 4 − 1 ) 3 = 49 − 9 = 40
a 26 = 7 + ( 26 −1)( −3)
20. (b) We have, a = 5 and d = 10
= 7 + 25( −3) = 7 − 75 = − 68
∴ a 31 = a + 30d = 5 + 30 × 10 = 305
14. (b) Let nth term of the given AP be 210.
Let nth term of the given AP be 130 more than its 31st term.
Here, first term, a = 21
and common difference, Then, an = 130 + a 31
d = 42 − 21 = 21 and an = 210 ⇒ a + ( n − 1)d =130 + 305
Q a n = a + (n − 1) d ⇒ 5 + 10( n − 1) = 435
⇒ 210 = 21 + ( n − 1 ) 21 ⇒ 10( n − 1) = 430
⇒ 210 = 21 + 21 n − 21 ⇒ n − 1 = 43
⇒ 210 = 21 n ⇒ n = 44
⇒ n = 10 Hence, 44th term of the given AP is 130 more than its
31st term.
Hence, the 10th term of an AP is 210.
21. (c) AP 5, 8, 11, …… , 47
15. (c) Given, the common difference of AP i.e. d = 5
a = 5, d = 8 − 5 = 3
Now, a18 − a13 = a + (18 − 1 ) d − [a + (13 − 1 ) d]
Let nth term = 47
[Q an = a + ( n − 1 ) d]
= a + 17 × 5 − a − 12 × 5 an = 47
= 85 − 60 = 25 ⇒ a + ( n − 1)d = 47
16. (a) Given, a18 − a14 = 32 ⇒ 5 + ( n − 1)( 3) = 47
⇒ 5 + 3n − 3 = 47
⇒ a + (18 − 1 ) d − [a + (14 − 1 )d] = 32 [Q an = a + ( n −1 ) d]
⇒ 3n = 45 ⇒ n = 15
⇒ a + 17d − a − 13d = 32
Now, 2nd term from last = 14th term from starting
⇒ 4d = 32
= a + 13d
∴ d=8 = 5 + 13 × 3
Which is the required common difference of an AP. = 5 + 39 = 44
36 CBSE Term II Mathematics X (Basic)

22. (b) Let AP = a1 , a 2 , a 3 , … (ii) (b) Number of rose plants in 7th row = a 7
∴ S1 = a1 , S2 = a1 + a 2 = a + 6d = 43 + 6 ( − 2 ) = 43 − 12 = 31
∴ S2 − S1 = (a1 + a 2 ) − a1 = a 2 Number of rose plants in 13th row = a 13
∴ S2 − S1 = Second term of the AP. = a + 12d = 43 + 12 ( − 2 ) = 43 − 24 = 19
23. (a) Given, AP is10, 6, 2 , ... ∴ Required difference = 31 − 19 = 12
(iii) (d) Here, n = 15
Here, first term a = 10, common difference, d = − 4
16 ∴ a 15 = a + 14d = 43 + 14 ( − 2 ) = 43 − 28 = 15
∴ S 16 = [2a + (16 − 1 ) d] (iv) (c) Number of rose plants in 6th row
2
 n  = a 6 = a + 5d
Q Sn = 2 {2a + ( n − 1) d} = 43 + 5 ( − 2 )
= 8 [2 × 10 + 15 ( − 4)] = 43 − 10 = 33
= 8 ( 20 − 60 ) = 8 ( − 40 ) = − 320 (v) (a) Number of rose plants in 5th row
24. (a) Given, a = − 5 and d = 2 = a 5 = a + 4d
6 = 43 + 4 ( − 2 )
∴ S 6 = [ 2 a + (6 − 1 ) d ] = 43 − 8 = 35
2
 n  Number of rose plants in 8th row
Q Sn = 2 {2a + ( n − 1) d} = a 8 = a + 7d
= 3 [ 2 ( − 5) + 5 ( 2 )] = 43 + 7 ( − 2 )
= 3 ( − 10 + 10 ) = 0 = 43 − 14 = 29
n ∴ Required sum = 35 + 29 = 64
25. (c)Q Sn = [2a + ( n − 1 ) d]
2 28. (i) (c) Let the number of terms of AP be n.
n Q Sum of first n terms of an AP,
399 = [2 × 1 + ( n − 1 ) d]
2 n
Sn = [2a + ( n − 1) d] ...(i)
798 = 2 n + n ( n − 1 ) d …(i) 2
and an = 20 ∴ Sum of first five terms of an AP,
⇒ a + ( n − 1 ) d = 20 [Q an = a + ( n − 1 ) d] 5
S 5 = [2a + ( 5 − 1) d] [from Eq.(i)]
⇒ 1 + ( n − 1 ) d = 20 ⇒ ( n − 1) d = 19 …(ii) 2
5
Using Eq. (ii) in Eq. (i), we get = ( 2 a + 4 d ) = 5 (a + 2 d )
798 = 2 n + 19 n 2
⇒ 798 = 21 n ⇒ S 5 = 5a + 10d ...(ii)
798 and sum of first seven terms of an AP,
∴ n= = 38 7
21 S 7 = [2 a + ( 7 − 1 ) d ]
26. (a) The first five multiples of 3 are 3, 6, 9, 12 and 15. 2
7
Here, first term, a = 3, common difference, d = 6 − 3 = 3 and = [2 a + 6 d ] = 7 (a + 3 d )
number of terms, n = 5 2
5  n  ⇒ S 7 = 7a + 21d ...(iii)
∴ S 5 = [2a + ( 5 − 1 ) d] Q Sn = {2a + ( n − 1) d} Now, by given condition,
2  2 
5 S 5 + S 7 = 167
= [2 × 3 + 4 × 3] ⇒ 5a + 10d + 7a + 21d = 167
2
5 ⇒ 12a + 31d = 167 ...(iv)
= (6 + 12 ) = 5 × 9 = 45
2 Given that, sum of first ten terms of this AP is 235.
27. (i) (c) Number of rose plants in 1st, 2nd and 3rd row ...... are ∴ S 10 = 235
10
43, 41, 39, ...... ⇒ [2a + (10 − 1 ) d] = 235
So, it forms an AP with first term, 2
a = 43 and common difference, ⇒ 5 ( 2a + 9d ) = 235
d = 41 − 43 = − 2 ⇒ 2a + 9d = 47 …(v)
Let n be the number of rows required. (ii) (a) On multiplying Eq. (v) by 6 and then subtracting it
into Eq. (iv), we get
∴ an = 11
12a + 54d = 282
⇒ a + ( n − 1) d = 11
12a + 31d = 167
⇒ 43 + ( n − 1) ( − 2 ) = 11 − −

⇒ − 2 ( n − 1) = − 32 23d = 115
⇒ n − 1 = 16 ⇒ n = 17 ⇒ d=5
CBSE Term II Mathematics X (Basic) 37

(iii) (d) Now, put the value of d in Eq. (v), we get (v) (d) The difference of the production during 7th year and
2a + 9 ( 5) = 47 ⇒ 2a + 45 = 47 4th year = T7 − T4
⇒ 2a = 47 − 45 = 2 ⇒ a = 1 = a + (7 − 1)d − [a + ( 4 − 1)d]
(iv) (b) a 4 = a + 3d = 6d − 3d = 3d = 3 × 2200 = 6600
= 1 + 3 ( 5) = 1 + 15 = 16 30. (i) (b) In first day, Veer takes 51 seconds to complete the
(v) (a) Sum of first twenty terms of this AP, 200 m race. But in each day he takes 2 seconds lesser
than the previous days.
20
S20 = [2a + ( 20 − 1 ) d] Thus, AP series will formed
2
51, 49, 47, …
= 10 [2 × (1 ) + 19 × ( 5)] = 10 ( 2 + 95)
(ii) (c) Since, Veer wants to achieve the race in 31 seconds.
= 10 × 97 = 970
Let Veer takes n days to achieve the target.
Hence, the required sum of its first twenty terms is 970.
∴ Tn = a + ( n − 1)d
29. (i) (b) Let the production of TV sets in first year be ‘a’ units.
Here, a = 51, d = 49 − 51 = − 2
Then, production in the next consecutive years are
a + d, a + 2d, …. ∴ 31 = 51 + ( n − 1)( −2 )
Thus, we get the sequence, a , a + d, a + 2d, … ⇒ ( n − 1)2 = 20 ⇒ ( n − 1) = 10 ⇒ n = 11
This is an AP sequence, whose first term = a Hence, he needs minimum 11 days to achieve the goal.
and common difference = d. (iii) (b) In an AP series, we get the series of odd terms.
Hence, term 30 is not an AP.
Given, T6 = 16000 and T9 = 22600
(iv) (a) Given, an = 2 n + 3
∴ a + (6 − 1)d = 16000
∴Common difference = an + 1 − an
and a + (9 − 1)d = 22600 [Q Tn = a + ( n − 1)d ]
= 2( n + 1) + 3 − ( 2 n + 3)
⇒ a + 5d = 16000 …(i)
= 2n + 2 + 3 − 2n − 3 = 2
and a + 8d = 22600 …(ii)
(v) (a) Given, terms 2 x , x + 10, 3x + 2 are in AP.
On subtracting Eq. (i) from Eq. (ii), we get
2 x + ( 3x + 2 )
3d = 22600 − 16000 ∴ x + 10 =
2
⇒ 3d = 6600 ⇒ 2 x + 20 = 5x + 2 ⇒ 3x = 18 ⇒ x = 6
⇒ d = 2200
31. (i) (a) Since, he pays first installment of ` 1000 and next
Put d = 2200 in Eq. (i), we get consecutive months he pay the installment are
a + 5 × 2200 = 16000 1100, 1200, 1300, … .
⇒ a = 16000 − 11000 = 5000 Thus, we get the AP sequence,
Hence, the production during first year is 5000 sets. 1000, 1100, 1200, …
(ii) (b) The production during 8th year is Here, a = 1000, d = 1100 − 1000 = 100
T8 = a + ( 8 − 1) d Now, T30 = a + ( 30 − 1) d
= 5000 + 7 × 2200 = 1000 + 29 × 100 = 1000 + 2900 = 3900
= 5000 + 15400 = 20400 Hence, the amount paid by him in 30th installment is
Hence, production during 8th year is 20400 sets. ` 3900.
(iii) (d) The production during first 3 years, 30
(ii) (b) Now, S30 = [2a + ( 30 − 1)d]
3 2
S3 = [2a + ( 3 − 1)d]
2 = 15( 2 × 1000 + 29 × 100)
3 = 15( 2000 + 2900)
= [2 × 5000 + 2 × 2200]
2 = 15 × 4900 = ` 73500
= 3[5000 + 2200] (iii) (c) After 30th installment, he still have to pay
= 3 × 7200 = 21600 = 118000 − 73500 = 44500
Hence, production during first 3 years is 21600 sets. (iv) (a) The amount in last 40th installment is
(iv) (b) Let in nth year, the production is 29200 T40 = a + ( 40 − 1)d
Q Tn = a + ( n − 1)d = 1000 + 39 × 100
∴ 29200 = 5000 + ( n − 1) 2200 = 1000 + 3900
⇒ ( n − 1)2200 = 24200 = ` 4900
24200 (v) (b) The ratio of 1st installment to the last installment
⇒ ( n − 1) =
2200 1000 10
is i.e. .
⇒ n − 1 = 11 ⇒ n = 12 4900 49
Hence, production is ` 29200 in 12th year.
38 CBSE Term II Mathematics X (Basic)

Subjective Questions and a 3 − a 2 = (a + b ) 2 − (a 2 + b 2 )


−3 5 = a 2 + b 2 + 2ab − (a 2 + b 2 ) = 2ab …(ii)
1. Here, a 1 = − 1, a 2 = , a 3 = − 2 and a 4 =
2 2 From Eqs. (i) and (ii), we get
−3 1 a 2 − a1 = a 3 − a 2
Now, a 2 − a1 = +1= −
2 2 Hence, given terms are in AP.
3 1
a3 − a2 = − 2 + = − 7. Given, AP is −27 , − 22 , − 17 , − 12 …… .
2 2
5 9 Here, a = − 27 and d = − 22 + 27 = 5
a4 − a3 = + 2 =
2 2 Since, nth term, an = a + ( n − 1)d
Clearly, the difference of successive terms is not same, On putting n = 11, we get
although, a 2 − a1 = a 3 − a 2 but a 3 − a 2 ≠ a 4 − a 3 ., therefore it a11 = a + (11 − 1)d
does not form an AP. = − 27 + 10( 5) [Q a = −27 and d = 5]
2. Given , 3y − 1, 3y + 5 and 5y + 1 are three consecutive terms = − 27 + 50 = 23
of an AP. Hence, 11th term of given AP is 23.
⇒ ( 3y + 5) − ( 3y − 1) = ( 5y + 1) − ( 3y + 5) 8. Q nth term of an AP, an = a + ( n − 1 ) d
⇒ 6 = 2y − 4 ∴ a 30 = a + ( 30 − 1 ) d = a + 29d
⇒ 2 y = 10 ⇒ y = 5 and a 20 = a + ( 20 − 1 ) d = a + 19d ...(i)
3. Let the angles are (a − d)° , a ° , (a + d)°. Now, a 30 − a 20 = ( a + 29d ) − ( a + 19d ) = 10d
Then, we get and from given AP common difference,
(a − d) + a + (a + d) = 180° d = − 7 − ( − 3) = − 7 + 3 = − 4
and a + d = 2 (a − d ) ∴ a 30 − a 20 = 10 ( − 4) = − 40 [from Eq. (i)]
⇒ 3a = 180° ⇒ a = 60° 9. Let 0 be the nth term of given AP. i.e. an = 0.
and 60° + d = 2 (60° − d) Given that, first term a = 31,
⇒ 60° + d = 120° − 2d Common difference, d = 28 − 31 = − 3
⇒ 3d = 60° The nth term of an AP, is an = a + ( n − 1 ) d
⇒ d = 20° ⇒ 0 = 31 + ( n − 1 ) ( −3)
∴ The angles of an AP are ⇒ 3 ( n − 1 ) = 31 ⇒ n − 1 =
31
a − d = 60° − 20° = 40° 3
a = 60° 31 34 1
∴ n= +1= = 11
and a + d = 60° + 20° = 80° 3 3 3
Hence, angles of an AP are 40°, 60°, 80°. Since, n should be positive integer. So, 0 is not a term of the
4. No, because the total fare (in `) after each kilometre is given AP.
15, (15 + 8), (15 + 2 × 8), (15 + 3 × 8),… or15, 23, 31, 39,… 10. Let four numbers in AP are a , a + d, a + 2d, a + 3d. Then,
Let t1 = 15, t 2 = 23, t 3 = 31 and t 4 = 39 (a ) + (a + d) + (a + 2d) + (a + 3d) = 50 ⇒ 4a + 6d = 50
Now, t 2 − t1 = 23 − 15 = 8 ⇒ 2a + 3d = 25 …(i)
and (a + 3d) = 4 (a )
t 3 − t 2 = 31 − 23 = 8
⇒ a =d …(ii)
t 4 − t 3 = 39 − 31 = 8
On solving Eq. (i) and Eq. (ii), we get
Since, all the successive terms of the given list have same
a =d = 5
difference i.e. common difference = 8
∴ The four numbers in AP are
Hence, the total fare after each killometre form an AP.
a = 5, a + d = 5 + 5 = 10
5. We know that, if a , b , c are in AP, then
a + 2d = 5 + 10 = 15, a + 3d = 5 + 15 = 20
2b = a + c
Hence, four numbers in AP are 5, 10, 15 and 20.
Since, ( 3k − 2 ) , ( 4k − 6) and ( k + 2 ) are in AP.
11. Let the first term, common difference and number of terms
∴ 2( 4k − 6) = 3k − 2 + k + 2 of an AP are a , d and n, respectively.
⇒ 8k − 12 = 4k Given that, first term (a ) = 12.
⇒ 8k − 4k = 12 Now by condition,
⇒ 4k = 12 7th term (T7 ) = 11th term (T11 ) − 24
12
⇒ k= =3 [Q nth term of an AP, Tn = a + (n − 1) d]
4
⇒ a + (7 − 1) d = a + (11 − 1) d − 24
Hence, value of k is 3.
⇒ a + 6d = a + 10d − 24
6. Let a1 = (a − b )2 , a 2 = a 2 + b 2 and a 3 = (a + b )2 . ⇒ 24 = 4d ⇒ d = 6
Now, a 2 − a 1 = a 2 + b 2 − (a − b )2 ∴ 20th term of AP, T20 = a + ( 20 − 1) d = 12 + 19 × 6 = 126
= a 2 + b 2 − (a 2 + b 2 − 2ab ) = 2ab …(i) Hence, the required 20th term of an AP is 126.
CBSE Term II Mathematics X (Basic) 39

12. Let the first term, common difference and number of terms 17. Here, the first number is 11, which divided by 4 leave
of an AP are a , d and n, respectively. remainder 3 between 10 and 300. Last term before 300 is
Given that, 9th term of an AP, T9 = 0 299, which divided by 4 leave remainder 3.
[Q nth term of an AP, Tn = a + ( n − 1) d] ∴ Required AP is11, 15, 19, 23,..., 299
⇒ a + (9 − 1 ) d = 0 ⇒ a + 8 d = 0 ⇒ a = − 8 d ... (i) Here, first term (a ) = 11, common difference d = 15 − 11 = 4
Now, its19th term, T19 = a + (19 − 1) d Q nth term, an = a + ( n − 1 ) d = l [last term]
= − 8d + 18d [from Eq. (i)] ⇒ 299 = 11 + ( n − 1 ) 4
⇒ T19 = 10d ... (ii) ⇒ 299 − 11 = ( n − 1 ) 4
and its 29th term, T29 = a + ( 29 − 1) d ⇒ 4 ( n − 1 ) = 288 ⇒ ( n − 1 ) = 72
= − 8d + 28d [from Eq. (i)] ∴ n = 73
= 20d = 2 × (10d) 18. Let first term of an AP is a and common difference is d.
⇒ T29 = 2 × T19 [from Eq. (ii)] The nth term of an AP is
Hence, its 29th term is twice its 19th term. Hence proved. an = a + ( n − 1)d
13. Let first term and common difference of an AP are a and d. According to the given condition,
According to the given condition, m × am = n × an
a12 = 47 ∴ m × [a + ( m − 1)d] = n × [a + ( n − 1)d]
⇒ a + 11d = 47 ...(i) ⇒ a ( m − n) = [( n 2 − m2 ) + ( − n + m)]d
and a16 = 1 + 2a 8 [by given condition] ⇒ a( m − n) = [( n − m) ( n + m) + ( m − n)]d
⇒ [a + (16 − 1)d] = 1 + 2 [a + ( 8 − 1)d] ⇒ a = [− ( n + m) + 1] d ...(i)
⇒ a − d = −1 ...(ii) [divide both sides by m − n ]
On solving Eqs. (i) and (ii), we get Now, ( m + n)th term of an AP is
d = 4 and a = 3 am + n = a + ( m + n − 1)d
∴ a n = 3 + ( n − 1) 4 = 4 n − 1 = [− ( n + m) + 1]d + ( m + n − 1)d
 n 2 , where n is even =0 Hence proved.
14. We have, tn =  2
n − 1, where n is odd 19. Given, AP sequence is 1, 4, 7, 10, .... whose first term is a =1
For 19th term, i.e. for n = 19 which is odd, we take and common difference, d = 4 − 1 = 3.
tn = n 2 − 1 = (19)2 − 1 = 360 Q Sum of n terms of an AP is
n
15. Let the three parts of the number 207 are (a − d) , a and Sn = [2a + ( n − 1)d]
2
(a + d), which are in AP. 20
∴ S20 = [2 × 1 + ( 20 −1) × 3]
Now, by given condition, 2
Sum of these parts = 207 [put, d = 3, n = 20 and a = 1]
⇒ a − d + a + a + d = 207
= 10 [2 + 19 × 3] = 10 [2 + 57] = 590
⇒ 3a = 207
Hence, sum of the first 20 terms of an AP is 590.
a = 69
20. Given sequence in AP is 120, 116, 112, ...
Given that, product of the two smaller parts = 4623
Here, a = 120, d = 116 − 120 = − 4
⇒ a (a − d) = 4623
The nth term of an AP is
⇒ 69 ⋅ (69 − d) = 4623 ⇒ 69 − d = 67
a n = a + ( n − 1) d
⇒ d = 69 − 67 = 2
an = 120 + ( n − 1) ( − 4)
So, first part = a − d = 69 − 2 = 67,
For, first negative term, an < 0
second part = a = 69
∴ 120 + ( n − 1) ( − 4) < 0
and third part = a + d = 69 + 2 = 71,
Hence, required three parts are 67, 69, 71. ⇒ 4 ( n − 1) > 120
16. Given AP, −2 , − 4, − 6,..., − 100 ⇒ ( n − 1) > 30
⇒ n > 31
Here, first term ( a ) = − 2, common difference
( d ) = − 4 − ( − 2 ) = − 2 and the last term ( l ) = − 100. ∴ The first negative term is 32.
We know that, the nth term an of an AP from the end is 21. Here, a = 18 and d = −2
an = l − ( n − 1) d, where l is the last term and d is the Let n terms are taken, so that their sum is zero.
common difference. Then, we have
∴ 12th term from the end, Sn = 0
a12 = − 100 − (12 − 1) ( − 2 ) n
⇒ [2a + ( n − 1)d] = 0
= − 100 + (11) ( 2 ) 2
= − 100 + 22 = − 78. ⇒ 2a + ( n − 1)d = 0
⇒ 2 × 18 + ( n − 1)( −2 ) = 0
Hence, the 12th term from the end is – 78.
⇒ n − 1 = 18 ⇒ n = 19
40 CBSE Term II Mathematics X (Basic)

22. First 8 multiples of 3 are 3, 6, 9, 12, 15, 18, 21, 24. 28. Given, S 14 = 1050 and a = 10
∴The sum of first 8 multiples of 3 Let a be the first term and d be the common difference of
n 8 given AP.
= [a + l] = [3 + 24] = 4 × 27 = 108
2 2 Then , S 14 = 1050
23. The annual salary received by Subha Rao in the years 14  n 
⇒ [2a + 13d] = 1050 Q Sn = 2 [2a + ( n − 1)d
1995, 1996, 1997 etc., is ` 5000, ` 5200, ` 5400, …, ` 7000 2
Hence, the list of numbers 5000, 5200, 5400, …, 7000 forms ⇒ 7[2(10) + 13d] = 1050 [Q a = 10 given]
an AP
⇒ 20 + 13d = 150
Q a 2 − a1 = a 3 − a 2 = 200 ⇒ 13d = 130
Let nth term of an AP, an = 7000 ⇒ d = 10
⇒ 7000 = a + ( n − 1)d [Q an = a + ( n − 1)d] Now, a 21 = a + 20d
⇒ 7000 = 5000 + ( n − 1)( 200) = 10 + 20(10)
⇒ 200( n − 1) = 7000 − 5000 = 2000 = 10 + 200 = 210
2000
⇒ n −1 = = 10 Hence, 21st term of given AP is 210.
200
29. Given, a1 = 8, d1 = 20, a 2 = −30, d2 = 8
⇒ n = 10 + 1 = 11
Sn = S2n
Thus, 11th year of his service or in 2005 Subha Rao received
an annual salary ` 7000. n 2n
[2 × 8 + ( n −1) × 20] = [2 × ( −30) + ( 2 n − 1) × 8]
24. Ramkali’s savings in the subsequent weeks are respectively 2 2
` 5, ` 5 + ` 1.75, ` 5 + 2 × ` 1.75, ` 5 + 3 × 1.75 … ⇒ [16 + ( n − 1)20] = 2 [−60 + ( 2 n − 1)8]
In nth week her saving will be ` 5 + (n −1) × ` 1.75 ⇒ 16 + 20n − 20 = −120 + 32 n − 16
⇒ 5 + ( n − 1) × 1.75 = 20.75 [given] ⇒ 12 n = 132 ⇒ n = 11
⇒ ( n − 1) × 1.75 = 20.75 − 5 = 15.75 30. Here, a1 = 2 , a 2 = 8 = 2 2 , a 3 = 3 2
15.75
⇒ n −1 = =9 ∴ a = 2 , d = a 2 − a1 = 2 2 − 2 = 2
1.75
10
⇒ n = 9 + 1 = 10 ∴ S 10 = [2 × 2 + (10 − 1)( 2 )]
2
1 + 3 + 5 + K upto n terms = 5[2 2 + 9 2 ] = 55 2
25. Given, =9
2 + 5 + 8 + ... upto 8 terms
31. The odd numbers between 100 and 300 are 101, 103, 105,
n
[2(1) + ( n − 1)2] ……, 299 which form an AP.
2 n( 2 n)
⇒ =9 ⇒ =9 Here, first term (a ) = 101, last term ( l ) = 299
8 8 ( 25)
[2( 2 ) + ( 8 − 1)3] and common difference (d) = 103 − 101 = 2
2
Let there are n numbers in the AP.
⇒ n 2 = 9 × 100 ⇒ n 2 = 900 ⇒ n = 30
Then, nth term (an ) = a + ( n − 1)d = l
26. Q nth term of an AP, ⇒ 101 + ( n − 1)2 = 299
an = Sn − Sn − 1 ⇒ 2( n − 1) = 198
= 3n 2 + 5n − 3 ( n − 1 ) 2 − 5 ( n − 1 ) ⇒ n − 1 = 99
[Q Sn = 3n 2 + 5n (given)] ⇒ n = 100
= 3n + 5n − 3n − 3 + 6 n − 5n + 5
2 2 n
Now, sum of n terms Sn = (a + l )
a n = 6n + 2 …(i) 2
ak = 6 k + 2 = 164 [Q ak = 164 (given)] 100
or ∴Sum of 100 terms, S 100 = (101 + 299)
⇒ 6k = 164 − 2 = 162 2
∴ k = 27 = 50 × 400
27. In the AP, d = 5, S 10 = 75, a = ? = 20000
n 32. The two-digit numbers divisible by 6 are 12, 18, 24, …, 96.
Sn = [2a + ( n − 1)d]
2 Clearly, these numbers are in AP.
10 Here, a = 12 and d = 18 − 12 = 6
⇒ S 10 = [2a + (10 − 1)( 5)]
2 Let the AP contains n terms.
⇒ 75 = 5[2a + 9 × 5] Then, an = 96 ⇒12 + ( n − 1) × 6 = 96 [Q an = a + ( n −1)d]
⇒ 15 = 2a + 45 ⇒ 6n + 6 = 96
⇒ 2a = 15 − 45 ⇒ 6n = 96 − 6 = 90
⇒ 2a = − 30 ⇒ n = 15
⇒ a = −15 Hence, there are 15 two-digit numbers divisible by 6.
CBSE Term II Mathematics X (Basic) 41

33. Q Sum of n terms of an AP, = 7 (18 + 3 × 18)


n
Sn = [2a + ( n − 1) d ] …(i) = 7 (18 + 54)
2 = 7 × 72 = 504
8 37. Given, first term of an AP, a = 5
∴ S 8 = [2 a + ( 8 − 1 ) d ]
2 Common difference, d = 3
= 4 ( 2a + 7d ) = 8a + 28d nth term of an AP, an = 50
4
and S 4 = [2 a + ( 4 − 1 ) d ] ∴ a + ( n − 1)d = 50
2
⇒ 5 + ( n − 1)3 = 50
= 2 ( 2a + 3d ) = 4a + 6d
⇒ ( n − 1)3 = 50 − 5
Now, S8 − S4 = 8a + 28d − 4a − 6d = 4a + 22d …(ii)
45
12 ⇒ n −1 =
and S 12 = [2a + (12 − 1 ) d] = 6 ( 2a + 11 d ) 3
2
⇒ n − 1 =15
= 3 ( 4a + 22d) = 3 (S8 − S4 ) [from Eq.
(ii)] ⇒ n = 15 + 1 =16
∴ S 12 = 3(S8 − S4 ) Hence proved. ∴ The sum of nth term of an AP is
n
34. For finding, the sum of last ten terms, we write the given AP Sn = [2a + ( n − 1)d]
2
in reverse order.
16
i.e. 126, 124, 122 , ..., 12 , 10, 8 ∴ S16 = [2 × 5 + (16 −1) × 3]
2
Here, first term (a ) = 126,
= 8 [10 + 15 × 3]
common difference,
= 8 [10 + 45]
(d) = 124 − 126 = − 2
10  n  = 8 × 55 = 440
∴ S 10 = [2a + (10 − 1 ) d] Q Sn = [2a + ( n − 1) d] 38. Given, AP = 24, 21, 18 ……
2  2 
= 5 {2 (126) + 9 ( −2 )} ∴ First term, a = 24
= 5 ( 252 − 18) and common difference, d = 21 − 24 = −3
We know that,
= 5 × 234 n
= 1170 Sn = [2a + ( n − 1)d]
2
35. Given AP is 3, 5, 7, 9 …… n
⇒ 78 = [2 × 24 + ( n − 1)( −3)]
Here, a = 3, d = 5 − 3 = 7 − 5 = 2 2
n ⇒ 78 × 2 = n[48 − 3n + 3]
∴ Sn = [2a + ( n − 1)d]
2 ⇒ 156 = n [51 − 3n]
n
⇒ 120 = [2 × 3 + ( n − 1)2] ⇒ 156 = 51n − 3n 2
2
⇒ 3n 2 − 51n + 156 = 0
⇒ 240 = n[6 + 2 n − 2] ⇒ 240 = n( 2 n + 4)
⇒ n 2 −17 n + 52 = 0
⇒ 2 n 2 + 4n − 240 = 0
⇒ n −13n − 4n + 52 = 0
2
⇒ n 2 + 2 n −120 = 0 [divide by 2]
⇒ n ( n − 13) − 4( n − 13) = 0
⇒ n 2 + 12 n − 10n − 120 = 0 ⇒ ( n − 4) ( n − 13) = 0
⇒ n( n + 12 ) − 10( n + 12 ) = 0 ⇒ n − 4 = 0 or n − 13 = 0
⇒ ( n + 12 )( n − 10) = 0 ∴ n = 4 and 13
⇒ n = −12 or 10 Hence, the number of terms is either 4 or 13.
∴ n ≠ −12 39. Let a and d be the first term and common difference of an
Hence, n = 10 AP. Then,
36. For finding, the sum of first seven numbers which are multiples S 14 = 1050 and T4 = 40 [given]
of 2 as well as of 9. Take LCM of 2 and 9 which is 18. 14
⇒ [2a + (14 − 1) d] = 1050 and a + ( 4 − 1) d = 40
So, the series becomes 18, 36, 54,... 2
Here, first term (a ) = 18, ⇒ 7 [2a + 13d] = 1050 and a + 3d = 40
common difference (d) = 36 − 18 = 18 ⇒ 2a + 13d = 150 …(i)

n
Sn = [2a + ( n − 1) d] and a + 3d = 40 …(ii)
2 Multiply Eq. (ii) by 2 and subtract Eq. (ii) from Eq. (i),
7
S7 = [2 (18) + (7 − 1) 18] 13d − 6d = 150 − 80
2 ⇒ 7d = 70
7
= [36 + 6 × 18] ⇒ d = 10
2
42 CBSE Term II Mathematics X (Basic)

Put d = 10 in Eq. (i), we get Now, subtracting Eq. (iv) from Eq. (iii),
2a + 13 × 10 = 150 a + 25 d = 0
⇒ 2a = 150 − 130 a + 10 d = 3
− − −
⇒ 2a = 20 15d = − 3
⇒ a = 10 1
∴ The 20th term of an AP is ⇒ d=−
5
a 20 = a + ( 20 − 1)d Put the value of d in Eq. (iii), we get
= 10 + 19 × 10  1
a + 25  −  = 0
= 10 + 190 = 200  5
40. Let the first term, common difference and number of terms ⇒ a − 5=0
of the AP 9, 7, 5, ... are a1 , d1 and n1 , respectively. ⇒ a=5
i.e. first term (a1 ) = 9 and common difference (d1 ) Now, put the value of a , d in Eq. (v), we get
= 7 − 9 = − 2.
− 1 / 5 = 5 + ( n − 1) ( − 1 / 5)
∴ Its nth term, T′n = a1 + ( n − 1) d1
⇒ − 1 = 25 − ( n − 1)
⇒ Tn′ = 9 + ( n − 1) ( − 2 ) ⇒ − 1 = 25 − n + 1
⇒ Tn′ = 9 − 2 n + 2 ⇒ n = 25 + 2 = 27
⇒ Tn′ = 11 − 2 n ...(i) Hence, the common difference and number of terms are
Let the first term, common difference and the number of − 1 / 5 and 27, respectively.
terms of the AP 24, 21, 18, ... are a 2 , d2 and n 2 , respectively. 42. Let a and d be the first term and common difference of the
i.e. first term, (a 2 ) = 24 and common difference given AP, respectively.
(d2 ) = 21 − 24 = − 3. Given a4 = 0
∴ Its nth term, Tn′′ = a 2 + ( n − 1) d2 ⇒ a + 3d = 0
⇒ Tn′′ = 24 + ( n − 1) ( − 3) ⇒ a = − 3d …(i)
⇒ Tn′′ = 24 − 3n + 3 Now, a 25 = a + 24d
⇒ Tn′′ = 27 − 3n ...(ii) = − 3d + 24d [from Eq. (i)]
Now, by given condition, ⇒ a 25 = 21d …(ii)
nth terms of the both APs are same, Also, a11 = a + 10d
i.e. Tn′ = Tn′′ = − 3d + 10d [From Eq. (i)]
11 − 2 n = 27 − 3n [from Eqs. (i) and (ii)] ⇒ a11 = 7d
⇒ n = 16 ⇒ 3a11 = 21d …(iii)
∴ nth term of first AP, From Eqs. (ii) and (iii), we get
Tn′ = 11 − 2 n = 11 − 2 (16) a 25 = 3a11 Hence proved.
= 11 − 32 = − 21 43. Let a be the first term and d be the common difference of
and nth term of second AP, the AP.
Tn′′ = 27 − 3n Then, a17 = 2a 8 + 5 (given)
∴ a + 16d = 2(a + 7d) + 5
= 27 − 3 (16) = 27 − 48 = − 21
⇒ a + 16d = 2a + 14d + 5 [Q an = a + ( n −1)d]
Hence, the value of n is 16 and that term i.e. nth term is –21.
⇒ a − 2d = − 5 ...(i)
41. Let the first term, common difference and number of terms
of an AP are a, d and n, respectively. Also, a11 = 43 (given)
We know that, if last term of an AP is known, then ⇒ a + 10d = 43 ...(ii)
l = a + ( n − 1) d ...(i) From Eqs. (i) and (ii), we get
and nth term of an AP is −5 + 2d + 10d = 43
Tn = a + ( n − 1) d 48
...(ii) ⇒ 12d = 48 ⇒ d = =4
Given that, 26th term of an AP = 0 12
⇒ T26 = a + ( 26 − 1) d = 0 [from Eq. (ii)] Putting d = 4 in Eq. (i), we get
⇒ a + 25 d = 0 ...(iii) a − 2 × 4 =− 5 ⇒ a =− 5 + 8 = 3
∴ an = a + ( n − 1)d = 3 + ( n −1)4
11th term of an AP = 3
= 3 + 4n − 4 = 4n − 1
⇒ T11 = a + (11 − 1) d = 3 [from Eq. (ii)]
Hence, the required nth term of AP is ( 4n − 1).
⇒ a + 10d = 3 ... (iv)
44. AP : 23, 25, 27, …
and last term of an AP = − 1 / 5 First term (a ) = 23
⇒ l = a + ( n − 1) d [from Eq. (i)] Common difference (d) = 25 − 23 = 2
⇒ − 1 / 5 = a + ( n − 1) d ...(v) an (nth term) = a + ( n − 1)d
CBSE Term II Mathematics X (Basic) 43

= 23 + ( n − 1)2 Now, sum of given AP is


= 23 + 2 n − 2 = 2 n + 21 Sn =
n
[2a + ( n − 1)d]
AP : 5, 8, 11, 14, …… 2
First term (a′ ) = 5 x+3 x + 3  
∴ 1326 =  2 ×1 +  − 1 × 4
Common difference (d′ ) = 8 − 5 = 3 4×2   4  
a′n (n th term) = a′ + ( n − 1)d′  x −1 
⇒ 8 × 1326 = ( x + 3) 2 + × 4
= 5 + ( n − 1) 3  4 
= 5 + 3n − 3 ⇒ 8 × 1326 = ( x + 3) [2 + x − 1]
= 3n + 2 ⇒ 10608 = ( x + 3) ( x + 1)
Since, an = a′n [given] ⇒ 10608 = x 2 + 4x + 3
⇒ 2 n + 21 = 3n + 2 ⇒ n = 19
⇒ x 2 + 4x − 10605 = 0
a 1
45. Given, S6 = 42 and 10 = ⇒ x + (105 − 101)x − 10605 = 0
2
a 30 3
Let a be the first term and d be the common difference of [by spliting middle term]
given AP. ⇒ x ( x + 105) − 101 ( x + 105) = 0
Sum of the first n terms is given by ( x + 105) ( x − 101) = 0
n x = 101, 105
Sn = [2a + ( n − 1)d]
2 47. (i) Here, first term (a ) = 1
6 and common difference
⇒ S6 = [2a + 5d]
2 (d ) = ( − 2 ) − 1 = − 3
⇒ 42 = 3( 2a + 5d) Q Sum of n terms of an AP,
⇒ 2a + 5d = 14 …(i) n
Sn = [2a + ( n − 1) d]
a10 1 2
Also, =
a 30 3 n
⇒ Sn = [2 × 1 + ( n − 1) × ( − 3)]
a + 9d 1 2
⇒ =
a + 29d 3 n
⇒ Sn = ( 2 − 3n + 3)
⇒ 3a + 27d = a + 29d 2
n
⇒ 2a = 2d ⇒ Sn = ( 5 − 3n)
2
⇒ a =d …(ii)
...(i)
On putting a = d from Eq. (ii) in Eq. (i), we get
We know that, if the last term ( l) of an AP is known, then
2d + 5d = 14
l = a + ( n − 1) d
⇒ 7d = 14
⇒ − 236 = 1 + ( n − 1) ( − 3)
⇒ d=2
[Q l = − 236, given]
∴ a =2
⇒ − 237 = − ( n − 1) × 3
Hence, first term of the AP = 2 and 13th term of the
AP = a + 12d = 2 + 24 = 26 ⇒ n − 1 = 79
⇒ n = 80
46. Given, equation is1 + 5 + 9 + 13 + K + x = 1326
Now, put the value of n in Eq. (i), we get
Here, first term is a1 = 1
80
last term is an = l = x Sn = [5 − 3 × 80]
2
Difference of two consecutive terms,
5 − 1 = 4 and 9 − 5 = 4, which is same. = 40 ( 5 − 240)
Thus, given series is an AP. = 40 × ( − 235)
Then, nth term of given AP is = − 9400
an = a + ( n − 1)d Hence, the required sum is − 9400.
∴ x = 1 + ( n − 1 )4 Alternate Method
⇒ ( n − 1) 4 = x − 1 Given, a = 1, d = − 3 and l = − 236
x −1 ∴ Sum of n terms of an AP,
⇒ n −1 = n
4 Sn = [a + l]
x −1 2
⇒ n= +1 80
4 = [1 + ( − 236)] [Q n = 80]
x+3 2
⇒ n=
4 = 40 × ( − 235)
= − 9400
44 CBSE Term II Mathematics X (Basic)

1 Now, subtract Eq. (ii) from Eq. (iii), we get


(ii) Here, first term, a = 4 −
n a + 8d = − 30
Common difference, a + 3d = − 15
− − +
 2  1  − 2 1 −1
d = 4 −  − 4 − = + = 5d = − 15
 n  n n n n
⇒ d=−3
Q Sum of n terms of an AP,
Put the value of d in Eq. (ii), we get
n
Sn = [2a + ( n − 1 ) d] a + 3 ( − 3) = − 15
2
⇒ a − 9 = − 15
n  1  −1  ⇒ a = − 15 + 9
⇒ Sn = 2  4 −  + ( n − 1 )   
2   n  n 
⇒ a = −6
n 2 1 Q Sum of first n terms of an AP,
= 8 − − 1 + 
2  n n n
Sn = [2a + ( n − 1) d]
n 1  n  7 n − 1 2
= 7 −  = ×  
2  n 2  n  ∴ Sum of first 17 terms of an AP,
7n − 1 17
S 17 = [2 × ( − 6) + (17 − 1) ( − 3)]
=
2 2
4 2 1 17
48. Given AP sequence is − , − 1, − , ...., 4 = [− 12 + (16) ( − 3)]
3 3 3 2
4 17
Here, first term (a ) = − , = ( − 12 − 48)
3 2
4 1 17
common difference (d ) = − 1 + = = × ( − 60 )
3 3 2
1 13 = 17 × ( − 30 )
and the last term ( l ) = 4 =
3 3 = − 510
Q nth term of an AP,
Hence, the required sum of first 17 terms of an AP is − 510.
l = an = a + ( n − 1 ) d
50. Given first term of each sum of an AP is 1 and common ratio
13 4 1
⇒ = − + (n − 1) of each sum are 1, 2 and 3, respectively.
3 3 3 n
∴ S 1 = [2(1) + ( n − 1)1]
⇒ 13 = − 4 + ( n − 1 ) 2
⇒ n − 1 = 17 ⇒ n = 18 [even] n
= [2 + n − 1]
 n n  2
So, the two middle most terms are   th and  + 1 th.
 2 2  n
= ( n + 1)
18 18  2
i.e.   th and  + 1 th terms
 2 2  ∴
n
S 2 = [2(1) + ( n − 1)2]
i.e. 9th and 10th terms. 2
n
4  1  −4 + 8 4 = [2 + 2 n − 2 ]
∴ a 9 = a + 8d = − + 8  = = 2
3  3 3 3
−4 = n2
 1  −4 + 9 5
and a10 = a + 9d = +9  = = n
3  3 3 3 and S 3 = [2(1) + ( n − 1)3]
2
So, sum of the two middle most terms = a 9 + a10
n
4 5 9 = [2 + 3n − 3]
= + = =3 2
3 3 3
n
49. Let the first term, common difference and the number of = ( 3n − 1)
2
terms in an AP are a, d and n, respectively.
We know that, the nth term of an AP, LHS = S 1 + S 3
Tn = a + ( n − 1) d ... (i) n n
= ( n + 1) + ( 3n − 1)
∴ 4th term of an AP, 2 2
T4 = a + ( 4 − 1 ) d = − 15 [given] n
= [n + 1 + 3n − 1]
2
⇒ a + 3d = − 15 ...(ii)
n
and 9th term of an AP, = × 4n
2
T9 = a + (9 − 1 ) d = − 30 [given]
⇒ a + 8d = − 30 ...(iii) = 2 n 2 = 2S 2 Hence proved.
CBSE Term II Mathematics X (Basic) 45

51. Given, S4 = 40 and S14 = 280 Then, S7 = 49


7 n
Let a be the first term and d be the common difference of ⇒ [2a + 6d] = 49 [Q Sn = [2a + ( n − 1)d]
given AP. Then, 2 2
4 2
S4 = 40 ⇒ [2a + ( 4 − 1)d] = 40 ⇒ 2a + 6d = 49 ×
2 7
 n  ⇒ 2a + 6d = 14
Q Sn = 2 {2a + ( n − 1)d} ⇒ a + 3d = 7 …(i)
⇒ 2[2a + 3d] = 40 ⇒ 2a + 3d = 20 …(i) and S17 = 289
and S14 = 280 17
⇒ [a + 16d] = 289
14 2
⇒ [2a + (14 − 1)d] = 280 2
2 ⇒ 2a + 16d = 289 ×
⇒ 2a + 13d = 40 …(ii) 17
On subtracting Eq. (i) from Eq. (ii), we get ⇒ 2a + 16d = 34
10d = 20 ⇒ d = 2 ⇒ a + 8d = 17 …(ii)
On substituting d = 2 in Eq. (i), we get On subtracting Eq. (i) from Eq. (ii), we get
⇒ 2a + 3 × 2 = 20 5d = 10 ⇒ d = 2
⇒ 2a = 14 On substituting d = 2 in Eq. (i), we get
a =7 a =1
n n n
Now, Sn = [2a + ( n − 1)d] = [2(7 ) + ( n − 1)2] Now, Sn = [2a + ( n − 1)d]
2 2 2
n n
= [14 + 2 n − 2] = n[6 + n] = 6n + n 2 = [2 + ( n − 1 )2 ]
2 2
Hence, the sum of first n terms is n 2 + 6n. n
= [2 + 2 n − 2 ]
2
52. Let a and d be the first term and common difference of an AP.
n
Given that, a11 : a18 = 2 : 3 = [2 n]
2
a + 10d 2 = n2
⇒ =
a + 17 d 3 Hence, the sum of first n terms is n 2 .
⇒ 3a + 30d = 2a + 34d 54. Given that, the AP is a , b , c.
⇒ a = 4d …(i) Here, first term = a, common difference = b − a
Now, a 5 = a + 4d = 4d + 4d = 8d [from Eq. (i)] and last term, l = an = c
and a 21 = a + 20d = 4d + 20 d = 24d [from Eq. (i)] Q an = l = a + ( n − 1 ) d
∴ a5 : a 21 = 8d : 24d = 1 : 3 ⇒ c = a + ( n − 1) ( b − a )
Now, sum of the first five terms, c −a
5 ⇒ (n − 1) =
S 5 = [2 a + ( 5 − 1 ) d ] b −a
2 c −a
5 ⇒ n= +1
= [2 ( 4 d ) + 4 d ] [from Eq. (i)] b −a
2
c − a + b − a c + b − 2a
5 5
= ( 8d + 4d ) = × 12d = 30 d ⇒ n= = …(i)
2 2 b −a b −a
and sum of the first 21 terms, ∴ Sum of an AP,
21 n
S21 = [2a + ( 21 − 1 )d] Sn = [2 a + ( n − 1 ) d ]
2 2
21 ( b + c − 2a )   b + c − 2a  
= [2 ( 4d ) + 20d] [from Eq. (i)] =  2a +  − 1  ( b − a )
2 2 (b − a)   b −a  
21
= ( 28d ) = 294 d ( b + c − 2a )  c −a 
2 =  2a + ⋅ ( b − a )
So, ratio of the sum of the first five terms to the sum of the 2 (b − a)  b −a 
first 21 terms ( b + c − 2a )
= ( 2a + c − a )
S 5 : S21 = 30 d : 294 d = 5 : 49 2 (b − a)
53. Given, ( b + c − 2a )
S7 = 49 and S17 = 289 = ⋅ (a + c ) Hence proved.
2( b − a )
Let a be the first term and d be the common difference of
given AP.
46 CBSE Term II Mathematics X (Basic)

1 2 56. (i) Now, she takes ` 1 on day 1, ` 2 on day 2, ` 3 on day 3


55. Given AP is 20, 19 , 18 ,... .
3 3 and so on till the end of the month, from this money.
1 58 58 − 60 −2 i.e. 1 + 2 + 3 + 4 + ... + 31.
Here, a = 20 and d = 19 − 20 = − 20 = =
3 3 3 3 which form an AP in which terms are 31 and first term
Let n terms of given AP be required to get sum 300. (a ) = 1, common difference (d) = 2 − 1 = 1
n ∴ Sum of first 31 terms = S31
We know that, Sn = [2a + ( n − 1)d]
2 Sum of n terms,
n  −2   n
⇒ 300 = 2( 20) + ( n − 1)   Sn = [2a + ( n − 1) d]
2   3  2
31
[Qa = 20and d = − 2 / 3] ∴ S31 = [2 × 1 + ( 31 − 1) × 1]
2
 2 2 31 × 32
⇒ 600 = n 40 − n +  =
31
( 2 + 30) = = 31 × 16 = 496
 3 3 2 2
1
⇒ 600 = [120n − 2 n + 2 n]
2 So, Kanika takes ` 496 till the end of the month from this
3 money.
⇒ 600 × 3 = 122 n − 2 n 2 (ii) Let her pocket money be ` x.
⇒ 1800 + 2 n 2 − 122 n = 0 Now, she spent ` 204 of her pocket money and found that
at the end of the month she still has ` 100 with her.
⇒ 2 [n 2 − 61n + 900] = 0
Now, according to the condition,
⇒ n 2 − 61n + 900 = 0 [divide by 2]
( x − 496) − 204 = 100
⇒ n − 36n − 25n + 900 = 0
2
⇒ x − 700 = 100
⇒ n ( n − 36) − 25( n − 36) = 0 ∴ x = ` 800
⇒ ( n − 36)( n − 25) = 0 ⇒ n = 36 or 25 Hence, ` 800 was her pocket money for the month.
Since, a is positive and d is negative, so both values of n (iii) Here, a = 1, d = 1, n = 13
are possible. Now, a n = a + ( n − 1) d
Hence, sum of 25 terms of given AP ⇒ a 13 = 1 + 12(1) = 1 + 12 = 13
= Sum of 36 terms of given AP = 300. So, Kanika, saved ` 13 till January 13th, 2008.
Chapter Test
Multiple Choice Questions (iv) Total amount paid in 13th and 17th
installment is
1. The list of numbers − 10, − 6, − 2, 2, ... is (a) ` 380
[NCERT Exemplar] (b) ` 300
(a) an AP with d = − 16 (b) an AP with d = 4 (c) ` 360
(c) an AP with d = − 4 (d) not an AP (d) ` 340
2. In an AP, if a = 3.5, d = 0 and n = 101, then an will be (v) If he increases the installment by ` 6 every
month, then the amount he will pay in
(a) 0 (b) 3.5 53th installment is
(c) 103.5 (d) 104.5 (a) ` 314 (b) ` 360
(c) ` 412 (d) ` 416
3. Is an sequence defined by an = 2n2 + 1 forms an
Short Answer Type Questions
AP?
(a) Yes 7. Two AP’s have the same common difference.
(b) Not The first term of one AP is 2 and that of the
(c) Cannot be determined other is 7.
(d) None of the above
The difference between their 10th terms is the
4. The sum of first 20 terms of an AP in which same as the difference between their 21st
a = 1 and 20th term = 58 is terms, which is the same as the difference
(a) 590 (b) 580 between any two corresponding terms?
[NCERT Why?
Exemplar]
(c) 570 (d) 560
8. Determine the AP whose fifth term is 19 and
5. The 10th term of an AP is 52 and 16th term is 82, the difference of the eighth term from the
then 32nd term of the AP is [NCERT Exemplar] thirteenth term is 20.
(a) 152 (b) 159
(c) 162 (d) 156 9. Find the sum of all the 11 terms of an AP
whose middle most term is 30.
Case Based MCQs
Long Answer Type Questions
6. Kartik starts repaying a loan as first installment
of ` 100. He increases the installment by ` 5 every 10. An AP consists of 37 terms. The sum of the
month. three middle most terms is 225 and the sum
of the last three terms is 429. Find the AP.
(i) AP formed from the given situation is
(a) 105, 110, 115, ...... (b) 100, 105, 110, ...... 11. If sum of first 6 terms of an AP is 36 and that
(c) 95, 100, 105, ...... (d) 110, 115, 120, ...... of the first 16 terms is 256, then find the sum
(ii) The amount Kartik will pay in 30th of first 10 terms.
installment is 12. Which term of the AP : 121, 117, 113, ... is its
(a) ` 265 (b) ` 235
second negative term?
(c) ` 255 (d) ` 245
(iii) If Kartik pays ` 795, then it is 13. The sum of the third and the seventh terms of
(a) 140th installment an AP is 6 and their product is 8. Find the sum
(b) 150th installment of first sixteen terms of the AP.
(c) 160th installment
(d) 170th installment
14. Solve the equation −4 + (−1) + 2 + ..... + x = 437.

Answers
1. (b) 2. (b) 3. (b) 4. (a) 5. (c) 6. (i) (b) (ii) (d) (iii) (a) (iv) (d) (v) (c) For Detailed Solutions
8. 3, 7, 11, 15 9. 330 10. 3, 7, 11, 15 Scan the code
11. 100 12. 33rd term 13. 20 or 76 14. x = 50
48 CBSE Term II Mathematics X (Basic)

CHAPTER 03

Circles

In this Chapter...
l Circle
l Tangent of a Circle
l Theorem Related to Tangent of a Circle

A circle is a collection of all points in a plane which are at a Semi-circle


constant distance i.e. radius from a fixed point i.e. centre.
A diameter of a circle divides it into two equal parts or
In the given figure, O is the centre of circle and OA is the radius of in two equal arcs. Each of these two arcs is called a
the circle. Also, AB is the diameter of the circle. semi-circle.

Circumference
B A
O The length of the complete circle is called the
circumference of the circle.

Arc
Two or more circles having the same centre are called concentric
circles. A continuous piece of a circle is called an arc. In
adjoining figure, P and Q are two points on a circle
Some Important Terms Related to Circle which divide it into two parts, called the arcs. The
Chord larger part is called the major arc QRP and the smaller
part is called the minor arc PMQ.
A line segment joining any two points on the circumference of the
R
circle is called a chord of the circle. If this chord passes through the
centre, then this chord (or diamter) is the longest chord of the circle. Major arc

O
P Q
Minor arc
A B
M
CBSE Term II Mathematics X (Basic) 49

Segment Tangent to a Circle


The region between a chord and either of its arcs is called a
A line which touches the circle at a point, is called tangent to
segment of the circular region or simply a segment of the
a circle.
circle. The segment formed by minor arc along with chord, is
called minor segment and the segment formed by major arc, is In the figure, O is the centre of circle, AB is a tangent line and
called the major segment. P is a point of contact.
l
There is only one tangent at a point of the circle.
l
A circle can have maximum two parallel tangents which
Major
can be drawn to the opposite sides of the centre.
segment

A B
O

Minor segment
A B (Tangent line)
P
Sector (Point of contact)

The region between an arc and the two radii, joining the ends
of the arc to the centre, is called a sector. Length of a Tangent
The length of the segment of the tangent, between the given
Major sector point (on the tangent) and the point of contact, is called the
O
length of tangent from the given point.

Minor
A sector B O

The sector formed by minor arc, is called minor sector and


the sector formed by major arc, is called major sector.
A B
Segment of the tangent between
Important Results Related to Circle Tangent line the given point B and point of contact A
(i) The perpendicular drawn from the centre of a circle to
a chord bisects it and vice-versa. In the above figure, AB is called the length of tangent.
(ii) Equal chords of a circle are equidistant from the centre. ∴ Length of tangent to the circle from an exterior point,
(iii) The angle subtended by an arc (or corresponding AB =
chord) at the centre of the circle is twice the angle (Distance of exterior point from centre) − (Radius) 2
2

subtended by the same arc at any point on the


remaining part of the circle. Number of Tangent from a Point on a Circle
C (i) If point P lies outside the circle, then two tangents can
θ be drawn to the circle, i.e. PT1 and PT2 .
ent)
O ang
T1 (T

A B P

(iv) Equal chords of a circle subtend equal angles at T2 (T


ang
the centre. ent)

(v) The angle in a semi-circle is a right angle.


(ii) If point P lies on the circle, then there is one and only
(vi) Angles in the same segment of a circle are equal. one tangent to a circle passing through point P.
(vii) The sum of any pair of opposite angles of a cyclic
quadrilateral is 180°.
(viii) If two circles intersect at two points, then the line
through the centres is the perpendicular bisector of the
common chord.
P Tangent
50 CBSE Term II Mathematics X (Basic)

(iii) If a point P lies inside the circle, then there is no Important Results Related to Tangent to a Circle
tangent to a circle passing through a point lying
(i) If two circles touch internally or externally, then point of
inside the circle.
contact lies on the straight line through the two centres.

No Tangent

P O O′
O O′ P
P

(ii) A pair of tangents drawn at two points of a circle are either


parallel or they intersect each other at a point outside the
Theorems Related to Tangent of Circle circle.
Theorem 1 The tangent at any point of a circle is (iii) If two tangents drawn to a circle are parallel to each other,
perpendicular to the radius through the point of contact. then the line segment joining their point of contact is a
diameter of the circle.
O
(iv) If two tangents are drawn to a circle from an external point,
then
(a) They subtend equal angles at the centre,
A
P
B i.e. ∠POA = ∠POB.
A
Here, O is centre of circle and AB is tangent of circle at P
and it is point of contact and OP is radius. P O
∴ OP ⊥ AB.
Theorem 2 A perpendicular drawn from the end point of B
radius is tangent to the circle. If OP ⊥ AB, then AB is (b) They are equally inclined to the segment joining
tangent to circle.
the centre to that point,
i.e. ∠APQ = ∠BPQ.
O

A B
P

Theorem 3 The lengths of two tangents drawn from an


external point to a circle are equal.
A (v) The opposite sides of a quadrilateral circumscribing a circle
subtend supplementary angles at the centre of the circle.
O P D C

B
O
Here, P is exterior point and PA and PB are tangents.
A B
∴ PA = PB
CBSE Term II Mathematics X (Basic) 51

Solved Examples
Example 1. Prove that a tangent to a circle is From Eqs. (i) and (ii), we get
perpendicular to the radius through the point of TP = TQ
contact. Hence, T is the mid-point of the line segment PQ.
Sol. Given A circle with centre O and a tangent AB at a point
P on the circle.
Example 3. In figure, PQ is tangent to the circle with
To prove OP ⊥ AB centre O, at the point B. If ∠AOB =100 °, then find
Construction Take any point Q, other than P on the tangent ∠ABP.
AB and join OQ.

O
A 10

O
Q
B
R
P
A P Q B
Sol. Given, ∠AOB = 100° ,
Proof Here, Q is a point on the tangent AB, other than the
point of contact P. So, Q lies outside the circle (if Q lies inside
the circle, then AB becomes a secant and not a tangent to the A
circle). 100°
O
Let OQ intersects the circle at R .
Then, OP = OR [radii of the circle] B
Now, OQ = OR + RQ P Q
⇒ OQ > OR
In ∆OAB,
⇒ OQ > OP or OP < OQ [QOP = OR]
OA = OB [radii of the circle]
Thus, OP is shorter than any other segment joining O to any
point of AB. Also, we know that the shortest distance ⇒ ∠OBA = ∠OAB
between a point and a line is perpendicular distance from [angles opposite to equal sides are equal] …(i)
the point to the line. In ∆OAB,
So, OP is perpendicular to AB. ∠AOB + ∠OAB + ∠OBA = 180°
i.e. OP ⊥ AB Hence proved. [by angle sum property of triangle]
⇒ 100° + ∠OBA + ∠OBA =180° [from Eq. (i)]
Example 2. In given figure, two circles touch each ⇒ 2 ∠OBA =180° − 100°
other at the point C. Prove that the common 80°
tangent to the circles at C, bisects the common ⇒ ∠OBA =
2
tangent at P and Q.
⇒ ∠OBA = 40° …(ii)
P
T Q We know that, radius of circle is perpendicular to the
tangent.
A C B ∴ ∠OBP = 90° ⇒ ∠OBA + ∠ABP = 90°
⇒ 40° + ∠ABP = 90°
⇒ ∠ABP = 90° − 40° ⇒ ∠ABP = 50°
Sol. We know that, tangents drawn from an external point are of Example 4. In below figure, PA is a tangent from
equal length. Therefore, according to the given figure,
an external point P to a circle with centre O.
TP = TC …(i) [Q point T is external]
If ∠POB = 115°, find ∠APO.
and TQ = TC …(ii) [Q point T is external]
A
P
T Q
P O
A B 115°
C
B
52 CBSE Term II Mathematics X (Basic)

Sol. Given, ∠POB = 115° ⇒ AP 2 = OA 2 − OQ2


A [QOP = OQ = radii of a circle] …(ii)
P Now, in right angled ∆OQA,
O OA 2 = OQ2 + AQ2
115°
⇒ AQ2 = OA 2 − OQ2 ...(iii)
B
From Eqs. (ii) and (iii), we get
Since, AB is a straight line. AP 2 = AQ2
∴ ∠POB + ∠AOP = 180°
⇒ AP = AQ Hence proved.
⇒ 115° + ∠AOP = 180°
⇒ ∠AOP = 180° − 115° Example 6. Prove that the angle between the two
⇒ ∠AOP = 65° tangents drawn from an external point to a circle is
We know that, radius line is perpendicular to the tangent. supplementary to the angle subtended by the line
segment joining the points of contact at the centre.
∴ ∠PAO = 90°
Sol. Let PQ and PR be two tangents drawn from an external point
In ∆AOP,
∠PAO + ∠AOP + ∠APO = 180° P to a circle with centre O.
Q
[Q sum of all angles of a triangle is 180°]
∴ 90° + 65° + ∠APO = 180°
⇒ ∠APO = 180° − (65° + 90° ) P O
= 180° − 155° = 25°
Example 5. Prove that the length of tangents drawn
R
from an external point to a circle are equal.
Sol. Let AP and AQ are two tangents drawn from a point A to a To prove ∠QOR = 180° − ∠QPR
circle with centre O. or ∠QOR + ∠QPR = 180°
P Proof In ∆OQP and ∆ORP,
PQ = PR [Q tangents drawn from an
O external point are equal in length]
A
OQ = OR [radii of circle]
OP = OP [common sides]
Q ∴ ∆OQP ≅ ∆ORP
[by SSS congruence rule]
To prove AP = AQ
Then, ∠ QPO = ∠ RPO [by CPCT]
Construction Join OP, OQ and OA. and ∠ POQ = ∠ POR [by CPCT]
Proof We know that, a tangent at any point of a circle is ⇒ ∠ QPR = 2 ∠ OPQ
 …(i)
perpendicular to the radius through the point of contact. and ∠ QOR = 2 ∠ POQ 
Here, AP is a tangent and OP is the radius of the circle
through P. Now, in right angled ∆OQP,
∴ OP ⊥ AP ∠ QPO + ∠ QOP = 90°
Similarly, OQ ⊥ AQ ⇒ ∠ QOP = 90° − ∠ QPO
⇒ ∠OPA = ∠OQA = 90° ...(i) ⇒ 2 ∠QOP = 180° − 2 ∠QPO
[multiplying both sides by 2]
First Method
⇒ ∠ QOR = 180° − ∠ QPR [from Eq. (i)]
In ∆OPA and ∆OQA, we have
⇒ ∠ QOR + ∠ QPR = 180° Hence proved.
OP = OQ [radii of a circle]
∠OPA = ∠OQA = 90° [from Eq. (i)] Example 7. In figure, find the perimeter of ∆ABC,
OA = OA [common sides] if AP =12 cm.
So, ∆OPA ≅ ∆OQA [by RHS congruence A
rule]
∴ AP = AQ [by CPCT]
Second Method D
B C
In right angled ∆OPA,
P Q
OA 2 = OP 2 + AP 2
[by Pythagoras theorem]
⇒ AP 2 = OA 2 − OP 2
CBSE Term II Mathematics X (Basic) 53

Sol. Given, AP = 12 cm ∴ AC = AR + RC
⇒ AQ = AP = 12 cm ⇒ 11 = 4 + RC
A ⇒ RC =11 − 4 = 7 cm
Now, BC = BQ + QC
= 3 + 7 = 10 cm
D
B C Hence, length of BC is 10 cm.
P Q Example 9. In the given figure, from an external point
P, two tangents PQ and PR are drawn to a circle of
radius 4 cm with centre O. If ∠QPR = 90 °, then
find the length of PQ.
[Q tangents drawn from an external point Q
are equal in lengths]

4 cm
P
Also, BD = BP
[Q B is an external point] …(i) O R
and CD = CQ
[QC is an external point] …(ii)
Now, AP = AB + BP
⇒ 12 = AB + BD [from Eq. (i)] …(iii) Sol. We know that, if pair of tangents are drawn from an external
point P, then line joining from centre O to the point P,
and AQ = AC + CQ
bisects the angle P.
⇒ 12 = AC + CD [from Eq. (i)] …(iv)
Perimeter of ∆ABC Q
= AB + BC + AC 4 cm
= AB + BD + DC + AC 45°
P
= 12 + 12 [from Eqs. (iii) and (iv)] O
90°
= 24 cm
Hence, perimeter of a ∆ABC is 24 cm. R

Example 8. In below figure, ∆ABC is circumscribing a ∴ ∠OPQ =


∠QPR
circle, the length of BC is …… cm. 2
A 90°
= = 45°
2
m

Also, radius of circle OQ is perpendicular to the tangent line


4c

QP.
11

Now, in right angled ∆OQP,


cm

P R
OQ
tan 45° =
m

QP
3c

4
B Q C ⇒ 1=
QP
Sol. We know that, the tangents drawn from an external point to ⇒ QP = 4 cm
a circle are equal. Therefore,
A
Hence, length of PQ is 4 cm.
Example 10. In the given figure, if tangents PA and PB
m

from an external point P to a circle with centre O,


4c
4c

are inclined to each other at an angle of 80°, then


P R find ∠AOB.
m

A
7c
3c

P
80º

B 3 cm Q 7 cm C
O
BP = BQ , [point B is an external]
AP = AR [point A is an external] B
and CQ = CR [point C is an external]
54 CBSE Term II Mathematics X (Basic)

Sol. Given, ∠APB = 80°. Example 12. In given figure, two tangents TP and TQ
We know that, line drawn from centre of a circle to the are drawn to a circle with centre O from an external
tangent is perpendicular. point T. Prove that ∠PTQ = 2∠OPQ.
Since, OA⊥PA and OB⊥PB.
P
Then, ∠OAP = ∠OBP = 90° …(i)
A
T
P O
80º
O Q

B Sol. Given, TP and TQ are two tangents of a circle with centre O


and points P and Q are point of contact.
To prove ∠ PTQ= 2∠ OPQ

Since, OAPB is a quadrilateral. Proof Let ∠PTQ = θ


By using angle sum property of a quadrilateral, As we know that, the length of tangents drawn from an
∠AOB + ∠OBP + ∠APB + ∠OAP = 360° external point to a circle are equal.
⇒ ∠AOB + 90° + 80° + 90° = 360° So, ∆TPQ is an isosceles triangle.
⇒ ∠AOB = 360° − 260° Therefore, according to the given figure,
1 θ
⇒ ∠AOB = 100° ∠TPQ = ∠TQP = (180° − θ ) = 90°−
2 2
Example 11. In given figure, PA and PB are tangents to As we know that, the tangents at any point of a circle is
the circle with centre O, such that ∠APB = 50 °, perpendicular to the radius through the point of contact.
then the measure of ∠OAB is ...... P
A
T q
O
P 50°
O
Q
B
∴ ∠OPT = 90°
Sol. Given, ∠APB = 50° Now, ∠OPQ = ∠OPT − ∠TPQ
A  θ  θ ∠PTQ
= 90° − 90° −  = =
 2 2 2
P 50° O ⇒ ∠PTQ = 2 ∠OPQ Hence proved.

Example 13. In figure, a quadrilateral ABCD is drawn


B
to circumscribe a circle. Prove that
Since, P is an external point of a circle. AB + CD = BC + AD
Therefore, PA = PB A
[Q tangents drawn from an external to a circle are equal] B
⇒ ∠PBA = ∠PAB
[Q angles opposite to equal sides are equal] ...(i)
In ∆APB,
D
∠APB + ∠PBA + ∠PAB = 180° C
[Q sum of all angles of a triangle is 180°]
Sol. Given A quadrilateral ABCD is circumscribing a circle.
∴ 50° + 2 ∠PAB = 180° [from Eq. (i)]
To prove AB + CD = AD + BC
⇒ 2 ∠PAB = 130°
Proof Let P , Q, R and S be the point of contact.
⇒ ∠PAB = 65° ...(ii)
A P
Also, radius OA is perpendicular to the tangent of a circle. B
Therefore,
∠OAP = 90° S
Q
⇒ ∠OAB + ∠PAB = 90°
⇒ ∠OAB + 65° = 90° [from Eq. (ii)] D
R C
⇒ ∠OAB = 90° − 65° = 25°
CBSE Term II Mathematics X (Basic) 55

We know that, the length of tangents drawn from an external Now, ∠OPQ = 90° − 60° = 30°
point to a circle are equal. [Q ∠OPT = 90°, as radius line OP is perpendicular
∴ AP = AS to the tangent]
[Q both are tangents to a circle from point A] …(i) ⇒ ∠OQP = 30°
Similarly, BP = BQ, …(ii) [angles opposite to equal sides are equal]
CR = CQ …(iii) In ∆OPQ, using angle sum property of a triangle,
and DR = DS …(iv) ∠POQ + ∠OPQ + ∠OQP = 180°
Adding Eqs. (i), (ii), (iii) and (iv), we get ⇒ ∠POQ + 30° + 30° = 180°
( AP + BP ) + (CR + DR ) = ( AS + BQ) + (CQ + DS) ⇒ ∠POQ = 180° − 60° = 120°
⇒ AB + CD = ( AS + DS) + (BQ + CQ)
⇒ ∠PQ′ Q = 60°
⇒ AB + CD = AD + BC Hence proved.
[angle subtended by an arc at centre is twice the angle
Example 14. Prove that the tangents at the extremities subtended at remaining part of circle]
of any chord of a circle make equal angles with the ⇒ ∠PRQ = 180°−∠PQ′ Q = 120°
chord. [Q opposite angles are supplementary in a cyclic
Sol. Let AB be a chord of a circle having centre O. Let AP and BP quadrilateral PQ′ QR]
be the tangents at A and B, which intersect at point P. Example 16. In given figure, AB is a chord of circle
To prove ∠PAC = ∠PBC with centre O, AOC is diameter and AT is tangent
Construction Join points C and P. at A. Prove that ∠BAT = ∠ACB.
Proof We know that, tangents drawn from an external point C
are equal.
A
O B

O C
P
A T
B Sol. Given AB is a chord of a circle, AOC is a diameter of the
∴ In ∆PCA and ∆PCB, circle having centre O and line AT is tangent at A.
PA = PB, [QP is an external point of a circle] To prove ∠BAT = ∠ACB
∠APC = ∠BPC C
[Q AP and BP are equally inclined to OP]
and PC = PC [common sides]
O B
∴ ∆PAC ~ ∆PBC [by SAS similarity rule]
⇒ ∠PAC = ∠PBC [by CPCT]
A T
Example 15. In given figure, PQ is a chord of a circle
and PT is tangent at P such that ∠QPT = 60 °, then Proof We know that, diameter of a circle subtends 90° to the
the measure of ∠PRQ is ....... semi-circle.
∴ ∠ABC = 90°
Q Let ∠ACB = θ,
O
then ∠CAB = 180° − (90° + θ)
60° [by using angle sum property of triangle]
R
P ⇒ ∠CAB = 90° − θ ...(i)
T We know that, radius of a circle is perpendicular to the
Sol. Take a point Q′ on circle and join PQ′ and QQ′. tangent.
∴ ∠OAT = 90°
Q′ ⇒ ∠OAB + ∠BAT = 90°
Q ⇒ ∠CAB + ∠BAT = 90° [Q ∠OAT = ∠CAT]
O ⇒ 90° − θ + ∠BAT = 90°
⇒ ∠BAT = θ
º R
60 ⇒ ∠BAT = ∠ACB Hence proved.
A P T
56 CBSE Term II Mathematics X (Basic)

Chapter
Practice
PART 1 P
50°
R

Objective Questions
O
l
Multiple Choice Questions Q
1. A line intersecting a circle in two points is called a
[CBSE 2020 (Basic)] 7. In the given figure, PA is a tangent from an external
(a) secant point P to a circle with centre O. If ∠POB = 115°,
(b) tangent then ∠APO is
(c) chord
(d) None of the above A
2. In the given circle in figure, number of tangents
O
parallel to tangent PQ is [CBSE 2020 (Basic)]
115°
P
B
Q P

(a) 25° (b) 20° (c) 30° (d) 65°


8. In the given figure, AB is a tangent to the
circle with centre at O from an external point A.
If OA = 6 cm and OB = 3 3 cm, then the length of
(a) 0 (b) many (c) 2 (d) 1
the tangent is [CBSE 2020 (Basic)]
3. The lengths of the tangents drawn from an external
point to a circle are ……… [CBSE 2020 (Basic)] B
(a) AP = 2 AQ
(b) AP = AQ
1
(c) AP = AQ O A
2
(d) None of the above
4. A chord of a circle of radius 10 cm, subtends a right
angle at its centre. The length of the chord (in cm) (a) 3 cm (b) 3 3 cm (c) 9 cm (d) 33 cm
is [CBSE 2020 (Basic)]
5
9. PQ is a tangent drawn from a point P to a circle with
(a) (b) 5 2 (c)10 2 (d)10 3 centre O and QOR is a diameter of the circle such
2
that ∠POR = 135°, then ∠OPQ is
5. PQ is a tangent to a circle with centre O at the (a) 60° (b) 45° (c) 30° (d) 90°
point P. If ∆OPQ is an isosceles triangle, then
∠OPQ is equal to 10. If the two tangents inclined at an angle of 60° are
(a) 30° (b) 45° (c) 60° (d) 90° drawn to a circle of radius 3 cm, then the length of
each tangent is [CBSE 2020 (Basic)]
6. In figure, if O is the centre of a circle, PQ is a chord
3 3
and the tangent PR at P makes an angle of 50° with (a) 3 cm (b) cm
PQ, then ∠POQ is equal to [NCERT Exemplar] 2
(a) 100° (b) 80° (c) 90° (d) 75° (c) 3 3 cm (d) 6 cm
CBSE Term II Mathematics X (Basic) 57

11. AB and CD are two parallel tangents to a circle of 18. PA is a tangent to the circle with centre O.
radius 5 cm. The distance between the tangents is If BC = 3 cm, AC = 4 cm and ∆ACB ~ ∆PAO, then
[CBSE 2020 (Basic)] OA is equal to [CBSE 2013]
(a) 50 cm (b) 10 cm (c) 5 cm (d) 2 5 cm B

12. If tangents PA and PB drawn from an external point C


P to a circle with centre O are inclined to each other
O
at an angle of 80°, then find ∠POA. [CBSE 2020 (Basic)]
(a) 50° (b) 100°
(c) 120° (d) None of these
A P
13. In the given figure, find the value of x°.
A 5
(a) 2.7 cm (b) 5 cm (c) 5 cm (d) cm
2
x° 19. If the tangents PA and PB from an external point P
30°
B to a circle with centre O are inclined to each other
O
at an angle of 80°, then ∠POA equals
[CBSE 2020 (Basic)]
(a) 50° (b) 60° (c) 70° (d) 80°
(a) 130° (b) 75° (c) 120° (d) 60° 20. In the adjoining figure, PQ is a chord of a circle
14. QP is a tangent to a circle with centre O at a point P with centre O and PT is a tangent at P such that
on the circle. If ∆OPQ is isosceles, then ∠OQP ∠QPT = 60 °, then ∠PRQ =
equals [CBSE 2020 (Basic)]
(a) 30° (b) 45° (c) 60° (d) 90° Q

15. At one end A of a diameter AB of a circle of radius O


5 cm, tangent XAY is drawn to the circle. The
R
length of the chord CD parallel to XY and at a 60°
distance 8 cm from A is [NCERT Exemplar] A P T
(a) 4 cm (b) 5 cm (c) 6 cm (d) 8 cm (a) 120° (b) 160° (c) 130° (d) 150°
16. In figure, AB is a chord of the circle and AOC is its 21. In figure, if PQR is the tangent to a circle at Q,
diameter such that ∠ ACB = 50 °. If AT is the tangent whose centre is O, AB is a chord parallel to PR and
to the circle at the point A, then ∠BAT is equal to ∠BQR = 70 °, then ∠AQB is equal to
[NCERT Exemplar] [NCERT Exemplar]
C D B
A

O B O

70°
A T P Q R
(a) 45° (b) 60° (c) 50° (d) 55° (a) 20° (b) 40° (c) 35° (d) 45°
17. In the adjoining figure, AD = 8 cm, AC = 6 cm and 22. In figure, AP , AQ and BC are tangents to the circle
TB is the tangent at B to the circle with centre O. with centreO. If AB = 5 cm, AC = 6 cm and BC = 4 cm,
If BT is 4 cm, then OT = [CBSE 2013] then the length of AP (in cm) is [CBSE 2020 (Basic)]
A A

R
B C
C D
O P Q
O

B 4 cm T
(a) 41 cm (b) 41 cm (c) 40 cm (d) 40 cm (a) 15 (b) 10 (c) 9 (d) 7.5
58 CBSE Term II Mathematics X (Basic)

23. Tangents AP and AQ are drawn to circle with centre l


Case Based MCQs
O from an external point A, then ∠PAQ is equal to
∠OPQ ∠OPQ ∠OPQ 29. A playground is in the shape of a triangle with right
(a) 2∠OPQ (b)
2
(c)
3
(d)
4 angle at B , AB = 3 m and BC = 4 m. A pit was dig
inside it such that it touches the walls AC, BC and
24. In the given figure, two tangents AB andAC are
AB at P , Q and R, respectively such that AP = x m.
drawn to a circle with centre O such that
∠BAC = 120 °, then OA is equal to A

O
P
r
B C
120° R
O
A
(a) 2AB (b) 3AB (c) 4AB (d) 5AB B Q C

25. In the given figure, O is the centre of a circle, BOA


is its diameter and the tangent at the point P meets
BA extended at T. If ∠PBO = 30 °, then ∠PTA = Based on the above information, answer the
[CBSE 2016] following questions.
P
(i) The value of AR =
30°
(a) 2x m (b) x / 2 m
B T
O A (c) x m (d) 3x m
(ii) The value of BQ =
(a) 2x m (b) ( 3 − x ) m
(a) 40° (b) 50° (c) 30° (d) 20°
(c) ( 2 − x ) m (d) 4x m
26. In figure, O is the centre of circle. PQ is a chord
and PT is tangent at P which makes an angle of (iii) The value of CQ =
50° with PQ. ∠POQ is [CBSE 2020 (Basic)] (a) ( 4 + x ) m (b) ( 5 − x ) m
T (c) (1 + x ) m (d) Both (b) and (c)
Q (iv) Which of the following is correct?
50°
(a) Quadrilateral AROP is a square
P O (b) Quadrilateral BROQ is a square
(c) Quadrilateral CQOP is a square
(d) None of the above
(a) 130° (b) 90° (c) 100° (d) 75° (v) Radius of the pit is
27. In given figure, AB is diameter of a circle with centre (a) 1 m (b) 3 m
O and AT is tangent. If ∠AOQ = 58 ° , then ∠ATQ is (c) 4 m (d) 5 m
[CBSE 2015] 30. A student draws two circles that touch each other
B
externally at point K with centres A and B and radii
6 cm and 4 cm, respectively as shown in the figure.
O
T 3c
58° Q m

A T X K
P Q
6 cm A 4 cm B Y
(a) 52° (b) 58° (c) 61° (d) 62°
8
cm

28. In figure, if ∠AOB = 125°, then ∠COD is equal to


S
[NCERT Exemplar]
A Based on the above information, answer the
B following questions.
125°
(i) The value of PA is
O
(a) 10 cm
(b) 5 cm
C (c) 13 cm
D
(a) 62.5° (b) 45° (c) 35° (d) 55° (d) Can’t be determined
CBSE Term II Mathematics X (Basic) 59

(ii) The value of BQ is 5. If a number of circles touch a given line segment


(a) 4 cm (b) 5 cm PQ at a point A, then their centres lie on the
(c) 6 cm (d) 18 cm perpendicular bisector of PQ. Why or why not?
(iii) The value of PK is [NCERT Exemplar]
(a) 13 cm (b) 15 cm 6. Two concentric circles are of radii 5 cm and 3 cm.
(c) 16 cm (d) 18 cm Find the length of chord of the larger circle which
(iv) The value of QY is touches the smaller circle. [CBSE 2020 (Basic)]
(a) 2 cm (b) 5 cm 7. If a chord AB subtends an angle of 60° at the
(c) 1 cm (d) 3 cm centre of a circle, then find the angle between the
(v) If two circles touch externally, then the number of tangents at A and B. [NCERT Exemplar]
common tangents can be drawn is 8. In the given figure, XP and XQ are tangents from
(a) 1 (b) 2
X to the circle with centre O. R is a point on the
(c) 3 (d) None of these
circle and AB is tangent at R. [CBSE 2020 (Basic)]
P
PART 2 A

Subjective Questions O R X

B
l
Short Answer Type Questions Q
1. In figure, find the length of the tangent PQ drawn Prove that XA + AR = XB + BR.
from the point P to a circle with centre at O, given
that OP =12 cm and OQ = 5 cm. [CBSE 2020 (Basic)] 9. Two tangents TP and TQ are drawn to a circle
with centre O from an external point T. Prove that
∠PTQ = 2∠OPQ. [CBSE 2020 (Basic)]
10. If from an external point B of a circle with centre
O
O, two tangents BC and BD are drawn, such that
12 cm 5 cm ∠ DBC = 120 °, prove that BC + BD = BO i.e.
BO = 2 BC.
P Q 11. In the figure, two circles touch each other at a point
2. If figure, PT is tangent to a circle centred at O. Find C. Prove that the common tangent to the circles at C,
the value of ∠OTP, if ∠POT = 75°. [CBSE 2020 (Basic)] bisects the common tangent at P and Q.
[CBSE 2020 (Basic)]
P P
R
Q

C
O T

12. In figure, common tangents AB and CD to two


3. If AB is a chord of a circle with centre O, AOC is a circles intersect at E. Prove that AB = CD.
diameter and AT is the tangent at A as shown in [NCERT Exemplar]
figure. Prove that ∠ BAT = ∠ ACB. [NCERT Exemplar] A
D
C

E
O B
B
C

A
13. If PA and PB are two tangents drawn from a point
T
P to a circle with centre O touching it at A and B,
4. Prove that the tangents drawn at the ends of a prove that OP is perpendicular bisector of AB.
diameter of a circle are parallel. [CBSE 2020 (Basic)] [CBSE 2008]
60 CBSE Term II Mathematics X (Basic)

14. Tangents AP and AQ are drawn to circle with centre 22. In the given figure, AD is a diameter of a circle with
O from an external point A. Prove that centre O and AB is a tangent at A. C is a point on the
∠PAQ = 2∠OPQ. [CBSE 2013, 12,11, 09] circle such that DC produced intersects the
15. Prove that the opposite sides of a quadrilateral tangent at B and ∠ABD = 50 °. Find ∠COA. [CBSE 2015]
D
circumscribing a circle subtend supplementary
angles at the centre of the circle. [CBSE 2020 (Basic)]
16. ∆ABC is a right angled triangle with ∠B = 90 °, O C
BC = 3 cm and AB = 4 cm. A circle with centre O
and radius r cm has been inscribed in ∆ABC. Find 50°
the radius of the incircle. B
A
17. The radii of two concentric circles are 13 cm and 23. Tangents PQ and PR are drawn to a circle such that
8 cm. AB is a diameter of the bigger circle. BD is a ∠RPQ = 30 °. A chord RS is drawn parallel to the
tangent to the smaller circle touching it at D. tangent PQ. Find ∠RQS. [CBSE 2015]
Find the length of AD. [CBSE 2010]
24. PA and PB are the tangents to a circle, which
18. In figure, a circle is inscribed in a ∆ABC touching circumscribes an equilateral ∆ABQ. If ∠PAB = 60 °,
BC, CA and AB at P , Q and R respectively, as shown in the figure, prove that QP bisects AB at
A right angle. [CBSE 2015]
A

R Q 60°
Q P
M

B P C
B
If AB =10 cm, AQ = 7 cm, CQ = 5 cm, find the
length of BC. [CBSE 2020 (Basic)] 25. Two circles with centres O and O′ of radii 3 cm and
4 cm, respectively intersect at two points P and Q,
l
Long Answer Type Questions such that OP and O′ P are tangents to the two
circles. Find the length of the common chord PQ.
19. Prove that the tangent at any point of a circle is [NCERT Exemplar]
perpendicular to the radius through the point of 26. If an isosceles ∆ABC in which AB = AC = 6 cm, is
contact. [CBSE 2020 (Basic)] inscribed in a circle of radius 9 cm, find the area of
20. AC and AD are tangents at C and D, respectively. the triangle.
If ∠BCD = 44 ° , then find∠CAD , ∠ADC, ∠CBD 27. In a figure, the common tangents AB and CD of
and ∠ACD. two circles with centres O and O′ intersect at E.
C Prove that the points O, E and O′ are collinear.
A
[NCERT Exemplar]
A
44° D
O
D O O′
E

C B
B
28. In figure, O is the centre of a circle of radius 5 cm,
21. In figure, a circle touches all the four sides of a
T is a point such that OT =13 and OT intersects the
quadrilateral ABCD. If AB = 6 cm, BC = 9 cm and
circle at E, if AB is the tangent to the circle at E,
CD = 8 cm, then find the length of AD.
find the length of AB. [NCERT Exemplar]
C P
A
D

O T
E

B
A B
Q
CBSE Term II Mathematics X (Basic) 61

l
Case Based Questions (a) 6 cm (b) 12 cm
(c) 15 cm (d) 10 cm
29. Dheeraj loves geometry. So, he was curious to know A
more about the concepts of circles. His grand father
is a mathematicians. So, he reached to his grand
father to learn something interesting about tangents
and circles. His grand father gave him knowledge F
on circles and tangents and ask him to solve the
R O E
following questions.

B D C

(iii) In the given figure, what is the length of CD?


A 4 cm 2 cm
B
P

5 cm
10 cm
S Q
R
C
D
(a) 11 cm (b) 9 cm
(c) 7 cm (d) 13 cm
(iv) If PA and PB are two tangents to a circle with
centre O from an external point P such that
∠OPB = 50 °, then find ∠BPA
(i) In the given figure, AP, AQ and BC are tangents to
(a) 60° (b) 50°
the circle such that AB = 7 cm, BC = 4 cm and
(c) 120° (d) 100°
AC = 9 cm. Find AP
A
(v) In the given figure, P is an external point from,
which tangents are drawn to two externally
touching circles. If PA = 11 cm, then find PC.
P
D
B C
P Q

A
B C
(a) 12 cm (b) 15 cm
(c) 13 cm (d) 10 cm
(ii) A circle of radius 3 cm is inscribed in a right (a) 3.5 cm (b) 4 cm
angled ∆BAC such that BD = 9 cm and DC = 3 cm. (c) 11 cm (d) Can’t be determined
Find the length of AB.
62 CBSE Term II Mathematics X (Basic)

SOLUTIONS
Objective Questions ∴ ∆OPQ is an isosceles triangle.
1. (a) When there are two common points P and Q between a Q
circle and a line AB.

A P O
P

∠OQP = 90°
OP = QP
Q ∴ ∠POQ = ∠OPQ
B In ∆OPQ, ∠POQ + ∠OQP + ∠OPQ = 180°
⇒ 2 ∠OPQ = 180° − 90° [Q ∠POQ = ∠OPQ]
In this case, line AB intersects the circle at two points, so ⇒ 2 ∠OPQ = 90° ⇒ ∠OPQ = 45°
this line is called secant of the circle.
6. (a) Given, ∠ QPR = 50°
2. (d) If a point lies on the circle, then there is one and only
We know that, the tangent at any point of a circle is
one line passing through the point which touch the circle in
perpendicular to the radius through the point of contact.
one point.
∴ ∠OPR = 90°
Hence, there is one and only one tangent to the circle
passing through the point lying on the circle and this point ⇒ ∠OPQ + ∠QPR = 90° [from figure]
will be its point of contact. ⇒ ∠OPQ = 90° − 50° = 40° [Q ∠QPR = 50°]
3. (b) Now, OP = OQ = Radius of circle
P ∴ ∠OQP = ∠ OPQ = 40°
[since, angles opposite to equal sides are equal]
r
In ∆OPQ, ∠ O + ∠ P + ∠ Q = 180°
O A
[since, sum of all angles of a triangle = 180°]
r ⇒ ∠ O = 180° − ( 40° + 40° ) [Q ∠P = 40° = ∠Q]
Q = 180° − 80° = 100°
7. (a) Here, ∠OAP = 90° [Q tangent at any point of a circle is
In ∆APO and ∆AQO,
perpendicular to the radius]
OP = OQ (radius)
Now, ∠AOP + ∠BOP = 180°
∠OPA = ∠OQA = 90°
⇒ ∠AOP + 115° = 180°
OA = OA (common)
⇒ ∠AOP = (180° − 115° ) = 65°
From RHS rule,
And also, ∠OAP + ∠AOP + ∠APO = 180°
∆APO ~= ∆AQO
[angle sum property of triangle]
AP = AQ
⇒ 90°+ 65°+∠APO = 180°
4. (c) In ∆ABC,
⇒ 155° + ∠APO = 180°
Using Pythagoras theorem, AB2 + AC 2 = BC 2
⇒ ∠APO = 180°−155° = 25°
8. (a) Given, OA = 6 cm, OB = 3 3 cm

B
A
cm
cm 90° 10
10 O A

B C

⇒ (10)2 + (10)2 =BC 2 ⇒100 + 100 = BC 2 We know that, AB ⊥ OB,


⇒ 200 = BC ⇒ BC = 200 = 10 2
2 ∴By Pythagoras theorem, OA 2 = OB2 + AB2

∴The length of the chord is10 2 cm. ⇒ (6)2 = ( 3 3 )2 + AB2


5. (b) Since, PQ is a tangent to a circle from a point P and ⇒ 36 = 27 + AB2 ⇒ AB2 = 9 ⇒ AB = 3
centre of circle is O. Hence, the length of the tangent AB is 3 cm.
CBSE Term II Mathematics X (Basic) 63

9. (b) Given, PQ is a tangent from point P, centre O and QOR Join OA and OB,
as diameter. Clearly, OA ⊥ PA and OB ⊥ PB
R
[Q radius of a circle is perpendicular to the
tangent at the point of contact]
135° ∴ ∠OAP = 90° and ∠OBP = 90°
O
Now, in quadrilateral OAPB,
∠OAP + ∠APB + ∠OBP + ∠AOB = 360°
[Q sum of all interior angles = 360°]
P
Q ⇒ 90°+ 80° + 90°+ ∠AOB = 360° ⇒ ∠AOB = 100°
In ∆PQO, ∠ROP = ∠OPQ + 90° Since, tangents drawn to a circle from an external point
[Q exterior angle of a triangle is equal to the sum subtend equal angles at the centre.
of opposite angles] 1 1
∴ ∠POA = ∠POB ⇒ ∠POA = ∠AOB = × 100° = 50°
∠OPQ = 135° − 90° = 45° 2 2
10. (c) Let PA and PB be two tangents to the circle with centre O.
A 13. (c) Given, ∠OBA = 30°
In ∆ABO, x ° = ∠ABO + 90°
3 cm
30º [Q external angle = sum of opposite internal angles]
P O
30º x° = 30° + 90° = 120°
14. (b) We know that, the radius and tangents are perpendicular
B at their point of contact.
P
We know that, radius is perpendicular to the tangent at the Q
point of contact.
∴ OA ⊥ AP
∠APB 60° O
Also, ∠OPA = = = 30° [Q OP bisects∠APB ]
2 2
AP
Now, in ∆OAP, = cot 30°
OA Now, in isosceles ∆POQ
AP
⇒ = 3 ∠POQ + ∠OPQ + ∠OQP = 180°
3
⇒ 2 ∠OQP + 90° = 180°
∴ AP = 3 3 cm
⇒ ∠OQP = 45°
11. (b) Let the two parallel tangents AB and CD touch the circle 15. (d) First, draw a circle of radius 5 cm having centre O.
of radius 5 cm at M and N respectively. A tangent XY is drawn at point A.
B D
C
B
5 cm

X E
5 cm cm
M
O
N cm O 3 D
5
cm
A 8

A C Y
∴The distance between the tangents is A chord CD is drawn, which is parallel to XY and at a
= MN = MO + ON = 5 cm + 5 cm = 10 cm distance of 8 cm from A.
Also, AE = 8 cm. Join OC
where MO = ON = radius of circle = 5 cm
Now, in right angled ∆OEC,
12. (a) Given, PA and PB are two tangents drawn from a
point P, such that ∠APB = 80°. OC 2 = OE 2 + EC 2
P 80° ⇒ EC 2 = OC 2 − OE 2
B
[by Pythagoras theorem]
= 52 − 32
[Q OC = radius = 5 cm, OE = AE − AO = 8 − 5 = 3 cm]
= 25 − 9 = 16 ⇒ EC = 4 cm
O Hence, length of chord CD = 2 CE = 2 × 4 = 8 cm
A
[since, perpendicular from centre to the
chord bisects the chord]
64 CBSE Term II Mathematics X (Basic)

16. (c) In figure, AOC is a diameter of the circle. We know that, PA = PB[Lengths of tangent drawn from
diameter subtends an angle 90° at the circle. external point is equal]
So, ∠ ABC = 90° OP = OP [common]
In ∆ACB, ∠A + ∠B + ∠C = 180° ∴ ∆OAP ~= ∆OBP [SSS congruence]
[since, sum of all angles of a triangle is 180°] So, ∠OPA = ∠OPB
⇒ ∠ A + 90° + 50° = 180° [CPCT]
1 1
⇒ ∠A + 140 = 180 So, ∠OPA = ∠APB = × 80° = 40°
2 2
⇒ ∠ A = 180° − 140° = 40°
In ∆OPA,
∠A or ∠OAB = 40°
∠POA + ∠OPA + ∠OAP =180°
Now, AT is the tangent to the circle at point A. So, OA is
perpendicular to AT. [Angles sum property of triangle]
∴ ∠ OAT = 90° [from figure] ⇒ ∠POA + 40° + 90° = 180°
⇒ ∠ OAB + ∠ BAT = 90° ⇒ ∠POA + 130° = 180°
On putting ∠OAB = 40°, we get ⇒ ∠POA = 50°
⇒ ∠ BAT = 90° − 40° = 50° 20. (a) Take a point Q′ on circle and join PQ′ and QQ′.
Hence, the value of ∠BAT is 50°.
Q′
17. (b) Clearly, ∠CAD = 90° [angle in a semi-circle]
Q
So, in ∆ACD, CD2 = AC 2 + AD2 = 36 + 64 = 100 O
[by Pythagoras theorem]
⇒ CD = 10 cm 60
º R
Therefore, OC = OD = OB = 5 cm [Q radius of a circle] P
A T
Since, ∠OBT = 90°
Now, ∠OPQ = 90°−60°= 30° [Q ∠OPT = 90°]
[angle between radius and tangent]
⇒ ∠OQP = 30° [angles opposite to equal sides are equal]
So, in ∆OBT, OT2 = OB2 + BT2 ⇒ ∠POQ = 120°
= 25 + 16 = 41 [by Pythagoras theorem] [angle sum property of a triangle]
⇒ OT = 41 cm ⇒ ∠PQ′ Q = 60°
18. (d) In ∆ACB, [angle subtended by an arc at centre is twice the angle
∠BCA = 90° [angle in a semi-circle] subtended at remaining part of circle]
⇒ ∠PRQ = 120° [Q opposite angles are
∴ AB2 = AC 2 + BC 2 [by Pythagoras theorem]
supplementary in a cyclic quadrilateral PQQ′ R]
⇒ AB2 = 42 + 32
21. (b) Given, AB|| PR
⇒ AB2 = 16 + 9 cm
⇒ AB2 = 25 cm A D B
⇒ AB = 5 cm
5
⇒ OA = cm O
2
19. (a) Given PA and PB are tangents to circle and ∠APB = 80°
70°
To find ∠POA
P Q R
Construction Join OA, OB and OP.
∴ ∠ ABQ = ∠ BQR = 70° [alternate angles]
A
Also, QD is perpendicular to AB and QD bisects AB.
In ∆QDA and ∆QDB,
40º
P
40º O ∠QDA = ∠QDB [each 90°]
AD = BD
QD = QD [common side]
B
∴ ∆ ADQ ~ ∆ BDQ
Proof Since, PA is tangent [by SAS similarity criterion]
OP ⊥ PA Then, ∠QAD = ∠ QBD [by CPCT] ...(i)
[∴Tangent at any point of circle is perpendicular Also, ∠ABQ = ∠BQR [alternate interior angle]
to the radius through point of contact]
∴ ∠ABQ = 70° [Q ∠BQR = 70°]
∴ ∠OAP = 90°
Hence, ∠QAB = 70° [from Eq. (i)]
In ∆OAP and ∆OBP,
Now, in ∆ ABQ, ∠ A + ∠ B + ∠ Q = 180°
OA = OB [Both radius]
⇒ ∠ Q = 180° − ( 70° + 70° ) = 40°
CBSE Term II Mathematics X (Basic) 65

22. (d) We know that tangent segments to a circle from the same = 180° − ( 30° + 120° )
external point are congruent. Therefore, we have [Q ∠BPT = 90° + 30° = 120° ]
AP = AQ, BP = BR and CQ = CR = 180° − 150°
A = 30°
26. (c) ∠OPQ = ∠TPO − ∠TPQ = 90° − 50° = 40°
R ∴ OP = OQ [radii of circle]
B C
∴ ∠OPQ = ∠OQP = 40°
P Q
[Q equal opposite sides have equal angles]
O
∴ ∠POQ = 180° − ( ∠OPQ + ∠OQP )
= 180° − ( 40° + 40° )
= 180° − 80° = 100°
Now, AB + BC + AC = 5 + 4 + 6 = 15 1 1
⇒ AB + BR + RC + AC = 15 cm 27. (c) ∠ABQ = ∠AOQ = ( 58° ) = 29°
2 2
⇒ AB + BP + CQ + AC = 15 cm
⇒ AP + AQ =15 cm [angle subtended by an arc at the centre is twice the angle
⇒ 2 AP = 15 cm ⇒ AP = 7.5 cm [Q AP = AQ] subtended at remaining part of circle]
and ∠BAT = 90° [angle between radius and tangent]
23. (a) Here, AP = AQ
In ∆ABT, we get,
⇒ ∠AQP = ∠APQ = x (say)
∠ATQ = 180° − ( 29°+90° ) = 61°
[Q angles opposite to equal sides of a triangle are equal]
[angle sum property of a triangle]
P
28. (d) We know that, the opposite sides of a quadrilateral
circumscribing a circle subtend supplementary angles at the
O A centre of the circle.
i.e. ∠AOB + ∠COD = 180°
Q ⇒ ∠ COD = 180° − ∠AOB
= 180° − 125° = 55°
In ∆APQ, ∠PAQ = 180° − ( ∠APQ + ∠AQP )
29. Here, in right angled ∆ABC, AB = 3 m and BC = 4 m.
= 180° − ( x + x ) = 180° − 2x
Q OP ⊥ AP ∴By Pythagoras theorem,
∴ ∠OPA = 90° AC = ( AB)2 + (BC )2
⇒ ∠OPQ + ∠APQ = 90°
= ( 3)2 + ( 4)2 = 9 + 16 = 25 = 5 m
⇒ ∠OPQ + x = 90°
Also, AP = x m
⇒ ∠OPQ = 90° − x (i) (c) AR = AP = x m … (i)
⇒ 2 ∠OPQ = 180°−2 x [multiplying by 2] [since, length of tangents drawn from an
⇒ ∠PAQ = 2 ∠OPQ external point are equal]
24. (a) In ∆OAB and ∆OAC, we have (ii) (b) BQ = BR = AB − AR = ( 3 − x ) m [using Eq. (i)]
∠OBA = ∠OCA = 90° (iii) (d) CQ = CP = AC − AP = ( 5 − x ) m
OA = OA [common] Also, CQ = BC − BQ = BC − BR
and OB = OC [radii of circle] = 4 − ( 3 − x) = 1 + x
So, by RHS congruence criterion, (iv) (b) Since, CQ = 5 − x = 1 + x
∆OBA ≅ ∆OCA ⇒ 4 = 2x ⇒ x = 2
⇒ ∠OAB = ∠OAC ∴ AR = AP = 2 m, BR = BQ = 1 m
1 and CP = CQ = 3 m
= × 120° = 60°
2 A
In ∆OBA, we have
AB
cos60° =
OA P
r
1 AB
⇒ = ⇒ OA = 2 AB R O
2 OA
25. (c) ∠OPB = 30°
B C
[Q angles opposite to equal sides are equal] Q
and ∠OPT = 90° Also, OQ ⊥ BQ and OR ⊥ BR
Now, ∠PTA = 180°−( ∠OBP + ∠BPT) ∴ BROQ is a square.
[angle sum property of a ∆BPT] (v) (a) Radius of the pit, OR = BR = 1 m
66 CBSE Term II Mathematics X (Basic)

30. Here, AS = 6 cm, BT = 4 cm [Q radii of circles] ⇒ ∠OTP + 165° = 180°


(i) (c) Since, radius at point of contact is perpendicular to ⇒ ∠OTP = 180° − 165°
tangent. ⇒ ∠OTP = 15°
∴ By Pythagoras theorem, we have 3. Since, AC is a diameter line, so angle in semi-circle makes
PA = PS2 + AS2 an angle 90°.
∴ ∠ ABC = 90°
= 82 + 62 In ∆ ABC, ∠ CAB + ∠ ABC + ∠ ACB = 180°
= 64 + 36 [Q sum of all interior angles of any triangle is 180°]
= 100 = 10 cm ⇒ ∠ CAB + ∠ ACB = 180° − 90° = 90° …(i)
(ii) (b) Again, by Pythagoras theorem, we have Since, diameter of a circle is perpendicular to the tangent.
BQ = TQ2 + BT2 = 32 + 42 i.e. CA ⊥ AT
∴ ∠ CAT = 90°
= 9 + 16 = 25 = 5 cm ⇒ ∠ CAB + ∠ BAT = 90° …(ii)
(iii) (c) PK = PA + AK = 10 + 6 = 16 cm From Eqs. (i) and (ii), we get
(iv) (c) QY = BQ − BY = 5 − 4 = 1 cm ∠ CAB + ∠ ACB = ∠ CAB + ∠ BAT
(v) (b) If two circles touch externally, then the number of ⇒ ∠ ACB = ∠ BAT Hence proved.
common tangents can be drawn is 2.
4. Let AB be a diameter of a given circle and LM and PQ be
Subjective Questions the tangent lines drawn to the circle at points A and B,
respectively.
1. Given, OP = 12 cm and OQ = 5 cm
To Prove LM|| PQ
Proof We know that, the tangent at any point of a circle is
perpendicular to the radius through the point of contact.
O ∴ OA ⊥ PQ and OB ⊥ LM
12 cm 5 cm B
L M

P Q
O
We know that, the tangent drawn from an external point to a
circle is always perpendicular to the radius
i.e. tangent to the circle make an angle of 90° with radius. P Q
A
∴ ∠OQP = 90°
⇒ AB ⊥ PQ and AB ⊥LM
So, using pythagoras theorem in ∆OQP,
⇒ ∠PAB = 90° and ∠ABM = 90°
OP 2 = OQ2 + PQ2
∠PAB = ∠ABM [each 90°]
12 2 = 52 + PQ2 But these are alternate angles.
PQ2 = 144 − 25 ∴ PQ|| LM
PQ2 = 119 Hence, the tangents drawn at the ends of a diameter of a
circle are parallel. Hence proved.
PQ = 119 cm
5. Given that, PQ is any line segment and S1 , S2 , S3 , S4 , ...
2. Given, ∠POT = 75°, ∠OTP = ? circles are touch a line segment PQ at a point A. Let the
centres of the circles S1 , S2 , S3 , S4 , ... be C1 , C 2 , C 3 , C 4 ,...
P respectively.

75º
O T
S4
C4 S3
C3 S2
C2 S1
We know that, tangent from an external point to the circle C1
make an angle of 90° with radius. Means radius and tangent
P Q
to the circle are always perpendicular. A
In ∆OPT, To prove Centres of these circles lie on the perpendicular
∠POT + ∠OTP + ∠OPT = 180° bisector of PQ.
[sum of interior angles in a triangle is 180°] Now, joining each centre of the circles to the point A on the
⇒ 75° + ∠OTP + 90° = 180° line segment PQ by a line segment, i.e.
C1 A , C 2 A , C 3 A , C 4 A ,... so on.
CBSE Term II Mathematics X (Basic) 67

We know that, if we draw a line from the centre of a circle to and ∠ OBA + ∠ABC = 90°
its tangent line, then the line is always perpendicular to the ⇒ ∠ BAC = 90° − 60° = 30°
tangent line. But it not bisect the line segment PQ. and ∠ ABC = 90° − 60° = 30°
So, C1 A ⊥ PQ [for S 1 ]
In ∆ ABC, ∠ BAC + ∠ CBA + ∠ ACB = 180°
C 2 A ⊥ PQ [for S2 ]
[since, sum of all interior angles of a triangle is 180°]
C 3 A ⊥ PQ [for S3 ]
⇒ ∠ ACB = 180° − ( 30° + 30° ) = 120°
C 4 A ⊥ PQ [for S4 ]
8. Given, XP and XQ are tangents of the circle with centre O
... so on.
and R is a point on the circle.
Since, each circle is passing through a point A. Therefore, all
the line segments C1 A , C 2 A , C 3 A , C 4 A,..., so on are coincident. We know that, lengths of tangents drawn from an external
point to a circle are equal.
So, centre of each circle lies on the perpendicular line of PQ
but they do not lie on the perpendicular bisector of PQ. ∴ XP = XQ [Q X is an external point]…(i)
Hence, a number of circles touch a given line segment PQ at AP = AR [Q A is an external point]…(ii)
a point A, then their centres lie. and BQ = BR [Q B is an external point]…(iii)
6. Let O be the centre of the concentric circle of radii 5 cm and Now, from Eq. (i),
3 cm respectively. Let AB be a chord of the larger circle XP = XQ
touching the smaller circle at P. ⇒ XA + AP = XB + BQ
⇒ XA + AR = XB + BR [Using Eqs. (ii) and (iii)]
Hence proved.
9. TP = TQ ⇒ ∠TPQ = ∠TQP = x (say)
O [Q angles opposite to equal side are equal]
P
5 cm 3 cm
A P B
T O

Then, AP = PB and OP ⊥ AB Q
Applying pythagoras theorem in ∆OPA, we get In ∆TPQ ,
OA 2 = OP 2 + AP 2 ∠PTQ = 180° − ( ∠TPQ + ∠TQP )
⇒ 25 = 9 + AP 2 [angle sum property of a triangle]
⇒ AP 2 = 16 = 180° − ( x + x )
= 180° − 2x
⇒ AP = 4 cm
∴ OP ⊥ TP
∴ AB = 2 AP = 8 cm
[Q radius is perpendicular to the tangent
7. Since, a chord AB subtends an angle of 60° at the centre of a at the point of contact]
circle.
∴ ∠OPT = 90°
⇒ ∠OPQ + ∠TPQ = 90°
⇒ ∠OPQ + x = 90°
O ⇒ ∠OPQ = 90° − x
60° ∴ ∠PTQ = 2 ∠OPQ Hence proved.
A B 10. Two tangents BD and BC are drawn from an external point B.
C

B 120°
O
C

i.e. ∠ AOB = 60° D


As, OA = OB = Radius of the circle
∴ ∠ OAB = ∠ OBA = 60° To prove BO = 2BC
The tangent at points A and B is drawn, which intersects Given, ∠ DBC = 120°
at C. Join OC , OD and BO.
We know, OA ⊥ AC and OB ⊥ BC. Since, BC and BD are tangents.
∴ ∠ OAC = 90° and ∠OBC = 90° ∴ OC ⊥ BC and OD ⊥ BD
⇒ ∠ OAB + ∠ BAC = 90° We know, OB is a angle bisector of ∠DBC.
68 CBSE Term II Mathematics X (Basic)

∴ ∠ OBC = ∠ DBO = 60° Now, in ∆ACP and ∆BCP,


In right angled ∆OBC, AP = BP
BC [Q length of tangents drawn from an external point
cos60° =
OB to a circle are equal]
1 BC PC = PC [common sides]
⇒ =
2 OB and ∠APO = ∠BPO [from Eq. (i)]
⇒ OB = 2 BC ∴ ∆ACP ≅ ∆BCP [by SAS congruence rule]
Also, BC = BD Then, AC = BC [by CPCT]
[tangents drawn from external point to circle are equal] and ∠ACP = ∠BCP [by CPCT]
∴ OB = BC + BC 1
= × 180° = 90°
⇒ OB = BC + BD 2
11. As we know that the tangents drawn from an external point [Q AB is a straight line]
to a circle are equal in length. Hence, OP is perpendicular bisector of AB. Hence proved.
P 14. P
R
Q
C
A O

Q
Therefore, PR = RC ...(i) AP = AQ
and RQ = RC ...(ii) ⇒ ∠APQ = ∠AQP = x [say]
From Eqs. (i) and (ii), we get [Q angles opposite to equal sides are equal]
PR = RQ In ∆APQ, ∠PAQ = 180° − ( ∠APQ + ∠AQP )
Hence, R is the mid-point of PQ and it bisects the common [angle sum property of a triangle]
tangent at P and Q.
= 180° − ( x + x ) = 180° − 2x
12. Given Common tangents AB and CD of two circles Q OP ⊥ AP
intersecting at E.
[Q radius is perpendicular to the tangent
To prove AB = CD
at the point of contact]
A
D ∴ ∠OPA = 90°
⇒ ∠OPQ + ∠APQ = 90°
⇒ ∠OPQ + x = 90°
E ⇒ ∠OPQ = 90° − x
∴ ∠PAQ = 2 ∠OPQ Hence proved.
B
C
15. Let ABCD is a quadrilateral circumscribing circle with
Proof EA = EC ...(i) centre O.
[the lengths of tangents drawn from an external Let circle touches the sides of a quadrilateral at points
point to a circle are equal] E , F , G and H.
EB = ED ...(ii) D
On adding Eqs. (i) and (ii), we get G
EA +EB = EC + ED C
⇒ AB = CD Hence proved. 7 6
13. Let OP intersect AB at a point C. 8 5
H F
Clearly, ∠APO = ∠BPO …(i) 1
3
4
[QO lies on bisector of ∠APB] 2
B
A E
A
P
C O
To prove ∠AOB + ∠COD = 180°
and ∠AOD + ∠BOC =180°
B
Construction Join OE , OF , OG and OH.
Proof We know that, two tangents drawn from an external
point to a circle subtend equal angles at the centre
CBSE Term II Mathematics X (Basic) 69

∠1 = ∠ 2 OD =
1
AE ⇒ AE = 2 × OD = 2 × 8 = 16 cm
∠3 = ∠4 2
∠ 5 = ∠6 [QOD = radius of smaller circle = 8 cm]
∠7 = ∠ 8 …(i) In right angled ∆ODB,
Also, we know that the sum of all angles subtended at a point OB2 = OD2 + BD2 [by Pythagoras theorem]
is 360°. ⇒ BD2 = 169 − 64 = 105
∴∠1 + ∠ 2 + ∠ 3 + ∠ 4 + ∠ 5 + ∠6 + ∠7 + ∠ 8 = 360° …(ii) ⇒ BD = 105 cm = DE [QBD = DE]
⇒ 2( ∠ 2 + ∠ 3 + ∠6 + ∠7 ) = 360° Now, in right angled ∆AED,
⇒ ( ∠ 2 + ∠ 3) + ( ∠6 + ∠7 ) = 180° AD2 = AE 2 + ED2 [by Pythagoras theorem]
⇒ ∠AOB + ∠COD = 180° ⇒ AD = (16)2 + ( 105 )2
Similarly, we have
= 256 + 105 = 361 = 19 cm
2( ∠1 + ∠ 8 + ∠ 4 + ∠ 5) = 360°
[from Eqs. (i) and (ii)] 18. Given, AB = 10 cm, AQ = 7 cm, CQ = 5 cm and BC = ?
A
⇒ ( ∠1 + ∠ 8) + ( ∠ 4 + ∠ 5) = 180°
⇒ ∠AOD + ∠BOC = 180°
Hence proved.
R Q
16. Let D, E and F are the points, where the incircle touches the
sides AB, BC and CA, respectively. Join OA, OB and OC.
A
B P C
A circle is given inscribed in ∆ABC, such that the circle
F touches the sides of the triangle.
r Tangents drawn to a circle from an external point are equal.
r O
D AR = AQ = 7 cm, CP = CQ = 5 cm
r
Now, BR = AB − AR = 10 − 7 = 3 cm
B
E
C
∴ BR = BP = 3 cm
In ∆ ABC, AC 2 = AB2 + BC 2 [by Pythagoras theorem] ∴ BC = (BP + CP ) = 3 + 5 = 8 cm
The length of BC is 8 cm.
= 42 + 32 = 16 + 9 = 25
19. Given A circle with centre O and a tangent AB at a point
∴ AC = 5 cm
P on the circle.
[taking positive square root, as length cannot be negative]
To prove OP ⊥ AB
1 1 4r 2
Now, ar (∆OAB ) = × OD × AB = × r × 4 = cm , Construction Take any point Q, other than P on the tangent
2 2 2
1 1 3r 2 AB and join OQ.
ar (∆OBC ) = × OE × BC = × r × 3 = cm
2 2 2
1 1 5r 2
and ar (∆OAC ) = × OF × AC = × r × 5 = cm
2 2 2
O
∴ ar (∆ABC ) = ar (∆OAB ) + ar (∆OBC ) + ar (∆OAC )
1 4r 3r 5r 1 12 r
⇒ AB × BC = + + ⇒ ×3×4 = R
2 2 2 2 2 2
⇒ r = 1 cm A P Q B
17. Produce BD to meet the bigger circle at E. Join AE. Proof Here, Q is a point on the tangent AB, other than the
Then, ∠AEB = 90° [Q angle in semi-circle]
point of contact P.
A
So, Q lies outside the circle [if Q lies inside the circle, then
AB becomes a secant and not a tangent to the circle].
O
Let OQ intersects the circle at R .
B D E ∴ OP = OR [radii of the same circle]
Now, OQ = OR + RQ ⇒ OQ > OR
Clearly, OD ⊥ BE [QBE is tangent to the smaller ⇒ OQ > OP or OP < OQ [QOP = OR]
circle at D and OD is its radius] Thus, OP is shorter than any other segment joining O to any
∴ BD = DE point of AB.
[QBE is a chord of the bigger circle and OD ⊥ BE] Also, we know that the shortest distance between a point
Now, in ∆AEB, O and D are the mid-points of AB and BE, and a line is perpendicular distance from the point to the
respectively. line. So, OP is perpendicular to AB.
Therefore, by mid-point theorem, we have i.e. OP ⊥ AB Hence proved.
70 CBSE Term II Mathematics X (Basic)

20. ∠OCA = 90° [angle between tangent and radius] 23. Let O be the centre of circle.
Now, ∠OCA = ∠OCD + ∠ACD Q
⇒ ∠ACD = ∠OCA − ∠OCD
⇒ ∠ACD = 90° − 44° = 46°
S O 30° P
As, AC = AD
[tangents drawn from an external point A
are equal in length] R
So, ∠ADC = ∠ACD = 46°
[Q angles opposite to the equal sides are equal] Join OQ and OR. Then,
Also, ∠CAD + ∠ADC + ∠ACD = 180° OQ ⊥ PQ and OR ⊥PR
[angle sum property of a ∆ACD] [Q tangent is perpendicular to the radius
at the point of contact]
⇒ ∠CAD = 180°− ( 46° + 46° ) = 88°
So, ∠ROQ + ∠RPQ = 180°
Again, ∠COD = 180°− ∠CAD = 92 °
[Q sum of all interior angles of
Further, ∠OBD = ∠ODB [OB = OD radii of circle]
quadrilateral is 360°]
In ∆OBD, use exterior angle theorem
⇒ ∠ROQ = 150°
exterior angle ∠COD = ∠OBD + ∠ODB 1
= ∠OBD + ∠OBD But ∠RSQ = ∠ROQ
2
⇒ 2∠OBD = ∠COD [exterior angle theorem] 1
1 = × 150° = 75°
⇒ ∠CBD = × 92 °= 46° 2
2 Now, on extending QO to intersect RS at A, we get
Hence, ∠CAD = 88°, ∠ADC = 46°, ∠CBD = 46° and ∠OQP = ∠QAS = 90° [alternate interior angle]
∠ACD = 46°
[Q PQ|| RS and ∠OQP = 90°]
21. We know that, tangents drawn from an external points to a
Therefore, from ∆QSA,
circle are equal in length.
∠SQA = 180° − 90° − 75° = 15°
R C Also, ∠PQR + ∠PRQ + ∠QPR = 180°
D ⇒ ∠PQR + ∠PQR + 30° = 180°
Q [Q ∠PQR = ∠PRQ because PQ = PR]
S
150°
⇒ ∠PQR = = 75°
B 2
A P ⇒ ∠AQR = ∠AQP − ∠PQR
Let AP = AS = x = 90° − 75° = 15°
Similarly, BP = BQ , CQ = CR and RD = DS So, ∠RQS = ∠SQA + ∠AQR
Since, AP = x ⇒ BP = AB − AP = 6 − x [Q AB = 6 cm] = 15° + 15° = 30°
Now, BP = BQ = 6 − x 24. Clearly, ∠QAB = 60° and ∠QBA = 60°
⇒ CQ = BC − BQ = 9 − (6 − x ) = 3 + x [Q ∆ABQ is an equilateral]
Now, CQ = CR = 3 + x So, ∠PAQ = ∠PAB + ∠QAB = 120°
⇒ RD = CD − CR = 8 − ( 3 + x ) [Q CD = 8cm] Similarly, ∠PBQ = 120°
⇒ RD = 5 − x [Q∠PAB = ∠PBA, as PA = PB] …(i)
Now, RD = DS = 5 − x Now, in ∆PAQ and ∆PBQ,
AD = AS + SD = x + 5 − x = 5 PA = PB
AD = 5 cm [tangents drawn from external point]
Hence, the length of AD is 5 cm. ⇒ AQ = BQ [∆ABQ is an equilateral]
22. ∠ DAB = 90° ⇒ ∠PAQ = ∠PBQ [each120°, shown above]
In ∆ABD, ∠ DAB + ∠ ABD + ∠ ADB = 180° So, ∆PAQ ≅ ∆PBQ [by SAS similarity rule]
⇒ ∠ ADB = 180° − 140° = 40° ⇒ ∠APQ = ∠BPQ [by CPCT] …(ii)
In ∆ODC, OD = OC [radii of same circle] Let QP intersect AB at M.
⇒ ∠OCD = ∠CDO = 40°
Now, in ∆PAM and ∆PBM,
[Q angles opposite to equal sides are equal]
∠APM = ∠BPM [from Eq. (ii)]
∴ ∠DOC + ∠OCD + ∠ CDO = 180°
[Q sum of all angles in a triangle is 180°] ⇒ PA = PB
⇒ ∠ DOC = 100° [tangents drawn from an external point]
Since, AD is a straight line. ⇒ PM = PM [common side]
∴ ∠ DOC + ∠ COA = 180° ⇒ ∠ COA = 80° So, ∆PAM ≅ ∆PBM [by SAS congruence rule]
CBSE Term II Mathematics X (Basic) 71

⇒ AM = BM AB = AC [given]
and ∠AMP = ∠BMP [by CPCT] ...(iii) BO = CO [radii of same circle]
But ∠AMP + ∠BMP = 180° AO = AO [common side]
∴ ∆ABO ≅ ∆ ACO [by SSS congruence rule]
⇒ ∠AMP + ∠AMP = 180°
⇒ ∠1 = ∠ 2 [CPCT]
⇒ ∠AMP = 90° Hence Proved.
Now, in ∆ABM and ∆ACM,
25. Here, two circles are of radii OP = 3 cm and PO′ = 4 cm. AB = AC [given]
These two circles intersect at P and Q. ∠1 = ∠ 2 [proved above]
P AM = AM [common side]
3 cm 4 cm ∴ ∆AMB ≅ ∆AMC [by SAS congruence rule]
O ⇒ ∠ AMB = ∠ AMC …(i) [CPCT]
N O′
Also, ∠ AMB + ∠ AMC = 180° [linear pair]
Q ⇒ ∠ AMB + ∠ AMB = 180° [from Eq. (i)]
⇒ ∠ AMB = 90°
Here, OP and PO′ are two tangents drawn at point P.
We know that a perpendicular from centre of circle bisects
∠ OPO′ = 90°
the chord. So, OA is perpendicular bisector of BC.
[tangent at any point of circle is perpendicular to radius
through the point of contact] Let AM = x, then OM = 9 − x [QOA = radius = 9 cm]
Join OO′ and PN. In right angled ∆AMC,
In right angled ∆OPO′, AC 2 = AM2 + MC 2 [by Pythagoras theorem]
(OO′ )2 = (OP )2 + (PO′ )2 [by Pythagoras theorem] i.e. (Hypotenuse)2 = (Base)2 + (Perpendicular)2
i.e. ( Hypotenuse) = (Base) + (Perpendicular)
2 2 2
⇒ MC 2 = 62 − x 2 …(i)
= ( 3)2 + ( 4)2 = 25 and in right angled ∆OMC,
⇒ OO′ = 5 cm OC 2 = OM2 + MC 2 [by Pythagoras theorem]
Also, PN ⊥ OO′
⇒ MC 2 = 92 − (9 − x )2 …(ii)
Let ON = x, then NO′ = 5 − x
From Eqs. (i) and (ii),
In right angled ∆OPN,
(OP )2 = (ON )2 + ( NP )2 [by Pythagoras theorem] 6 2 − x 2 = 9 2 − (9 − x ) 2

⇒ ( NP )2 = 32 − x 2 = 9 − x 2 …(i) ⇒ 36 − x 2 = 81 − ( 81 + x 2 − 18 x )
⇒ 36 = 18x ⇒ x = 2
and in right angled ∆PNO′,
∴ AM = x = 2
(PO′ )2 = (PN )2 + ( NO′ )2 [by Pythagoras theorem]
In right angled ∆ABM,
⇒ ( 4)2 = (PN )2 + ( 5 − x )2
AB2 = BM2 + AM2 [by Pythagoras theorem]
⇒ (PN )2 = 16 − ( 5 − x )2 …(ii)
62 = BM2 + 2 2
From Eqs. (i) and (ii), we get
9 − x 2 = 16 − ( 5 − x )2 ⇒ BM2 = 36 − 4 = 32 ⇒ BM = 4 2
⇒ 7 + x 2 − ( 25 + x 2 − 10x ) = 0 ⇒10x = 18 ∴ BC = 2 BM = 2 × 4 2 = 8 2 cm
∴ x = 1.8 1
∴ Area of ∆ABC = × Base × Height
Again, in right angled ∆OPN, 2
OP 2 = (ON )2 + ( NP )2 [by Pythagoras theorem] 1
= × BC × AM
⇒ 32 = (1.8)2 + ( NP )2 2
1
⇒ ( NP )2 = 9 − 3.24 = 5.76 = × 8 2 × 2 = 8 2 cm2
∴ ( NP ) = 2.4 2
∴ Length of common chord, PQ = 2 PN = 2 × 2.4 = 4.8 cm Hence, the required area of ∆ABC is 8 2 cm2 .
26. In a circle, ∆ABC is inscribed. 27. In the given figure, join AO, OC and O′ D, O′ B.
Join OB, OC and OA. Now, in ∆EO′ D and ∆EO′ B,
Conside ∆ABO and ∆ACO O′ D = O′ B [radius]
A
m O′ E = O′ E [common side]
6c 6 cm
1 2 ED = EB
B C
M [since, tangents drawn from an external point to the circle
9 9 are equal in length]
O
72 CBSE Term II Mathematics X (Basic)

= ∠AED + 180° − ∠AED = 180°


A
D ∴ ∠AEO + ∠AED + ∠DEO′ = 180°
So, OEO′ is straight line.
O O′ Hence, O, E and O′ are collinear. Hence proved.
E
B 28. Given, OT = 13 cm and OP = 5 cm
C
Since, if we draw a line from the centre to the tangent of the
circle, then it is always perpendicular to the tangent i.e.
∴ ∆EO′ D ≅ ∆EO′ B [by SSS similarity rule] OP ⊥ PT.
⇒ ∠O′ ED = ∠O′ EB [by CPCT] P
O′ E is the angle bisector of ∠DEB. …(i) A
Similarly, OE is the angle bisector of ∠AEC.
Now, in quadrilateral DEBO′, O E
T
∠O′ DE = ∠O′ BE = 90°
[since, CED is a tangent to the circle and O′ D is the radius, B
i.e. O′ D ⊥ CED] Q
⇒ ∠O′ DE + ∠O′ BE = 180° In right angled ∆OPT,
∴ ∠DEB + ∠DO′ B = 180° OT2 = OP 2 + PT2
[since, DEBO′ is cyclic quadrilateral] …(ii) [by Pythagoras theorem,
Since, AB is a straight line. (hypotenuse)2 = (base)2 + (perpendicular)2 ]
∴ ∠AED + ∠DEB = 180°
⇒ PT2 = (13)2 − ( 5)2
⇒ ∠AED + 180° − ∠DO′ B = 180° [from Eq. (ii)]
= 169 − 25 = 144
⇒ ∠AED = ∠DO′ B …(iii)
⇒ PT = 12 cm
Similarly, ∠AED = ∠AOC ...(iv) Since, the length of pair of tangents from an external point T
Again from Eq. (ii), is equal.
∠DEB = 180° − ∠DO′ B ∴ QT = 12 cm
Divided by 2 on both sides, we get Now, TA = PT − PA
1 1 ⇒ TA = 12 − PA ...(i)
∠DEB = 90° − ∠DO′ B and TB = QT − QB
2 2
⇒ TB = 12 − QB ...(ii)
1
⇒ ∠DEO′ = 90° − ∠DO′ B …(v) Again, using the property,length of pair of tangents from an
2 external point is equal.
[since, O′ E is the angle bisector of ∠DEB i.e. ∴ PA = AE and QB = EB ...(iii)
1
∠DEB = ∠DEO′] ∴ OT = 13 cm
2 ∴ ET = OT − OE [Q OE = 5 cm = radius]
Similarly, ∠AEC = 180° − ∠AOC ⇒ ET = 13 − 5 ⇒ ET = 8 cm
Divided by 2 on both sides, we get Since, AB is a tangent and OE is the radius.
1 1 ∴ OE ⊥ AB
∠AEC = 90° − ∠AOC
2 2 ⇒ ∠OEA = 90°
1 ∴ ∠AET = 180° − ∠OEA [linear pair]
⇒ ∠AEO = 90° − ∠AOC …(vi) ⇒ ∠AET = 90°
2
[since, OE is the angle bisector of ∠AEC Now, in right angled ∆AET,
1 ( AT)2 = ( AE )2 + (ET)2
i.e. ∠AEC = ∠AEO]
2 [by Pythagoras theorem]
Now, ⇒ (PT − PA )2 = ( AE )2 + ( 8)2
 1  ⇒ (12 − PA )2 = (PA )2 + ( 8)2
∠AED + ∠DEO′ + ∠AEO = ∠AED + 90° − ∠DO′ B [from Eq. (iii)]
 2 
⇒ 144 + (PA )2 − 24 ⋅ PA = (PA )2 + 64
 1 
+ 90° − ∠AOC 10
 2  ⇒ 24 ⋅ PA = 80 ⇒ PA = cm
3
1
= ∠AED + 180° − ( ∠DO′ B + ∠AOC ) ∴ AE =
10
cm [from Eq. (iii)]
2 3
1
= ∠AED + 180° − ( ∠AED + ∠AED) Similarly
10
BE = cm
2 3
[from Eqs. (iii) and (iv)] 10 10 20
1 Hence, AB = AE + EB = + = cm
= ∠AED + 180° − ( 2 × ∠AED) 3 3 3
2
CBSE Term II Mathematics X (Basic) 73

Hence,the required length AB is


20
cm . So, CD = DR + CR = 6 + 3 = 9 cm
3 (iv) (d) Here ∠OAP = 90°
29. (i) (d) We have, AP = AQ, BP = BD, CQ = CD … (i) A
[Q tangents drawn from an external point
are equal in length]
P O
Now, AB + BC + AC = 7 + 4 + 9 = 20 cm 50°
⇒ AB + BD + CD + AC = 20 cm
⇒ AP + AQ = 20 cm B
⇒ 2 AP = 20 cm ⇒ AP = 10 cm
In ∆AOP and ∆BOP,
(ii) (c) Let AF = AE = x cm ∠OAP = ∠OBP [90° each]
[Q tangents drawn from an external point to
OA = OB [radii of circle]
a circle are equal in length]
PA = PB
Given, BD = FB = 9 cm, CD = CE = 3 cm
[tangents drawn from an external point are equal]
In ∆ABC, AB2 = AC 2 + BC 2
∴ ∆AOP ~ ∆BOP [by SAS similarity]
⇒ ( AF + FB)2 = ( AE + EC )2 + (BD + CD)2 ∴ ∠APO = ∠OPB [by CPCT]
⇒ ( x + 9)2 = ( x + 3)2 + 12 2 = 50°
⇒ x 2 + 81 + 18x = x 2 + 9 + 6x + 144 ∴ ∠BPA = 50° + 50° = 100°
⇒ 18x + 81 = 6x + 9 + 144 (v) (c) For bigger circle, PA = PB … (i)
⇒ 12 x = 72 ⇒ x = 6 cm [Qtangents drawn from an external point are
equal in length] … (ii)
∴ AB = 6 + 9 = 15 cm
Similarly, for smaller circle, PB = PC
(iii) (b) As we know that, tangents drawn from an external
point are equal in length. Therefore, AP = AS = 4 cm From Eqs. (i) and (ii), we get
PA = PB = PC = 11 cm
∴ DS = DR = 10 − 4 = 6 cm
And BP = BQ = 2 cm. So, CR = CQ = 5 − 2 = 3 cm
Chapter Test
Multiple Choice Questions
1. Two concentric circles are of radii 10 cm and (iv) In PA and PB are two tangents, drawn to a circle
8 cm, then the length of the chord of the larger with centre O from P such that ∠PBA = 60°, then
circle, which touches the smaller circle is ∠OAB is
(a) 6 cm (b) 12 cm (a) 50° (b) 25° (c) 30° (d) 130°
(c) 18 cm (d) 9 cm
(v) In the adjoining figure, if PC is the tangent at A
2. From a point P, which is at a distance of 13 cm of the circle with ∠PAB = 62° and ∠AOB = 132°,
from the centre O of a circle of radius 5 cm, the then ∠ABC is
pair of tangents PQ and PR to the circle is B
drawn. Then, the area of the quadrilateral
PQOR is O
(a) 60 cm2 (b) 65 cm2
(c) 30 cm2 (d) 32.5 cm2 62°
P A C
3. If two tangents inclined at an angle 60° are
(a) 18° (b) 20°
drawn to a circle of radius 3 cm, then the
(c) 60° (d) Can’t be determined
length of each tangent is
Short Answer Type Questions
3
(a) 3 cm (b) 6 cm
2 5. Two tangents PQ and PR are drawn from an
(c) 3 cm (d) 3 3 cm external point to a circle with centre O. Prove
Case Based MCQs that QORP is a cyclic quadrilateral.
4. For revision of chapter circles, a teacher 6. In figure, AB and CD are common tangents
planned a game with some questions written to two circles of unequal radii. Prove that
on the paper, which are to be answered by the AB = CD.
students. For each correct answer, a student A
will get a prize. Some of the questions are given B
below.
Answer the questions to check your
knowledge. D
C
(i) In the given figure, x + y is
(a) 60° O Long Answer Type Questions
(b) 90° x
(c) 120°
7. A is a point at a distance 13 cm from the centre
y
(d) 145° O of a circle of radius 5 cm. AP and AQ are the
A C
tangents to the circle at P and Q. If a tangent
(ii) In the given figure, PQ and PR are two tangents BC is drawn at a point R lying on the minor arc
to the circle, then ∠ROQ is PQ to intersect AP at B and AQ at C, find the
S perimeter of the ∆ABC.
R
8. If a chord and a tangent intersect externally,
O then the product of the lengths of the
segments of the chord is equal to the square of
the length of the tangent from the point of
20°
P contact to the point of intersection.
Q
(a) 30° (b) 60° (c)
Answers
C
105° (d) 160° 1. (b) 2. (a) 3. (d)
(iii) In the adjoining figure, 45° 4. (i) (b) (ii) (d) (iii) (b) (iv) (c) (v) (b) 7. 24 cm
AB is a chord of the circle O B
and AOC is its diameter such
that ∠ACB = 45°, For Detailed Solutions
T
then ∠BAT is A Scan the code
(a) 35°
(b) 45°
(c) 125°
(d) 110°
CBSE Term II Mathematics X (Basic) 75

CHAPTER 04

Constructions

In this Chapter...
l Division of a Line Segment Internally in the Given Ratio
l Construction of a Tangent to a Circle at a Point that lies on it
l Construction of Tangent to a Circle from a Point Outside the Circle
l Construction of Tangents to a Circle When Angle

Constructions 1 Justification
Division of a Line Segment Internally in the Given Ratio Since, Am C||Ap B, so use the basic proportionality theorem in
∆ABAp .
To divide a line segment AB (say) internally in the given ratio
AAm AC
m : n, where m and n are both positive integers, we use the Then, = …(i)
following steps Am Ap CB
Step I Draw the given line segment AB and any ray AX, By using construction, the ratio is
making an acute angle with the line segment AB. AAm m
This ray AX can be drawn above or below AB. = …(ii)
A m A p ( p − m)
Step II Mark m + n = p points
∴From Eqs. (i) and (ii),
(i.e. A 1 , A 2 , ..., A m , ..., A p ) on the ray AX, such that AC m
AA 1 = A 1 A 2 = ... = A p − 1 A p =
CB ( p − m)
Step III Join BA p .
Alternate Method
Step IV Through the point Am , draw a line parallel to A p B To divide a line segment in the given ratio m : n, where
(by making an angle equal to ∠AA p B at A m ) which m and n are both positive integers, we can also use the
intersects the line segment AB at point C. Thus, following steps.
point C divides the line segment AB internally in the Step I Draw the given line segment AB (say) and any ray
ratio m : n, i.e. AC : CB = m : n. AX making an acute angle with the line
X segment AB.
n
)
m+ Step II Draw another ray BY|| AX by making
(A
Ap
∠ABY = ∠BAX.
Am Step III Mark m points i.e. A1 , A 2 , ..., Am on AX
A2 and n points i.e. B1 , B 2 , ..., Bn on BY such that
A1
AA1 = A1 A 2 = ... = Am − 1 Am
A B
C = BB1 = B1B 2 = ... = Bn − 1Bn
76 CBSE Term II Mathematics X (Basic)

Step IV Join Am Bn which intersects line segment AB at the So, in ∆ABA5 , by basic proportionality theorem, we get
AA 3 AC
point C. = ...(i)
A 3 A5 CB
Now, C is the required point which divides line
By construction, we have
segment AB internally in the ratio m : n. AA 3 3 3
X = = ...(ii)
Am
A 3 A5 ( 5 − 3 ) 2
Am – 1 On equating Eqs. (i) and (ii), we get
A2 AC 3
=
A1 BC 2
This shows that C divides AB internally in the ratio 3 : 2.
A B
C
B1
Construction 2
B2 Construction of a Tangent to a Circle at a Point
that lies on it
Bn – 1
Bn We can construct a tangent to a circle at a point that lies on it
Y
by two cases which are given below
Justification Case I By using the centre of circle
Step V Use the condition of similarity of two triangles in To construct a tangent to a circle by using the centre, we use
AAm AC the following steps.
∆AAm C and ∆BBnC. Then, = ...(i)
BBn BC Step I Take a point O as centre and draw a circle of
Step VI Write the ratio by using construction, given radius.
AAm m Step II Take a point P on the circle, at which we want to
= ..(ii)
BBn n draw tangent.
AC m Step III Join OP, which is the radius of circle.
Step VII Equating Eqs. (i) and (ii), we get =
BC n
Step IV Take OP as base and construct ∠OPT = 90 ° at P.
Example 1. Draw a line segment AB = 8 cm and divide it
internally in the ratio 3 : 2 and also justify it. Step V Draw a ray PT and produce TP to T′ to get the
Sol. Steps of Construction required tangent TPT′.
(i) First, draw line segment, AB = 8 cm and draw a ray AX,
which makes an acute angle with line segment AB.
(ii) Mark m + n = 3 + 2 = 5 points i.e. A1 , A 2 , A 3 , A 4 and A5
on the ray AX such that O
AA1 = A1 A 2 = A 2 A 3 = A 3 A 4 = A 4 A5
(iii) Join BA5 .
T′ T
(iv) Through the point A 3 (Q m = 3 ), draw a line A 3C || A5B P
(by making an angle equal to ∠AA5 B at A 3 ), which
intersects the line segment AB at C. Example 2. Draw a circle of diameter AB = 5 cm with
X
A5
centre O and then draw a tangent to the circle at
A4 point A or B.
A3 Sol. Given, diameter of circle = AB = 5 cm and centre is O.
A2
A1 5
∴ Radius = OA = OB = = 2 . 5 cm
A B 2
C
8 cm Steps of Construction
Thus, point C divides the line segment AB internally in the (i) Take a point O as centre and draw a circle of radius
ratio 3 : 2. 2.5 cm.
Justification (ii) Draw diameter AOB.
Since, A 3C || A5B. (iii) Take OA as base and construct ∠OAT = 90° at A.
CBSE Term II Mathematics X (Basic) 77

(iv) Produce TA to T′ to get the required tangent TAT′. Construction 3


Similarly, we can draw a tangent at point B or any other
point on the circle. Construction of Tangents to a Circle from a Point
Outside the Circle
T
If a point lies outside the circle, then there will be two
tangents to the circle from this point.
2.5 cm
B A Case I When centre of circle is known
O
If centre of circle is known, then to draw tangents from a
given external point, we use the following steps
T′ Step I Draw a circle with centre O of given radius and take
Case II Without using the centre of circle a point P outside it.
To construct a tangent to a circle without using the Step II Join OP and bisect it. Let its mid-point be M. Then,
centre of circle, we use the following steps. MP = MO.
Step I Draw a circle of given radius and take a point P Step III On taking M as centre and MO or MP as radius,
(at which we want to draw tangent) on the circle. draw a dotted circle, which intersects the given
Step II Draw any chord PQ through the given point P on the circle at points Q and Q ′ (say).
circle.
Q
Step III Take a point R in either the major arc or minor arc
and join PR and QR.
Step IV On taking PQ as base, construct ∠QPY equal to P
M
O

∠PRQ and on the opposite side of R.


Step V Draw a ray PY and produce YP upto X to get the Q′
required tangent YPX.
R Step IV Join PQ and PQ′. Thus, PQ and PQ′ are the required
Q tangents drawn to the circle from the external point
P. Here, we observe that PQ = PQ ′.
Justification
X
Join OQ. Then, ∠PQO = 90 °, since it is constructed in the
P Y semi-circle of dotted circle. It shows that OQ ⊥ PQ. Also, OQ
Example 3. Draw a circle of radius 6 cm. Take a point P is radius of given circle, so PQ has to be a tangent of given
on it. Without using the centre of the circle, draw a circle. Similarly, PQ′ is also a tangent to the given circle.
tangent to the circle at point P. Example 4. Draw a circle of radius 3.5 cm. From a point
Sol. Given, radius of circle = 6 cm P, 6 cm from its centre, draw two tangents of the
Steps of Construction circle.
(i) Draw a circle of radius 6 cm and take a point P on the Sol. Given, a circle of radius 3.5 cm whose centre is O (say) and
circle. a point P, 6 cm away from its centre.
(ii) Draw a chord PQ through the point P on the circle.
(iii) Take a point R in the major arc and join PR and RQ.
A
(iv) On taking PQ as base, construct ∠QPY equal to
∠PRQ on the opposite side of the point R.
(v) Produce YP to X. Then, YPX is the required tangent at
point P. 6 cm
R O P
M C
3.5 cm

Q
X
B
P
Y
78 CBSE Term II Mathematics X (Basic)

Steps of Construction Sol. Given, radius of circle = 2 cm and distance between point P
(i) Draw a circle with O as centre and radius and centre = 6.5 cm
OC = 3.5 cm. Take a point P such that OP = 6 cm. Steps of Construction
(ii) Draw the bisector of OP which intersect OP at M. (i) Draw a circle of radius 2 cm with centre O.
(iii) Take M as centre and MO as radius, draw a dotted (ii) Take a point P outside it, such that its distance from
circle. Let this circle cuts the given circle at A and B. centre O is 6.5 cm.
(iv) Join PA and PB. (iii) Consider O and P as centre and draw arcs of radius
Hence, PA and PB are the required tangents. more than half of OP on both sides of OP which
intersect each other at R and S. Join RS which bisects
Case II When centre of circle is unknown
OP at M. Then, MP = MO.
If centre of the circle is unknown, then to draw tangents of
(iv) Consider M as centre and MO as radius, draw a dotted
the circle, by using the following steps circle which intersects given circle at Q and Q′.
Step I Firstly, draw the circle and then draw two (v) Join PQ and PQ′.
non-parallel chords of the circle.
Hence, we get the required tangents drawn from point
Step II Draw the perpendicular bisectors of both chords P to the given circle.
which intersect each other at a point, say O. Then, R
this point O gives the centre of given circle. Now,
Q
we further use the steps given in case I to draw
tangents.
Alternate Method 1.1 cm M 2 cm
P O
If centre of circle is unknown, then we can draw tangents 6.5 cm
without finding centre of the circle. For this, we use the
following steps of construction.
Q′
Step I Draw a circle of given radius and take a point P
outside it. S

D
Construction 4
Construction of Tangents to a Circle When Angle
T1 between Them is Given
Sometimes, angle between two tangents (or pair of tangents)
K is given and we have to draw these tangents. Then, we use
C B
P A the following steps of construction.
T2
Step I First, draw the given circle with centre O and radius
r cm.
Step II Through P, draw a line (i.e. secant) intersecting the
A
given circle at points A and B, respectively and
O
produce it to C in opposite direction of AB such that r
AP = CP. α Q

Step III Now, bisect the segment CB at K. Then, take K as α


centre and KB (or KC) as radius, draw a semi-circle. P R
Step IV At point P, draw PD ⊥ CB which cuts the semi-circle
Step II Draw any diameter say AOQ of this circle.
at D.
Step III Make given angle α at centre O with OQ (say) as
Step V Take P as centre and PD as radius draw arcs to
intersect the given circle at points T1 and T2 . base which intersect the circle at point R (say) or
Step VI Join PT1 and PT2 which are the required tangents. draw the radius OR meets the circle at R such that
∠QOR = α.
Example 5. Draw a circle of radius 2 cm with centre O and
take a point P outside the circle such that Step IV Now, draw perpendiculars to OA at A and to OR at
OP = 6.5 cm. From P, draw two tangents to the R, which intersect the tangents each other at a point
circle. say P.
CBSE Term II Mathematics X (Basic) 79

Then, AP and RP are the required pair of tangents to given (iii) Draw the radius OR meets the circle at R such that
circle, inclined at an angle α, i.e. angle between pair of ∠QOR = 30°.
tangents is α.
Justification P
By construction, ∠OAP = 90 ° and OA is radius. 4 cm
O
So, PA is a tangent to the circle. 30° Q
Similarly, PR is a tangent to the circle. E 30°
Also, ∠AOR = 180 ° − ∠QOR [Q AOQ is a straight line] N R
D
= 180 ° − α
Now, in quadrilateral AORP, (iv) Draw PD ⊥ PQ and RE ⊥ OR, which intersect each
other at point N. Then, NP and NR are the required
∠APR + ∠PAO + ∠AOR + ∠PRO = 360 ° tangents to the given circle inclined to each other at an
⇒ ∠APR + 90 ° + 180 ° − α + 90 ° = 360 ° angle of 30°.
⇒ ∠APR = α Justification
Example 6. Draw a pair of tangents to a circle of radius By construction, ∠OPN = 90° and OP is radius.
4 cm which are inclined to each other at an angle of ∴ PN is a tangent to the circle.
30°. Similarly, NR is a tangent to the circle.
Sol. Given, a circle of radius 4 cm. We have to construct a pair of Now, ∠POR = 180° − 30° = 150°
tangents, which are inclined to each other at an angle of 30°. [QPOQ is a straight line and ∠QOR = 45°]
Steps of Construction In quadrilateral OPNR,
(i) Draw a circle with O as centre and radius 4 cm. ∠OPN = 90° , ∠POR = 150° and ∠ORN = 90°
(ii) Draw any diameter POQ of this circle. ∴ ∠PNR = 360° − (90° + 150° + 90° ) = 30°
80 CBSE Term II Mathematics X (Basic)

Chapter
Practice
6. To divide a line segment AB in the ratio 4 : 7, a ray
PART 1 AX is drawn first such that ∠ BAX is an acute angle
and then points A 1 , A 2 , A 3 , . . . are located at equal
Objective Questions distances on the ray AX and the point B is joined to
(a) A12 (b) A11
(c) A10 (d) A 9
l
Multiple Choice Questions
7. To divide a line segment AB in the ratio 5 : 6, draw a
1. To divide a line segment AB in ratio m : n
ray AX such that ∠ BAX is an acute angle, then draw
(m and n are positive integers), draw a ray AX to
a ray BY parallel to AX and the points
that ∠BAX is an acute angle and the mark point on
A 1 , A 2 , A 3 , . . . and B 1 , B 2 , B 3 , . . . are located to
ray AX at equal distances such that the minimum
equal distances on ray AX and BY, respectively.
number of these points is
Then, the points joined are
(a) greater of m and n (b) m + n
(a) A5 and B6 (b) A 6 and B5
(c) m + n − 1 (d) mn
(c) A 4 and B5 (d) A5 and B4
2. To divide a line segment AB in the ratio 5 : 7, first 8. To divide a line segment AB in the ratio 6 : 7, a ray
a ray AX is drawn, so that ∠ BAX is an acute angle AX is drawn first such that ∠BAX is an acute angle
and then at equal distances points are marked on and then points A 1 , A 2 , A 3 , … are located equal
the ray AX such that the minimum number of distances on the ray AX and the point B is joined
these points is with
(a) 8 (b) 10 (c) 11 (d) 12
(a) A12 (b) A13 (c)A10 (d)A11
3. To divide a line segment AB in the ratio 3 : 5 first a 9. In the given figure, find the ratio, when P divides
ray AX is drawn so that ∠BAX is an acute angle AB internally.
and then at equal distances points are marked on
A3 X
the ray AX such that the minimum number of A2
these points is A1
(a) 8 (b) 9 (c) 10 (d) 11 A B
P
4. To divide a line segment AB in the ratio 4 : 5, first B1
B2
a ray AX is drawn making ∠BAX an acute angle B3
Y B4
and then points A 1 , A 2 , A 3 , . . at equal distances
are marked on the ray AX and the point B is joined (a) 3 : 2 (b) 2 : 3 (c) 4 : 3 (d) 3 : 4
to
10. From the following ratios, a line segment cannot be
(a) A 4 (b) A 5
divided into A ratio.
(c) A 9 (d) A 7
1 1 1
5. The ratio of division of the line segment AB by the (a) A → 5: (b) A → :
5 5 5
point P from A in the following figure is [CBSE 2012] 2 5 1
(c) A → : (d) A → : 1
A5 5 2 5
A4
A3 11. To draw a pair of tangents to a circle, which are
A2 inclined to each other at an angle of 60°, it is
A1
required to draw tangents at end points of those two
A
P B
radii of the circle, the angle between them should be
(a) 135° (b) 90° (c) 60° (d) 120°
(a) 2 : 3 (b) 3 : 2
(c) 3 : 5 (d) 2 : 5
CBSE Term II Mathematics X (Basic) 81

12. A pair of tangents can be constructed from a 7. Draw a circle of radius 3 cm. Take a point P outside
point P to a circle of radius 3.5 cm, situated at a the circle at a distance of 7 cm from the centre O of
distance of 3 cm from the centre. the circle and draw two tangents to the circle.
(a) True [CBSE 2020 (Basic)]
(b) False 8. Draw a line segment AB of length 9 cm. Taking A as
(c) Can’t determined centre, draw a circle of radius 5 cm and taking B as
(d) None of the above
centre, draw another circle of radius 3 cm. Construct
13. A pair of tangents can be constructed to a circle tangents to each circle from the centre of the other
inclined at an angle of 170°. circle.
(a) True
9. Draw a circle with the help of circular solid ring.
(b) False
(c) Can’t determined
Construct a pair of tangents from a point P outside the
(d) None of the above circle. Also, justify the construction.
l
Long Answer Type Questions
PART 2 10. Draw a circle of radius 4 cm. Construct a pair of
tangents to it, the angle between which is 60°.
Subjective Questions Also, justify the construction. Measure the distance
between the centre of the circle and the point of
l
Short Answer Type Questions intersection of tangents.

1. Draw a line segment of length 7 cm and divide it 11. Construct a tangent to a circle of radius 1.8 cm from a
point on the concentric circle of radius 2.8 cm and
in the ratio 2 : 3. [CBSE 2020 (Basic)]
measure its length. Also, verify the measurement by
2. Draw a circle of diameter AB = 6 cm with centre actual calculation.
O and then draw a tangent to the circle at point A 12. Draw a circle of radius 2.8 cm. From an external point
or B.
P, draw tangents to the circle without using the centre
3. Draw a circle of radius 5 cm. Take a point P on it. of the circle.
Without using the centre of the circle, draw a
13. Draw a pair of tangents to a circle of radius 3 cm,
tangent to the circle at point P.
which are inclined to each other at an angle of 45°.
4. Draw a circle of radius 6 cm and draw a tangent
14. Let ABC be a right angled triangle, in which
to this circle, making an angle of 30° with a line
AB = 6 cm, BC = 8 cm and ∠ B = 90 °. BD is the
passing through the centre.
perpendicular from B on AC. The circle through
5. Draw a circle of radius 4 cm. Take a point P at a B , C and D is drawn. Construct the tangents from A to
distance of 8 cm from the centre and construct a this circle. Also, justify the construction.
pair of tangents from point P to the circle.
[CBSE 2020 (Basic), 2019] 15. Draw a circle of radius 3 cm. Take two points P and Q
on one of its extended diameter each at a distance of
6. Draw a circle of radius 1cm. From a point P, 7 cm from its centre. Draw tangents to the circle from
2.2 cm apart from the centre of the circle, draw these two points P and Q. [NCERT]
tangents to the circle.
82 CBSE Term II Mathematics X (Basic)

SOLUTIONS
Objective Questions 11. (d) The angle between them should be 120° because in that
1. (b) To divide a line segment in the ratio m : n, the maximum case the figure formed by the intersection point of pair of
tangent, the two end points of those two radii (at which
number of the points to mark are m + n.
tangents are drawn) and the centre of the circle is a
2. (d) We know that, to divide a line segment AB in the ratio quadrilateral.
m : n, first draw a ray AX, which makes an acute angle
∠BAX, then marked m + n points at equal distance. P
Here, m = 5, n = 7
So, minimum number of these points = m + n = 5 + 7 = 12.
3. (a) Minimum number of points = 3 + 5 = 8 θ R
O 60°
4. (c) Here, 4 + 5 = 9 points are located at equal distances on
the ray AX, so B is joined to last point A 9 .
Q
5. (b) The ratio of division of the line segment AB by the point
P from A is AP : BP = 3 : 2 . From figure, it is quadrilateral
6. (b) Here, minimum 4 + 7 = 11 points are located at equal ∠POQ + ∠ PRQ = 180°
distances on the ray AX and then B is joined to last point [Q sum of opposite angles are 180°]
is A 11 . 60° + θ = 180°
7. (a) Given, a line segment AB and we have to divide it in the ∴ θ = 120
ratio 5 : 6. Hence, the required angle between them is 120°.
Y
B
6
12. (b) False, since, the radius of the circle is 3.5 cm
B
5 B i.e. r = 3. 5 cm and a point P is situated at a distance
4
B
3
of 3 cm from the centre i.e. d = 3 cm
B
2 B We see that, r > d
1
A i.e. a point P lies inside the circle. So, no tangent can be
C B
A1 drawn to a circle from a point lying inside it.
A2 13. True
A3
A4
A5 X

170°
Steps of Construction
1. Draw a ray AX, making an acute ∠BAX.
2. Draw a ray BY parallel to AX by making ∠ABY equal to
∠BAX.
If the angle between the pair of tangents is always greater
3. Now, locate the points A 1 , A 2 , A 3 , A 4 and A 5 ( m = 5) than 0 or less than 180°, then we can construct a pair of
on AX and B1 , B2 , B3 , B4 , B5 and B6 ( n = 6 ) such that all tangents to a circle.
the points are at equal distance from each other. Hence, we can draw a pair of tangents to a circle inclined at
4. Join B6 A5 , which intersect AB at a point C. an angle of 170°.
Then, AC : BC = 5 : 6
Subjective Questions
8. (b) A 6 + 7 i.e. A13 is joined to the point B.
1. Steps of construction
9. (d) From given figure, it is clear that there are three points at (i) Draw AB = 7 cm
equal distances on AX and four points at equal distances on
(ii) Draw a rayAX and a ray BY making an acute ∠BAX
BY. Here, P divides AB on joining A 3B4 . So, P divides AB
and ∠ABY, respectively, such that ∠ABY = ∠BAX.
internally in the ratio 3 : 4.
Y
B3

10. (c) Since,


B2

1 1 1
= 5 :1 = 1 :1
B1

(a) 5 : (b) :
5 5 5 A
C
B
2 5 1 A1
(c) : =2 2 :5 (d) :1 = 1 : 5
5 2 5 A2

Since, (a), (b) and (d) are the ratio of both integers. So, it is
possible to divide a line segment into these points.
Hence, option (c) is correct. X
CBSE Term II Mathematics X (Basic) 83

(iii) Along AX, mark points A1 , A 2 , such that AA1 = A1 A 2 . (ii) Draw a radius OA and produce it to B.
(iv) Along BY, mark points B1 , B2 , B3 , such that (iii) Construct an ∠AOP equal to the complement of 30°
BB1 = B1B2 = B2B3 . i.e. equal to 60°.
(v) Draw A 2B3 which intersecting AB at C. (iv) Draw a perpendicular to OP at P, which intersects OB at
Thus, point C obtained is the required point which divides point Q.
AB in the ratio 2 : 3. Hence, PQ is the required tangent such that ∠OQP = 30°.
2. Given, diameter of circle = AB = 6 cm and centre is O. 5. Step of construction
6 (i) Taking point O as center draw a circle of radius 4 cm.
∴ Radius = OA = OB = = 3 cm
2 (ii) Now, name a point P which is 8 cm away from point O. Join
Steps of Construction OP.
(i) Take a point O as centre and draw a circle of radius 3 cm. (iii) Draw a perpendicular bisector of OP, name the
intersection point of bisector and OP as O′.
(ii) Draw diameter AOB.
(iv) Now, draw a circle considering O′ as center and O′ P as the
(iii) Take OA as base and construct ∠OAT = 90° at A. radius.
(iv) Draw a ray AT and produce TA to T′ to get the required
tangent TAT′. Q
Similarly, we can draw a tangent at point B or any other
point on the circle.
T
O′
O P

3 cm
B A
O
R

T′ (v) Name the intersection point of circles as Q and R.


(vi) Join PQ and PR. There are the required tangents.
3. Given, radius of circle = 5 cm
6. Given, radius of circle = 1 cm and distance between point P
Steps of Construction and centre = 2.2 cm.
(i) Draw a circle of radius 5 cm and take a point P on the circle. Steps of Construction
(ii) Draw a chord PQ through the point P on the circle. (i) Draw a circle of radius 1 cm with centre O.
(iii) Take a point R in the major arc and join PR and RQ. (ii) Take a point P outside it such that its distance from
(iv) On taking PQ as base, construct ∠QPY equal to ∠PRQ on centre O is 2.2 cm.
the opposite side of the point R.
(iii) Take O and P as centre and draw arcs of radius more
(v) Draw a ray PY and produce YP to X. Then, YPX is the
than half of OP on both sides of OP, which intersect
required tangent at point P.
each other at R and S. Join RS, which bisects OP at M.
R Then, MP = MO.
R
Q
Q
X

P 1.1 cm M 1 cm
P O
Y 2.2 cm
4. Steps of Construction
(i) Draw a circle with centre O and radius 6 cm.
P Q′
B S
30° Q
60°
A (iv) On taking M as centre and MO as radius, draw a dotted
6 cm circle, which intersects given circle at Q and Q′.
O
(v) Join PQ and PQ′. Thus, we get the required tangents
drawn from point P to the given circle.
84 CBSE Term II Mathematics X (Basic)

7. Step of construction (ii) Draw perpendicular bisectors of AB and CD, which


(i) Taking point O as center draw a circle of radius 3 cm. intersect each other at point O. Then, O is the centre of
(ii) Now, name a point P which is 7 cm away from point O. the circle.
Join OP. (iii) Now, take a point P outside the circle and join OP.
(iii) Draw a perpendicular bisector of OP, name the (iv) Draw bisector of OP. Let its mid-point be M.
intersection point of bisector and OP as O′.
(v) On taking M as centre and MP as radius, draw a dotted
(iv) Now, draw a circle considering O′ as center and O′ P as the circle which intersect the given circle at T and T′.
radius.
(vi) Join PT and PT′.
(v) Name the intersection point of circles as Q and R.
Then, PT and PT′ are the required pair of tangents drawn to
(vi) Join PQ and PR . These are the required tangents. the circle from P.
Justification
Q Join OT.
Then, ∠PTO = 90° [angle in semi-circle of dotted circle]
This shows that OT ⊥ PT.
O′
O P Also, OT is radius of given circle, so PT has to be a tangent of
given circle. Similarly, PT′ is also a tangent of given circle.
10. Steps of Construction
R (i) Take a point O on the plane of the paper and draw a circle
with centre O and radius OA = 4 cm.
8. Given, a line segment AB = 9 cm, two circles with centres A (ii) At O construct radii OA and OB such that ∠AOB equal to
and B of radii 5 cm and 3 cm, respectively. 120° i.e. supplement of the angle between the tangents.
We have to construct two tangents to each circle from the (iii) Draw perpendiculars to OA and OB at A and B,
centre of the other circle. respectively. Suppose these perpendiculars intersect at P.
Steps of Construction Then, PA and PB are required tangents.
(i) Draw a line segment AB = 9 cm.
A
(ii) Draw a circle with centre A and radius 5 cm and another
circle with centre B and radius 3 cm. 4 cm
(iii) Now, bisect AB. Let O be the mid-point of AB.
60° 120° O
(iv) Take O as centre and AO as radius and draw a dotted P
circle, which intersects the two given circles at N , Q, M
and P.
(v) Join AN , AQ, BM and BP. These are the required tangents B

to each circle from the centre of the other circle.


The distance between the centre of the circle and the point
M of intersection of tangents is 8 cm.
N
Justification
5 cm 3c
O m In quadrilateral OAPB, we have
A 9 cm B ∠OAP = ∠OBP = 90°
and ∠AOB = 120°
Q
P ∴ ∠OAP + ∠OBP + ∠AOB + ∠APB = 360°
⇒ 90° + 90° + 120° + ∠APB = 360°
9. Steps of Construction ∴ ∠APB = 360° − (90° + 90° + 120° )
(i) First, draw a circle with the help of given circular solid = 360° − 300° = 60°
ring and then draw two non-parallel chords AB and CD. 11. Given, two concentric circles of radii 2.8 cm and 1.8 cm with
T A
common centre say, O.

D A
P C1 C2
M O P O
B M

T′ B
C
CBSE Term II Mathematics X (Basic) 85

Steps of Construction (ii) Draw any diameter POQ of this circle.


(i) Draw two circles with common centre O and radii (iii) Draw the radius OR meets the circle at R such that
2.8 cm and 1.8 cm, respectively. ∠QOR = 45°.
(ii) Take a point P on the outer circle and join OP.
(iii) Draw bisector of OP. Let mid-point of OP be M. P
(iv) Taking M as centre and PM as radius, draw a dotted circle, 3 cm
which intersects the inner circle at two points say A and B. O
45° Q
(v) Join AP and BP. Then, AP and BP are required tangents.
On measuring the lengths, we get PA = PB = 2.14 cm 45°
Calculation E
N R
Join OA. Then, OA = 1.8 cm [radius of inner circle C1 ] D
OP = 2.8 cm [radius of outer circle C 2 ] (iv) Draw PD ⊥ PQ and RE ⊥ OR,
and ∠PAO = 90°
which intersects each other at point N.
[Q angle in semi-circle of constructed circle]
Then, NP and NR are the required tangents to the given
So, in ∆PAO, by Pythagoras theorem,
circle inclined to each other at an angle of 45°.
OP 2 = OA 2 + AP 2 Justification
⇒ (2.8)2 = (1.8)2 + AP 2 By construction, ∠OPN = 90° and OP is radius.
⇒ 7.84 = 3.24 + AP 2 ∴ PN is a tangent to the circle.
⇒ AP 2 = 7.84 − 3.24 = 4.6 Similarly, NR is a tangent to the circle.
⇒ AP = 2.14 cm Now, ∠POR = 180° − 45° = 135°
[taking positive square root, as length cannot be negative] [QPOQ is a straight line and ∠QOR = 45°]
⇒ PA = PB = 2.14 cm In quadrilateral OPNR,
Hence, the length of tangents is 2.14 cm. ∠OPN = 90° , ∠POR = 135°
12. Given, a circle of radius 2.8 cm and we have to draw and ∠ORN = 90°
tangents without using the centre. ∴ ∠PNR = 360° − (90° + 135° + 90° ) = 45°
Steps of Construction 14. Given, ABC is a right angled triangle, in which AB = 6 cm,
(i) First, draw a circle of radius 2.8 cm and take a point P BC = 8 cm, ∠B = 90° and BD is perpendicular to AC.
outside the circle. Then, ∠ADB = ∠CDB = 90°
(ii) Through P, draw a secant PAB, which intersects the Steps of Construction
circle at A and B and extend it to C in opposite (i) Draw the line segments AB = 6 cm and BC = 8 cm
direction of AB such that PC = PA. perpendicular to each other. Join AC. Thus, ∆ABC is
the given right angled triangle.
D
(ii) Draw perpendicular bisector of BC, which meets BC
M at O.
(iii) With O as centre and OB as radius draw a circle, which
intersects AC at D, then ∠BDC = 90°. Thus, BD is
O A perpendicular to AC.
C B
P (iv) With A as centre and AB as radius draw an arc, cutting
the circle at M.
N
(v) Join AM. Thus, AB and AM are required tangents.
(iii) Now, bisect BC and take its mid-point as O. Draw a
X
semi-circle with centre O and radius OB (or OC ).
(iv) Draw PD ⊥ BC, which intersects the semi-circle at D. A
(v) With centre P and radius PD draw two arcs, which
D M
intersects the given circle at points M and N. 6 cm
(vi) Join PM and PN. Thus, PM and PN are the required
tangents to the given circle.
B
13. Given, a circle of radius 3 cm. We have to construct a pair of O C
tangents, which are inclined to each other at an angle of 45°. 8 cm
Steps of Construction
(i) Draw a circle with O as centre and radius 3 cm.
86 CBSE Term II Mathematics X (Basic)

Justification Steps of Construction


Since ∆ABC is right angled triangle with ∠ABC = 90° (i) Draw a circle of radius 3 cm with centre at O.
∴ BO ⊥ AB. (ii) Produce its diameter on both sides and take points P and Q
Also, BO is the radius of circle. So, AB has to be tangent of on it such that OP = OQ = 7 cm
the circle. Similarly, AM is also a tangent to the circle. (iii) Draw bisector of OP and OQ, which intersect OP and OQ at
15. Given, two points P and Q on the extended diameter of a E and F, respectively.
circle with radius 3 cm such that OP = OQ = 7 cm (iv) Now, take E as centre and OE as radius, draw a dotted circle
We have to construct the tangents to the circle from the which intersects the given circle at two points M, N. Again,
given points P and Q. take F as centre and OF as radius, draw another dotted circle
which intersects the given circle at two points P′ and Q′.
(v) Join PM, PN , QP′ and QQ′. These are the required tangents
M P′
from P and Q to the given circle.
Justification
3 cm
O Join OM and ON. The ∠OMP is the angle that lies in the
P Q semi-circle of the dotted circle with centre E. Therefore,
E F
7 cm 7 cm ∠OMP = 90° ⇒ OM ⊥PM
Since, OM is radius of the circle. So, MP has to be a tangent to
N Q′ the circle. Similarly, PN, QP′ and QQ′ are also tangents to the
given circle.
Chapter Test
Multiple Choice Questions these two radii of the circle, the angle between
1. To divide a line segment AB in the ratio 2 : 5, two radii is
(a) 105°
first a ray AX is drawn, so that ∠BAX is an acute
(b) 70°
angle and then at equal distances, then the
(c) 125°
number of points located on the ray AX is (d) 135°
[CBSE 2011]
Short Answer Type Questions
(a) 7 (b) 10
(c) 2 (d) 5 5. Draw a line segment AB = 6. 5 cm and divide it
2. To divide a line segment AB in the ratio 7 : 5, internally in the ratio 3 : 5.
first a ray AX is drawn, so that ∠BAX is acute 6. Draw two tangents at the end points of the
angle and then at equal distance points are diameter of a circle of radius 3.5 cm. Are these
marked. Then, the minimum number of these tangents parallel?
points is
Long Answer Type Questions
(a) 5 (b) 35
(c) 7 (d) 12 7. Draw two concentric circles of radii 3 cm and
3. By geometrical construction, it is possible to 5 cm. Taking a point on outer circle construct
the pair of tangents to the other. Measure the
1
divide a line segment in the ratio 3 : . length of a tangent and verify it by actual
3 calculation.
(a) True (b) False
8. Draw a line segment AB of length 7 cm. Taking
(c) Can’t determined (d) None of these A as centre, draw a circle of radius 3 cm and
4. To draw a pair of tangents to a circle, which are taking B as centre, draw another circle of radius
inclined to each other at an angle of 55°, it is 2 cm. Construct tangents to each circle from
required to draw tangents at the end points of the centre of the other circle.

Answers
1. (a) 2. (d) 3. (a) 4. (c) 6. These are parallel. 7. 4 cm For Detailed Solutions
Scan the code
88 CBSE Term II Mathematics X (Basic)

CHAPTER 05

Applications of
Trigonometry

In this Chapter...
l Line of Sight & Horizontal Line
l Angle of Elevation
l Angle of Depression

Line of Sight Some Important Points


The line of sight is the line drawn from the eye of an observer to (i) The angle of elevation of a point P as seen from a
the point where the object is viewed by the observer. point O is always equal to the angle of depression of
O as seen from P.
Horizontal Line
P
Angle of
The line which goes parallel from eye to ground, is called depression
horizontal line. ht
f sig
o
Angle of Elevation e
Lin Angle of
The angle of elevation of an object viewed, is the angle formed by elevation
A
O
the line of sight with the horizontal, when it is above the horizontal Horizontal line
level, i.e. the case when we raise our head to look at the object.
(ii) The angles of elevation and depression are always
P(object)
acute angles.
ht
sig (iii) If the observer moves towards the perpendicular
of
e line (tower/building), then angle of elevation
Lin
Angle of increases and if the observer moves away from the
O elevation
Horizontal line
A perpendicular line (tower/building), then angle of
Eye elevation decreases.
(iv) If the height of tower is doubled and the distance
Angle of Depression between the observer and foot of the tower is also
The angle of depression of an object Horizontal line doubled, then the angle of elevation remains same.
O A
viewed, is the angle formed by the line Angle of (v) If the angle of elevation of Sun, above a tower
depression
of sight with the horizontal, when it is Lin decreases, then the length of shadow of a tower
e
below the horizontal level, i.e. the case of
s ig increases and vice-versa.
when we lower our head to look at the ht
object. P(object)
CBSE Term II Mathematics X (Basic) 89

Solved Examples
Example 1. In figure, a tightly stretched rope of length In right angled ∆ACB,
20 m is tied from the top of a vertical pole to the AB
tan 30° =
ground. Find the height of the pole, if the angle BC
made by the rope with the ground is 30°. 1 h
⇒ =
B 3 30
30 3
⇒ h= ×
3 3
20 m
[multiply numerator and denominator by 3]
30 × 3
30º =
A C 3
= 10 3 m
Sol. Let AB = h be the height of the pole.
Hence, height of the tower is10 3 m.
Given, length of rope, BC = 20 m
B Example 3. The ratio of the length of a vertical rod and
the length of its shadow is 1 : 3. Find the angle of
hm 20 m elevation of the Sun at that moment?
Sol. Let AB be the vertical rod and BC be its shadow and θ be
the angle of elevation of the Sun.
30º
A C A

In right angled ∆ACB,


Vertical rod

Perpendicular
sin 30° =
Hypotenuse
1 h
∴ =
2 20
20 q
⇒ h= = 10 m B C
2 Shadow
Hence, height of the pole is 10 m.
We have, AB : BC =1 : 3
Example 2. In figure, the angle of elevation of the top Let AB = x, then BC = x 3
of a tower from a point C on the ground, which is AB
30 m away from the foot of the tower, is 30°. Find In ∆ABC, tanθ =
BC
the height of the tower. x 1
A
⇒ tanθ = =
x 3 3
⇒ tan θ = tan 30°
⇒ θ = 30°

Example 4. A vertical tower stands on a horizontal


30° plane and is surmounted by a vertical flag-staff of
C height 6 m. At a point on the plane, the angle of
B 30 m
elevation of the bottom and top of the flag-staff are
Sol. Let height of a tower be AB = h m 30° and 45°, respectively. Find the height of the
A tower. (take, 3 = 1.73)
Sol. Let BC = h be the height of the tower, CD = 6 m be the
h height of the flag-staff and A is any point on the ground.
Consider, AB = x m.
30° Given, the angle of elevation from point A to the points C
B 30 m C and D are ∠CAB = 30° and ∠DAB = 45°.
90 CBSE Term II Mathematics X (Basic)

D Also, let AB = x m.
In right angled ∆ABD,
6m
Perpendicular BD
C tan60° = =
Base AB
BC + CD
⇒ 3= [Q tan 60° = 3]
h x
h + 16.
⇒ 3=
30° 45° x
A xm B 3x = h + 1 . 6
In right angled ∆ABC, ⇒ h = 3x − 16
. …(i)
BC
tan 30° = In right angled ∆CBA,
AB BC
1 h tan 45° =
⇒ = AB
3 x h
⇒ x= 3hm …(i) ⇒ 1= [Q tan 45° = 1]
x
Now, in right angled ∆ABD, ⇒ x=h
BD
tan 45° = On putting x = h in Eq. (i), we get
AB
h = 3 h − 16 .
6+ h
⇒ 1= [QBD = BC + CD = 6 + h] ⇒ h ( 3 − 1) = 16
.
x
⇒ x =6+ h …(ii) 16
. 3 +1
⇒ h= × [rationalising]
⇒ 3h = 6 + h [from Eq. (i)] ( 3 − 1) 3 +1
⇒ h( 3 − 1) = 6 1 .6( 3 + 1)
= [Q (a + b )(a − b ) = a 2 − b 2 ]
6 3 +1 ( 3 )2 − (1)2
⇒ h= × (rationalisation)
( 3 −1) 3+1 1 .6
= ( 3 + 1)
6( 3 + 1) 2
= [Q(a − b )(a + b ) = a 2 − b 2 ]
( 3 )2 − (1)2 = 0.8 (173
. + 1)
. + 1) 6 × 2.73
6(173 = 0.8 (2.73)= 2.184 m
= =
3 −1 2 Hence, the height of the pedestal is 2.184 m.
= 3 × 2.73 = 819
. m Example 6. From a point on the ground, the angles of
Hence, height of tower is 8.19 m. elevation of the bottom and the top of a transmission
Example 5. A statue 1.6 m tall, stands on the top of a tower fixed at the top of a 20 m high building are
pedestal. From a point on the ground, the angle of 45° and 60°, respectively. Find the height of the
elevation of the top of the statue is 60° and from the tower. (use 3 = 1.73 )
same point the angle of elevation of the top of the Sol. Let AB = 20 m be the height of the building and BC = h m be
pedestal is 45°. Find the height of the pedestal. the height of transmission tower. The angles of elevation
(use 3 = 1.73) from a ground point D to the points B and C are
Sol. Let BC = h m be the height of the pedestal and CD = 1.6 m ∠ADB = 45° and ∠ADC = 60°
be the length of the statue, which is standing on the C
pedestal.
hm
D
B

C 20 m

45° 60°
h A
D

A In right angled ∆ADB,


xm B AB
tan 45° =
Again, let point A be a fixed point on the ground such that AD
the angles of elevation of the top of the statue and bottom of 20
⇒ 1=
the statue (i.e. top of the pedestal) are AD
∠DAB = 60° and ∠CAB = 45°. ⇒ AD = 20 m …(i)
CBSE Term II Mathematics X (Basic) 91

Now, in right angled ∆ADC, ⇒ h = 25 3 m


AC
tan60° = Hence, height of the pole is 25 3 m and distances of the
AD point from the poles are 25 m and 75 m.
AB + BC
⇒ 3= [Q from Eq. (i)] Example 8. The angle of elevation of the top of a
20
building from the foot of a tower is 30° and the
⇒ 20 3 = 20 + h
angle of elevation of the top of a tower from the foot
⇒ h = 20( 3 −1) = 20(173
. −1) of the building is 60°. If the tower is 50 m high,
= 20 × 073
. = 1460
. m then find the height of the building.
Hence, height of the transmission tower is 14.6 m. Sol. Let AB = 50 m, CD = h be the height of the tower and
building. Then,
Example 7. Two poles of equal heights are standing B
opposite to each other on either side of the road,
which is 100 m wide. From a point between them D
on the road, the angles of elevation of the top of the 50 m
poles are 60° and 30°, respectively. Find the height hm
of the poles and the distance of the point from the
poles. 30º 60º
A C
Sol. Let AB = 100 m be the width of the road. On both sides of
∠CAD = 30° and ∠ACB = 60°
the road, poles AE = BD = h m are standing. Let C be any
point on AB such that from point C, angles of elevation are In right angled ∆ACD ,
∠BCD = 60° and ∠ACE = 30° CD 1 h
tan 30° = ⇒ =
E D AC 3 AC
⇒ AC = 3 h …(i)
h hm In right angled ∆CAB,
AB
tan60° =
30º 60º AC
A (100 – x) m C xm B 50
100 m ⇒ 3=
AC
Let BC = x m, then AC = AB − BC = (100 − x ) m 50
⇒ 3= [from Eq. (i)]
In right angled ∆CAE, 3h
Perpendicular 50
tan 30° = ⇒ h= = 16.67 m
Base 3
1 AE 1 h
∴ = ⇒ = Hence, the height of the building is 16.67 m.
3 AC 3 (100 − x )
(100 − x ) Example 9. From the top of a 7 m high building the
⇒ h= angle of elevation of the top of a tower is 60° and
3
the angle of depression of its foot is 45°. Determine
⇒ h 3 = 100 − x …(i)
the height of the tower.
and in right angled ∆BCD,
Sol. Let AB = 7 m be the height of the building and EC be the
BD
tan60° = height of tower.
BC
h A is the point from where elevation of tower is 60° and the
⇒ 3= angle of depression of its foot is 45°.
x
Here, EC = DE + CD
⇒ h = 3x …(ii)
Also, CD = AB = 7 m
Put h = 3x in Eq. (i), we get and BC = AD
3x × 3 = 100 − x E
⇒ 3x + x = 100 ⇒ 4x = 100
100
⇒ x= = 25 m
4 60°
A D
∴ BC = 25 m and AC = 100 − x = 100 − 25 = 75 m 45°
Put x = 25 in Eq. (i), we get
7m

⇒ h 3 = 100 − 25
75 3 75 3
⇒ h= × = B 45° C
3 3 3
92 CBSE Term II Mathematics X (Basic)

Now, in right angled ∆ABC, C′ B + BM


⇒ 3=
AB PM
tan 45° = 10 + h + 10
BC ⇒ 3= [from Eq. (i)]
3h
7
⇒ 1= ⇒ 3h = 20 + h
BC
⇒ 2 h = 20
⇒ BC = 7 m ⇒ h = 10 m
Also, in right angled ∆ADE, Now, the height of the cloud from the surface of lake
DE =BC
tan60° = = BM + h
AD
= 10 + 10 = 20 m
DE
⇒ 3= [Q AD = BC]
7 Example 11. From a point on a bridge across a river,
⇒ DE = 7 3 m the angles of depression of the banks on opposite
sides of the river are 30° and 45°, respectively. If
∴ Height of the tower,
the bridge is at a height of 30 m from sea level, then
EC = DE + CD
find the width of the river. (use 3 = 1 . 73)
= (7 3 + 7 ) m
Sol. Let A be a point on the bridge and points B and D are on the
= 7( 3 + 1) m opposite side of the banks. Then, angles of depression from
Hence, height of the tower is 7( 3 + 1) m. point A to the opposite banks are
∠EAB = 30° and ∠FAD = 45°
Example 10. If the angle of elevation of a cloud from a ⇒ ∠CBA = 30° and ∠CDA = 45°
point 10 m above a lake is 30° and the angle of [Q alternate angles are equal]
depression of its reflection in the lake is 60°, find
A F
the height of the cloud from the surface of lake. E
30º 45º Bridge
Sol. Let AB be the surface of the lake and P be the point of
observation such that AP = 10 m. Let C be the position of the
cloud and C′ be the reflection in the lake, then CB = C′ B. 30 m
C
30º 45º
h B C D
(River bank) (Opposite
30° river bank)
P M
60° In right angled ∆ACB,
10 m 10 m
AC
tan 30° =
A B BC
1 30
⇒ =
3 BC
(10 +h)m
⇒ BC = 30 3 m
and in right angled ∆ACD,
C′ AC
tan 45° =
CD
Let CM = h, then CB = 10 + h
AC
⇒ C′ B = 10 + h ⇒ 1=
30
In right angled ∆CMP,
CM ⇒ AC = 30 m
tan 30° = Hence, width of the river is
PM
1 h BD = BC + CD
⇒ =
3 PM = 30 3 + 30
⇒ PM = 3h ...(i) = 30 × 1.73 + 30
In right angled ∆PMC′, = 51.9 + 30
C′ M = 81.9
tan 60° =
PM Hence, width of the river is 81.9 m.
CBSE Term II Mathematics X (Basic) 93

Example 12. From the top of a 7 m building, the angle Given, speed of plane is 720 km/h and time of flight is 10 s.
of elevation of the top of a cable tower is 60° and Also, given ∠AOC = 60° and ∠BOD = 30°
the angle of depression of its foot is 45°. Determine In right angled ∆OCA,
the height of the tower. (use 3 = 1.73). cot60° =
OC
Sol. Let AB = 7 m be the height of the building and DE = h m be AC
the height of cable tower. 1 OC
⇒ =
E 3 h
h
⇒ OC = m
3
In right angled ∆ODB,
60º OD
B C cot 30° =
45º hm BD
OD
⇒ 3=
7m h
⇒ OD = h 3
Now, CD = OD − OC
45º
A D h 2h
xm =h 3− = m
3 3
Then, ∠CBE = 60° 2h
and ∠CBD = 45° ⇒ ∠ADB = 45° (alternate angle) Thus, distance covered by aeroplane in 10 s is m.
3
Let distance between two towers be
Distance
AD = BC = x m Q Speed of aeroplane =
CE Time
In right angled ∆BCE, tan60° = 2h
BC
5  5 
⇒ 3=
CE
⇒x =
CE
… (i) ∴ 720 × = 3 Q1 km = 18 m / s
x 18 10
3
and in right angled ∆ADB, ⇒ 40 × 5 × 10 3 = 2 h
AB 7 ⇒ h = 1000 3 m
tan 45° = ⇒1 = ⇒ x = 7 m
AD x Hence, height at which the aeroplane is flying is1000 3 m.
Put x = 7 in Eq. (i), we get
CE Example 14. A straight highway leads to the foot of a
7= ⇒ CE = 7 3 m
3 tower. A man standing at the top of the tower
Now, height of cable tower, observes a car at an angle of depression of 30°,
h = DC + CE which is approaching the foot of the tower with a
= 7 + 7 3 = 7(1 + 3 ) uniform speed. After covering a distance of 50 m,
= 7(1 + 173
. ) = 7 × 2 .73 = 1911
. m the angle of depression of the car becomes 60°.
Find the height of the tower. (use 3 = 1.73).
Example 13. The angle of elevation of an aeroplane
from point O on the ground is 60°. After a flight of Sol. Let AB = h m be the height of the tower. Let C be the initial
10 s, on the same height, the angle of elevation position of the car and D be the final position of the car, when
from point O becomes 30°. If the aeroplane is flying it covers a distance, CD = 50 m.
at the speed of 720 km/h, find the constant height at E B
which the aeroplane is flying. 30°
60°
Sol. Let OX be the horizontal ground, A and B be the two
positions of the plane and O be the points of observation. h
Let height of an aeroplane from A to the ground is
AC = BD = h m
30° 60°
A B
D C A
50 m x
h
h Given, the angle of depressions from point B to the points C
and D are
30º
∠EBC = 60° and ∠EBD = 30°
60º ⇒ ∠BCA = 60° and ∠BDA = 30° [alternate angles]
X
O C D
94 CBSE Term II Mathematics X (Basic)

In right angled ∆CAB, A F


45º
AB 60º
tan60° =
AC
h
⇒ 3=

60 Ö3 m
x 45º
h B x E
⇒ x= m …(i)
3
And in right angled ∆DAB, hm
AB
tan 30° =
AD 60º
1 h C x D
⇒ =
3 50 + x Given, the angle of depressions from point A are
⇒ 50 + x = h 3 ∠ FAE = 45° and ∠FAD = 60°
⇒ x = h 3 − 50 …(ii) ⇒ ∠BEA = 45° and ∠CDA = 60° [alternate angles]
From Eqs. (i) and (ii), we have In right angled ∆AEB,
AB AB
h
= h 3 − 50 tan 45° = ⇒1=
3 BE x
⇒ x = AB …(i)
⇒ h = 3h − 50 3
and in right angled ∆ADC,
⇒ 2 h = 50 3
AC 60 3 60 3
⇒ h = 25 3 tan60° = ⇒ 3= ⇒ x=
CD x 3
⇒ h = 25 × 1 .73 ⇒ x = 60 m …(ii)
⇒ h = 43. 25 m ∴ From Eqs. (i) and (ii), we get
Hence, height of the tower is 43.25 m. AB = x = 60 m
Example 15. The angles of depression of the top and Now height of cliff,
bottom of a tower as seen from the top of a 60 3 m h = AC − AB
high cliff are 45° and 60°, respectively. Find the = 60 3 − 60
height of the tower. (use 3 = 1.73) = 60( 3 − 1)
Sol. Let AC = 60 3 m be the height of the cliff, DE = h m be the = 60( 1.73 − 1)
height of the tower and distance between tower and cliff be = 60 × 0.73 = 43.8 m
CD = BE = x m. Hence, height of the cliff is 43.8 m.
CBSE Term II Mathematics X (Basic) 95

Chapter
Practice
6. A circus artist is climbing a 20 m long rope, which
PART 1 is tightly stretched and tied from the top of a
vertical pole to the ground, then the height of pole,
Objective Questions if the angle made by the rope with the ground level
is 30°, is
l
Multiple Choice Questions (a) 5 m (b) 10 m (c) 15 m (d) 20 m

1. The angle of elevation of the Sun when the shadow 7. A ladder, leaning against a wall, makes an angle of
60° with the horizontal. If the foot of the ladder is
of a pole h m high is 3 h m long is
9.5 m away from the wall. The length of the ladder is
(a) 0° (b) 30° [NCERT Exemplar]
(c) 45° (d) 60° (a) 10 m (b) 16 m (c) 18 m (d) 19 m
2. If a pole 6 m high casts a shadow 2 3 m long on the 8. A ramp for disabled people in a hospital have slope
ground, then the Sun’s elevation is 30°. If the height of the ramp be 1 m, then the
(a) 60° (b) 45° length of ramp is
(c) 30° (d) 90° (a) 2 m (b) 0.5 m (c) 2 3 m (d) 1 m
3. If 300 3 m high tower makes an angle of elevation
9. A kite is flying at a height of 80 m above the
at a point on ground which is 300 m away from its
ground. The string attached to the kite is
foot, then the angle of elevation is
temporarily tied to a point on the ground. The
(a) 0° (b) 30°
inclination of the string with ground is 60°, then the
(c) 45° (d) 60°
length of the string is
4. From the top of a 60 m high tower, the angle of (a) 62.37 m (b) 92.37 m
depression of a point on the ground is 30°. The (c) 52.57 m (d) 72.57 m
distance of the point from the foot of tower is
10. The length of a string between a kite and a point on
(a) 180 m (b) 60 3 m
the ground is 85 m. If the string makes an angle θ
(c) 150 m (d) 30 3 m 15
with the ground level such that tanθ = , then the
5. In the given figure, the angle of elevation of the top 8
of a tower AC from a point B on the ground is 60°. height of kite is
[CBSE 2020 (Basic)] (a) 75 m (b) 78.05 m
C (c) 226 m (d) None of these
11. A tower stands near an airport. The angle of
20 m elevation θ of the tower from a point on the ground
is such that its tangent is 5/12. The height of the
tower, if the distance of the observer from the tower
60º is 120 m is [CBSE 2015]
A B (a) 40 m (b) 50 m
(c) 60 m (d) 70 m
If the height of the tower is 20 m, find the distance
of the point from the foot of the tower. 12. The top of two poles of height 20 m and
20 14 m are connected by a wire. If the wire makes an
(a) 20 3 m (b) m angle of 30° with the horizontal, then the length of
3
20 the wire is
(c) m (d) None of these
3 (a) 12 m (b) 10 m (c) 8 m (d) 6 m
96 CBSE Term II Mathematics X (Basic)

13. An observer, 1.5 m tall is 20.5 m away from a tower (ii) Value of DF is equal to
22 m high, then the angle of elevation of the top of h h
(a) m (b) h 3 m (c) m (d) h m
the tower from the eye of the observer is 3 2
(a) 30° (b) 45° (c) 60° (d) 90° (iii) Value of h is
14. The angle of elevation of the top of the tower from a (a) 2 (b) 3( 3 + 1)
point, which is 40 m away from the base of the (c) 4 (d) 3( 3 − 1)
tower in the horizontal level, is 45°. Find the height (iv) Height of the Parachute from the ground is
of the tower. (a) 4 m (b) 3( 4 − 3 )
(a) 70 m (b) 60 m (c) 40 m (d) 30 m
(c) 8 m (d) 3( 4 + 3 )
15. The angle of elevation of the top of a building
150 m high, from a point on the ground is 45°. The (v) If the Parachute is moving towards the building,
distance of the point from foot of the building is then both angles of elevation will
(a) 120 m (b) 130 m (c) 140 m (d) 150 m (a) remain same (b) increases
(c) decreases (d) Can’t be determined
16. The angle of depression of the car parked on the
road from the top of a 150 m high tower is 30°. The 20. A cyclist is climbing through a 20 m long rope
distance of the car from the tower is [CBSE 2014]
which is highly stretched and tied from the top of a
150 vertical pole to the ground as shown below
(a) 150 m (b) 75 m (c)150 3 m (d) m
3
17. From a point on the ground, the angles of elevation
of the bottom and the top of a transmission tower
fixed at the top of a 20 m high building are 45° and
60° respectively, then the height of the tower is
(a) 14.64 m (b) 28.64 m (c) 38.64 m (d) 19.64 m
18. A bridge on a river makes an angle of 45° with its Based on the above information, answer the
edge. If the length along the bridge from one edge following questions.
to the other is 150 m, then the width of the river is (i) The height of the pole, if angle made by rope with
(a) 107.75 m (b) 105 m (c) 75 m (d) 106.05 m the ground level is 60°, is
(a) 15 m (b)10 3 m
l
Case Based MCQs 10 15
(c) m (d) m
19. There are two balcony in a house. First balcony is 3 2
at a height of 3 m above the ground and other (ii) If the angle made by the rope with the ground
balcony is 6 m vertically above the lower balcony. level is 60°, then the distance between artist and
Ankit and Radha are sitting inside the two balcony pole at ground level is
at points G and F, respectively. At any instant, the 10
(a) m (b)10 2 m
angles of elevation of a Parachute from these 2
balcony are observed to be 60° and 45° as shown (c) 10 m (d)10 3 m
below (iii) If the angle made by the rope with the ground
E level is 45°. The height of the pole is
(a) 2.5 m (b) 10 m
hm
F 45° D (c) 7.5 m (d)10 2 m
6m (iv) If the angle made by the rope with the ground
G 60° C level is 45° and 3 m rope is broken, then the height
3m of the pole is
A B 17
(a) m (b) 7 m
Based on the above information, answer the 2
following questions. (c) 14 m (d) 7 2 m
(i) Who is more closer to the Parachute. (v) Which mathematical concept is used here?
(a) Ankit (a) Similar triangles
(b) Radha (b) Pythagoras theorem
(c) Both are at equal distance (c) Application of trigonometry
(d) Can’t be determined (d) None of the above
CBSE Term II Mathematics X (Basic) 97

21. A group of students of class X visited India Gate on respectively. If the distance between the peaks of
an educational trip. The teacher and students had two mountains is 1937 km, and the satellite is
interest in history as well. The teacher narrated that vertically above the mid-point of the distance
India Gate, official name Delhi Memorial, between the two mountains. (use 3 = 1.73)
originally called All-India War Memorial,
monumental sandstone arch in New Delhi, F
dedicated to the troops of British India who died in A
G
wars fought between 1914 and 1919.The teacher
also said that India Gate, which is located at the
H P
eastern end of the Rajpath (formerly called the Q R
Kingsway), is about 138 feet (42 m) in height. B D C I S
Nanda devi Mullayanagiri
(i) The distance of the satellite from the top of Nanda
Devi is
(a) 1139.4 km (b) 1119.65 km
(c) 1937 km (d) 1025.36 km
(ii) The distance of the satellite from the top of
Mullayanagiri is
(a) 1139.4 km (b) 577.52 km
(c) 1937 km (d) 1025.36 km
(i) What is the angle of elevation if they are standing
(iii) The distance of the satellite from the ground is
at a distance of 42 m away from the monument?
(a) 1139.4 km (b) 567.64 km
(a) 30° (b) 45°
(c) 1937 km (d) 1025.36 km
(c) 60° (d) 0°
(iv) What is the angle of elevation, if a man is standing
(ii) They want to see the tower at an angle of 60°. So,
at a distance of 7816 m from Nanda Devi?
they want to know the distance where they should
(a) 30° (b) 45°
stand and hence find the distance.
(c) 60° (d) 0°
(a) 25.24 m (b) 20.12 m
(c) 42 m (d) 24.24 m (v) If a mile stone very far away, makes 45° to the top
of Mullayanagiri mountain. Hence, find the
(iii) If the altitude of the Sun is at 60°, then the height
distance of this mile stone from the mountain.
of the vertical tower that will cast a shadow of
(a) 1118.327 m (b) 566.976 m
length 20 m is
20 (c) 1930 m (d) 1025.36 m
(a) 20 3 m (b) m
3
(c)
15
3
m (d)15 3 m PART 2
(iv) The ratio of the length of a rod and its shadow is Subjective Questions
1 : 1. The angle of elevation of the Sun is
(a) 30° (b) 45° l
Short Answer Type Questions
(c) 60° (d) 90°
(v) The angle formed by the line of sight with the 1. A ladder 25 m long just reaches the top of a
horizontal when the object viewed is below the building 24 m high from the ground. What is the
horizontal level is distance of the foot of ladder from the base of the
(a) corresponding angle building? [CBSE 2020 (Basic)]
(b) angle of elevation 2. A straight tree is broken due to thunderstorm. The
(c) angle of depression broken part is bent in such a way that the peak of
(d) complete angle the tree touches the ground at an angle of 60° at a
22. A Satellite flying at height h is watching the top of distance of 2 3 m. Find the whole height of the tree.
the two tallest mountains in Uttarakhand and 3. Determine the height of a mountain, if the
Karnataka ,they being Nanda Devi (height 7,816m) elevation of its top at an unknown distance from the
and Mullayanagiri (height 1,930 m). The angles of base is 30° and at a distance 10 km farther off from
depression from the satellite to the top of Nanda the mountain, along the same line, the angle of
Devi and Mullayanagiri are 30° and 60°, elevation is 15°. (take tan15° = 0. 27)
98 CBSE Term II Mathematics X (Basic)

4. There is a flag staff on a tower of height 20 m. At a depression of the base of the hill is 30°. Calculate
point on the ground, the angles of elevation of the the distance of the hill from the ship and height of
foot and top of the flag are 45° and 60°, respectively. the hill. [CBSE 2016]
Find the height of the flag staff. 15. The angle of elevation of an aeroplane from a point
5. If the length of the shadow of a tower is increasing, on the ground is 45°. After flying for 15 s, the angle
then the angle of elevation of the Sun is also of elevation changes to 30°. If the aeroplane is
increasing. Why or why not? flying at a constant height of 2500 m, then find the
6. The angle of elevation of the top of a building from average speed of the aeroplane. [CBSE 2013]
the foot of a tower is 30° and the angle of elevation 16. The angle of elevation of the top of a building from
of top of the tower from foot of the building is 60°. the foot of a tower is 30°. The angle of elevation of the
If the tower is 60 m high, then find the height of the top of the tower from the foot of the building is 60°. If
building. [CBSE 2020 (Basic)] the tower is 60 m high, find the height of the building
[CBSE 2020 (Basic)]
7. A player sitting on the top of a tower of height 20 m
observes the angle of depression of a ball lying on
17. An aeroplane, when flying at a height of 4000 m
from the ground, passes vertically above another
the ground as 60°. Find the distance between the
aeroplane at an instant when the angles of elevation
foot of the tower and the ball.
of two planes from the same point on the ground
8. Two poles of equal heights are standing opposite to are 60° and 45°, respectively. Find the vertical
each other, on either side of the road which is 90 m distance between the aeroplanes at that instant.
wide. From a point between them on the road, the
angles of elevation of the top of the poles are 60º 18. There is a small island in the middle of a 100 m
and 30º respectively. Find the height of the poles. wide river and a tall tree stands on the island. P and
[CBSE 2020 (Basic)] Q are points directly opposite to each other on two
banks and in line with the tree. If the angles of
9. If a man standing on a platform 3 m above the
elevation of the top of the tree from P and Q are
surface of a lake observes a cloud and its reflection
respectively 30° and 45°, then find the height of the
in the lake, then the angle of elevation of the cloud
is equal to the angle of depression of its reflection. tree. [take, 3 = 1.732]
State true or false. Justify.
10. From the top of a hill, the angles of depression of
l
Long Answer Type Questions
two consecutive kilometre stones due East are 19. An aeroplane is at an altitude of 1200 m. If two
found to be 30° and 45°. Find the height of the hill. ships are sailing towards it in the same direction.
[CBSE 2015] The angles of depression of the ships as observed
11. The angles of elevation of the top of a tower from from the aeroplane are 60° and 30°, respectively.
two points at a distance of 4 m and 9 m from the Find the distance between both ships.
base of the tower and in the same straight line with 20. The angles of depression of two consecutive kilometre
it, are θ and ( 90 º − θ ) respectively. Prove that the stones on the road on right and left of an aeroplane
height of the tower is 6 m. [CBSE 2020 (Basic)] are 60° and 45°, respectively as observed from the
12. From the top of a 75 m high lighthouse from the sea aeroplane. Find the height of the aeroplane.
level, the angles of depression of two ships are 30° 21. The angle of elevation of the top of a tower at a
and 45°. If the ships are on the opposite sides of the distance of 120 m from a point A on the ground
lighthouse, then find the distance between the two is 45°. If the angle of elevation of the top of a flag
ships. [CBSE 2020 (Basic)] staff fixed at the top of the tower, at A is 60°, then
13. From a point on the ground, the angles of elevation find the height of the flag staff. [use, 3 =1.73]
of the bottom and the top of a transmission tower, [CBSE 2014]
fixed at the top of a 20 m high building, are 45° and 22. A balloon is connected to an electric pole. It is
60° respectively. Find the height of the tower. inclined at 60° to the horizontal by a cable of length
(use 3 = 1.73) [CBSE 2020 (Basic)] 215 m. Determine the height of the balloon from
14. A man standing on the deck of a ship, which is 10 m the ground. Also, find the height of the balloon, if
above the water level. He observes that the angle of the angle of inclination is changed from 60° to 30°.
[CBSE 2015]
elevation of the top of a hill is 60° and the angle of
CBSE Term II Mathematics X (Basic) 99

23. A man in a boat rowing away from a light house 100 m 31. From the top of a 7 m high building, the angle of
high takes 2 min to change the angle of elevation of the elevation of the top of a cable tower is 60°, and the
light house from 60° to 45°. Find the speed of boat. angle of depression of its foot is 45°. Find the height
24. The angle of elevation of the top of a tower from of the tower. Given that 3 = 1.732
certain point is 30°. If the observer moves 20 m [CBSE 2020 (Basic)]
towards the tower, the angle of elevation of the top 32. The lower window of a house is at a height of 2 m
increases by 15°. Find the height of the tower. above the ground and its upper window is 4 m
25. A TV tower stands vertically on the bank of a canal. vertically above the lower window. At any instant
From a point on the other bank directly opposite the the angles of elevation of a balloon from these
tower, the angle of elevation of the top of the tower is windows are observed to be 60° and 30°,
60°. From another point on the same bank, which is respectively. Find the height of the balloon above
20 m away from this point, on the line joining this the ground.
point to the foot of the tower, the angle of elevation l
Case Based Questions
of the top of the tower is 30°. Find the width of the
canal. [CBSE 2020 (Basic)] 33. A girl 8 m tall spots a parrot sitting on the top of a
26. A vertical tower stands on a horizontal plane and is building of height 58 m from the ground. The
surmounted by a vertical flag staff of height h. At a angle of elevation of the parrot from the eyes of
point on the plane, the angles of elevation of the girl at any instant is 60°. The parrot flies away
bottom and the top of the flag staff are α and β horizontally in such a way that it remained at a
respectively. Prove that the height of the tower is constant height from the ground. After 8 s, the
angle of elevation of the parrot from the same
 h tan α  point is 30°.
 .
 tan β − tan α 
27. The angle of elevation of the top of a tower 30 m
high from the foot of another tower in the same
plane is 60° and the angle of elevation of the top of
the second tower from the foot of the first tower is
58 m

30°. Find the distance between the two towers and


also the height of the tower. 60°
28. A person standing on the bank of a river observes 30° 8m
that the angle of elevation of the top of a tree
standing on opposite bank is 60°. When he moves
Based on the above information, answer the
30 m away from the bank, he find the angle of
following questions. (Take 3 =1.73)
elevation to be 30°. Find the height of the tree and
width of the river. [take 3 =1.732] [CBSE 2020 (Basic)] (i) Find the distance of first position of the parrot
from the eyes of the girl.
29. A statue 3.6 m tall, stands on top of a pedestal. From
(ii) If the distance between the position of parrot
a point on the ground, the angle of elevation of the
increases, then the angle of elevation decreases.
top of the statue is 60° and from the same point, the Justify with girl.
angle of elevation of the top of the pedestal is 45°.
Find the height of the pedestal. [CBSE 2020 (Basic)]
(iii) Find the distance between the girl and the
building.
30. The angle of elevation of the top of a vertical tower (iv) How much distance covers parrot covers?
from a point on the ground is 60°. From another
point 10 m vertically above the first, its angle of (v) Find the speed of the parrot in 8s.
elevation is 45°. Find the height of the tower.
100 CBSE Term II Mathematics X (Basic)

SOLUTIONS
Objective Questions 4. (b) Let the distance of the foot of tower from the point be
1. (b) Let the angle of elevation of the Sun is θ. QR = x m.
A P
30°

60 m
h
30°
θ Q x R
B C
√3h
and height PR = 60 m and ∠PQR = 30°
Given, height of pole = h PR 60
AC h In ∆PRQ, tan 30° = =
Now, in ∆ABC, tanθ = = QR x
BC 3h 1 60
1 ⇒ = ⇒ x = 60 3 m
⇒ tan θ = = tan 30° 3 x
3 Base
5. (a) In ∆ABC, cot60° =
⇒ θ = 30° Height
Hence, the angle of elevation of the Sun is 30°. 1 AB
2. (a) Let BC = 6 m be the height of the pole and AB = 2 3 m ⇒ =
3 AC
be the length of the shadow on the ground. 1 AB
⇒ =
Let the Sun makes an angle θ on the ground. 3 20
C Sun 20
⇒ AB = m
3
Hence, the distance of the point from the foot of the tower is
6m 20
m.
θ 3
A B
2√3 m 6. (b) Let AB be the vertical pole and CA be the 20 m long rope
such that its one end A is tied from the top of the vertical pole
BC
Now, in ∆BAC, tan θ = AB and the other end C is tied to a point C on the ground.
AB
A
6 3 3
⇒ tan θ = = ⋅ [by rationalising]
2 3 3 3
3 3 m
⇒ tan θ = = 3 = tan 60° [Q tan 60° = 3] 20
3
∴ θ = 60°
Hence, the Sun’s elevation is 60°. 30°
3. (d) Let AB be the tower whose height is 300 3 m, C B
i.e. AB = 300 3 m. Again, let C be the point at a distance of In ∆ABC, we have
300 m from the foot of the tower, i.e. AC = 300 m. AB
Here, the angle of elevation is unknown, so let it be θ. sin 30° =
AC
Since, here base and perpendicular are given. 1 AB
B ⇒ =
2 20
⇒ AB = 10 m
300√3 m Hence, the height of the pole is 10 m.
7. (d) Let the length of ladder AB = h m.
θ B
A C
300 m
So, in right angled ∆BAC,
Perpendicular AB 300 3
tan θ = = = h
Base AC 300
⇒ tan θ = 3 = tan 60°
∴ θ = 60° 60°
A 9.5 m C
Hence, the required angle of elevation is 60°.
CBSE Term II Mathematics X (Basic) 101

Here, AC = 9. 5 m, ∠BAC = 60° ⇒ sinθ =


15
AC 1 9.5 17
In ∆ABC, cos 60° = ⇒ = B
AB 2 h
⇒ h = 2 × 9.5 = 19 m
Hence, length of ladder is 19 m.

m
85
8. (a) Let XZ be the length and YZ be the height of the ramp.
Z

θ
A C
1m
BC
In ∆ABC, sinθ =
30° AB
X Y 15 BC
⇒ =
Then, ∠ZXY = 30° and YZ = 1 m 17 85
In right angled ∆XYZ, ⇒ BC = 75 m
Perpendicular YZ Hence, height of kite is 75 m
sin 30° = =
Hypotenuse XZ 11. (b) Let BC = h m be the height of the tower and A be the
1 1 point on the ground such that, ∠BAC = θ.
⇒ = ⇒ XZ = 2 m
2 XZ C
Hence, the length of ramp is 2 m.
9. (b) Let C be the position of the kite and AC be the length of
the string which makes an angle of 60° with the ground. The hm
height of the kite from the ground is BC = 80 m.
C θ
A 120 m B
Then, AB = 120 m
80 m In right angled ∆ABC,
Perpendicular BC h
tanθ = = =
60° Base AB 120
A B 5 h  5 
⇒ = Q tan θ = 12 , given
In right angled ∆ABC, 12 120

sin60° =
Perpendicular BC
= ⇒ h = 50 m
Hypotenuse AC Hence, height of the tower is 50 m.
3 80 12. (a) Here, CD = 20 m
⇒ = [height of big pole]
2 AC
AB = 14 m [height of small pole]
80 × 2 3
⇒ AC = × [by rationalising] D
3 3
160 3 160 × 1732. 6m
= = = 92 . 37 m
3 3
Hence, the length of the string is 92.37 m. B 30°
x E 20 m
10. (a) Given, length of the string of the kite,
AB = 85 m
14 m 14 m
15
and tanθ =
8
8
⇒ cotθ = A C
15
∴ DE = CD − CE
Q cosec 2θ − 1 = cot 2 θ
⇒ DE = CD − AB [Q AB = CE]
64 ⇒ DE = 20 − 14 = 6 m
∴ cosec 2θ − 1 =
225 DE
64 289 In ∆BDE, sin 30° =
⇒ cosec 2θ = 1 + = BD
225 225 1 6
⇒ = ⇒ BD = 12 m
289 17 2 BD
⇒ cosec θ = =
225 15 ∴ Length of wire = 12 m
102 CBSE Term II Mathematics X (Basic)

13. (b) Let BE = 22 m be the height of the tower and ⇒


150
1=
⇒ BC = 150 m
AD = 1. 5 m be the height of the observer. The point D be the BC
observer’s eye. Draw DC || AB. Hence, the distance of the point from foot of the building is
E 150 m.
16. (c) Let AB=150 m be the height of the tower and angle of
depression is ∠DAC = 30°.

20.5 m
22 m
A
D 30º

D θ 150 m
C
20.5 m
1.5 m
30º
A B
20.5 m C B

Then, AB = 20. 5 m = DC Then, ∠ACB = ∠DAC = 30° [alternate angles]


and EC = BE − BC = BE − AD Now, in right angled ∆ABC,
= 22 − 1. 5 = 20. 5 m [Q BC = AD] Perpendicular AB
tan 30° = =
Let θ be the angle of elevation made by observer’s eye with Base BC
the top of the tower i.e. ∠EDC = θ. 1 150
⇒ =
In right angled ∆DCE, 3 BC
Perpendicular CE 20. 5 ⇒ BC = 150 3 m
tan θ = = =
Base DC 20. 5 Hence, the distance of the car from the tower is150 3 m.
⇒ tanθ = 1 17. (a) Let the height of the building be, BC = 20 m
⇒ tan θ = tan 45° and height of the tower be CD = x m
⇒ θ = 45° D
14. (c) In the figure, let AB be a tower which has height h m.
The angle of elevation from point C at a distance of 40 m xm
from point B is ∠ACB = 45° and BC = 40 m.
A C

20 m
hm
45°
60°
45° A ym B
C B
40 m Let the point A be at a distance y m
Then, in right angled ∆ABC, from the foot of the building.
Perpendicular AB Now, in ∆ABC,
tan 45° = = BC
Base BC = tan 45° = 1
h h AB
⇒ tan 45° = ⇒1= [Q tan 45° = 1] 20
40 40 ⇒ =1
∴ h = 40 m y
Hence, the height of tower is 40 m. ⇒ y = 20 m
15. (d) Let AB = 150 m be the height of building and C be a i.e. AB = 20 m
BD
point on the ground such that ∠ACB = 45°. Now, in ∆ABD, = tan60° = 3
AB
A
BD
⇒ = 3
AB
150 m 20 + x
⇒ = 3
20
45° ⇒ 20 + x = 20 3
C B ⇒ x = 20 3 − 20
In right angled ∆ABC, = 20( 3 − 1)
Perpendicular AB
tan 45° = = = 20(1732
. − 1)
Base BC ⇒ x = 20 × 0732
. = 1464
. m
CBSE Term II Mathematics X (Basic) 103

18. (d) Let BC be the width of the river and A, B be the ends of 20. (i) (b) Let in ∆ABC, AC will be rope and AB be a vertical
river such that AB = 150 m = Length of the bridge [given] pole.
and ∠BAC = 45°. A

B edge of river

m
20
River River
45°
edge of river
A C 60°
C B
In right angled ∆ACB,
Perpendicular Then, AC = 20 m, ∠C = 60°, ∠B = 90°
sin 45° =
Hypotenuse AB
In ∆ABC, sin60° =
1 BC BC 1 AC
⇒ = = [Qsin 45° = ]
2 AB 150 2 3 AB
=
150 2 2 20
∴ BC = × [by rationalising] AB = 10 3 m
2 2
150 BC 1 BC
= 2 = 75 2 (ii) (c) cos60° = ⇒ =
2 AC 2 20
= 75 × 1.414 [Q 2 = 1.414] BC = 10 m
= 106.05 m (approx.) AB
(iii) (d) sin 45° = [∠C = 45°]
Hence, width of the river is 106.05 m. AC
1 AB 20
19. (i) (b) Radha is more closer to the Parachute because the = ⇒ = AB
angles of elevation of Parachute from these balcony are 2 20 2
observed to be 45°. So, Radha is more closer to the 20 2
Parachute than Ankit. ⇒ AB = × [by rationalising]
2 2
DE
(ii) (d) In ∆DEF, ∠D = 90°, ∠DFE = 45°, tan 45° = = 10 2 m
DF
DE (iv) (a) Length of rope = 20 − 3 = 17 m
⇒ 1= AB
DF sin 45° = [∠C = 45°]
⇒ DE = DF = h m AC
1 AB 17
(iii) (b) In ∆EGC, ∠EGC = 60°, ∠C = 90° = ⇔ AB = m
CE 2 17 2
tan60° = A
CG
CD + DE h+6
3= ⇒ 3= [Q CG = DF]
CG DF
m

h+6
17

⇒ 3=
h 45°
6 6 C B
⇒ 3 =1 + ⇒ ( 3 − 1) =
h h (v) (c) in this, mathematical concept trigonometric ratio is
6 3 +1 used here, which is application of trigonometry.
∴ h= × [by rationalising]
( 3 − 1) 3 +1 21. (i) (b) Let AB be the monument of height 42 m and C is the
6( 3 + 1) point where they are standing, such that BC = 42 m.
= [Q (a + b )(a − b ) = a 2 − b 2 ] A
( 3 )2 − (1)2
6 ( 3 + 1) 6 ( 3 + 1)
= = = 3 ( 3 + 1) m
3 −1 2
42 m
(iv) (d) Height of the Parachute from the ground is BE, then
BE = BC + CD + DE
θ
BE = 3 + 6 + 3( 3 + 1) C B
42 m
= 9 + 3 ( 3 + 1)
Now, in ∆ABC,
=9+ 3 3 + 3 AB
= 12 + 3 3 tanθ =
BC
= 3( 4 + 3) m 42
⇒ tanθ = =1
(v) (c) If the Parachute is moving towards the building, then 42
both angles of elevation will decreases. ⇒ tanθ = 1 ⇒ θ = 45°
104 CBSE Term II Mathematics X (Basic)

(ii) (d) In ∆ABC, 22. As it is given that satellite is the mid-point of the two
A mountain hills i.e. I is the mid-point of DS.
F
30° 60°

42 m
A 30°
G
60°
C B
60°
7816 m H P
AB
tan60° = 968500 m 968500 m 1930 m
BC
42
3= D I S
BC
42 42 3 1937000 m
BC = = × [by rationalising]
3 3 3 (i) (b) We have, AG = DI = 968500 m
42 3 Now, in ∆FAG,
= = 14 3 AG
3 cos 30° =
= 14 × 1732
. = 24.24 m AF
(iii) (a) Let AB = h be the height of the tower. 3 968500
⇒ =
2 AF
A
968500 × 2 1937000
⇒ AF = =
3 1.73
= 111965318
. m
hm
= 111965
. km
(ii) (c) We have, HP = IS = 968500 m
60°
C B Now, in ∆FHP,
20 m
HP
AB h cos60° =
Now, in ∆ABC, tan60° = ⇒ 3= ⇒ h = 20 3 m FP
BC 20 1 968500
=
(iv) (b) Let h and x be the height and length of shadow of the 2 FP
vertical tower. FP = 968500 × 2 = 1937000 m = 1937 km
A
FG 1 FG
(iii) (b) In ∆FAG, tan 30° = ⇒ =
AG 3 968500
968500
hm ⇒ FG = = 559826. 59 m
3
= 559.82 km
θ
C B ∴Height of satellite from ground = FI = FG + GI
xm = 559.82 + 7.816
Now, in ∆ABC, [Q GI = AD = 7816 m = 7.816 km]
AB h = 567.64 km
tanθ = ⇒ tanθ =
BC x (iv) (b) Let E be the position of man.
⇒ tanθ = 1 [Q h : x = 1 : 1] A
⇒ θ = 45°
(v) (c) The angle of depression of the object viewed, is the
angle formed by the line of sight with the horizontal,
when it is below the horizontal level.
θ
Horizontal line D E
O 7816 m
Angle of depression
Then, DE = 7816 m
Li

AD 7816
ne

In ∆ADE, tanθ = = =1
of

DE 7816
sig
ht

[Q height of mountain AD = 7816 m]


P (object)
∴ θ = 45°
CBSE Term II Mathematics X (Basic) 105

(v) (c) Let T be the point where mile stone is kept. Again, in right angled ∆CBD,
BC
P tan 60° =
BD
BC
⇒ 3= [Q tan60° = 3 and BD = 2 3 m]
1930 m 2 3
⇒ BC = 3 × 2 3 = 6 m

T
45°
S
Now, AB = AC + BC
= 6.928 + 6 = 12.928 m (approx.)
PS 1930
So, In ∆PST, tan 45° = ⇒1= Hence, whole height of the tree is 12.928 m.
TS TS
3. Let AB = h km be the height of the mountain. Let C be a
⇒ TS = 1930 m
point at a distance of x km from the base of the mountain
Subjective Questions such that ∠ACB = 30° and let D be a point at a distance of
10 km from C along the same line. Then, ∠ADB = 15°
1. Let the ladder BC reaches the building at C and the height of
building where the ladder reaches be AC . and AD = AC + DC = ( x + 10) km
B
C

h km
m

24 m
25

15° 30°
D A
C

B A In right angled ∆BAC,


According to the question, BC = 25 m, AC = 24 m AB 1 h  1 
tan 30° = ⇒ = Q tan 30° = 3 
Now, in right angled ∆ABC, AC 3 x  
⇒ x=h 3 ...(i)
BC 2 = AB2 + AC 2
In right angled ∆BAD,
⇒ 252 = AB2 + 242 AB
tan 15° =
⇒ AB = 625 − 576 = 49
2 AD
h
⇒ AB = 7 m ⇒ 0.27 = [given, tan15° = 0.27]
x + 10
2. Let AB be the tree whose part AC breaks and touches the
ground at D. ⇒ 0.27 ( x + 10) = h ...(ii)
Then, BD = 2 3 m [given] On putting x = 3h from Eq. (i) in Eq. (ii), we get
0.27( 3h + 10) = h
and AC = CD
⇒ 0.27 × 3h + 0.27 × 10 = h
A
⇒ h(1 − 0.27 × 3 ) = 0.27 × 10
⇒ h(1 − 0.27 × 1732
. ) = 2 .7 [Q 3 = 1732
. ]
⇒ h (1 − 0.47 ) = 2 .7
⇒ 0. 53h = 2 .7
C
2.7
⇒ h= = 5.09 ≈ 5 km
0.53
Hence, the height of mountain is 5 km.
4. Let AB be the tower and AC be the flag staff on the tower.
60°
B D Let D be a point on the ground such that the angles of
2√3 m elevation of foot A and top C of the flag staff are 45° and 60°,
In right angled ∆CBD, respectively.
BD C
cos 60° =
CD A
1 2 3  1 
⇒ = Q cos 60° = 2 and BD = 2 3 m
2 CD
⇒ CD = 2 × 2 3 = 4 3
60°
= 4 × 1.732 = 6.928 m [Q 3 = 1732
. ] 45°
B D
∴ AC = CD = 6.928 m
106 CBSE Term II Mathematics X (Basic)

Then, we have AB = 20 m, ∠ADB = 45° and ∠CDB = 60° 6. Let AB be the building and CD be the tower.
In right angled ∆ABD, C
AB  Perpendicular 
tan 45° = Q tanθ = 
BD Base A
20
⇒ 1= 60 m
BD
⇒ BD = 20 m [Q tan 45° = 1]
and in right angled ∆CBD, 60º 30º
BC BC B D
tan60° = ⇒ 3= [Q tan 60° = 3]
BD 20 In ∆CDB,
⇒ BC = 20 3 = 20 × 1732
. [Q 3 = 1.732] CD
= tan60°
= 34.64 m (approx.) BD
Now, AC = BC − AB = 34.64 − 20 60°
⇒ = 3
= 14.64 m (approx.) BD
60°
Hence, the height of the flag staff is 14.64 m. ⇒ BD =
5. To understand the fact of this question, consider the 3
following example In ∆ABD,
AB
I. A tower 2 3 m high casts a shadow 2 m long on the = tan 30°
ground, when the Sun’s elevation is 60°. BD
60 1 60
BC 2 3 AB = × = = 20
In ∆ACB, tan θ = = 3 3 3
AB 2
C Therefore, the height of building is 20 m.
7. Let AB = 20 m be the height of tower and let the ball lying
on the ground at point C.
2√3 m Given, angle of depression,
θ
∠TAC = 60° = ∠ACB [alternate angles]
A 2m B A
T
60°
⇒ tan θ = 3 = tan 60°
∴ θ = 60°
20 m
II. A same hight of tower casts a shadow 4m long from
preceding shadow, when the Sun’s elevation is 30°.
AB
In ∆APB, tan θ = 60°
PB C B
AB In right angled ∆ABC,
=
PC + CB AB
tan60° =
2 3 2 3 BC
⇒ tan θ = =
4+ 2 6 20
⇒ 3=
A BC
20 20
⇒ BC = = = 11.55 m
3 1732
.
2√3 m Hence, the distance between the foot of the tower and the
ball is 11.55 m.
θ 60°
P B 8. Let h be the height of the poles.
4m C 2m
Now. according to the question
3 3 3 A E
⇒ tan θ = ⋅ =
3 3 3 3
1
⇒ tan θ = = tan 30° h h
3
∴ θ = 30° 60º 30º
Hence, we conclude from above two examples that if the B C D
length of the shadow of a tower is increasing, then the 90 cm
angle of elevation of the Sun is decreasing.
CBSE Term II Mathematics X (Basic) 107

In ∆ABC , tan60° =
AB
=
h 10. Let AB = h km be the height of the hill and C, D be two
BC BC consecutive stones such that CD = 1 km.
h h Let BC be x km, then BD = BC + CD = ( x + 1) km
3= ⇒ BC = …(i)
BC 3 30°
A
ED h X
In ∆CDE , tan 30° = =
CD CD 45°
1 h
= ⇒ CD = 3h …(ii)
3 CD
h km
On adding Eqs. (i) and (ii), we get
h
BC + CD = + 3h
3 45° 30°
 1   3 + 1 4h B x km C 1 km D
⇒ 90 = h 3 +  = h =
 3  3  3 (x+1) km

90 × 3 45 3 Now, ∠ ADB = ∠XAD = 30° [alternate angles]


⇒ h= = m
4 2 and ∠ ACB = ∠XAC = 45° [alternate angles]
45 3 In right angled ∆ABC,
Hence, height of the pole is m.
2 Perpendicular AB
tan 45° = =
9. From figure, we observe that, a man standing on a platform Base BC
at point P, 3 m above the surface of a lake observes a cloud at h
⇒ 1 = ⇒x = h …(i)
point C. Let the height of the cloud from the surface of the x
platform is h and angle of elevation of the cloud is θ1 . AB
Now, in right angled ∆ABD, tan 30° =
Now at same point P, a man observes a cloud reflection in BD
the lake at this time the height of reflection of cloud in lake 1 h  1 
is ( h + 3) because in lake platform height is also added to ⇒ = Q tan 30° = 3 
3 x +1  
reflection of cloud.
1 h
So, angle of depression is different in the lake from the angle ⇒ = [from Eq. (i)]
of elevation of the cloud above the surface of a lake. 3 h +1
C ⇒ h + 1 = 3 h ⇒ h( 3 − 1) = 1
1 3 + 1  3 + 1
⇒ h= × =  km
hm 3 −1 3 +1  2 
3 +1
θ1 Hence, height of the hill is km.
P M 2
θ2 3m
Q O 11. Let the height of the tower be h m.
Now, according to the question
A
h

θ R
CM h h
In ∆MPC, tan θ1 = =
PM PM
tan θ1 1
⇒ = …(i)
h PM θ
90º–θ
RM
In ∆RPM, tan θ 2 = D C 4 B
PM 9
OR + OM h + 3 AB h
= = In ∆ABC , tanθ = = …(i)
PM PM BC 4
tan θ 2 1 AB h
⇒ = …(ii) In ∆ABD, tan(90° − θ ) = = = cotθ …(ii)
h + 3 PM BD 9
From Eqs. (i) and (ii), Multiplying Eqs. (i) and (ii),
tan θ1 tan θ 2  h + 3 h h h2
= ⇒ tan θ 2 =   tan θ1 tan θ × cot θ = × ⇒ 1 =
h h+3  h  4 9 36
So, θ1 ≠ θ 2 ⇒ h 2 = 36 ⇒ h = 6 m
Hence, it is a false statement. Hence, the height of the tower is 6m.
108 CBSE Term II Mathematics X (Basic)

12. Let PM be the light house of height 75 m. h = 20( 314


. − 1)
And let A and B be two ships on either side of lighthouse = 20 × 2.14
such that the angle of depression of A and B are 30° and 45°, = 42.8 m
respectively. 14. Let a man is standing on the deck of a ship at point A such
Let AM = x m and BM = y m that AB = 10 m and let CD be the hill.
Then, ∠XYB = ∠MBP = 45° [alternate angles] Then, ∠EAD = 60°
and ∠YPA = ∠MAP = 30° and ∠CAE = ∠BCA = 30° [alternate angles]
x
P
y Let BC = x m = AE and DE = h m
45º 30º D

hm
75 m

45º 30º
B M A 60°
ym xm A E
30°
In right angled ∠AMP, 10 m 10 m
P PM 30°
tan 30° = = B C
xm
B AM
1 75  1  In right angled ∆AED,
⇒ = Q tan 30° = 3  Perpendicular DE h
3 x   tan 60° = = =
Base EA x
⇒ x = 75 3 m h
⇒ 3 = ⇒h= 3x
In right angled ∆BMP, x
PM In right angled ∆ABC,
tan 45° =
BM AB 1 10  1 
75 tan 30° = ⇒ = Q tan 30° = 3 
⇒ 1= [Q tan 45° = 1] BC 3 x  
y
⇒ x = 10 3 m
⇒ y = 75 m
⇒ h = 10 3 × 3 = 30 m
Now, distance between the two ships = AB
= x + y = 75 3 + 75 ∴The height of hill, CD = h + 10 = 30 + 10 = 40 m
= 75( 3 + 1) m Hence, The distance of the hill from the ship is10 3 m and
height of the hill is 40 m.
13. Let height of the tower be h m and the distance between the
15. Let OX be the horizontal ground; A and B be the two
point and foot of the building be x m.
positions of the plane and O be the point of observation.
A A B

h
2500 m

45°
O X
20 C D
Here, AC = BD = 2500 m, ∠AOC = 45°
45º 60º
D x B and ∠BOD = 30°
BC In right angled ∆OCA,
In ∆BCD, tan 45° = Base OC
CD cot 45° = =
perpendicular AC
20
1= OC
x ⇒ 1= [Q cot 45° = 1]
x = 20 m AC
AC ⇒ OC = AC = 2500 m
In ∆ACD, tan60° = In right angled ∆ODB,
CD
OD OD
h + 20 cot 30° = ⇒ 3=
3= BD 2500
20
⇒ OD = 2500 3 m
h = 20( 3 −1)
CBSE Term II Mathematics X (Basic) 109

Now, CD = OD − OC = 2500 3 − 2500 ⇒ 3=


4000
[Q tan 60° = 3]
= 2500( 3 − 1) = 2500(1732
. − 1) AO
4000
= 2500 × 0732
. = 1830 m ⇒ AO = ...(i)
3
Thus, distance covered by plane in 15 s is1830 m.
In right angled ∆AOQ,
1830 60 × 60
∴Speed of plane = × = 439.2 km/h OQ
15 1000 tan 45° =
OA
16. Let AB be height of the building and CD be height of tower OQ
such that CD = 60 m ⇒ 1= [∴ tan 45° = 1]
OA
C
⇒ OA = OQ ...(ii)
From Eqs. (i) and (ii), we get
A
4000
OQ = m
3
∴vertical distance between the aeroplanes
4000
60º 30º = PQ = OP − OQ = 4000 −
3
B D
 1   1 
In right angled ∆ABD, ∠B = 90° = 4000 1 −  = 40001 − 
 3 . 
 1732
AB
tan 30° = = 4000 (1 − 0.577 )
BD
1 AB = 4000 × 0.423 = 1692 m
⇒ = 18. Let OA be the tree of height h m.
3 BD
⇒ BD = AB 3 …(i) Given, PQ = 100 m and angles of elevation are ∠APO = 30°
and ∠OQA = 45°.
and similarly in right angled ∆BCD, A
∠D = 90°
CD
tan60° = hm
BD
60° 30° 45°
⇒ 3= P Q
BD O
100 m
60°
BD = …(ii) Perpendicular OA
3 In right angled ∆POA, tan 30° = =
Base OP
From Eqs. (i) and (ii), we get 1 h  1 
60 ⇒ = Q tan 30° = 3 
AB 3 = 3 OP  
3
60 1 ⇒ OP = 3h ...(i)
⇒ AB = ×
3 3 Now, in right angled ∆QOA,
60 OA
⇒ AB = = 20 m tan 45° =
3 OQ
Hence, the height of the building is 20 m. h
⇒ 1= [Q tan 45° = 1]
17. Let P and Q be the positions of two aeroplanes, where P is OQ
vertically above Q and OP = 4000 m. ⇒ OQ = h ...(ii)
P
On adding Eqs. (i) and (ii), we get
OP + OQ = 3 h + h
⇒ PQ = ( 3 + 1)h [Q OP + OQ = PQ]
Q 4000 m
⇒ 100 = ( 3 + 1)h [QPQ = 100 m, given]
100 3 −1
60° ⇒ h= × [by rationalising]
A 45°
O 3 +1 3 −1
100 (1732
. − 1)
Here, ∠PAO = 60° and ∠QAO = 45° =
2
Now, in right angled ∆AOP,
Perpendicular OP = 50 × 0732
. = 366. m
tan60° = = Hecne, height of the tree is 36.6 m.
Base AO
110 CBSE Term II Mathematics X (Basic)

19. Let the aeroplane be at B and two ships be at C and D such Also, BC = 1 km
that their angles of depression from B are 60° and 30°, Let BD = x km, then
respectively. Then, the angles of elevation of B from D and C DC = BC − BD = (1 − x ) km …(i)
are 30° and 60°, respectively.
In right angled ∆ADB,
B
Perpendicular AD
30° tan 45° = =
60° Base BD
AD
⇒ 1= [Q tan 45° = 1]
1200 m x
⇒ AD = x
30° 60° and in right angled ∆ADC,
A AD
D
ym
C
xm tan 60° =
DC
We have, AB = 1200 m x
⇒ 3=
Let AC = x m and CD = y m. 1−x
In right angled ∆BAC, we have [Q tan 60° = 3 and from Eq. (i)]
Perpendicular AB
tan60° = = ⇒ 3 − 3x = x
Base AC
1200 ⇒ 3 = 3x + x
⇒ 3= [Q tan 60° = 3]
x ⇒ ( 3 + 1) x = 3
1200 3
⇒ x= × [rationalising] ⇒ x=
3
3 3 3 +1
1200 3
⇒ x= = 400 3 m ...(i) 3 3 −1
3 = ×
3 +1 3 −1
In right angled ∆BAD, we have
AB AB [by rationalising]
tan 30° = = [Q AD = DC + CA]
AD DC + CA 3− 3
=

1
=
1200 ( 3 )2 − (1)2
3 x +y [Q (a + b )(a − b ) = a 2 − b 2 ]
 1 
3 − 3 3 − 1.732
⇒ x + y = 1200 3 Q tan 30° = 3 
  = = [Q 3 = 1.732]
2 2
⇒ y = 1200 3 − x …(ii) 1.268
= = 0.634 km
On putting the value of x from Eq. (i) in Eq. (ii), we get 2
y = 1200 3 − 400 3 Hence, the height of the aeroplane is 0.634 km.
= 800 3 21. Let height of the tower, BC = h m and height of the flagstaff
= 800 × 1732
. [Q 3 = 1.732] CD = H m.
∴ BD = BC + CD = ( h + H)m …(i)
= 13856
. m
Given, AB=120 m, ∠CAB = 45° and ∠DAB = 60°
Hence, the distance between both ships is13856 . m. D
20. Let A be the aeroplane and AD be its height. Again, let B Hm
and C be two consecutive kilometre stones on the road on
the left and right of plane A and the angles of depression of C
C and B from plane A are 60° and 45°, respectively. (h+H) m
A
P Q
45° 60° hm

45° 60°
A
120 m B
In right angled ∆ABC, we get
45° 60° BC  Perpendicular 
B C tan 45° = Q tanθ = 
x km (1 – x) km AB Base
D
1 km h
⇒ 1= [Q tan 45° = 1]
Then, ∠ABC = ∠PAB = 45° [alternate angles] 120
and ∠ACB = ∠QAC = 60° [alternate angles] ⇒ h = 120 m …(ii)
CBSE Term II Mathematics X (Basic) 111

BD 23. Let the height of the light house AB be 100 m. C and D be


Now, in right angled ∆ABD, we get tan60° =
AB the positions of man when angle of elevation changes from
h+H 60° to 45°, respectively. The man has covered a distance CD
⇒ 3= …(iii)
120 in 2 min.
[Q tan60° = 3 and from Eq. (i)] Distance CD
Q Speed = ⇒ Speed = ...(i)
Time 2
From Eqs. (ii) and (iii),
120 + H In right angled ∆ABC,
3= A
120
⇒ 120 3 = 120 + H
⇒ H = 120 ( 3 − 1) 100 m
Light house
= 120(1732
. − 1)
= 120 × 0732
. = 87.84 m 45° 60°
D
Hence, height of flag staff is 87.84 m. C B
22. Let D be the position of the balloon, when it is inclined at Perpendicular AB
tan 60° = =
angle of 60° and AB be the height of the pole. Base BC
D (Balloon) 100
⇒ 3= [Q tan 60° = 3]
BC
100 3
⇒ BC = m ...(ii)
C 3
B In right angled ∆ABD,
AB
30°
tan 45° =
60° BD
E 100
A ⇒ 1= [Q tan 45° = 1]
BD
Given, length of cable, DE = 215 m
⇒ BD = 100 m
In right angled ∆EAD,
Perpendicular AD Now, CD = BD − BC
sin 60° = = 100 3
Hypotenuse ED = 100 −
3
3 AD  3
⇒ = Q sin60° =   3 − 3
2 215  2  100  
CD  3  50
215 3 and Speed = = = ( 3 − 3 ) m/min
⇒ AD = m 2 2 3
2
24. Let the height of the tower be h.
Hence , initial height of the balloon from the ground is
Also, SR = x m, ∠PSR = θ
215 3
m. Given that, QS = 20 m
2 and ∠PQR = 30°
Again, in right angled ∆EAD, P
Base AE AE
cos 60° = = =
Hypotenuse DE 215
h
1 AE  1
⇒ = Q cos 60° = 2 
2 215
30° θ
215 Q xm R
⇒ AE = m …(i) 20 m S
2
Now, in ∆PSR,
Now, the angle of inclination is changed, say ∠CEA = 30°.
PR h
In right angled ∆EAC, tan θ = =
SR x
Perpendicular AC
tan 30° = = h
Base EA ⇒ tan θ =
x
1 AC × 2
⇒ = h
3 215 ⇒ x= ...(i)
tan θ
⇒ 2 3AC = 215
Now, in ∆PQR,
215 PR PR
⇒ AC = m tan 30° = =
2 3 QR QS + SR
112 CBSE Term II Mathematics X (Basic)

⇒ tan 30° =
h and in right angled ∆ABC,
20 + x AB h
tan60° = ⇒ 3= [Q tan60° = 3]
h h BC x
⇒ 20 + x = =
tan 30° 1 / 3 ⇒ h = 3x …(ii)
⇒ 20 + x = h 3 On putting h = 3x in Eq. (i), we get
h 20 + x = 3( 3x )
⇒ 20 + =h 3 [from Eq. (i)] …(ii)
tan θ ⇒ 20 + x = 3x
Since, after moving 20 m towards the tower the angle of ⇒ 2 x = 20
elevation of the top increases by 15°. ⇒ x = 10 m
i.e. ∠PSR = θ = ∠PQR + 15° On putting x = 10 in Eq. (ii), we get
⇒ θ = 30° + 15 = 45° h = 3(10)
h
∴From Eq. (i) 20 + =h 3 ⇒ h = 10 3 m
tan 45°
h Hence, the height of the tower is10 3 m and width of the
⇒ 20 + = h 3 canal is 10 m.
1
⇒ 20 = h 3 − h 26. Let the height of the tower be H and OR = x

⇒ h ( 3 − 1) = 20 Given that, height of flag staff = h = FP and ∠PRO = α,


∠FRO = β
20 3 +1
∴ h= ⋅ [by rationalisation] F
3 −1 3 +1
flag staff h
20 ( 3 + 1)
⇒ = P
3 −1
20 ( 3 + 1)
= H
2
⇒ = 10 ( 3 + 1) m β
α
Hence, the required height of tower is 10 ( 3 + 1) m. R x O

25. Let BC = x m be the width of the canal Now, in ∆PRO, tan α =


PO H
=
and AB = h m be the height of the tower. RO x
Given,∠ACB = 60° and ∠ADB = 30° H
⇒ x= …(i)
tan α
A
FO
and in ∆FRO, tan β =
RO
FP + PO
=
RO
hm
h+H
⇒ tan β =
x
h+H
⇒ x= …(ii)
30º 60º tan β
D C B
20 m xm From Eqs. (i) and (ii),
H h+H
In right angled ∆ABD, =
tan α tan β
P AB
tan 30° = = ⇒ H tan β = h tan α + H tan α
B DB
1 h ⇒ H tan β − H tan α = h tan α
⇒ =
3 DC + CB
⇒ H (tan β − tan α) = h tan α
1 h tan α
[Q tan 30° = andDB = DC + CB] ⇒ H=
3 tan β − tan α
1 h
⇒ = h tan α
3 20 + x Hence, the required height of tower is
tan β − tan α
⇒ 20 + x = 3 h …(i)
Hence proved.
CBSE Term II Mathematics X (Basic) 113

27. Let distance between the two towers = AB = x m In right angled ∆ACD, we have
and height of the other tower = PA = h m tan 30° =
DC

1
=
h
Given that, height of the tower = QB = 30 m and AC 3 x + 30
∠QAB = 60°, ∠PBA = 30°
⇒ x + 30 = h 3
Q
⇒ x = 3h − 30 …(ii)

P On comparing Eqs. (i) and (ii), we get


h
30 m = 3h − 30
hm
3
⇒ h = 3h − 30 3
60° 30°
A B
xm ⇒ 2 h = 30 3
QB 30
Now, in ∆QAB, tan60° = = ⇒ h = 15 3 = 15 × 1732
. = 2598
. m
AB x
30 Hence, the height of the tree is 25.98 m.
⇒ 3=
x Now, substituting the value of h = 15 3 in Eq. (i), we get
30 3 15 3
∴ x= ⋅ [by rationalising] x= = 15 m
3 3 3
30 3 Hence, width of the river is 15 m.
= = 10 3 m
3 29. Let the height of pedestal BD be h m, and angle of elevation
and in ∆PBA, of C and D at a point A on the ground be 60° and 45°,
PA h respectively.
tan 30° ==
AB x C
1 h
⇒ = [Q x = 10 3 m]
3 10 3
1.6 m
⇒ h = 10 m
Hence, the required distance and height are10 3 m and
D
10 m, respectively.
28. Let CD be the tree of height h m. Let B be the position of a
H
man standing on the opposite bank of the river. After moving
30 m away from point B, let new position of man be A i.e.
45º 60º
AB = 30 m. A B
The angles of elevation of the top of the tree from points A
and B are 30° and 60°, respectively. It is also given that the height of the statue CD be 3.6 m
∴ ∠CAD = 30° and ∠CBD = 60°, let BC = x m i.e. ∠CAB = 60°
D
∠DAB = 45° and CD = 16
. m
In right triangle ABD, we have
BD
tan 45° =
AB
h
1=
hm AB
⇒ AB = h …(i)
In right triangle ABC, we have
BC
tan60° =
30° 60° AB
A 30 m B xm C BD + DC
⇒ 3=
AB
In right angled ∆BCD, we have
h + 36
.
tan60° =
CD
⇒ 3=
h ⇒ 3=
BC x AB
h h + 36
.
⇒ x= …(i) ⇒ AB = …(ii)
3 3
114 CBSE Term II Mathematics X (Basic)

Comparing Eqs. (i) and (ii), we get 10 3 3 +1


∴ H= ⋅ [by rationalisation]
Let height of the pedestal BD be h m, 3 −1 3 +1
h + 36
. 10 3 ( 3 + 1)
h= =
3 3 −1
⇒ 3h = h + 36
. 10 3 ( 3 + 1)
=
⇒ 3h − h = 36
. 2
⇒ = 5 3 ( 3 + 1)
⇒ h( 3 − 1) = 36
.
= 5( 3 + 3) m.
36
. 3 +1
⇒ h= × Hence, the required height of the tower is 5 ( 3 + 3) m.
3 −1 3 +1
31. Let AD = 7 m be the height of the building and BC = h m
. ( 3 + 1)
36 be the height of the cable tower. From the top of the
⇒ h=
3 −1 building D, the angles of elevation and depression are
∠CDE = 60° and ∠EDB = 45°.
. ( 3 + 1)
36
⇒ h= From the point D, draw a line DE || AB.
2
Then, ∠EDB = ∠ABD = 45° [alternate angles]
⇒ h = 1.8( 3 + 1)
C
Hence, the height of pedestal = 1.8( 3 + 1) m
30. Let the height of vertical tower be,
OT = H and OP = AB = x m
Given that, AP = 10 m 60°
E hm
D
and ∠TPO = 60° , ∠TAB = 45° 45°
T
7m 7m
(H – 10) m

45°
A B
xm
45° H
A xm B
Also, let AB = DE = x m be the distance between building
10 m

10 m

and tower.
60° In right angled ∆BAD,
P O P AD
xm tan 45° = =
B AB
Now, in ∆TPO, 7
OT H ⇒ 1= [Q tan 45°= 1 ]
tan60° = = x
OP x ⇒ x =7m …(i)
H H
⇒ 3= ⇒ x= ...(i) and in right angled ∆CED,
x 3 CE CB − BE
tan60° = = [Q CE = CB − BE]
and in ∆TAB, DE AB
TB H − 10 h −7
tan 45° = = ⇒ 3= [Q tan 60° = 3]
AB x x
H − 10 ⇒ h −7 = x 3
⇒ 1=
x
⇒ h=x 3+7
⇒ x = H − 10
H ⇒ h =7 3 + 7 [from Eq. (i)]
⇒ = H − 10 [from Eq. (i)]
3 ⇒ h = 7( 3 + 1)
H ⇒ h = 7(1732
. + 1)
⇒ H− = 10
3 ⇒ h = 7 × 2.732 ⇒ h = 19124
. m
 1  Hence, the height of the tower is 19.124 m.
⇒ H 1 −  = 10
 3 32. Let the height of the balloon above the ground is H
 3 − 1 and OP = W2R = W1Q = x
⇒ H  = 10
 3  Given that, height of lower window from above the ground
= W2P = 2 m = OR
CBSE Term II Mathematics X (Basic) 115

Height of upper window from above the lower window 33. (i) Distance of first position of parrot from the eyes of girl
= W1 W2 = 4 m = QR = AC
∴ BQ = OB − (QR + RO) E C

=H − ( 4 + 2 ) =H − 6
and ∠BW1 Q = 30°
⇒ ∠BW2R = 60°

58 m
Balloon 60°
D A
B 30° 8m
B
(H – 6) G H F

In ∆ABC,
BC
w1 30° sin60° =
Q AC
xm
Upper Hm CH − BH
window ⇒ AC =
4m sin60°
58 − 8 100
w2 60° x = = m
Lower R 3/2 3
window 2m
(ii) If the distance increases, then the angle of elevation
P O
xm decreases.
(iii) Distance between girl and building = AB
Now, in ∆BW2R,
Now, in ∆ABC,
BR BQ + QR
tan60° = = BC
W2R x tan60° =
AB
(H − 6 ) + 4 ⇒ 3 AB = 50
⇒ 3=
x 50
H−2 ⇒ AB = m
⇒ x= ...(i) 3
3 DE
and in ∆BW1Q, (iv) In ∆AED, tan 30° =
AD
BQ
tan 30° = ⇒ AD = 3 BC = 50 3 m
W1Q
[QED = BC = 58 − 8 = 50]
H −6 1
⇒ tan 30° = = Now, distance between two position of parrot = EC
x 3
= BD = AD − AB
⇒ x = 3 (H − 6 ) ...(ii)
 50 
=  50 3 − m
From Eqs. (i) and (ii),  3
(H − 2 ) 50( 3 − 1)
3 (H − 6 ) = =
3 173
.
3 (H − 6 ) = H − 2 100
= = 57.80 m
⇒ 3H − 18 = H − 2 173
.
⇒ 2H = 16 Distance covered
(v) Speed of parrot =
⇒ H=8 Time taken
So, the required height is 8 m.  57.80
=  m/s
Hence, the required height of the balloon above the ground  8 
is 8 m. = 7.225 m/s
Chapter Test
Multiple Choice Questions Based on the above information, answer the following
1. A circus artist is climbing from the ground along a questions.
rope stretched from the top of a vertical pole and tied (i) Measure of ∠ACD is equal to
at the ground. The height of the pole is 12 m and the (a) 30° (b) 45°
angle made by the rope with ground level is 30°. The (c) 60° (d) 90°
distance covered by the artist in climbing to the top of (ii) If ∠YAB = 45°, then ∠ABD is also 45°, Why?
the pole is (a) vertically opposite angles
(a) 12 m (b) 6 m (b) alternate interior angles
(c) 24 m (d) 32 m (c) alternate exterior angles
(d) corresponding angles
2. A ladder 15 m long just reaches the top of a vertical
wall. If the ladder makes an angle of 60° with the wall, (iii) Length of CD is equal to
then the height of the wall is (a) 90 m (b) 50 3 m
15 (c) 50 / 3 m (d) 100 m
(a) 30 m (b) m
2 (iv) Length of BD is equal to
(c) 15 m (d) 25 m (a) 50 m (b) 100 m
(c) 100 2 m (d) 100 3 m
3. A kite is flying at a height of 30 m from the ground.
The length of string from the kite to the ground is (v) Length of AC is equal to
60 m. Assuming that there is no slack in the string, (a) 100 / 3 m (b) 100 3 m
then the angle of elevation of the kite at the ground is (c) 50 m (d) 100 m
(a) 30° (b) 45° Short Answer Type Questions
(c) 60° (d) None of these 7. The angle of elevation of the top of a tower is 30°. If
4. The tops of two poles of height 30 m and 24 m are the height of the tower is doubled, then the angle of
connected by a wire. If the wire makes an angle of 45° elevation of its top will also be doubled. State true or
with the horizontal, then find the length of the wire. false. Explain.
(a) 14 m (b) 3 m 8. A peacock is sitting on the top of a tree. It observes a
(c) 4 m (d) 6 2 m serpent on the ground making an angle of depression
5. An observer 3.5 m tall is 38.5 m away from a tower of 30°. The peacock catches the serpent in 12 s with the
42 m high. The angle of elevation of the top of the speed of 300 m/min. What is the height of the tree?
[CBSE 2015]
tower from the eye of the observer is
(a) 30° 9. The angles of elevation and depression of the top and
(b) 90° bottom of a light house from the top of a 60 m high
(c) 45° building are 30° and 60°, respectively. Find the
(d) 60° difference between the heights of the light house and
building.
Case Based MCQs
10. As observed from the top of a 100 m high light house
6. A boy is standing on the top of mountain. He from the sea-level, the angles of depression of two
observed that boat P and boat Q are approaching ships are 30° and 45°. If one ship is exactly behind the
towards mountain from opposite directions. He finds other on the same side of the light house, find the
that angle of depression of boat P is 60° and angle of distance between the two ships. [CBSE 2018]
depression of boat Q is 45°. He also knows that height
Long Answer Type Questions
of the mountain is 50 m.
A 11. Two ships are sailing in the sea on either side of the
X Y light house. The angles of depression of two ships as
60° 45°
observed from the top of the light house are 60° and
45°, respectively. If the distance between the ships is
50 m
 3 + 1
P Q C B
100   m, then find the height of the light house.
D  3 

Answers
For Detailed Solutions
1. (c) 2. (b) 3. (a) 4. (d) 5. (c) 6. (i) (c) (ii) (b) (iii) (c) (iv) (a) (v) (a)
Scan the code
7. False 8. 30 m 9. 20 m 10. 100 ( 3 − 1) m 11. 100 m
CBSE Term II Mathematics X (Basic) 117

CHAPTER 06

Surface Areas
and Volumes

In this Chapter...
l Solid Figures
l Surface Area
l Volume
l Combination of Two Figures
l Conversion of Solid from One Shape to Another

Solid Figures Different Types of Solid Figures


The objects having definite shape, size and occupies a fixed 1. Cuboid
amount of space in three dimensions are called solids such as A cuboid is a solid figure having 6 rectangular faces. Let its
cube, cuboid, cylinder, cone, sphere and hemisphere, etc. length = l units, breadth = b units and height = h units.

Surface Area (SA) b

Surface area of a solid body is the area of all of its surfaces


together and it is always measured in square unit. h

e.g. A cube has 6 surfaces and each surface is in a square


shape. Therefore, its surface area will be 6a 2 sq units, where
l

a 2 is the area of each surface of the cube. Then,


Volume (i) Total surface area of cuboid (TSA)
= 2 ( lb + bh + hl) sq units
Space occupied by an object/solid body is called the volume
of that particular object/solid. Volume is always measured in (ii) Lateral surface area of cuboid = 2( l + b)h sq units
cube unit. or Lateral surface area = Area of the 4 vertical faces
e.g. Suppose, a cube has edge of length a units. Volume of a
(iii) Diagonal of the cuboid = l 2 + b 2 + h 2 units
cube is equal to the product of area of base and height of
a cube i.e. a 2 × a = a 3 cu units. (iv) Volume of cuboid = l × b × h cu units
118 CBSE Term II Mathematics X (Basic)

2. Cube Then,
Cube is a special case of cuboid which has 6 equal square faces. (i) Curved surface area (CSA)
= CSA of outer cylinder + CSA of inner cylinder
= 2 πRh + 2 πrh
a = 2π(R + r)h sq units
(ii) Total surface area (TSA)
a = CSA of hollow cylinder + Area of both ends
a = 2π ( R + r )h + 2π ( R 2 − r 2 )
Let its length = breadth = height = a units = 2 π(R + r)h + 2 π(R + r) (R − r)
∴ Each edge of cube = a units = 2π(R + r) [ h + R − r] sq units
Then, (iii) Total outer surface area = 2 πRh + 2 π(R 2 − r 2 ) sq units
(i) Total surface area (TSA) of a cube (iv) Volume of hollow cylinder
= 6 × (Edge ) 2 = 6a 2 sq units = Volume of outer cylinder
(ii) Lateral surface area of cube = 4 × (Edge ) 2 = 4a 2 sq units − Volume of inner cylinder
= πR 2 h − πr 2 h
(iii) Diagonal of a cube = 3 × Edge = 3 a units
= π(R 2 − r 2 )h cu units
(iv) Volume of a cube = (Edge ) 3 = a 3 cu units
5. Sphere
3. Right Circular Cylinder
Cylinder is a solid figure obtained by revolving the rectangle, A sphere is a solid generated by the revolution of a
say ABCD, about its one side, say BC. Let base radius of right semi-circle about its diameter. Let radius of sphere be r
circular cylinder be r units and its height be h units. Then, units.
B A
A
r

h r
O

r
D B
C
Then,
(i) Curved surface area (CSA) (i) Surface area (SA) of sphere = 4 πr 2 sq units
= Circumference of the base × Height = 2πrh sq units 4
(ii) Total surface area (TSA) (ii) Volume of sphere = πr 3 cu units
3
= Curved surface area (CSA) + Area of two ends
6. Spherical Shell
= 2 πrh + 2 πr 2 = 2 πr( h + r) sq units
If R and r are respectively the outer and inner radii of a
(iii) Volume of the cylinder = Area of base × Height spherical shell, then
= πr 2 h cu units (i) Outer surface area = 4 πR 2 sq units
4. Right Circular Hollow Cylinder (ii) Inner surface area = 4 πr 2 sq units
Let R units and r units be the external and internal radii of the 4
(iii) Volume of a hollow sphere = π(R 3 − r 3 ) cu units
hollow cylinder, respectively and h units be its height. 3
R
B
r
A
h O R

r
CBSE Term II Mathematics X (Basic) 119

7. Hemisphere e.g. A combined solid is formed by joining hemisphere and


A plane passing through the centre, cuts the sphere in two right circular cone.
equal parts, each part is called a hemisphere. Let radius of
hemisphere be r units. Then,
r O

(i) Curved surface area (CSA) of hemisphere = 2 πr 2 sq units (i) Surface area of combined solid figure
(ii) Total surface area (TSA) of hemisphere = CSA of cone + CSA of hemisphere
= CSA of hemisphere + Area of one end (ii) Volume of combined solid figure
= 2 πr 2 + πr 2 = 3 πr 2 sq units. = Volume of cone + Volume of hemisphere
2 While calculating the surface area, we have not added the
(iii) Volume of hemisphere = πr 3 cu units surface areas of the two individual solids, rather we have
3
added curved surface area because some part of the surface
8. Right Circular Cone area disappeared in the process of joining them. But this will
not be in the case, when we calculate the volume.
A right circular cone is a solid generated by the revolution of
a right angled triangle about one of its sides containing the Conversion of Solid from One Shape to
right angle as axis as shown in figure.
Another
Let height of a right circular cone be h units and its radius be
r units. Then, Sometimes, we need to convert solid figure of one shape to
another. When we come across objects which are converted
(i) Slant height of the cone,
from one shape to another or when a liquid which is
l = AC = r 2 + h 2 units originally filled in one container of a particular shape is
(ii) Curved surface area (CSA) of cone = πrl sq units poured into another container of a different shape or size, the
volume remains same. e.g.
(iii) Total surface area (TSA) of a cone
(i) If a solid metallic sphere is melted and recast into more
= Curved surface area (CSA) + Area of the base
than one spherical balls, then volume of metallic sphere
= πrl + πr 2 = πr( l + r) sq units = Sum of volumes of all spherical balls.
A (ii) If the Earth taken out by digging a well and spreading it
uniformly around the well to form an embankment in the
shape of a cylindrical shell from its original shape of right
l circular cylinder, then volume of embankment
h
= Volume of Earth taken out by digging a well.
Important Results or Formulae
B r O C
If a solid of one shape is converted into solid (or solids) of
1 2 another shape, then
(iv) Volume of cone = πr h cu units (i) Volume of the solid to be converted = Total volume of the
3
solids into which the given solid is to be converted
Combination of Two Solids (ii) Number of solids of a given shape in which a given solid
is to be converted
Sometimes, we have to find the curved surface area and
volume of a solid, which is a combination of two solids. Then, Volume of the solid to be converted
=
for finding the surface area, we add the curved surface areas Volume of one converted solid
of individual solids and for finding the volume of this solid,
we add the volumes of individual solids.
120 CBSE Term II Mathematics X (Basic)

Solved Examples
Example 1. Three metallic solid cubes whose edges are We know that,
3 cm, 4 cm and 5 cm are melted and formed into a Volume of cylinder = πr 2 h
single cube. Find the edge of the cube so formed.
Sol. Given, edges of three solid cubes are 3 cm, 4 cm and 5 cm,
respectively. 66 cm
10cm
∴ Volume of first cube = ( 3)3 = 27 cm3
[Q volume of cube = (side) 3 ]
Volume of second cube = ( 4)3 = 64 cm3 13 cm
and volume of third cube = ( 5)3 = 125 cm3 240 cm

∴ Sum of volume of three cubes = ( 27 + 64 + 125)


= 216 cm3
Let the edge of the resulting cube = R cm
∴ Total volume of iron pole = Volume of first cylinder
Then, volume of the resulting cube, R 3 = 216 ⇒ R = 6 cm
+ Volume of second cylinder
Example 2. The volume of a right circular cylinder with = π (13)2 × 240 + π (10)2 × 66
1
its height equal to the radius is 25 cm 3 . Find the = π [169 × 240 + 100 × 66]
7 = 3.14 [40560 + 6600]
22
height of the cylinder. (Use π = ) = 3.14 × 47160
7 = 148082.4 cm3
Sol. Let h and r be the height and radius of right circular
Hence, total mass of the iron pole
cylinder, respectively.
Given, height of cylinder = Radius of cylinder = 148082.4 × 8 g = 1184659.2 g
[given, 1 cm 3 ≈ 8 g]
i.e. h=r
1184659.2
Q Volume of cylinder = πr 2 h = kg
1000
1 22 1
∴ 25 = × h 2 × h [Q h = r and V = 25 , given]  1 
= 1184.66 kg Q1g = kg
7 7 7  1000 
176 22
⇒ = ×h 3
7 7 Example 4. A spherical metal ball of radius 8 cm is
⇒ h3 = 8 melted to make 8 smaller identical balls. The radius
⇒ h =2
3 3
⇒ h=2 [taking cube root]
of each new ball is ……… cm.
Sol. Let radius of larger sphere be R = 8 cm
Hence, height of cylinder is 2 cm.
and radius of smaller sphere be r cm
Example 3. An iron pole consists of a cylinder of height Let number of smaller sphere be n = 8
240 cm and base diameter 26 cm, which is According to the given condition,
surmounted by another cylinder of height 66 cm Volume of larger sphere = n × volume of smaller sphere
and radius 10 cm. Find the mass of the pole given 4 3 4
that 1 cm3 of iron has approximately 8 g mass. ∴ πR = n × πr 3
3 3
[take, π = 3.14] ∴ ( 8) 3 = 8 × r 3
Sol. Here, solid iron pole is a combination of two cylinders. ⇒ r 3 = 82 ⇒ r 3 = 64 = ( 4)3
For first cylinder,
⇒ r = 4 cm [taking cube root]
Height = 240 cm
Hence, radius of new ball is 4 cm.
Base diameter = 26 cm
26 Example 5. A solid is in the shape of a cone mounted
∴ Base radius = cm = 13 cm
2 on a hemisphere of same base radius. If the curved
For second cylinder, surface areas of the hemispherical part and the
Height = 66 cm conical part are equal, then find the ratio of the
Radius = 10 cm radius and the height of the conical part.
CBSE Term II Mathematics X (Basic) 121

Sol. Let radius, height and slant height of a cone are r, h and l, Example 7. In figure, a tent is in the shape of a cylinder
respectively. Then, radius of hemisphere will be r.
surmounted by a conical top. The cylindrical part is
2.1 m high and conical part has slant height
h l 2.8 m. Both the parts have same radius 2 m.
r Find the area of the canvas used to make the tent.
 22 
Use π = 7 

Now, curved surface area of cone C1 = πrl 2.8 m


and curved surface area of hemisphere, C 2 = 2 πr 2
According to the question,
C1 = C 2
2.1 m
∴ πrl = 2 πr 2 ⇒ l = 2 r …(i)
Also, l = r 2 + h 2
2m
⇒ ( 2 r ) = r 2 + h2 [Q from Eq. (i)]
On squaring both sides, we get Sol. Given radius of conical and cylindrical part is r = 2 m.

( 2 r ) = ( r + h ) ⇒ 4r = r + h
2 2 2 2 2 2 2 Slant height of cone is l = 2.8 m
And height of cylinder is h = 21. m
⇒ 3r 2 = h 2 ⇒ ( 3r ) 2 = h 2
Taking square root both sides, we get 2.8 m
r 1
3r = h ⇒ =
h 3
Hence, the ratio of the radius and height of the conical part
is 1 : 3. 2.1 m
Example 6. A solid is in the shape of a hemisphere
surmounted by a cone. If the radius of hemisphere 2m
and base radius of cone is 7 cm and height of cone
is 3.5 cm, find the volume of the solid. ∴The area of the canvas used, to make the tent = curve
surface area of cone + curve surface area of cylinder
 22 
 take, π = 7  = πrl + 2 πrh
22 22
= × 2 × 2.8 + 2 × × 2 × 21.
Sol. Given, radius of hemisphere and cone is r = 7 cm. 7 7
And height of cone ( h) = 3.5 cm = 17.6 + 26.4 = 44 cm 2
A Hence, the area of the canvas used to make the tent is
44 cm2 .
3.5 cm
7 cm Example 8. From a solid right circular cylinder of
B C
height 14 cm and base radius 6 cm, a right circular
7 cm cone of same height and same base radius is
removed. Find the volume of the remaining solid.
Sol. Given radius and height of cylinder are
1 2 1 22
Now, volume of cone V1 = πr h = × × (7 )2 × 3.5 r = 6 cm and h = 14 cm
3 3 7
= 179.67 cm 3
2 2 22
and Volume of hemisphere , V2 = πr 3 = × × (7 ) 3
3 3 7
14 cm
= 718.67 cm 3
∴ The volume of solid figure
= Volume of cone + Volume of hemisphere
= V1 + V2 6 cm
= 179.67 + 71867
. = 898.34 cm 3 Also, radius and height of cone will be
Hence, volume of solid shape is 898.34 cm 3 . r1 = 6 cm and h1 = 14 cm
122 CBSE Term II Mathematics X (Basic)

Now, volume of cylinder, Example 10. Two cones with same base radius 8 cm and
V1 = πr12 h height 15 cm are joined together along their bases.
22 Find the surface area of the shape so formed.
= × (6)2 × 14 = 1584 cm 3
7 Sol. If two cones with same base and height are joined together
1 along their bases, then the shape so formed is look like as
Volume of cone, V2 = πr12 h1
3 figure shown.
1 22
= × × (6)2 × 14 = 528 cm 3
3 7 8 cm
∴Volume of remaining solid
= Volume of cylinder − Volume of cone 15 cm
16 cm
= V1 − V2 8 cm
= 1584 − 528 = 1056 cm 3
Hence, volume of the remaining solid is 1056 cm 3 .
30 cm
Example 9. An ice-cream cone full of ice-cream having
radius 5 cm and height 10 cm as shown in figure Given that, radius of cone, r = 8 cm and height of cone,
5 cm h = 15 cm
So, surface area of the shape so formed
= Curved area of first cone
+ Curved surface area of second cone
= 2 ⋅ Surface area of cone [since, both cones are identical]
10 cm = 2 × πrl = 2 × π × r × r 2 + h 2
22 2 × 22 × 8 × 64 + 225
=2× × 8 × ( 8)2 + (15)2 =
7 7
44 × 8 × 289 44 × 8 × 17
= =
7 7
Calculate the volume of ice-cream, provided that its 5984
= = 854.85 cm 2
1 7
part is left unfilled with ice-cream.
6 = 855 cm 2 (approx.)
Sol. Given, ice-cream cone is the combination of a hemisphere Hence, the surface area of shape so formed is 855 cm 2 .
and a cone.
Also , radius of hemisphere = 5 cm
Example 11. The barrel of a fountain pen, cylindrical in
2 2 22 shape, is 7 cm long and 0.5 cm in diameter. A full
∴ Volume of hemisphere = πr 3 = × × ( 5) 3 barrel of ink in the pen can be used for writing 275
3 3 7
5500 words on an average. How many words would be
= = 261.90 cm3 written using a bottle of ink containing one-fourth
21
of a litre? [CBSE 2015, 14]
Now, radius of the cone = 5 cm
Sol. Given, height of cylindrical pen = 7 cm
and height of the cone = 10 − 5 = 5 cm
Diameter 0. 5

1
Volume of the cone = πr 2 h Radius = = cm
3 2 2
1 22 ∴Volume of barrel of a fountain pen = πr 2 h
= × × ( 5) 2 × 5 2
3 7 22  0. 5 22
= ×   ×7= cm3
2750 7  2  16
= = 13095
. cm3
21 It is given that, a pen can write 275 words by using the ink
22
Now, total volume of ice-cream cone cm3 .
= 261.90 + 130.95 = 392.85 cm3 16
∴ Volume of ink = 275 words
1 22
Since, part is left unfilled with ice-cream. ⇒ cm3 = 275 words
6 16
1 1 275 × 16 1
∴Required volume of ice-cream = 392.85 − 392.85 × ⇒ × 1000 cm3 = × × 1000 = 50000
6 4 22 4
1
= 392.85 − 65.475 [Q he will use L of ink to write words]
4
= 327.4 cm3 1
Hence, the pen can write 50000 words by L of ink.
4
CBSE Term II Mathematics X (Basic) 123

Example 12. 500 persons are taking a dip into a Sol. It is clear, from the figure, length = 20 m
cuboidal pond which is 80 m long and 50 m broad. 1
and width = m of each step.
What is the rise of water level in the pond, if the 2
1
average displacement of the water by a person is and height of Ist step which is in the bottom = m
4
0.04 m 3 ? 1 1
Sol. Let the rise of water level in the pond be h m when 500 ∴Height of second step = 2 × = m
4 2
persons are taking a dip into a cuboidal pond. 1 3
Height of third step = 3 × = m
4 4
h M M
1 10
Height of tenth step = 10 × = m
4 4
Water Total volume of the concrete used
1 1 1 2 1 3 1 10
= 20 × × + 20 × × + 20 × × + ... + 20 × ×
2 4 2 4 2 4 2 4
Given that, [Q volume of cuboid = l × b × h]
1 1
Length of the cuboidal pond = 80 m = 20 × × [1 + 2 + 3 + ... + 10]
2 4
Breadth of the cuboidal pond = 50 m
1 1 10 × 11  n ( n + 1) 
Now, volume for the rise of water level in the pond = 20 × × × Q1 + 2 + ... + n =
2 4 2  2 
= Length × Breadth × Height
= 137.5 m3
= 80 × 50 × h
= 4000 h m3 Example 14. A wall 24 m long, 0.4 m thick and 6 m
and the average displacement of the water by a person high is constructed with the bricks each of
= 004
. m3 dimensions 25 cm × 16 cm × 10 cm. If the mortar
1
So, the average displacement of the water by 500 persons occupies th of the volume of the wall, then find
= 500 × 004
. m3
10
the number of bricks used in constructing the wall.
Now, by given condition,
Sol. Given that, a wall is constructed with the help of bricks and
Volume for the rise of water level in the pond = Average mortar.
displacement of the water by 500 persons
∴Number of bricks
⇒ 4000 h = 500 × 004.
1 
500 × 004
. 20 1 (Volume of wall) −  th volume of wall 
∴ h= = = m 10 
4000 4000 200 = ...(i)
Volume of a brick
= 0.005 m Also, given that
= 0.005 × 100 cm Length of a wall ( l ) = 24 m ,
[Q1 m = 100 cm] Thickness of a wall ( b ) = 0.4 m,
= 0. 5 cm Height of a wall ( h) = 6 m
Hence, the required rise of water level in the pond is 0.5 cm. So, volume of a wall constructed with the bricks = l × b × h
Example 13. A small terrace at a hockey ground = 24 × 0.4 × 6
comprises of 10 steps each of which 20 m long and 24 × 4 × 6 3
= m
1 10
built of solid concrete. Each step has a rise of m 1 1 24 × 4 × 6
4 Now, th volume of a wall = ×
1 10 10 10
and a tread of m. Calculate the total volume of 24 × 4 × 6 3
2 = m
concrete required to build the terrace. 102
25
and Length of a brick ( l1 ) = 25 cm = m
100
16
Breadth of a brick ( b1 ) = 16 cm = m
1m 100
2 10
Height of a brick ( h1 ) = 10 cm = m
100
m

So, volume of a brick = l1 × b1 × h1


20

1m 1 25 16 10 25 × 16 3
4 m 3m = × × = m
2 4 100 100 100 105
124 CBSE Term II Mathematics X (Basic)

From Eq. (i), Sol. Suppose, the level of the water in the tank will rise by 7 cm
 24 × 4 × 6 24 × 4 × 6 in x h.
 − 
 10 100  Since, the water is flowing at the rate of 5 km/h.
Number of bricks =
 25 × 16 Therefore, length of the water flow in x h = 5x km = 5000x m
 
 105  [Q 1 km = 1000 m]
24 × 4 × 6 105 We have, diameter of cylindrical pipe = 14 cm
= ×9× 14 7
100 25 × 16 ∴ Radius of cylindrical pipe, r = = 7 cm = m
2 100
24 × 4 × 6 × 9 × 1000
= Volume of the water flowing through the cylindrical
25 × 16
pipe in
= 24 × 6 × 9 × 10 = 12960 22  7 
2

Hence, the required number of bricks used in constructing x h = πr 2 h = ×  × 5000x


7 100
the wall is 12960.
= 77 x m3
Example 15. Water is flowing at the rate of 5 km/h
Also, volume of the water that falls into the tank in x h
through a pipe of diameter 14 cm into a rectangular
= l×b×h
tank which is 50 m long and 44 m wide. Determine the 7
time in which the level of the water in the tank will rise = 50 × 44 ×
100
by 7 cm.
= 154 m3
14 cm  7 
Q l = 50 m, b = 44 m and h = radius = 100 m
 
Q Volume of the water flowing through the cylindrical pipe
in x h = Volume of water that falls in the tank in x h
7 cm
⇒ 77 x = 154
44 m ⇒ x=2
50 m Hence, the level of water in the tank will rise by 7 cm in 2 h.
CBSE Term II Mathematics X (Basic) 125

Chapter
Practice
7. A cylindrical pencil sharpened at one edge is the
PART 1 combination of
(a) a cone and a cylinder
Objective Questions (b) cube and a cylinder
(c) a hemisphere and a cylinder
l
Multiple Choice Questions (d) two cylinders

1. Three cubes each of side 5 cm are joined end to 8. A surahi is the combination of
end, then the surface area of the resulting solid is (a) a sphere and a cylinder
(a) 250 cm 2
(b) 180 cm 2 (b) a hemisphere and a cylinder
(c) 350 cm 2 (d) None of these (c) two hemispheres
(d) a cylinder and a cone
2. A solid ball is exactly fitted inside the cubical box of
side a. The volume of the ball is 9. Two cones have their heights in the ratio 1 : 3 and
1 4 radii in the ratio 3 : 1, then the ratio of their volumes
(a) πa 3 (b) πa 3
6 3 is
1 (a) 1 : 3 (b) 3 : 1
(c) πa 3 (d) None of these
3 (c) 2 : 3 (d) 3 : 2
3. The diameter of a solid metallic sphere is 16 cm. 10. The shape of a gilli, in the gilli-danda game (see
The sphere is melted and recast into solid spherical figure) is a combination of
balls of radius 2 cm. The number of balls are
[CBSE 2020 (Basic)]
(a) 18 (b) 16 (a) two cylinders (b) a cone and a cylinder
(c) 8 (d) 12 (c) two cones and a cylinder (d) two cylinders and a cone
4. A cone of radius 5 cm and slant height 13 cm, then 11. A plumbline (sahul) is the combination of
the height of cone is [CBSE 2020 (Basic)] (see figure)
(a) 12 cm (b) 16 cm
(c) 18 cm (d) 21 cm
5. Total surface area of a solid hemisphere is
[CBSE 2020 (Basic)]
(a) 3πr 2 (b) 2 πr 2
2
(c) 4πr 2 (d) πr 3
3 (a) a cone and a cylinder (b) a hemisphere and a cone
6. A solid cylinder of radius r and height h is placed (c) cube and a cylinder (d) sphere and cylinder
over other cylinder of same height and radius. 12. A solid cone of radius r and height h is placed over
The total surface area of the shape so formed is a solid cylinder having same base radius and height
(a) 4πr( h 2 + r 2 ) as that of a cone. The total surface area of the
(b) 4πr[h + r] combined solid is
(c) 4π ( h 2 + r 2 ) (a) πrl + 2 πrh (b) πr 2 ( l + 2 h)
(d) None of the above (c) πr [ r 2 + h 2 + 2 h + r] (d) None of these
126 CBSE Term II Mathematics X (Basic)

13. The capacity of a cylindrical vessel with a 20. A wooden article was made by scooping out a
hemispherical portion raised upward at the bottom hemisphere from each end of a solid cylinder, as
πr 2 shown in figure. If the height of the cylinder is
as shown in the figure is [ 3h − 2r ]. 10 cm and its base is of radius 3.5 cm. Find the total
3
surface area of the article.
r cm

h cm

1 2 2
(a) πr 2 [2 h − 3r] (b) πr [3h − 2 r] [CBSE 2018]
3 3
(a) 374 cm2 (b) 370 cm2
1 2
(c) πr [3h − 2 r] (d) None of these (c) 475 cm2 (d) None of these
3
21. A heap of rice is in the form of a cone of base
14. The diameter of a sphere is 6 cm. It is melted and
diameter 24 m and height 3.5 m. Find the volume of
drawn into a wire of diameter 2 mm.
the rice. How much canvas as cloth is required to
The length of the wire is
just cover the heap? [CBSE 2018]
(a) 12 m (b) 18 m
(a) 105.5 m2 (b) 471.42 m2
(c) 36 m (d) 66 m
(c) 173.5 m2 (d) None of these
15. During conversion of a solid from one shape to
another, the volume of the new shape will 22. A mason constructs a wall of dimensions
(a) increase (b) decrease
270 cm × 300 cm × 350 cm with the bricks each of
(c) remain unaltered (d) be doubled size 22.5 cm × 11.25 cm × 8.75 cm and it is assumed
1
16. A cylindrical bucket, 32 cm high and with radius of that space is covered by the mortar. Then, the
8
base 14 cm, is filled completely with sand.
22 number of bricks used to construct the wall is
Find the volume of the sand. (use π = ) (a) 11100 (b) 11200 (c) 11000 (d) 11300
7
[CBSE 2020 (Basic)] l
Case Based MCQs
2 2
(a) 2.032 m (b) 1.9712 m
23. To make the learning process more interesting
(c) 1.876 m 2 (d) 1.3285 m 2
creative and innovative Shavya’s class teacher
17. A 20 m deep well, with diameter 7 m is dug and the brings clay in the classroom, to teach the topic.
earth from digging is evently spread out to form a Surface Areas and Volumes. With clay, she forms a
platform 22 m by 14 m. The height of the platform cylinder of radius 4 cm and height 18 cm. Then, she
is moulds the cylinder into a sphere and ask some
(a) 2.5 m (b) 3.5 m question to students.
(c) 3 m (d) 2 m
18. If the radius of the base of a right circular cylinder
is halved, keeping the height same, then find the
ratio of the volume of the cylinder thus obtained to
the volume of original cylinder. [CBSE 2009]
1 1 1 1
(a) (b) (c) (d)
3 4 2 5
19. Marbles of diameter 1.4 cm are dropped into a
cylindrical beaker of diameter 7 cm containing
some water. The water level rises by 5.6 cm. When
marble dropped into the beaker, then the number of
marble is (i) The radius of the sphere so formed is
(a) 4 cm (b) 6 cm
(a) 150 (b) 160
(c) 7 cm (d) 8 cm
(c) 175 (d) 235
CBSE Term II Mathematics X (Basic) 127

(ii) The volume of the sphere so formed is 25. The Great Stupa at Sanchi is one of the oldest stone
(a) 905.14 cm3 (b) 903.27 cm3 structures in India, and an important monument of
(c) 1296.5 cm3 (d) 1156.63 cm3 Indian Architecture. It was originally
(iii) Find the ratio of the volume of sphere to the commissioned by the emperor Ashoka in the 3rd
volume of cylinder. century BCE. Its nucleus was a simple
(a) 2 : 1 (b) 1 : 2 hemispherical brick structure built over the relics
(c) 1 : 1 (d) 3 : 1 of the Buddha. It is a perfect example of
(iv) Total surface area of the cylinder is combination of solid figures. A big hemispherical
(a) 553.14 cm2 (b) 751.52 cm2 dome with a cuboidal structure mounted on it.
(c) 625 cm2 (d) 785.38 cm2
22
(take π = )
(v) During the conversion of a solid from one shape to 7 [CBSE Question Bank]
another the volume of new shape will
(a) be increase (b) be decrease
(c) remain unaltered (d) be double
24. Geeta and Meena have 10 and 6 CD respectively,
each of radius 4 cm and thickness 1 cm. They place
their CD one above the other to form solid
cylinders.

Chattra
Dome Harmika
Toranas Balustrade

Stairs

Based on the above information, answer the


following questions.
(i) Curved surface area of the cylinder made by Geeta
is (i) Calculate the volume of the hemispherical dome if
(a) 308.17 cm2 (b) 132 cm2 the height of the dome is 21 m.
(c) 154 cm2 (d) 251.42 cm2 (a) 19404 cu m (b) 2000 cu m
(ii) The ratio of curved surface area of the cylinder (c) 15000 cu m (d) 19000 cu m
made by Geeta and Meena is (ii) The formula to find the volume of sphere is
(a) 3 : 5 (b) 3 : 2 2 4
(a) π r3 (b) πr 3
(c) 5 : 3 (d) 5 : 7 3 3
(iii) The volume of the cylinder made by Meena is (c) 4π r 2 (d) 2 πr 2
(a) 301.44 cm3 (b) 144 cm3 (iii) The cloth require to cover the hemispherical dome
(c) 132 cm3 (d) 208.42 cm3 if the radius of its base is 14m is
(iv) The ratio of the volume of the cylinders made by (a) 1222 sq m (b) 1232 sq m
Geeta and Meena is (c) 1200 sq m (d) 1400 sq m
(a) 1 : 2 (b) 2 : 5 (iv) The total surface area of the combined figure
(c) 3 : 5 (d) 5 : 3 i.e. hemispherical dome with radius 14 m and
(v) When two CD Cassette are shifted from Geeta cuboidal shaped top with dimensions 8 m × 6 m
cylinder to Meena’s cylinder, then × 4 m is
(a) Volume of two cylinder become equal (a)1200 sq m (b) 1232 sq m
(b) Volume of Geeta’s cylinder > Volume of Meena’s (c) 1392 sq m (d) 1932 sq m
cylinder (v) The volume of the cuboidal shaped top is with
(c) Volume of Meena’s cylinder > Volume of Geeta’s dimensions mentioned in question (iv).
cylinder (a) 182.45 m 3 (b) 282.45 m 3
(d) None of the above (c) 292 m 3 (d) 192 m 3
128 CBSE Term II Mathematics X (Basic)

12. A solid metallic cylinder of diameter 12 cm and


PART 2 height 15 cm is melted and recast into 12 toys in
the shape of a right circular cone mounted on a
Subjective Questions hemisphere of same radius. Find the radius of the
hemisphere and total height of the toy, if the height
l
Short Answer Type Questions of the cone is 3 times the radius. [CBSE 2020 (Basic)]
1. Two identical cubes each of volume 64 cm 3 are 13. A rectangular water tank of base 11 m × 6 m
contains water upto a height of 5 m. If the water in
joined together end to end. What is the surface area
the tank is transferred to a cylindrical tank of radius
of the resulting cuboid?
3.5 m, find the height of the water level in the tank.
2. A cuboid measuring 1 cm, 2 cm, 4 cm is melted and
14. A copper rod of diameter 1 cm and length 8 cm is
recast into the shape of a cube. Find the length of
drawn into a wire of length 8 m of uniform
each side of the cube. [CBSE 2020 (Basic)]
thickness. Find the thickness of the wire.
3. 16 glass spheres each of radius 2 cm are packed into 15. The rain water from a roof of dimensions 22 m
a cuboidal box of internal dimensions 16 cm × 8 cm × 20 m drains into a cylindrical vessel having
× 8 cm and then the box is filled with water. Find diameter of base 2 m and height 3.5 m. If the rain
the volume of water filled in the box. water collected from the roof just fill the cylindrical
4. If a solid piece of iron in the form of a cuboid of vessel, then find the rainfall (in cm).
dimensions 49 cm × 33 cm × 24 cm, is moulded to 16. A cylindrical bucket, 32 cm high and with radius of
form a solid sphere. Then, find radius of the sphere. base 18 cm, is filled with sand. This bucket is
5. If the radii of two spheres are in the ratio 2 : 3, then emptied on the ground and a conical heap of sand is
the ratio of their respective volumes is …… . formed. If the height of the conical heap is 24 cm,
[CBSE 2020 (Basic)] then find the radius and slant height of the heap.
6. A solid is in the shape of a cone surmounted on a [CBSE 2020 (Basic)]
hemisphere. The radius of each of them being 17. The barrel of a fountain pen, cylindrical in shape, is
3.5 cm and the total height of the solid is 9.5 cm. 7 cm long and 5 mm in diameter. A full barrel of ink
Find the volume of the solid. [CBSE 2020 (Basic)] in the pin is used up on writing 3300 words on an
7. From a solid cube of side 7 cm, a conical cavity of average. How many words can be written in a
height 7 cm and radius 3 cm is hollowed out. Find bottle of ink containing one-fifth of a litre?
the volume of the remaining solid. 18. Water flows at the rate of 10 m min −1 through a
cylindrical pipe 5 mm in diameter. How long would
8. A hemispherical depression is cut out from one face
it take to fill a conical vessel whose diameter at the
of a cubical wooden block of edge 21 cm, such that
base is 40 cm and depth 24 cm?
the diameter of the hemisphere is equal to edge of
the cube. Determine the volume of the remaining 19. Water flows through a cylindrical pipe, whose inner
block. [CBSE 2020 (Basic)] radius is 1 cm, at the rate of 80 cms −1 in an empty
cylindrical tank, the radius of whose base is 40 cm.
9. From a solid cylinder whose height is 15 cm and the
What is the rise of water level in tank in half an
diameter is 16 cm, a conical cavity of the same
hour?
height and same diameter is hollowed out. Find the
total surface area of remaining solid. 20. A factory manufactures 120000 pencils daily. The
(give your answer in terms of π) [CBSE 2020 (Basic)] pencils are cylindrical in shape each of length 25
cm and circumference of base as 1.5 cm. Determine
10. Twelve solid spheres of the same size are made by the cost of colouring the curved surfaces of the
melting a solid metallic cylinder of base diameter
pencils manufactured in one day at ` 0.05 per dm 2 .
2 cm and height 16 cm. Find the diameter of each
sphere. 21. A well of diameter 10 m is dug 14 m deep. The
Earth taken out of it is spread evenly all around to a
11. A solid is composed of a cylinder with width of 5 m to form an embankment. Find the
hemispherical ends. If the whole length of the solid height of embankment.
is 104 cm and the radius of each hemispherical end
is 7 cm, then find the cost of polishing its surface at 22. A solid metallic cuboid of dimension 24 cm ×
the rate of ` 2 per dm 2 .  22  11 cm × 7 cm is melted and recast into solid cones
 take, π = 7  of base radius 3.5 cm and height 6 cm. Find the
number of cones of formed. [CBSE 2020 (Basic)]
CBSE Term II Mathematics X (Basic) 129

l
Long Answer Type Questions 30. Two solid cones A and B are placed in a cylindrical
tube as shown in the figure. The ratio of their
23. If a hollow cube of internal edge 22 cm is filled with capacities is 2 : 1. Find the heights and capacities
spherical marbles of diameter 0.5 cm and it is of cones. Also, find the volume of the remaining
1
assumed that space of the cube remains unfilled. portion of the cylinder.
8
21 cm
Then, the number of marbles that the cube can
accomodate is

6 cm
A B
24. A hollow sphere of external and internal diameters
8 cm and 4 cm respectively is melted into a cone of
base radius 4 cm. Find the height of the cone. 31. How many spherical lead shots of diameter 4 cm
[CBSE 2020 (Basic)]
can be made out of a solid cube of lead whose
25. A building is in the form of a cylinder surmounted by edge measures 44 cm.
19 3
a hemispherical vaulted dome and contains 41 m 32. A cone of height 24 cm and radius of base 6 cm is
21 made up of modelling clay. A child reshapes it in
of air. If the internal diameter of dome is equal to its the form of a sphere. Find the radius of the
total height above the floor, find the height of the sphere. [CBSE 2020 (Basic)]
building?
33. How many spherical lead shots each of diameter
26. A medicine-capsule is in the shape of a cylinder of
4.2 cm can be obtained from a solid rectangular
diameter 0.5 cm with two hemispheres stuck to each
lead piece with dimensions 66 cm, 42 cm and
of its ends. The length of entire capsule is
21 cm?
2 cm. The capacity of the capsule is
27. A rocket is in the form of a right circular cylinder 34. Find the number of metallic circular disc with
closed at the lower end and surmounted by a cone 1.5 cm base diameter and of height 0.2 cm to be
with the same radius as that of the cylinder. The melted to form a right circular cylinder of height
diameter and height of the cylinder are 6 cm and 10 cm and diameter 4.5 cm.
12 cm, respectively. If the slant height of the conical 35. A heap of rice is in the form of a cone of diameter
portion is 5 cm, then find the total surface area and 9 m and height 3.5 m. Find the volume of the rice.
volume of the rocket. [use π = 314]. How much canvas cloth is required to just cover
28. A toy is in the form of a cone of radius 3.5 cm heap?
mounted on a hemisphere of same radius. If the total 36. How many cubic centimetres of iron is required
height of the toy is 15.5 cm, find the total surface area to construct an open box whose external
of the toy. [CBSE 2020 (Basic)] dimensions are 36 cm, 25 cm and 16.5 cm
29. A wooden toy rocket is in the shape of a cone provided the thickness of the iron is 1.5 cm. If one
mounted on a cylinder, as shown in figure. The cubic centimetre of iron weights 7.5 g, then find
height of the entire rocket is 24 cm, while the height the weight of the box.
of the conical part is 4 cm. 37. Water in a canal 6 m wide and 1.5 m deep, is
flowing with a speed of 10 km/h. How much area
4 cm will it irrigate in 30 min, if 4 cm of standing water
24 cm is needed ? [CBSE 2020 (Basic)]

4 cm 6 cm
l
Case Base Questions
38. Mathematics teacher of a school took her 10th
standard students to show Ram Mandir. It was a
Base of cylinder part of their Educational trip. The teacher had
Base of cone interest in history as well. She narrated the facts
The base of the conical portion has a diameter of of Ram Mandir to students.
6 cm, while the base diameter of the cylindrical Ram mandir is a Hindu temple that is being built
portion is 4 cm. If the conical portion is to be painted in Ayodhya, which is in Uttar Pradesh. The
orange and the cylindrical portion yellow, then find temple construction is being supervised by the
the area of the rocket painted with each of these Shri Ram Janmabhoomi Teerth Kshetra.
colours. [take, π = 314
. ]
130 CBSE Term II Mathematics X (Basic)

Then the teacher said in this monuments one can After that they went for a jungle trek. The jungle
find combination of solid figures. She pointed that trek was enjoyable but tiring. As dusk fell, it was
there are cubical bases and in centre cylinder with time to take shelter. Each group of four students
the cone shape structure on the top is constructed. was given a canvas of area 551m2. Each group had
to make a conical tent to accommodate all the four
students. Assuming that all the stitching and
wasting incurred while cutting, would amount to 1
m2, the students put the tents. The radius of the
tent is 7 m. [CBSE Question Bank]

(i) Ram Mandir is constructed in the form of the


cubical base of 30 cm × 20 cm × 10 cm, then find
the area covered.
(ii) If the radius of the cylinder is 7 cm and Height of
the cylinder is 60 cm and the radius of the cone is Area = 551 m2
similar to that of cylinder and Height of the cone is r=7m
24 cm, then the ratio of curved surface area of
cylinder to curved surface area of the cone.
(iii) Given structure in based on the concept of (i) Find the volume of cylindrical cup.
(a) Area and perimeter (ii) Find the volume of hemispherical cup.
(iii) Find the height of the conical tent prepared to
(b) Surface area and volume
accommodate four students.
(c) Both (a) and (b)
40. On a Sunday, your Parents took you to a fair. You
(d) None of the above could see lot of toys displayed, and you wanted
39. Adventure camps are the perfect place for the them to buy a RUBIK’s cube and strawberry
children to practice decision making for themselves ice-cream for you.
without parents and teachers guiding their every Observe the figures and answer the questions.
move. Some students of a school reached for [CBSE Question Bank]
adventure at Sakleshpur. At the camp, the waiters
served some students with a welcome drink in a
cylindrical glass and some students in a
hemispherical cup whose dimensions are shown
below.

(i) Find the length of the diagonal if each edge


measures 6cm.
(ii) Find volume of the solid figure if the length of the
edge is 7cm.
(iii) What is the curved surface area of hemisphere
(ice-cream) if the base radius is 7 cm?
(iv) Find the slant height of a cone if the radius is 7 cm
d=7 cm
and the height is 24 cm
h=10.5 cm d=7 cm (v) Find the total surface area of cone with
hemispherical ice cream.
CBSE Term II Mathematics X (Basic) 131

SOLUTIONS
Objective Questions 6. (d) Since, the total surface area of cylinder of radius r and
1. (c) Here, on joining three cubes, we get a cuboid whose height h = 2 πrh + 2 πr 2
length, l = 5 + 5 + 5 = 15 cm, breadth, b = 5 cm and When one cylinder is placed over the other cylinder of same
height, h = 5 cm height and radius,
5 cm 5 cm 5 cm then height of the new cylinder = 2 h
and radius of the new cylinder = r
5 cm ∴Total surface area of the new cylinder = 2 πr( 2 h) + 2 πr 2
I II III
= 4πrh + 2 πr 2
5 cm = 2 πr( 2 h + r )
7. (a) Because the shape of sharpened pencil is
∴ Required surface area of the resulting solid
= Surface area of new cuboid
= 2 ( lb + bh + hl ) = 2 (15 × 5 + 5 × 5 + 5 × 15)
= +
= 2 (75 + 25 + 75) = 2 (175) = 350 cm 2
2. (a) Because solid ball is exactly fitted inside the cubical box = Cylinder + Cone
of side a. So, a is the diameter for the solid ball. 8. (a) Because the shape of surahi is
a
∴ Radius of the ball =
2
3
4 a 1
So, volume of the ball = π   = πa 3
3  2 6
= + = Sphere + Cylinder

3. (c) Diameter of a solid metallic sphere = 16 cm


and radius of a solid metallic sphere = 8 cm
Let the number of balls be n 9. (b) Let the radii of two cones are r1 , r2 and their heights are
Volume of solid metalic sphere h1 and h 2 .
n=
r 3 h 1
Volume of solid spherical balls Given, 1 = and 1 =
4 r2 1 h2 3
π (16)3
3 1 2
n= πr1 h1   2  
4 V1 r h
π ( 8) 3 Now, the ratio of their volumes, = 3 =  1  1
3 V2 1 πr 2 h  r2   h 2 
2 2
n=8 3
2
4. (a) Given, Radius of cone ( r ) = 5 cm  3  1
=   × 
Slant height of cone ( l ) = 13 cm  1  3
Let h cm be the height of cone. 1 3
= 9 × = = 3 :1
We know that, 3 1
l 2 = r 2 + h2 Hence, the ratio of their volumes is 3 : 1.
⇒ (13)2 = ( 5)2 + h 2 10. (c)
⇒ 169 = 25 + h 2
⇒ h 2 = 169 − 25 = + +
⇒ h = 144
2
= Cone + Cylinder + Cone
⇒ h = 12 cm = Two cones and a cylinder
[Q taking positive value only as height 11. (b)
cannot be negative]
∴ Height of cone = 12 cm
5. (a) Total surface area of a solid hemisphere
= Curved surface area + Flat surface area = + = Hemisphere + Cone
(Mean area of circular part)
= 2 πr 2 + πr 2
= 3π r 2
132 CBSE Term II Mathematics X (Basic)

12. (c) We know that, total surface area of a cone of radius, r 7


17. (a)Q Radius of the well = m = 3. 5 m
and height, h = Curved surface Area + area of base 2
= πrl + πr 2 ∴Volume of the earth dug out =
22
× ( 3. 5)2 × 20
7
where, l = h2 + r 2
22
and total surface area of a cylinder of base radius, r and = × 3. 5 × 3. 5 × 20 = 770 m3
7
height, h
Area of platform = ( 22 × 14) m2 = 308 m2
= Curved surface area + Area of both base
770
= 2 πrh + 2 πr 2 ∴ Height = = 2.5 m
308
Here, when we placed a cone over a cylinder, then one base
18. (b) Let the radius of original right circular cylinder be ‘r’ cm.
is common for both.
and height be ‘h’ cm. Then,
So, total surface area of the combined solid
Volume of original cylinder V1 = ( πr 2 h) cm 3
= πrl + 2 πrh + πr 2
Volume of circular cylinder, when radius is halved
= πr [l + 2 h + r] 2
 r πr 2 h 3
= πr  r 2 + h 2 + 2 h + r  V2 = π   h cm 3 = cm
   2 4
13. (c) We know that, capacity of cylindrical vessel = πr 2 h cm3 V2 πr 2 h 1 1
Thus, = × = .
2 V1 4 πr h 4
2
and capacity of hemisphere = πr 3 cm
3 4
19. (a) So, volume of one spherial marble = π (0.7)3
From the figure, capacity of the cylindrical vessel 3
2 1  4 
= πr 2 h − πr 3 = πr 2 [3h − 2 r] Q volume of sphere = πr 3
3 3  3 
14. (c) We have, diameter of metallic sphere = 6 cm 1.372
= π cm3
∴Radius of metallic sphere, r1 = 3 cm 3
Also, diameter of cross-section of cylindrical wire = 0.2 cm ∴ Volume of the raised water in beaker = π (3.5)2 × 5.6
∴Radius of cross-sections of cylindrical wire, r2 = 01
. cm [Q volume of cylinder = πr 2 h]
Let the length of the wire be h cm. = 68.6 π cm 3
Since, metallic sphere is converted into a cylindrical shaped Now, required number of marbles
wire of length h cm. Volume of the raised water in beaker
=
∴Volume of the metal used in wire = Volume of the sphere Volume of one spherical marble
4 68.6 π
⇒ πr22 h = πr 13 = × 3 = 150 marbles
3 1.372 π
2
1 4 20. (a) Given, wooden article is a combination of a cylinder and
⇒ π ×   × h = × π × 27
10 3 two hemispheres.
3.5 cm
1
⇒ π× × h = 36π
100
36π × 100
⇒ h=
π
10 cm
= 3600 cm = 36 m
[Q1m = 100cm]
15. (c) During conversion of a solid from one shape to another,
the volume of the new shape will remain unaltered.
16. (b) Given, a cylindrical bucket, with height h = 32 cm and
radius r = 14 cm. Here, height of the cylinder, h = 10 cm
So, Volume of sand = Volume of a cylinder Q Radius of base of the cylinder
= πr 2 h = Radius of hemisphere, r = 3.5 cm
22 Now, required TSA of the wooden article
= × (14)2 × 32 = 2 × CSA of one hemisphere + CSA of cylinder
7
= 2 × ( 2 πr 2 ) + 2 π r h
= 22 × 2 × 14 × 32
22
= 2 π r ( 2 r + h) = 2 × × 3.5 × (2 × 3.5 + 10)
= 19712 cm2 7
19712 22
= m2 = 19712
. m2 = × 7 × ( 7 + 10) = 22 × 17 = 374 cm2
100 × 100 7
CBSE Term II Mathematics X (Basic) 133

21. (b) Given diameter d = 24 m (iii) (c) Since the volume of sphere is equal to volume of
A cylinder, then the ratio of volume of the sphere to the
volume of cylinder = 1 : 1
(iv) (a) Total surface area of cylinder = 2πr( r + h)
22 22
3.5m =2× × 4( 4 + 18) = 2 × × 4 × 22
7 7
= 55314
. cm2
B C
24 m (v) (c) During the conversion of a solid from one shape to
another the volume of new shape will remain unaltered.
1 2
Now, volume of rice = volume of cone = πr h 24. We have radius of each CD cassette = 4 cm
3
and thickness of each cassette = 1 cm
1 22  d 24 
= × × (12 )2 × 3. 5 Qr = = = 12 m So, height of cylindrical made by Geeta, h1 = 10 × 1 = 10 cm
3 7  2 2 
and height of cylindrical made by Meena, h 2 = 6 × 1 = 6 cm
22 × 144 × 3.5
= (i) (d) Curved surface area of cylinder made by Geeta
21 22
11088 = 2 πrh = 2 × × 4 × 10 = 251.42 cm2
= = 528 m3 7
21
Curved surface area of 
Now, slant height l = h 2 + r 2 cylinder made by Geeta 
(ii) (c)∴Required ratio =  
= ( 3. 5)2 + (12 )2 = 12 . 25 + 144 Curved surface area of 
cylinder made by Meena 
= 156.25 = 12 . 5 m  
∴The canvas required to cover the heap = πrl 2 πrh1 h1 10
= = = = 5:3
22 3300 2 πrh 2 h 2 6
= × 12 × 12 . 5 = = 471. 42 m2
7 7 (iii) (a) Volume of cylinder made by Meena
22. (b) Volume of the wall = 270 × 300 × 350 = 28350000 cm3 = πr 2 h 2 =
22
× 4 × 4 × 6 = 301.44 cm3
[Q volume of cuboid = length × breadth × height] 7
1 (iv) (d) Required ratio
Since, space of wall is covered by mortar. Volume of the cylinder made by Geeta
8 =
Volume of the cylinder made by Meena
So, remaining space of wall = Volume of wall
− Volume of mortar πr 2 h1 h1
= = = 5:3
1 πr 2 h 2 h 2
= 28350000 − 28350000 ×
8 (v) (a) When two CD Cassette are shifted from Geeta’s
= 28350000 − 3543750 cylinder to Meena’s cylinder, then length of both
= 24806250 cm3 cylinders become equal.
So, volume of both cylinders become equal.
Now, volume of one brick = 22 . 5 × 11.25 × 875
.
= 2214.844 cm3 25. (i) (a) As, we know that hemisphere is a type of solid in
which radius is the height. So, radius = 21 m
[Q volume of cuboid = length × breadth × height] 2
24806250 ∴Required volume = πr 3
∴ Required number of bricks = = 11200 (approx.) 3
2214.844 2 22
Hence, the number of bricks used to construct the wall is = × × 21 × 21 × 21
3 7
11200. = 19404 cu m
23. (i) (b) Since, volume of sphere = Volume of cylinder 4
4 (ii) (b) Volume of sphere = πr 3
⇒ πR 3 = πr 2 h, where R , r are the radii of sphere and 3
3 (iii) (b) Given, radius ( r ) = 14 m
cylinder, respectively.
3 ∴Curved surface area of hemisphere dome = 2 πr 2
⇒ R3 = r 2 × h × 22
4 =2× × 14 × 14
4 × 4 × 18 × 3 7
⇒ R = 3
= 8 × 27 ⇒ R 3 = ( 2 × 3)3
4 = 1232 sq m
∴ R = 6 cm (iv) (c) Here, radius of hemispherical dome ( r ) = 14 m
4 4 22 Surface area of dome = 2 πr 2
(ii) (a) Volume of sphere = πR 3 = × ×6×6×6
3 3 7 22
= 90514
. cm3 =2× × 14 × 14 = 1232 m2
7
134 CBSE Term II Mathematics X (Basic)

and CSA of cuboidal shaped top = 2 × h( l + b ) + lb 4. Given, dimensions of the cuboid = 49 cm × 33 cm × 24 cm


= 2 × 4( 8 + 6) + 8 × 6 ∴ Volume of the cuboid = 49 × 33 × 24 = 38808 cm3
= 8(14) + 48 [Q volume of cuboid = length × breadth × height]
= 112 + 48 = 160 m2 Let the radius of the sphere is r, then
∴Total surface area = 1232 + 160 = 1392 m2 4
Volume of the sphere = πr 3
(v) (d) Volume of cuboidal shape = lbh = 8 × 6 × 4 = 192 m3 3
4
[Q volume of the sphere = π × (radius)3 ]
Subjective Questions 3
1. Let the length of a side of a cube = a cm According to the question,
a Volume of the sphere = Volume of the cuboid
a 4 3
⇒ πr = 38808
3
22 3
⇒ 4× r = 38808 × 3
a 7
38808 × 3 × 7
⇒ r3 = = 441 × 21
4 × 22
2a ⇒ r 3 = 21 × 21 × 21
Given, volume of the cube, a 3 = 64 cm 3 ⇒ a = 4 cm ∴ r = 21 cm
On joining two cubes, we get a cuboid whose Hence, the radius of the sphere is 21 cm.
length, l = 2a cm 5. Let radii of two spheres be 2k and 3k respectively.
4
breadth, b = a cm Since, volume of sphere = πr 3 cu units
3
and height, h = a cm
4
Now, surface area of the resulting cuboid π ( 2 k)3
Volume of first sphere 3
= 2 ( lb + bh + hl ) ∴ =
Volume of second sphere 4 π ( 3k)3
= 2 ( 2a ⋅ a + a ⋅ a + a ⋅ 2a ) 3
= 2 ( 2a 2 + a 2 + 2a 2 ) = 2 ( 5 a 2 ) 23 8
= 3 =
= 10 a 2 = 10 ( 4)2 = 160 cm2 3 27
2. Given a cuboid measuring1 cm × 2 cm × 4 cm 6. Height of the cone = Total height − Radius of the
hemisphere
∴ l = 1 cm, b = 2 cm, h = 4 cm
= 9. 5 − 3. 5 = 6 cm
Now, volume of cuboid = l × b × h
A
= 1 × 2 × 4 = 8 cm 3
After it is melted and recast into the shape of a cube volume
will remain unchange.
Let each side of the cube is a cm.
B r
∴ Volume of cube = a 3
∴ a3 = 8 = 2 × 2 × 2 r
⇒ a = 2 cm
C
∴ Length of each side of the cube is 2 cm.
3. Given, dimensions of the cuboidal = 16 cm × 8 cm × 8 cm Volume of the solid = Volume of the cone
+ Volume of the hemisphere
∴Volume of the cuboidal = 16 × 8 × 8 = 1024 cm3
1 2 2 3
Also, given radius of one glass sphere = 2 cm = πr h + πr
 3 3 
4 4 22
∴Volume of one glass sphere = πr 3 = × × ( 2 )3 1
= πr 2 [h + 2 r]
3 3 7 3
704
= = 33.523 cm3 = ×
1 22
× 3.5 × 3.5[6 + 2 × 3.5]
21 3 7
Now, volume of 16 glass spheres = 16 × 33.523 = 536.37 cm3 1 22
= × × 3.5 × 3.5 × 13
∴ Required volume of water = Volume of cuboidal 3 7
− Volume of 16 glass spheres = 166.83 cm3
= 1024 − 536.37 Thus, total volume of the solid is166.83 cm3 .
= 487.6 cm3
CBSE Term II Mathematics X (Basic) 135

7. Given that, side of a solid cube (a ) = 7 cm = 2 × π × 8 × 15 + π × ( 8)2 + π × 8 × 152 + 82


Height of conical cavity i.e. cone, h = 7 cm
[Q l = r 2 + h 2 = 82 + 152 ]
= 240π + 64π + 136π = 440π cm2
Hence, the total surface area is 440π cm2 .
10. Given, diameter of the cylinder = 2 cm
7 cm ∴Radius = 1 cm and height of the cylinder = 16 cm
[Q diameter = 2 × radius]
∴Volume of the cylinder = π × (1)2 × 16 = 16 π cm3
[Q volume of cylinder= π × (radius)2 × height]
3 cm
Now, let the radius of solid sphere = r cm
Since, the height of conical cavity and the side of cube is 4
equal that means the conical cavity fit vertically in the cube. Then, its volume = πr 3 cm3
3
Radius of conical cavity i.e. cone, r = 3 cm 4
⇒ Diameter = 2 × r = 2 × 3 = 6 cm [Q volume of sphere = × π × (radius)3 ]
3
Since, the diameter is less than the side of a cube that means
According to the question,
the base of a conical cavity is not fit inhorizontal face of
cube. Volume of the twelve solid sphere = Volume of cylinder
4
Now, volume of cube = (side) 3 = a 3 = (7 )3 = 343 cm 3 ⇒ 12 × πr 3 = 16 π
3
1
and volume of conical cavity i.e. cone = π × r 2 × h ⇒ r 3 = 1 ⇒ r = 1 cm
3
1 22 ∴ Diameter of each sphere, d = 2 r = 2 × 1 = 2 cm
= × × 3× 3×7 Hence, the required diameter of each sphere is 2 cm.
3 7
= 66 cm 3 11. Given, whole length of the solid = 104 cm

∴Volume of remaining solid = Volume of cube and the radius of each hemisphere = 7 cm
− Volume of conical cavity Therefore, the length of the cylindrical part of the solid
= 343 − 66 = 277 cm3 = (104 − 2 × 7 ) = 90 cm
For hemispherical portion,
Hence, the required volume of solid is 277 cm 3 .
Radius, r = 7 cm
8. Given, edge of wooden block (a )
For cylindrical portion,
= Diameter of the hemisphere
Radius, r = 7 cm
= 21 cm
Height, h = 90 cm
21
∴ Radius of the hemisphere, r = = 10.5 cm ∴ Total surface area of the solid
2
= 2 × Curved surface area of a hemisphere
Now, volume of remaining block
+ Curved surface area of cylindrical part
= Volume of box − Volume of hemisphere
2 2 22 = 2 [2 πr 2 ] + 2 πrh
= a 3 − πr 3 = ( 21)3 − × × (10.5)3 = 6835.5cm3
3 3 7 = 2 × [2 π (7 )2 ] + 2 π (7 ) (90)
9.  22  22
A 8 cm O 8 cm B =2× 2× × (7 ) 2 + 2 × × (7 ) (90)
 7  7
= 4 × 22 × 7 + 2 × 22 × 90 = 22 [28 + 180]
Conical
cavity = 4576 cm 2
15 cm
7 cm
7 cm
Remaining
solid

D O′ C 104 cm
90 cm
Total surface area of the remaining solid
= Curved surface area of the cylinder
+ Area of the base of the cylinder
7 cm
+ Curved surface area of the cone
7 cm
= 2 πrh + πr 2 + πrl
136 CBSE Term II Mathematics X (Basic)

Then, the cost of polishing at the rate of ` 2 per dm 2 15. Given, length of roof = 22 m and breadth of roof = 20 m
4576 × 2
=` = ` 91.52 [Q1 dm 2 = 100 cm 2 ] Let the rainfall be a cm.
100 a 22a 3
∴Volume of water on the roof = 22 × 20 × = m
12. Given, volume of cylinder = 12 × volume of toy 100 5
⇒ Volume of cylinder = 12 (volume of cone+ volume of Also, we have radius of base of the cylindrical vessel, r = 1 m
hemisphere) and height of the cylindrical vessel, h = 3.5 m
1 2 3 ∴Volume of water in the cylindrical vessel when it is just full
⇒ πr h = 12 ×  × πr1 h + πr1 
2 2
3 3   22 7
= πr 2 h =  × 1 × 1 ×  = 11 m3
[Q radius of cone = radius of hemisphere]  7 2
12 2 Now, volume of water on the roof = Volume of water in
⇒ r h = r12 × 3r + 12 × × r13
2
3 3 the vessel
⇒ 6 × 6 × 15 = 20r13 22 a
⇒ = 11
⇒ r13 = 27 ⇒ r1 = 3 cm 5
11 × 5
Now, h = 3r = 3 × 3 = 9 cm ∴ a= = 2.5 cm
Radius of the hemisphere = 3 cm 22
Total height of the toy = 9 + 3 = 12 cm [Q volume of cylinder=π × (radius) 2 × height]
13. Given, dimensions of base of rectangular tank = 11 m × 6 m Hence, the rainfall is 2.5 cm.
and height of water = 5 m 16. Here, sand in cylindrical bucket empties in the shape of
conical heap. It means volume of cylinder is equal to the
Volume of the water in rectangular tank = 11 × 6 × 5
volume of cone.
= 330 m3
Now, volume of cylinder = πr 2 h = π (18)2 × 32
Also, given radius of the cylindrical tank = 3.5 m
1 1
Let height of water level in cylindrical tank be h. and volume of cone = πr 2 h = × πr 2 × 24
3 3
Then, volume of the water in cylindrical tank = πr 2 h
According to the given condition,
= π (3.5)2 × h
Volume of cylinder = Volume of cone
22
= × 3.5 × 3.5 × h = 110
. × 3.5 × h = 38. 5 h m3 ∴
1
π (18)2 × 32 = π × r 2 × 24
7 3
According to the question, ⇒ (18)2 × 3 × 32 = r 2 × 24
330 = 38.5 h
324 × 3 × 32
[since, volume of water is same in both tanks] ⇒ r2 = = 1296 ⇒ r = 36 cm
330 3300 24
∴ h= =
38. 5 385 Now, slant height of cone, l = r 2 + h 2
∴ = 8. 57 m or 8.6 m = ( 36)2 + ( 24)2 = 1296 + 576 = 1872 = 43.3 cm
Hence, the height of water level in cylindrical tank is 8.6 m.
17. Given, length of the barrel of a fountain pen = 7 cm
14. Here, a rod of cylindrical shape is converted into a wire of
5 1  1 
cylindrical shape. and diameter = 5 mm = cm = cm Q1 mm = cm
10 2  10 
For rod,
1 1
Diameter = 1 cm ⇒ Radius, r1 = cm and length, ∴ Radius of the barrel = = 0.25 cm
2 2×2
h = 8 cm
2 Volume of the barrel = πr 2 h [since, its shape is cylindrical]
1
∴ Volume of the rod = πr12 h = π ×   × 8 = 2 π cm3 22
 2 = × (0.25)2 × 7
7
For wire,
= 22 × 0.0625 = 1.375 cm3
Length = 8 m = 800 cm [Q1m = 100cm]
1
Let r be the radius (in cm) of cross-section of the wire, then Also, given volume of ink in the bottle = of litre
Volume of wire = π × r 2 × 800 cm 3 5
1
Since, the rod is converted into wire, so = × 1000 cm = 200 cm3
3

Volume of wire = Volume of rod 5


1 1 Now,1.375 cm3 ink is used for writing number of words
⇒ π × r 2 × 800 = 2 π ⇒ r 2 = ⇒r = cm
400 20 = 3300
2 1 3300
Then, diameter = = cm ∴1 cm ink is used for writing number of words =
3
20 10 1.375
Hence, the diameter of the cross-section, i.e. the thickness ∴200 cm3 ink is used for writing number of words
1 3300
of the wire is cm, i.e. 1 mm [Q1cm = 10 mm] = × 200 = 480000
10 1.375
CBSE Term II Mathematics X (Basic) 137

18. Given, speed of water flow = 10 m min −1 = 1000 cm/min Now, cost of colouring 1 dm2 curved surface of the pencils
5  1  manufactured in one day = ` 0.05
and diameter of the pipe = 5 mm = cm Q1 mm = cm ∴Cost of colouring 45000 dm2 curved surface
10  10 
5 = 45000 × 005
. = ` 2250
∴ Radius of the pipe = = 0.25 cm 21. Here, a well is dug and Earth taken out of it is used to form
10 × 2
an embankment.
∴ Area of the face of pipe Given, Diameter of well = 10 m
22 10
= πr 2 = × (0.25)2 = 0.1964 cm2 ∴ Radius = = 5m
7 2
Also, given diameter of the conical vessel = 40 cm Also, depth = 14 m
40 ∴Volume of Earth taken out on digging the well
∴Radius of the conical vessel = = 20 cm
2 22
= πr 2 h = × ( 5)2 ×14
and depth of the conical vessel = 24 cm 7
1 1 22 = 1100 m 3
∴Volume of conical vessel = πr 2 h = × × ( 20)2 × 24
3 3 7
211200 10 m
= = 1005714
. cm3
21 5m 5m
Volume of the conical vessel h
∴Required time =
Area of the face of pipe × Speed of water
10057.14
=

14 m
0.1964 × 1000
20
= 51.20 min = 51 min × 60 s
100
= 51 min 12 s
19. Given, radius of tank, r1 = 40 cm
Let height of water level in tank in half an hour = h1 The embankment is in the form of cylindrical shell, so area
of embankment = π(R 2 − r 2 ) = π(102 − 52 )
Also, given internal radius of cylindrical pipe, r2 = 1 cm
22
and speed of water = 80 cm/s i.e. in 1s water flow = 80 cm = π (100 − 25) = × 75 m 2
7
∴ In 30 (min) water flow = 80 × 60 × 30 = 144000 cm
Since, Earth taken out from well is used to form
According to the question,
embankment.
Volume of water in cylindrical tank = Volume of water flow
∴ Volume of embankment
from the circular pipe in half an hour
= Volume of Earth taken out on digging the well
⇒ π r12 h1 = π r22 h 2
⇒ Area of embankment × Height of embankment
⇒ 40 × 40 × h1 = 1 × 1 × 144000
= Volume of Earth dugout
144000
∴ h1 = = 90 cm Volume of Earth dugout
40 × 40 ⇒ Height of embankment =
Area of the embankment
Hence, the level of water in cylindrical tank rises 90 cm in 1100
half an hour. = = 467
. m
22
20. Given, pencils are cylindrical in shape. × 75
7
Length of one pencil = 25 cm 22. Dimension of cuboid 24 cm × 11 cm × 7 cm
and circumference of base, 2πr =1.5 cm Length of cuboid ( l ) = 24 cm
1.5 × 7
⇒ r= = 0.2386 cm Breadth of cuboid ( b ) = 11 cm
22 × 2 Height of cuboid ( h) = 7 cm
Now, curved surface area of one pencil = 2πrh Volume of cuboid = l × b × h = 24 × 11 × 7 = 1848 cm3
22
=2× × 0. 2386 × 25 Radius of cone = 3.5 cm and height of cone = 6 cm
7
1 1 22
262.46 ∴ Volume of cone = πr 2 h = × × 3.5 × 3.5 × 6
= = 37.49 cm2 3 3 7
7
37.49  1  Volume of cone = 77 cm3
= dm2 Q 1 cm = dm
100  10  Thus, required number of cones =
Volume of cuboid
Volume of 1 cone
= 0.375 dm2
1848
∴Curved surface area of 120000 pencils = 0.375 × 120000 Total number of cones = = 24
77
= 45000 dm2
Hence, the total number of cones 24.
138 CBSE Term II Mathematics X (Basic)

23. Given, edge of the cube = 22 cm ∴ Radius of building (or dome) =


2r
=rm
∴ Volume of the cube = ( 22 ) = 10648 cm
3 3 2
[Q volume of cube = (side)3 ] Height of cylinder = 2r − r = r m
∴ Volume of the cylinder = π r 2 ( r ) = πr 3 m3
Also, given diameter of marble = 0.5 cm
2
0.5 and volume of hemispherical dome on cylinder = πr 3 m3
∴ Radius of a marble, r = = 0. 25 cm 3
2
[Q diameter =2 × radius] ∴ Total volume of the building = Volume of the cylinder
4 3 4 22 + Volume of hemispherical dome
Volume of one marble = πr = × × (0.25)3
3 3 7  3 2 3 3 5 3 3
=  πr + πr  m = πr m
4  3  3
[Q volume of sphere = × π × (radius)3 ]
3 According to the question,
1.375
= = 00655
. cm 3
Volume of the building = Volume of the air
21 5 3 19
1 ⇒ πr = 41
Filled space of cube = Volume of the cube − 3 21
8 5 3 880
× Volume of cube ⇒ πr =
3 21
1 7
= 10648 − 10648 × = 10648 × 880 × 7 × 3 40 × 21
8 8 ⇒ r3 = = =8
21 × 22 × 5 21 × 5
= 9317 cm3
⇒ r3 = 8 ⇒ r = 2 m
∴ Required number of marbles
Total space filled by marbles in a cube ∴ Height of the building = 2 r = 2 × 2 = 4 m
= 26. Given, diameter of cylinder = Diameter of hemisphere
Volume of one marble
= 0.5 cm
9317
= = 142244 (approx.) [since, both hemispheres are attach with cylinder]
0.0655
∴ Radius of cylinder ( r ) = radius of hemisphere
Hence, the number of marbles that the cube can accomodate 0.5
is 142244. (r) = = 0.25 cm
2
24. Given a hollow sphere of external and internal diameters [Q diameter = 2 × radius]
8 cm and 4 cm.
∴External and internal radius are 4 cm and 2 cm.
4 4 0.5 cm
Now, volume of sphere = π ( 4)3 − π ( 2 )3 0.25 0.25
3 3
4 4
= π[( 4)3 − ( 2 )3 ] = π[64 − 8] 2 cm
3 3 and total length of capsule = 2 cm
4π 224π ∴Length of cylindrical part of capsule,
= × 56 = cm3
3 3 h = Length of capsule − Radius of both hemispheres
After it is melted to cone, volume will remain same. = 2 − (0.25 + 0.25) = 1.5 cm
Now, capacity of capsule = Volume of cylindrical part
Let h be the height of cone and radius of base = 4 cm
+ 2 × Volume of hemisphere
1 1 16π 2
∴ Volume of cone = πr 2 h = π ( 4)2 h = h = πr 2 h + 2 × πr 3
3 3 3 3
16πh 224π [Q volume of cylinder = π × (radius)2
Now, = ⇒ h = 14 cm
3 3 2
× height and volume of hemisphere = π (radius)3 ]
Hence, height of the cone is 14 cm. 3
25. Let total height of the building = Internal diameter of the =
22 4
[(0.25) × 1.5 + × (0.25) ]
2 3
dome = 2r m 7 3
22
= [0.09375 + 0.0208]
7
r 22
= × 011455
. = 0.36 cm3
r r 7
Hence, the capacity of capsule is 0.36 cm 3 .
2r
27. Since, rocket is the combination of a right circular cylinder
r and a cone.
Given, diameter of the cylinder = 6 cm
6
∴ Radius of the cylinder = = 3 cm
2
CBSE Term II Mathematics X (Basic) 139

and height of the cylinder = 12 cm Then, r1 = 3.5 cm [Q r = r1 ]


∴ Volume of the cylinder = πr 2 h = 3.14 × ( 3)2 × 12 Now, the total surface area of the toy
= 339.12 cm3 = curved surface area of hemisphere
and curved surface area = 2πrh + curved surface area of cone
= 2 × 3.14 × 3 × 12 = 226.08 = 2 πr12 + πrl = 2 πr 2 + πrl [Q r1 = r]
A
= πr[2 r + l]
22
= × 3.5[2 × 3.5 + 12.5] = 11(7 + 12.5)
5 cm 7
h = 11 × 19.5 = 214.5cm2
29. Here, the given wooden toy rocket is combination of a cone
and a cylinder.
O 3 cm
B C For conical portion,
Diameter = 6 cm
6
12 cm

∴ Radius, r1 = cm = 3 cm
2
Height, h1 = 4 cm
Then, slant height, l = ( 3)2 + 42 [Q l = r 2 + h 2 ]
E D
= 9 + 16 = 25 = 5cm
6 cm
For cylindrical portion,
Now, in right angled ∆AOC, Diameter = 4 cm
h = 52 − 32 4
∴ Radius, r2 = cm = 2 cm
2
= 25 − 9 = 16 = 4
Height, h 2 = Total height of rocket − Height of cone
∴Height of the cone, h = 4 cm
= 24 − 4 [Q total height of rocket = 24 cm]
and radius of the cone, r = 3 cm
= 20 cm
Now, volume of the cone
1 1 Here, we have to find the area of the rocket painted with
= πr 2 h = × 3.14 × ( 3)2 × 4 orange and yellow colours separately.
3 3
Since, radius of base of cone is larger than radius of base of
113.04
= = 37.68 cm3 cylinder and cone is mounted on cylinder.
3 ∴ Area to be painted orange = Curved surface area of cone
and curved surface area = πrl = 314
. × 3 × 5 = 471
.
+ Area of base of cone − Area of base of cylinder
Hence, total volume of the rocket
[Q area of base of cylinder is common
= 339.12 + 37.68 = 376.8 cm3 in area of base of cone]
and total surface area of the rocket = πr1 l + πr12 − πr22
= CSA of cone + CSA of cylinder + Area of base of cylinder = 3.14 × 3 × 5 + 3.14 × (3)2 − 3.14 × (2)2
= 471
. + 22608
. + 28. 26
= 3.14 [15 + 9 −4]
= 301.44 cm 2 = 3.14 × 20 = 62.8 cm2
28. Let r cm be the radius, h cm be the height and l cm be the Now, area to be painted yellow
slant height of the cone, then r = 3.5 cm,
= Curved surface area of cylinder
h = (15.5 − 3.5) =12 cm
+ Area of base of the cylinder = 2 πr2 h 2 + π r2 2
∴ l = r 2 + h 2 = ( 3.5)2 + (12 )2 = 156.25 =12.5 cm
= 2 × 3.14 × 2 × 20 + 3.14 × (2)2
O
= 3.14 [80 + 4]
= 3.14 × 84 = 263.76 cm2
30. Let volume of cone A be 2 V and volume of cone B be V.
Again, let height of the cone A = h1 cm, then height of cone
15.5 cm
B = ( 21 − h1 ) cm
21 cm
6 cm
6 cm

A B
3.5 cm

Let r1 cm be the radius of the hemisphere. h1 21 – h1


140 CBSE Term II Mathematics X (Basic)

Given, diameter of the cone = 6 cm From Eq. (i),


6 44 × 44 × 44
∴ Radius of the cone = = 3 cm Number of spherical lead shots = × 21
2 4 × 22 × 8
1 1
Now, volume of the cone, A = 2 V = πr 2 h = π( 3)2 h1 = 11 × 21 × 11
3 3 = 121 × 21
1 3 = 2541
⇒ V = π 9 h1 = h1 π …(i)
6 2 Hence, the required number of spherical lead shots is 2541.
and volume of the cone, 32. Let h be height of cone and r be the radius of base.
1
B = V = π ( 3)2 ( 21 − h1 ) = 3π ( 21 − h1 ) …(ii) ∴ h = 24 cm r = 6 cm
3
From Eqs. (i) and (ii), Let R be the radius of sphere.
3
h1 π = 3π ( 21 − h1 )
2
⇒ h1 = 2 ( 21 − h1 )
h
⇒ 3h1 = 42 R
42
⇒ h1 = = 14 cm
3 r
∴ Height of cone, B = 21 − h1 = 21 − 14 = 7 cm
22 1 2 1
Now, volume of the cone, A = 3 × 14 × = 132 cm3 Volume of cone = πr h = π (6)2 ( 24) …(i)
7 3 3
[using Eq. (i)] After reshaping it into a sphere, the volume will remain
1 22 same.
and volume of the cone, B = × × 9 × 7 = 66 cm3
3 7 4
Now, volume of sphere = π(R )3 …(ii)
[using Eq. (ii)] 3
22 2 ∴ Now, Eq. (i) = Eq. (ii)
Now, volume of the cylinder = πr h =2
( 3) × 21
7 1 4
∴ π (6)2 ( 24) = πR 3
= 594 cm3 3 3
∴ Required volume of the remaining portion 62 × 24
⇒ R =
3

= Volume of the cylinder 4


− (Volume of cone A + Volume of cone B) = 62 × 6 = 63
= 594 − (132 + 66) = 396 cm3 ⇒ R =6
31. Given that, lots of spherical lead shots made out of a solid Hence, the radius of sphere is 6 cm.
cube of lead. 33. Given that, lots of spherical lead shots made from a solid
∴ Number of spherical lead shots rectangular lead piece.
Volume of a solid cube of lead ∴ Number of spherical lead shots
= ...(i) Volume of solid rectangular lead piece
Volume of a spherical lead shot = ...(i)
Given that, diameter of a spherical lead shot i.e. sphere = 4 cm Volume of a spherical lead shot
4 Also, given that diameter of a spherical lead shot
⇒ Radius of a spherical lead shot (r) = i.e. sphere = 4. 2 cm
2
4.2
r = 2 cm [Q diameter = 2 × radius] ∴ Radius of a spherical lead shot, r = = 21
. cm
So, volume of a spherical lead shot i.e. sphere 2
 1 
4
= π r3 Q radius = diameter
 2 
3
4 22 So, volume of a spherical lead shot i.e. sphere
= × × ( 2 )3
3 7 4
= π r3
4 × 22 × 8 3
= cm 3
21 4 22
= × × ( 21
. )3
Now, since edge of a solid cube (a ) = 44 cm 3 7
So, volume of a solid cube = (a )3 4 22
= × × 2.1 × 2.1 × 2.1
3 7
= ( 44)3
4 × 22 × 21 × 21 × 21
= 44 × 44 × 44 cm 3 =
3 × 7 × 1000
CBSE Term II Mathematics X (Basic) 141

Now, length of rectangular lead piece, l = 66 cm So, volume of rice =


1
π × r2h
Breadth of rectangular lead piece, b = 42 cm 3
Height of rectangular lead piece, h = 21 cm 1 22 9 9
= × × × × 3. 5
∴Volume of a solid rectangular lead piece i.e. cuboid 3 7 2 2
=l×b×h 6237
= = 74. 25 m 3
= 66 × 42 × 21 84
From Eq. (i), Now, canvas cloth required to just cover heap of rice
Number of spherical lead shots = Surface area of a heap of rice
66 × 42 × 21 = πrl
= × 3 × 7 × 1000 22
4 × 22 × 21 × 21 × 21 = × r × r 2 + h2
3 × 22 × 21 × 2 × 21 × 21 × 1000 7
= 2
4 × 22 × 21 × 21 × 21 22 9  9
= × ×   + ( 3.5)2
7 2  2
= 3 × 2 × 250
= 6 × 250 = 1500 11 × 9 81
= × + 12. 25
Hence, the required number of spherical lead shots is 1500. 7 4
34. Given that, lots of metallic circular disc to be melted to form 99 130 99
a right circular cylinder. Here, a circular disc work as a = × = × 32 . 5
7 4 7
circular cylinder.
= 14142
. × 5.7 = 80.61 m 2
Base diameter of metallic circular disc = 1. 5 cm
1. 5 Hence, 80.61 m 2 canvas cloth is required to just cover heap.
∴ Radius of metallic circular disc = cm
2 36. Let the length(l), breadth (b) and height (h) be the external
[Q diameter = 2 × radius] dimension of an open box and thickness be x.
and height of metallic circular disc i.e. = 0.2 cm
∴Volume of a circular disc = π × (Radius)2 × Height
2
1. 5 x x
= π ×   × 0. 2
 2
π x
= × 1. 5 × 1. 5 × 0. 2
4
Now, height of a right circular cylinder ( h) = 10 cm Given that,
and diameter of a right circular cylinder = 4. 5 cm external length of an open box ( l ) = 36 cm
4. 5 external breadth of an open box ( b ) = 25 cm
⇒ Radius of a right circular cylinder ( r ) = cm
2 and external height of an open box ( h) = 16.5 cm
∴ Volume of right circular cylinder = π r 2 h ∴ External volume of an open box = lbh
2 = 36 × 25 × 16. 5 = 14850 cm3
 4.5
= π   × 10 Since, the thickness of the iron ( x ) = 1. 5 cm
 2 
π So, internal length of an open box ( l1 ) = l − 2 x
= × 4. 5 × 4. 5 × 10 = 36 − 2 × 1. 5
4
∴ Number of metallic circular disc = 36 − 3 = 33 cm
Volume of a right circular cylinder Therefore, internal breadth of an open box ( b 2 ) = b − 2 x
=
Volume of a metallic circular disc = 25 − 2 × 1. 5
π = 25 − 3 = 22 cm
× 4. 5 × 4. 5 × 10
= 4 and internal height of an open box ( h 2 ) = ( h − x )
π = 16. 5 − 1. 5 = 15 cm
× 1. 5 × 1. 5 × 0. 2
4 So, internal volume of an open box
3 × 3 × 10 900 = ( l − 2 x) ⋅ ( b − 2 x) ⋅ ( h − x)
= = = 450
0. 2 2 = 33 × 22 × 15 = 10890 cm3
Hence, the required number of metallic circular disc is 450. Therefore, required iron to construct an open box
35. Given that, a heap of rice is in the form of a cone. = External volume of an open box
Height of a heap of rice i.e. cone ( h) = 3. 5 m − Internal volume of an open box
and diameter of a heap of rice i.e. cone = 9 m = 14850 − 10890 = 3960 cm 3
9
Radius of a heap of rice i.e. cone ( r ) = m Hence, required iron to construct an open box is 3960 cm 3 .
2
142 CBSE Term II Mathematics X (Basic)

7. 5
Given that, 1 cm 3 of iron weights = 7 . 5 g =
kg ⇒ L2 = 7 × 7 + 24 × 24
1000 ⇒ L2 = 49 + 576 = 625
= 00075
. kg
∴ L = 25 cm
∴ 3960 cm of iron weights = 3960 × 00075
3
. = 297. kg
22
C.S.A. of cone = πrl = × 7 × 25 = 550 cm2
37. Canal is the shape of cuboid, where 7
Breadth = 6 m, Height = 1.5 m C.S. A of the cylinder 2640
⇒ = = 24 : 5
C.S. A. of the cone 550
Sp (iii) (b) Given structure is based on the concept of surface
eed
= area and volume.
10
7 21
39. (i) Given, r = cm, h = 10.5 cm = cm
h = 1.5 cm 2 2
Volume of cylindrical cup = πr 2 h
b = 6 cm
22 7 7 21
Speed of canal = 10 km/h = × × ×
7 2 2 2
Length of canal in 1 h = 10 km 11 × 7 × 21
=
Length of canal in 60 min = 10 km 4
1 = 404.25cm3
Length of canal in 1 min = × 10 km
60 7
30 (ii) Given, r = cm
Length of canal in 30 min = × 10 km = 5 km = 5000 m 2
60 2 22 7 7 7
Now, Volume of canal = Length × Breadth × Height ∴ Volume of hemispherical cup = × × × ×
3 7 2 2 2
= ( 5000 × 6 × 1.5) m3
= 89.83 cm3
Now, Volume of water in canal = Volume of area irrigated
(iii) Area of canvas provided = 551 m2
Volume of water in canal = Area irrigated × Height
Area of remained after westage = 551 − 1 = 550 m2
( 5000 × 6 × 1.5) m3 = Area irrigated × 4 cm
4 So, area of conical tent = πrl
⇒ 5000 × 6 × 1.5 = Area irrigated × m Here, r = 7 m
100
22
5000 × 6 × 1.5 × 100 ∴ πrl = 550 ⇒ × 7 × l = 550 ⇒ l = 25 m
⇒ = Area irrigated 7
4
⇒ Area irrigated = 1125000 m2 Now, h = l 2 − r 2 = 625 − 49
1125000 = 576 = 24 m
⇒ Area irrigated = hec [∴10000m2 = 1 hec]
10000 40. (i) Given, edge of cube = 6 cm
⇒ Area irrigated = 112.50 hec ∴Diagonal of cube = 3 × edge of cube
38. (i) Length of the cuboid (L ) = 30 cm = 3 × 6 = 6 3 cm
Breadth of the cuboid (B) = 20 cm
(ii) Given edge of cube = 7 cm
Height of the cuboid (H) = 10 cm
∴Volume of cube = 7 × 7 × 7 = 343 cm3
Then,
Area covered by the cuboid (iii) Given, radius ( r ) = 7 cm
= L × B + 2H(L + B) ∴Curved surface area of hemisphere = 2 πr 2
= 30 × 20 + 2 × 10( 30 + 20) 22
=2× ×7 ×7
= 600 + 1000 7
= 1600 cm2 = 44 × 7 = 308 cm2
(ii) We know that, (iv) Given, radius ( r ) = 7 cm and height ( h) = 24 cm
C.S.A of the cylinder = 2 πRH1 Slant height ( l ) = ?
22 ∴ l 2 = r 2 + h 2 = (7 )2 + ( 24)2
C.S.A of the cylinder = 2 × × 7 × 60
7 = 49 + 576 = 625
= 2 × 22 × 60 ⇒ l = 625 = 25 cm
= 2640 cm2 (v) TSA of cone = πrl + 2 πr 2
In cone, height is given and radius of the cone is equal to 22 22
radius of the cylinder, = × 7 × 25 + 2 × ×7 ×7
7 7
Q L2 = H22 + R 2 = 550 + 308 = 858 cm2
Chapter Test
Multiple Choice Questions (iii) The curved surface area of the conical cavity
1. How many cubes of side 2 cm can be made from a so formed is
(a) 2250 cm 2 (b) 2200 cm 2
solid cube of side 10 cm?
(c) 1800 cm 2 (d) 2400 cm 2
(a) 100 (b) 125 (c) 175 (d) 200
3 (iv) External curved surface area of the cylinder
2. 2 cubes, each of volume 125 cm , are joined end to is
end. Find the surface area of the resulting cuboid.
(a) 2876 cm 2 (b) 1250 cm 2
(a) 100 cm 2 (b) 200 cm 2 (c) 225 cm 2 (d) 250 cm 2 (c) 4224 cm 2 (d) 3824 cm 2
3. The radius of a sphere (in cm) whose volume is (v) Volume of conical cavity is
12π cm 3 , is (a) 6232 cm 3
2/ 3 1/ 3
(a) 3 (b) 3 3 (c) 3 (d) 3 (b) 7248 cm 3
4. A solid spherical ball fits exactly inside the cubical box (c) 5380 cm 3
of side 2a. The volume of the ball is (d) 9856 cm 3
16 1 3 Short Answer Type Questions
(a) πa 3 (b) πa
3 6 7. A hemispherical depression is cut out from one face of
32 3 4
(c) πa (d) πa 3 a cuboidal block of side 7 cm such that the diameter of
3 3
the hemisphere is equal to the edge of the cube. Find
5. A cone and a cylinder have the same radii but the the surface area of the remaining solid.
height of the cone is 3 times that of the cylinder, then
the ratio of their volumes.
8. The capacity of a cylindrical glass tumbler is 125.6 cm 3 .
If the radius of the glass tumbler is 2 cm, then find its
(a) 1 : 2 (b) 2 : 1
height. (Use π = 3.14)
(c) 1 : 1 (d) None of these
9. In given figure, a solid toy is in the form of a
Case Based MCQs
hemisphere surmounted by a right circular cone. The
6. One day Aakash was going home from market saw a height of the cone is 2 cm and the diameter of the
carpenter working on wood. He found that he is base is 4 cm. Determine the volume of the toy.
carving out a cone of same height and same diameter  22 
take π =
from a cylinder. The height of the cylinder is 48 cm and  7 
base radius is 14 cm. While watching this, some
questions came into Aakash’s mind. Help Aakash to
find the answer of the following questions. 2 cm

14 cm
2 cm
48 cm

Long Answer Type Questions


10. From a solid cylinder whose height is 2.4 cm and
(i) After carving out cone from the cylinder, diameter 1.4 cm, a conical cavity of the same height
(a) Volume of the cylindrical wood will decrease and same diameter is hollowed out. Find the total
(b) Height of the cylindrical wood will increase surface area of the remaining solid to the nearest cm 2 .
(c) Volume of cylindrical wood will increase 11. A well of diameter 3 m is dug 14 m deep. The earth
(d) Radius of the cylindrical wood will decrease taken out of it has been spread evenly all around it in
(ii) Find the slant height of the conical cavity so the shape of a circular ring of width 4 m to form a
formed. platform. Find the height of the platform.
(a) 28 cm (b) 60 cm  22 
take π =
(c) 40 cm (d) 50 cm  7 

Answers
1. (b) 2. (d) 3. (c) 4. (d) 5. (c) 6. (i) (a) (ii) (d) (iii) (b) (iv) (c) (v) (d) For Detailed Solutions
7. 332.465 cm 2 8. 10 cm 9. 25.12 cm 3 10. 18 cm 2 11. 1.125 m Scan the code
144 CBSE Term II Mathematics X (Basic)

CHAPTER 07

Statistics

In this Chapter...
l Mean of Grouped Data
l Mode of Grouped Data
l Median of Grouped Data

Arithmetic Mean or Mean or Average In general, for the ith class interval, we have frequency fi
corresponding to the class mark xi . The sum of the values in
The arithmetic mean of a set of observations is obtained by
the last column gives us Σfi xi , so the mean x of the given data
dividing the sum of the values of all observations by the total
is given by
number of observations.
Σf x
Thus, the mean of n observations x1 , x 2 , x 3 , K , xn , is x = i i.
defined as Σfi
n
∑ xi 2. Assumed Mean Method
x + x 2 + x 3 + … + xn i = 1
Mean ( x ) = 1 = The cases, in which numerical values of xi and fi are large and
n n
computation of product of xi and fi becomes tedious and time
where, the Greek letter ‘Σ’ (sigma) means ‘Summation.’ consuming, assumed mean method is used. In this method,
Let x1 , x 2 , ... , xn be n observations with respective first of all, one among xi ’s is chosen as the assumed mean
frequencies f1 , f2 , ..., fn . This means observation x1 occurs f1 denoted by ‘a’. After that, the difference di between a and
times, x 2 occurs f2 times and so on. each of the xi ’ s, i.e. d i = xi − a is calculated.
n
∑ fi xi Then, arithmetic mean is given by
i=1 Σ fi xi Σfid i
∴ Mean ( x ) = = x =a+
n Σ fi Σfi
∑ fi
i=1 where, d i = xi − a
Method of Calculating Mode
Mean of Grouped Data The observation, which occurs most frequently among the
1. Direct Method
given observations, i.e. the value of the observation having
maximum frequency is called mode.
In this method, we find the class marks of each class
interval. These class marks would serve as the e.g. Mode of the numbers 2, 3, 4, 4, 6, 6, 6, 6, 7 and 9 is 6
representative of whole class and are represented by xi . because it is repeated maximum number of times, i.e. 4 times.
CBSE Term II Mathematics X (Basic) 145

Modal Class Case II If n is even, then


In a grouped frequency distribution, it is not possible to  n n 
Median = Mean of value of   th and  + 1 th
determine the mode by looking at the frequencies. So, here  2 2 
we first locate a class with the maximum frequency. This class observations
is called modal class. e.g.
1  n  n  
= × Value of   th +  + 1 th
Class interval 0-10 10-20 20-30 30-40 40-50 50-60 2  2  2  
Number of students 2 9 14 20 22 8 observations
e.g. If six girls of different heights are made to stand
Here, the highest frequency is of the class 40-50, which is 22. in a row, in descending order of their heights, then
Hence, the modal class is 40-50.
the mean height of third and fourth girl from either
Mode of Grouped Data end is the median height.
In grouped data, mode is a value that lies in the modal class Since, n = 6 is even.
n 6
and it is given by the formula, So, = = 3rd observation
 f1 − f0  2 2
Mode = l +  ×h n  6 6+2
 2 f1 − f0 − f2  and  + 1 = + 1 = = 4th observation
2  2 2
where, l = lower limit of the modal class ∴ Median = Mean of 3rd and 4th observations
h = size of the class intervals
(assuming all class sizes to be equal) Cumulative Frequency
f1 = frequency of the modal class The frequency of an observation in a data refers to how many
f0 = frequency of the class preceding the modal class times that observation occur in the data. Cumulative
frequency of a class is defined as the sum of all frequencies
f2 = frequency of the class succeeding the modal class upto the given class.
Less than type and more than type. Formation of these two
Median distributions can be understood with the help of following
Median is defined as the middle-most or the central example.
observation, when the observations are arranged either in e.g. Consider a grouped frequency distribution of marks
ascending or descending order of their magnitudes. obtained out of 100, by 58 students, in a certain examination,
Median divides the arranged series into two equal parts, i.e. as follows:
50% of the observations lie below the median and the
remaining are above the median. Marks Number of students
Let n be the total number of observations and suppose that 0-10 5
they are arranged in ascending or descending order. 10-20 7
Median of the data depends on the number of 20-30 4
observations (n). 30-40 2
Case I If n is odd, then
40-50 3
 n + 1
Median = Value of   th observation 50-60 6
 2 
60-70 7
e.g. If five girls of different heights are made to
70-80 9
stand in a row, in descending order of their heights,
80-90 8
then the height of the third girl from either end is
90-100 7
median height.
Since, n = 5 is odd. Cumulative frequency distribution of the less than type
 n + 1 5 +1 Here, the number of students who have scored marks less
∴ Median =   th observation = than 10 are 5. The number of students who have scored
 2  2
marks less than 20 includes the number of students who have
6
= = 3rd observation scored marks from 0-10 as well as the number of students
2 who have scored marks from 10-20.
146 CBSE Term II Mathematics X (Basic)

Thus, the total number of students with marks less than 20 is Let n be the total number of observations (sum of
5 + 7 , i.e. 12. So, the cumulative frequency of the class 10-20 frequencies), then median of the data depends on the number
is 12. of observations ( n ).
Similarly, on computing the cumulative frequencies of the  n + 1
If n is odd, then Median = Value of   th observation.
other classes, which is shown in the table.  2 
Number of students If n is even, then
Marks obtained
(cumulative frequency)  n n 
Median = Mean of   th and  + 1 th observations
Less than 10 5  2 2 
Less than 20 5 + 7 = 12  n 
1 n  
12 + 4 = 16 = × Value of   th +  + 1 th observations
Less than 30 2  2  2  
Less than 40 16 + 2 = 18
Here, for the value of observation, first look at the cumulative
Less than 50 18 + 3 = 21
frequency just greater than (and nearest to) the position of
Less than 60 21 + 6 = 27 required observations. Then, determine the corresponding
Less than 70 27 + 7 = 34 value of the observation.
Less than 80 34 + 9 = 43
Median for Grouped Data
Less than 90 43 + 8 = 51
Less than 100 51 + 7 = 58 In a grouped data, we may not find the middle observation by
looking at the cumulative frequencies, since the middle
Cumulative frequency distribution of the more than type observation will be some value in a class interval, so it is
For this type of distribution, we make the table for the necessary to find the value inside a class that divides the
number of students with scores, more than or equal to 0, whole distribution into two halves.
more than or equal to 10, more than or equal to 20 and so on. For this, we find the cumulative frequencies of all the classes
From the example, we observed that all 58 students have n
scored marks more than or equal to 0. and then determine , where n = number of observations.
2
There are 5 students scoring marks in the interval 0-10, it Now, locate the class whose cumulative frequency is greater
shows that there are 58 − 5 = 53 students getting more than or n
than (i.e. nearest to) and this class is called median class.
equal to 10 marks. In the same manner, the number of 2
students scoring 20 marks or above = 53 − 7 = 46 students, After finding the median class, use the following formula for
and so on. calculating the median.
Number of students N 
Marks obtained − cf
(cumulative frequency) 2 
Median = l +  ×h
More than or equal to 0 58
 f 
More than or equal to 10 58 − 5 = 53  
More than or equal to 20 53 − 7 = 46
where, l = lower limit of median class
More than or equal to 30 46 − 4 = 42
N = sum of frequencies
More than or equal to 40 42 − 2 = 40
cf = cumulative frequency of the class preceding
More than or equal to 50 40 − 3 = 37 the median class
More than or equal to 60 37 − 6 = 31
f = frequency of the median class
More than or equal to 70 31 − 7 = 24
h = class width (assuming class sizes to be equal)
More than or equal to 80 24 − 9 = 15
More than or equal to 90 15 − 8 = 7 Relationship among Mean,
Median and Mode
Median for Discrete Series There is an empirical relationship among the three measures
A series having observations x1 , x 2 , x 3 , K , xn with respective of central tendency, which is given by
frequencies f1 , f2 , f3 K , fn is known as discrete series. Mode = 3(Median) − 2(Mean)
3 (Median) − Mode
Method to Find the Median of the Discrete Series or Mean =
2
Firstly, we arrange the data in the ascending or descending Mode + 2 (Mean)
order of xi , then we find the cumulative frequencies of all the or Median =
observations. 3
CBSE Term II Mathematics X (Basic) 147

Solved Examples
Example 1. Find the mean of the following data. Sol. The table for given data is

x 10 30 50 70 89 Classes Frequency (fi ) Mid-value ( xi ) fi xi

f 7 8 10 15 10 2-4 6 3 18

Sol. Table for the given data is 4-6 8 5 40

xi fi fi xi 6-8 15 7 105

10 7 70 8-10 p 9 9p
30 8 240 10-12 8 11 88
50 10 500 12-14 4 13 52
70 15 1050 Σ fi xi
Σfi = p + 41
89 10 890 = 9p + 303
Total Σ fi = 50 Σ fi xi = 2750 Given, mean = 7.5
Σfi xi 9p + 303
Here, Σfi = 50 and Σfi xi = 2750 ∴ = 7. 5 ⇒ = 7. 5
Σfi p + 41
Σfi xi
∴ Mean ( x ) = ⇒ 9p + 303 = 7.5p + 307. 5
Σfi
⇒ 9p − 7.5p = 307.5 − 303
2750
=
= 55 ⇒ 1. 5p = 4. 5
50 4.5
Hence, mean of the given data is 55. ⇒ p= =3
1.5
Example 2. Calculate the mean of the scores of 20 Hence, value of p is 3.
students in a mathematics test
Example 4. The weights of tea in 70 packets are shown
Marks 10-20 20-30 30-40 40-50 50-60 in the following table
Number of students 2 4 7 6 1 Weight (in gm) Number of packets
200-201 13
Sol. We first, find the class marks xi of each class and then
proceed as follows 201-202 27
202-203 18
Marks Class marks ( x i ) Frequency ( fi ) fi xi
203-204 10
10-20 15 2 30 204-205 1
20-30 25 4 100 205-206 1
30-40 35 7 245
40-50 45 6 270 Find the mean weight of packets.
Sol. First, we find the class marks of the given data as follows.
50-60 55 1 55
Σ fi = 20 Σ fi xi = 700 Weight Number of Class Deviation fi di
(in gm) Packets ( fi ) marks ( xi ) (di = xi − a )
Σfi xi 700
Therefore, mean ( x ) = = = 35 200-201 13 200.5 −3 − 39
Σfi 20
201-202 27 201.5 −2 − 54
Hence, the mean of scores of 20 students in mathematics test
is 35. 202-203 18 202.5 −1 − 18
Example 3. Find the value of p, if the mean of the 203-204 10 a = 203.5 0 0
following distribution is 7.5. 204-205 1 204.5 1 1
Classes 2-4 4-6 6-8 8-10 10-12 12-14 205-206 1 205.5 2 2
Frequency (fi ) 6 8 15 p 8 4 N = ∑ fi = 70 ∑ fi di = − 108
148 CBSE Term II Mathematics X (Basic)

Here, assume mean (a ) = 203. 5 Sol. Here, the number of students who have scored marks less
∑ fi di than 10 are 10. The number of students who have scored
∴ Mean ( x ) = a +
∑ fi marks less than 20 includes the number of students who
have scored marks from 0-10 as well as the number of
108
= 203.5 − students who have scored marks from 10-20.
70
Thus, the total number of students with marks less than 20 is
= 203.5 − 1.54 10 + 8, i.e. 18. So, the cumulative frequency of the class 10-20
= 201.96 is 18.
Hence, the required mean weight is 201.96 gm. Similarly, on computing the cumulative frequencies of the
other classes, i.e. the number of students with marks less
Example 5. The following distribution gives cumulative than 30, less than 40, … less than 100, we get the
frequencies of ‘more than type’: distribution which is called the cumulative frequency
distribution of the less than type.
Marks obtained
5 10 15 20
(More than or equal to) Number of students
Marks obtained
Number of students (cumulative frequency)
30 23 8 2
(cumulative frequency) Less than 10 10

Change the above data into a continuous grouped Less than 20 10 + 8 = 18


frequency distribution. [CBSE 2015] Less than 30 18 + 7 = 25
Sol. Given, distribution is the more than type distribution. Less than 40 25 + 4 = 29
Here, we observe that, all 30 students have obtained marks Less than 50 29 + 6 = 35
more than or equal to 5. Further, since 23 students have
obtained score more than or equal to 10. So, 30 − 23 = 7 Less than 60 35 + 8 = 43
students lie in the class 5-10. Similarly, we can find the other Less than 70 43 + 5 = 48
classes and their corresponding frequencies. Now, we
Less than 80 48 + 9 = 57
construct the continuous grouped frequency distribution as
Less than 90 57 + 5 = 62
Class (Marks obtained) Number of students
Less than 100 62 + 8 = 80
5-10 30 – 23 = 7
Here, 10, 20, 30,…, 100 are the upper limits of the
10-15 23 – 8 = 15 respective class intervals.
15-20 8–2=6 Example 7. In a class of 72 students, marks obtained by
More than or equal to 20 2 the students in a class test (out of 10) are given
below:
Example 6. Consider a grouped frequency distribution
of marks obtained out of 100, by 70 students in a Marks obtained
1 2 3 4 6 7 9 10
certain examination, as follows: (Out of 10)
Number of students 3 5 12 18 23 8 2 1
Marks Number of students
0-10 10 Find the mode of the data.
Sol. The mode of the given data is 6 as it has the maximum
10-20 8 frequency, i.e. 23 among all the observations.
20-30 7 Example 8. The weight of coffee in 70 packets are
30-40 4 shown in the following table
40-50 6 Weight (in gm) Number of packets
50-60 8 200-201 12 ( f0 )
60-70 5 201-202 26 ( f1 )
70-80 9 202-203 20 ( f2 )
80-90 5 203-204 9

90-100 8 204-205 2
205-206 1
Form the cumulative frequency distribution of less
than type. Determine the modal weight.
CBSE Term II Mathematics X (Basic) 149

Sol. In the given data, the highest frequency is 26, which lies in Sol. In a given data, the highest frequency is 41, which lies in the
the interval 201-202 interval 10000-15000.
Here, l = 201, f1 = 26, f0 = 12 , f2 = 20 and h = 1 Here, l = 10000, f1 = 41, f0 = 26,
 f1 − f0 
∴ Mode = l +   ×h f2 = 16 and h = 5000
 2 f1 − f0 − f2   f1 − f0 
∴ Mode = l +   ×h
 26 − 12   2 f1 − f0 − f2 
= 201 +   ×1
 2 × 26 − 12 − 20
 41 − 26 
 14  14 = 10000 +   × 5000
= 201 +   = 201 +  2 × 41 − 26 − 16
 52 − 32  20
 15 
= 201 + 0.7 = 201.7 gm = 10000 +   × 5000
 82 − 42 
Hence, the modal weight is 201.7 gm.
 15 
Example 9. Find the mode of the following distribution = 10000 +   × 5000
 40
Marks 0-10 10-20 20-30 30-40 40-50 50-60 = 10000 + 15 × 125
Number of = 10000 + 1875
4 6 7 12 5 6
students = `11875
Hence, the modal income is ` 11875.
Sol. Given, distribution table is
Example 11. Find the median of the following data.
Marks Number of students
Marks obtained 20 29 28 42 19 35 51
0-10 4
10-20 6 Number of students 3 4 5 7 9 2 3

20-30 7 ( f0 ) Sol. Let us arrange the data in ascending order of xi and make a
30-40 12 ( f1 ) cumulative frequency table.
40-50 5 ( f2 ) Marks Number of Cumulative
obtained (xi ) students (fi ) frequency (cf)
50-60 6
19 9 9
The highest frequency in the given distribution is 12, whose
corresponding class is 30 - 40. 20 3 9 + 3 = 12
Thus, 30-40 is the required modal class. 28 5 12 + 5 = 17
Here, l = 30, f1 =12 , f0 = 7, f2 = 5 and h = 10 29 4 17 + 4 = 21
f1 − f0
∴ Mode = l + ×h 35 2 21 + 2 = 23
2 f1 − f0 − f2
12 − 7 42 7 23 + 7 = 30
= 30 + × 10
2 × 12 − 7 − 5 51 3 30 + 3 = 33
50 50
= 30 + = 30 + = 30 + 417
. = 3417
. Here, n = 33 (odd)
24 −12 12
 n + 1
Hence, mode of the given distribution is 34.17. ∴ Median = Value of   th observation
 2 
Example 10. The monthly income of 100 families are  33 + 1
given as below = Value of   th observation
 2 
Income (in `) Number of families = Value of 17th observation
0-5000 8 Corresponding value of 17th observation of cumulative
5000-10000 26 ( f0 ) frequency in xi is 28. Hence, median is 28.
10000-15000 41 ( f1 ) Example 12. 200 surnames were randomly picked up
15000-20000 16 ( f2 ) from a local telephone directory and the frequency
20000-25000 3 distribution of the number of letters in English
25000-30000 3 alphabets in the surnames was obtained as follows:
30000-35000 2 Number of letters 0-5 5-10 10-15 15-20 20-25
35000-40000 1 Number of surnames 20 60 80 32 8
Calculate the modal income. Find the median of the above data.
150 CBSE Term II Mathematics X (Basic)

Sol. The cumulative frequency table of given data is ( 45 − 40 − p)


= 50 + × 10
20
Number of Number of Cumulative
 5 − p 5−p
letters surnames ( fi ) frequency (cf) ⇒ 50 = 50 +   ⇒0 = [Median = 50]
 2  2
0-5 20 20
∴ p=5
5-10 60 20 + 60 = 80 (cf ) Also, 78 + p + q = 90 [given]
10-15 80 ( f ) 80 + 80 = 160 ⇒ 78 + 5 + q = 90
15-20 32 160 + 32 = 192 ⇒ q = 90 − 83
20-25 8 192 + 8 = 200 ∴ q =7

Total N = 200 Example 14. The median of the following data is 525.
Find the values of x and y, if total frequency is 100.
Since, the cumulative frequency just greater than 100 is 160
and the corresponding class interval is 10-15. 0- 100- 200- 300- 400- 500- 600- 700- 800- 900-
Class
N 200 100 200 300 400 500 600 700 800 900 1000
Given, N = 200; ∴ = = 100
2 2 Frequency 2 5 x 12 17 20 y 9 7 4
Here, l = 10, cf = 80, h = 5 and f = 80
N  Sol. Given, frequency table is
− cf
  100 − 80 
Now, median = l +  2  × h = 10 +  ×5 Class Frequency ( f1 ) Cumulative Frequency (cf )
 f   80 
  0-100 2 2
 20
= 10 +   × 5 = 10 + 1.25 = 11.25 100-200 5 7
 80
200-300 x 7+x
Example 13. The median of the following data is 50. 300-400 12 19 + x
Find the values of p and q, if the sum of all the
400-500 17 36 + x (cf )
frequencies is 90.
Marks Frequency 500-600 20 ( f ) 56 + x
20-30 p 600-700 y 56 + x + y
30-40 15 700-800 9 65 + x + y
40-50 25 800-900 7 72 + x + y
50-60 20 900-1000 4 76 + x + y
60-70 q
Given, total frequency is 100.
70-80 8
∴ 2 + 5 + x + 12 + 17 + 20 + y + 9 + 7 + 4 = 100
80-90 10 ⇒ 76 + x + y = 100
Sol. ⇒ x + y = 24 …(i)
Marks Frequency ( fi ) Cumulative frequency (cf ) It is given that the median is 525.
Clearly, 525 lies in the class 500-600. So, 500-600 is the
20-30 p p
median class.
30-40 15 15 + p
Here, l = 500, h = 100, f = 20 and cf = 36 + x
40-50 25 40 + p (cf ) ∴ N = 100
50-60 20 ( f ) 60 + p N
60-70 q 60 + p + q − cf
∴ Median = l + 2 ×h
70-80 8 68 + p + q f
80-90 10 78 + p + q 50 − 36 − x
⇒ 525 = 500 + × 100
20
Given, N = 90 ⇒ 525 = 500 + (14 − x ) × 5
N 90 ⇒ 525 = 500 + 70 − 5x
∴ = = 45
2 2 ⇒ 5x = 570 − 525
which lies in the interval 50-60. 45
⇒ 5x = 45 ⇒ x = =9
Here, l = 50, f = 20, cf = 40 + p and h = 10 5
N  Put x = 9 in Eq. (i), we get
 − cf
2  9 + y = 24 ⇒ y = 24 − 9 = 15
∴ Median = l + ×h
f Hence, x = 9 and y = 15
CBSE Term II Mathematics X (Basic) 151

Chapter
Practice
(c) mid-points of the class
PART 1 (d) frequencies of the class marks

Objective Questions 8. If the arithmetic mean of the following distribution


is 47, then the value of p is
l
Multiple Choice Questions Class interval 0-20 20-40 40-60 60-80 80-100

1. Which of the following is a measure of central Frequency 8 15 20 p 5


tendency?
(a) 10 (b) 11 (c) 13 (d) 12
(a) Frequency (b) Cumulative frequency
(c) Mean (d) Class-limit 9. The times (in seconds) taken by 150 atheletes to run
2. While computing mean of grouped data, we assume a 110 m hurdle race are tabulated below
that the frequencies are 13.8- 14- 14.2- 14.4- 14.6- 14.8-
Class
(a) evenly distributed over all the class 14 14.2 14.4 14.6 14.8 15
(b) centred at the class marks of the class 2 4 5 71 48 20
Frequency
(c) centred at the upper limits of the class
(d) centred at the lower limits of the class The number of atheletes who completed the race in
3. Mode is the value of the observation having less than 14.6 s is
_________ frequency. [CBSE 2020 (Basic)] (a) 11 (b) 71 (c) 82 (d) 130
(a) highest (b) lowest 10. For the following distribution
(c) equal (d) None of these
Marks Number of students
4. The median and mode respectively of a frequency
Below 10 3
distribution are 26 and 29. Then, its mean is
[CBSE 2020 (Basic)] Below 20 12
(a) 27.5 (b) 24.5 (c) 28.4 (d) 25.8 Below 30 27
5. Consider the following distribution Below 40 57

Class Interval 0-5 5-10 10-15 15-20 20-25 Below 50 75


Below 60 80
Frequency 10 15 12 20 9
The modal class is
The sum of lower limits of the median class and the
(a) 10-20 (b) 20-30 (c) 30-40 (d) 50-60
modal class is [CBSE 2020 (Basic)]
(a) 15 (b) 25 (c) 30 (d) 35 11. Consider the following distribution
6. If x i ’s are the mid-points of the class intervals of Marks obtained Number of students
grouped data, f i ’s are the corresponding More than or equal to 0 63
frequencies and x is the mean, then Σ ( f i x i − x ) is More than or equal to 10 58
equal to More than or equal to 20 55
(a) 0 (b) −1 (c)1 (d) 2 More than or equal to 30 51
Σf d
7. In the formula x = a + i i , for finding the mean More than or equal to 40 48
Σf i More than or equal to 50 42
of grouped data d i ’s are deviation from a of
(a) lower limits of the class The frequency of the class 30-40 is
(b) upper limits of the class (a) 3 (b) 4 (c) 48 (d) 51
152 CBSE Term II Mathematics X (Basic)

12. For the following distribution 18. The mean, mode and median of grouped data will
Number of Number of
always be
Marks Marks (a) same
students students
Below 10 3 Below 40 57 (b) different
(c) depends on the type of data
Below 20 12 Below 50 75
(d) None of the above
Below 30 28 Below 60 80
19. The mean and median of a distribution are 14 and
The modal class is 15 respectively. The value of mode is
(a) 0-20 (b) 20-30 (c) 30-40 (d) 50-60 (a) 16 (b) 17
13. A student noted the number of cars passing through (c) 13 (d) 18
a spot on a road for 100 periods each of 3 min and
summarised in the table given below.
l
Case Based Study
Number of cars Frequency 20. Analysis of Water Consumption in a Society
0-10 7 An inspector in an enforcement squad of department
10-20 14
of water resources visit to a society of 100 families
and record their monthly consumption of water on
20-30 13
the basis of family members and wastage of water,
30-40 12 which is summarise in the following table.
40-50 20
Monthly Consumption 0- 10- 20- 30- 40- 50-
50-60 11 Total
(in kWh) 10 20 30 40 50 60
60-70 15
Number of Families 10 x 25 30 y 10 100
70-80 8

Then, the mode of the data is


(a) 34.7 (b) 44.7 (c) 54.7 (d) 64.7
14. Mode of the following grouped frequency
distribution is
Class 3-6 6-9 9-12 12-15 15-18 18-21 21-24
Frequency 2 5 10 23 21 12 3

(a) 13.6 (b) 15.6 (c) 14.6 (d) 16.6


15. If the number of runs scored by 11 players of a
cricket team of India are 5, 19, 42, 11, 50, 30, 21, 0,
52, 36, 27, then median is
(a) 30 (b) 32 (c) 36 (d) 27
16. Consider the following frequency distribution
Class 0-5 6-11 12-17 18-23 24-29
Frequency 13 10 15 8 11 Based on the above information, answer the
following questions.
The upper limit of the median class is
(i) The value of x + y is
(a) 17 (b) 17.5 (c) 18 (d) 18.5 (a) 50 (b) 42
17. Consider the following frequency distribution (c) 25 (d) 200
Class 65- 85- 105- 125- 145- 165- 185- (ii) If the median of the above data is 32, then x is
85 105 125 145 165 185 205 equal to
(a) 10 (b) 8
Frequency 4 5 13 20 14 7 4
(c) 9 (d) None of these
The difference of the upper limit of the median (iii) What will be the upper limit of the modal class?
class and the lower limit of the modal class is (a) 40 (b) 60
(a) 0 (b) 19 (c) 20 (d) 38 (c) 65 (d) 70
CBSE Term II Mathematics X (Basic) 153

(iv) If A be the assumed mean, then A is always (iv) If a machine work for 10 h in a day, then
(a) > (Actual mean) approximate time required to complete the work
(b) < (Actual Mean) for a machine is
(c) = (Actual Mean) (a) 3 days (b) 4 days
(d) Can’t say (c) 5 days (d) 6 days
(v) The class mark of the modal class is (v) The measure of central tendency is
(a) 25 (b) 35 (a) Mean (b) Median
(c) 30 (d) 45 (c) Mode (d) All of these
21. As the demand for the products grew a 22. Direct income in India was drastically impacted
manufacturing company decided to purchase more due to the COVID-19 lockdown. Most of the
machines. For which they want to know the mean companies decided to bring down the salaries of the
time required to complete the work for a worker. employees upto 50%.
The following table shows the frequency The following table shows the salaries (in percent)
distribution of the time required for each machine received by 50 employees during lockdown.
to complete a work.
Salaries received (in %) 50-60 60-70 70-80 80-90
Time (in hours) 15-19 20-24 25-29 30-34 35-39 Number of employees 18 12 16 4
Number of machines 20 35 32 28 25

Based on the above information, answer the


following questions.
(i) Total number of persons whose salary is reduced
by more than 20% is
Based on the above information, answer the
(a) 40 (b) 46 (c) 30 (d) 22
following questions.
(ii) Total number of persons whose salary is reduced
(i) The class mark of the modal class 30-34 is
by atmost 40% is
(a) 17 (b) 22
(a) 32 (b) 40
(c) 27 (d) 32
(c) 46 (d) 18
(ii) If xi ’s denotes the class mark and fi ’s denotes the
(iii) The modal class is
corresponding frequencies for the given data, then
(a) 50-60 (b) 60-70
the value of Σxi fi equals to
(c) 70-80 (d) 80-90
(a) 3600
(b) 3205 (iv) The median class of the given data is
(c) 3670 (a) 50-60 (b) 60-70
(d) 3795 (c) 70-80 (d) 80-90
(iii) The mean time required to complete the work for a (v) The empirical relationship among mean, median
worker is and mode is
(a) 27.10 h (a) 3 Median = Mode + 2 Mean
(b) 23 h (b) 3 Median = Mode − 2 Mean
(c) 24 h (c) Median = 3 Mode − 2 Mean
(d) None of the above (d) Median = 3 Mode + 2 Mean
154 CBSE Term II Mathematics X (Basic)

11. Find the mode of the following distribution.


PART 2 25-30 30-35 35-40 40-45 45-50 50-55
Classes
Subjective Questions Frequency 20 36 53 40 28 14

12. The following table shows the ages of the patients


l
Short Answer Type Questions
admitted in a hospital during a year
1. Find the class marks of the class 15-35 and class [CBSE 2020 (Basic)]
45-60. Age (in year) 5-15 15-25 25-35 35-45 45-55 55-65
2. What is the arithmetic mean of first n natural Number of
numbers? 60 110 210 230 150 50
Patients
3. Find the mean for the following distribution.
[CBSE 2020 (Basic)] Find the mode of the distribution.
4-7 8-11 12-15 16-19 13. The following distribution shows the transport
Classes
expenditure of 100 employees : [CBSE 2020 (Basic)]
Frequency 5 4 9 10
Expenditure 200-400 400-600 600-800 800-1000 1000-1200
4. Find the mean of the following distribution: (in `)
20-50 50-80 80-110 110-140 140-170 170-200 Number of
Classes 21 25 19 23 12
employees
Frequency 5 8 15 6 12 4
Find the mode of the distribution.
[CBSE 2020 (Basic)]
14. Find the mean for the following distribution.
5. The cumulative frequency table is useful in [CBSE 2020 (Basic)]
determining [CBSE 2020 (Basic)]
Classes 5-15 15-25 25-35 35-45
(a) Mean (b) Median
(c) Mode (d) All of these Frequency 2 4 3 1

6. The mean of the following data is 14. Find the value 15. Given below is a cumulative frequency distribution
of k. showing the marks secured by 50 students of a
x 5 10 15 20 25 class
7 k 8 4 5 Marks Below 20 Below 40 Below 60 Below 80 Below 100
f
Number of 17 22 29 37 50
7. The mode of the following frequency distribution is students
36. Find the missing frequency ( f ). [CBSE 2020 (Basic)]
Form the frequency distribution table for the data.
Classes 0-10 10-20 20-30 30-40 40-50 50-60 60-70
16. The following table shows the cumulative
Frequency 8 10 f 16 12 6 7
frequency distribution of marks of 800 students in
8. Find the median for the given frequency an examination.
distribution: [CBSE 2020 (Basic)] Marks Number of students
Classes 40-45 45-50 50-55 55-60 60-65 65-70 70-75 Below 10 10
Frequency 2 3 8 6 6 3 2 Below 20 50
Below 30 130
9. An NGO working for welfare of cancer patients, Below 40 270
maintained its records as follows:
Below 50 440
Age of patients (in years) 0-20 20-40 40-60 60-80 Below 60 570
Number of patients 35 315 120 50 Below 70 670
Below 80 740
Find mode. [CBSE 2016]
Below 90 780
10. Find the mode of the following distribution. Below 100 800
Classes 10-15 15-20 20-25 25-30 30-35 35-40
Construct a frequency distribution table for the
Frequency 45 30 75 20 35 15 data above.
CBSE Term II Mathematics X (Basic) 155

17. The following distribution of weights (in kg) of 21. If median =137 units and mean = 137.05 units, then
40 persons. find the mode.
Weight (in kg) Number of persons
40-45 4
l
Long Answer Type Questions
45-50 4 22. The weights (in kg) of 50 wrestlers are recorded in
50-55 13 the following table.
55-60 5 Weight (in kg) Number of wrestlers
60-65 6 100-110 4
65-70 5 110-120 14
70-75 2 120-130 21
75-80 1 130-140 8
140-150 3
Construct a cumulative frequency distribution
(of the less than type) table for the data above. Find the mean weight of the wrestlers.
18. Form the frequency distribution table from the 23. If mode of the following series is 54, then find the
following data value of f .
Marks (Out of 90) Number of candidates
Class interval 0-15 15-30 30-45 45-60 60-75 75-90
More than or equal to 80 4
More than or equal to 70 6 Frequency 3 5 f 16 12 7
More than or equal to 60 11 Find the modal class in which the given mode lies
More than or equal to 50 17 and find the value of f by using the formula,
More than or equal to 40 23  f1 − f 0 
Mode = l +   ×h.
More than or equal to 30 27  2f 1 − f 0 − f 2 
More than or equal to 20 30
More than or equal to 10 32 24. Find the mode of the following distribution.
More than or equal to 0 34 Classes 0-20 20-40 40-60 60-80 80-100
Frequency 10 8 12 16 4
19. From the following distribution, find the median
Classes Frequency 25. The following are the ages of 300 patients getting
500-600 36 medical treatment in a hospital on a particular day
600-700 32 10-20 20-30 30-40 40-50 50-60 60-70
Age (in years)
700-800 32
Number of patients 60 42 55 70 53 20
800-900 20
900-100 30 Form
20. Size of agricultural holdings in a survey of 200 (i) less than type cumulative frequency distribution.
families is given in the following table (ii) more than type cumulative frequency distribution.
Size of agricultural Number of 26. Find the unknown entries a, b, c, d, e and f in the
holdings (in hectare) families following distribution of heights of students in a class
0-5 10 Height (in cm) Frequency Cumulative frequency
5-10 15 150-155 12 a
10-15 30 155-160 b 25
15-20 80 160-165 10 c
20-25 40 165-170 d 43
25-30 20 170-175 e 48
30-35 5 175-180 2 f

Compute median and modal class of the holdings. Total 50


156 CBSE Term II Mathematics X (Basic)

27. The maximum bowling speeds (in km/h) of 32. The table below shows the salaries of 280 persons.
33 players at a cricket coaching centre are given as
follows Salary (in ` thousand) Number of persons
5-10 49
Number of
Speed (in km/h) 10-15 133
players
15-20 63
85-100 11
20-25 15
100-115 9
25-30 6
115-130 8
30-35 7
130-145 5 35-40 4

Calculate the median bowling speed. 40-45 2

28. Obtain the median for the following frequency 45-50 1


distribution.
Calculate (i) median of the data,
x 1 2 3 4 5 6 7 8 9 (ii) mode of the data.
y 8 10 11 16 20 25 15 9 6 l
Case Based Questions
29. Weekly income of 600 families is tabulated below 33. The men’s 200 m race event at the 2020
Weekly income (in `) Number of families Tokyo Olympic took place 3 and 4 August.
A stopwatch was used to find the time that it took a
0-1000 250
group of Athletes to run 200 m.
1000-2000 190
Time (in seconds) 0-20 20-40 40-60 60-80 80-100
2000-3000 100
3000-4000 40 Number of Students 8 10 13 6 3

4000-5000 15
5000-6000 5
Total 600

Compute the median income.


30. A survey regarding the heights (in cm) of 51 boys of
Class X of a school was conducted and the following
data was obtained:
Heights (in cm) Number of boys
(i) Estimate the mean time taken by a student to
Less than 140 4 finish the race.
Less than 145 11
Time
Less than 150 29 Number of Students
(in seconds)
Less than 155 40
0-20 8
Less than 160 46
20-40 10
Less than 165 51
40-60 13
Find the median height. 60-80 6
31. Find the missing frequencies in the 80-100 3
following frequency distribution table, if N =100
and median is 32. CBSE 2019 (ii) What is the sum of lower limits of median class and
Marks obtained 0-10 10-20 20-30 30-40 40-50 50-60 Total modal class.
(iii) How many students finished the race within
Number of students 10 ? 25 30 ? 10 100 1 min?
CBSE Term II Mathematics X (Basic) 157

SOLUTIONS
Objective Questions 8. (d) Let us construct the following table for finding the
1. (c) Mean is the measure of central tendency. arithmetic mean
2. (b) In computing the mean of grouped data, the frequencies Class Frequency ( fi ) Class mark
are centred at the class marks of the class. ( xi )
fi xi
interval
3. (a) Highest 0-20 8 10 80
4. (b) Given, the median and mode of a frequency distribution
are 26 and 29. 20-40 15 30 450
We know that, 40-60 20 50 1000
3 Median = 2 Mean + Mode 60-80 p 70 70p
⇒ 2 Mean = 3 Median − Mode
3Median − Mode 80-100 5 90 450
⇒ Mean =
2 Total Σfi = 48 + p Σfi xi = 1980 + 70p
3 × 26 − 29
⇒ Mean = Σfi xi
2 Now, x=
78 − 29 Σfi
= 1980 + 70p
2 =
49 48 + p
=
2 1980 + 70p
⇒ 47 =
= 24.5 48 + p
5. (b) The cumulative frequency table of given data is ⇒ 2256 + 47 p = 1980 + 70p
⇒ 276 = 23p
Cumulative ⇒ p = 12
Class Interval Frequency
Frequency
9. (c) The number of atheletes who completed the race in less
0-5 10 10 than 14.6 = 2 + 4 + 5 + 71 = 82
5-10 15 10 + 15 = 25 10. (c)

10-15 12 25 + 12 = 37 Marks Number of students Cumulative frequency


15-20 20 37 + 20 = 57 Below 10 3=3 3
20-25 9 57 + 9 = 66 10-20 12 − 3 = 9 12
Here, n = 66 20-30 27 − 12 = 15 27
n 66 30-40 57 − 27 = 30 57
∴ = = 33
2 2 40-50 75 − 57 = 18 75
Since, the cumulative frequency just greater than 33 is 37
and the corresponding class internal is 10-15. 50-60 80 − 75 = 5 80
∴ Median class = 10 − 15 Here, we see that the highest frequency is 30, which lies in
Here, the maximum frequency is 20 and the corresponding the interval 30-40.
class is 15-20. 11. (a) Given, the distribution table
So, this is the modal class.
∴ Sum of lower limits of the median class and the modal Marks obtained Number of students
class = 10 + 15 = 25 0-10 63 − 58 = 5
Σf x
6. (a)Q x= i i 10-20 58 − 55 = 3
n
20-30 55 − 51 = 4
∴ Σ ( fi xi − x ) = Σ fi xi − Σx
= nx − nx [Q Σx = nx ] 30-40 51 − 48 = 3
=0 40-50 48 − 42 = 6
7. (c) We know that, 50 42
di = xi − a
Frequency of the modal class 30-40 is 3 from the above table.
i.e. di ’s are the deviation from a of mid-points of the classes.
158 CBSE Term II Mathematics X (Basic)

12. (c) Let us first construct the following frequency distribution 17. (c)
table.
Class Frequency Cumulative frequency
Marks Number of students
65-85 4 4
0-10 3
85-105 5 9
10-20 9
105-125 13 22
20-30 16
125-145 20 42
30-40 29
145-165 14 56
40-50 18
165-185 7 63
50-60 5
185-205 4 67
Since, the maximum frequency is 29 and the class
N 67
corresponding to this frequency is 30-40. Here, = = 33. 5 which lies in the interval 125 -145.
So, the modal class is 30-40. 2 2
13. (b) Here, modal class is 40-50. Since, it has maximum Hence, upper limit of median class is 145.
frequency which is 20. Here, we see that the highest frequency is 20 which lies in
125-145. Hence, the lower limit of modal class is 125.
Here, l = 40, f1 = 20, f0 = 12, f2 = 11 and h = 10
∴ Required difference = Upper limit of median class
 f1 − f0 
∴ Mode = l +   ×h − Lower limit of modal class
 2 f1 − f0 − f2 
= 145 − 125 = 20
 20 − 12  80 18. (c) No, the value of these three measures can be the same, it
= 40 +   × 10 = 40 +
 40 − 12 − 11 17 depends on the type of data.
= 40 + 4.7 = 44.7 19. (b) Given, mean = 14 and median = 15
14. (c) We observe that the class 12-15 has maximum frequency. By using empirical relationship,
Therefore, this is the modal class. Mode = 3 Median − 2 Mean = 3 × 15 − 2 × 14 = 45 − 28 = 17
We have, l = 12, h = 3 , f1 = 23 , f0 = 10 and f2 = 21 20. (i) (c) Given, x and y are the frequencies of class intervals
f1 − f0
∴ Mode = l + ×h 10-20 and 40-50, respectively. Then,
2 f1 − f0 − f2 10 + x + 25 + 30 + y + 10 = 100
23 − 10 ⇒ x + y = 25 …(1)
= 12 + ×3
46 − 10 − 21 (ii) (c) Median is 32, which lies in 30-40.
13 13 So, the median class is 30-40.
= 12 + × 3 = 12 + = 12 + 2.6 = 146.
15 5 Here, l = 30, h = 10, f = 30, N = 100
15. (d) Arranging the terms in ascending order, and cf = 10 + x + 25 = x + 35
0, 5, 11, 19, 21, 27, 30, 36, 42, 50, 52 N 
 − cf 
Here, n = 11 (odd) Now, median = l +  2  ×h
 n + 1  f 
∴ Median =   th  
 2 
{50 − ( x + 35)} 
11 + 1 ⇒ 30 + × 10 = 32
Median value =   th = 6th value = 27  30 
 2 
(15 − x )
⇒ 30 + = 32 ⇒ (15 − x ) = 6 ⇒ x = 9
16. (b) Given, classes are not continuous, so we make 3
continuous by subtracting 0.5 from lower limit and adding Put x = 9 in Eq. (1), we get
0.5 to upper limit of each class.
y = 16
Class Frequency Cumulative frequency Hence, x = 9 and y = 16
−0.5-5.5 13 13 (iii) (a) Since, the maximum frequency is 30, so the modal
class is 30-40.
5.5-11.5 10 23
Hence, upper limit of the modal class is 40.
11.5-17.5 15 38 (iv) (d) The value of assumed mean can be less, more or equal
17.5-23.5 8 46 than the actual mean.
23.5-29.5 11 57 (v) (b) The modal class is 30-40.
30 + 40
N 57 ∴ Class mark =
Here, = = 28. 5, which lies in the interval 11.5-17.5. 2
2 2 70
= = 35
Hence, the upper limit is 17.5. 2
CBSE Term II Mathematics X (Basic) 159

21. (i) (d) Class mark of class 30-34 Subjective Questions


30 + 34 1. We know that,
=
2 Lower limit + Upper limit
Class mark =
64 2
=
2 15 + 35 50
∴ Class mark of 15-35 = = = 25
= 32 2 2
(ii) (d) Let’s make the table 45 + 60 105
Class mark of 45-60 = = = 52.5
2 2
Class marks Frequency
Classes xi fi Sum of all the observations
( xi ) ( fi ) 2. Arithmetic mean =
Number of observations
15-19 17 20 340 1+ 2 +K+ n
=
20-24 22 35 770 n
n
25-29 27 32 864 [2 × 1 + ( n − 1)1]
30-34 32 28 896 = 2
n
35-39 37 25 925 [Q1 + 2 + K + n is an AP series whose first term is a = 1 and
common difference is d = 1. We know that, the
Total Σfi = 140 Σxi fi = 3795
n
sum of nth term of an AP is Sn = [2a + ( n − 1)d]]
Given, Σxi fi = 3795 2
Σx f 2 + n −1 n + 1
(iii) (a)∴ Mean time ( x ) = i i = =
Σfi 2 2
3795 3. Since, given data is not continuous, so we subtract 0.5 from
= the lower limit and add 0.5 in the upper limit of each class.
140
Now, we first find the class mark xi of each class and then
= 2710
. proceed as follows
2710
.
(iv) (a)∴ Approximate time = Class marks Frequency
10 Class fi xi
= 2.710
( xi ) ( fi )
~− 3 days 3.5-7.5 5.5 5 27.5
(v) (d) Measure of central tendency are mean, median and 7.5-11.5 9.5 4 38
mode.
11.5-15.5 13.5 9 121.5
22. (i) (b)∴Required number of employees = 18 + 12 + 16
= 46 15.5-19.5 17.5 10 175
(ii) (a)∴Required number of employees = 12 + 16 + 4 Σ fi = 28 Σ fi xi = 362
= 32
Σfi xi
(iii) (a) The maximum frequency is 18 and the corresponding Therefore, ( x ) mean =
Σfi
class is 50-60.
362
Hence, modal class is 50-60. = = 12 .93
28
(iv) (b) Consider the table
Hence, mean of the given data is 12.93.
Salaries received Number of Cumulative 4.
(in %) employees ( fi ) frequency (cf )
Frequency Mid-value fi xi
50-60 18 18 Class interval
( fi ) ( xi )
60-70 12 18 + 12 = 30 20-50 5 35 175
70-80 16 30 + 16 = 46 50-80 8 65 520
80-90 4 46 + 4 = 50 80-110 15 95 1425
Total Σfi = 50 110-140 6 125 750
140-170 12 155 1860
N 50
Here, = = 25 170-200 4 185 740
2 2
The cumulative frequency more than 25 lies in 60-70. ∑ fi = 50 ∑ fi xi = 5470
Hence, median class is 60-70.
Σfi xi 5470
(v) (a) As we know, Mode = 3 Median − 2 Mean Mean = = = 109.4
Σfi 50
∴ 3 Median = Mode + 2 Mean
160 CBSE Term II Mathematics X (Basic)

5. A cumulative frequency table is useful in determining the 8.


median. Classes Frequency Cumulative frequency
6. Table for the given data is 40-45 2 2
xi fi fi xi 45-50 3 5
50-55 8 13 (cf )
5 7 35
55-60 6 (f) 19
10 k 10k
60-65 6 25
15 8 120
65-70 3 28
20 4 80 70-75 2 30
25 5 125 N = 30
Total Σ fi = k + 24 Σ fi xi = 10 k + 360 We have, N = 30
N 30
Here, Σfi = k + 24 and Σfi xi = 10k + 360 ∴ = = 15
2 2
Given, mean = 14 N
The cumulative frequency just greater than is 19 and the
Σfi xi 2
∴ = 14
Σfi corresponding class is 55-60.
10k + 360 Thus, 55-60 is the median class such that
⇒ = 14 Here, l = 55, f =6, cf = 13 and h = 5
k + 24
N
⇒ 10k + 360 = 14( k + 24) − cf
15 − 13
∴ Median = l + 2 × h = 55 + ×5
⇒ 10k + 360 = 14k + 336 f 6
⇒ 14k − 10k = 360 − 336 2 5
= 55 + × 5 = 55 + = 5667 .
⇒ 4k = 24 6 3
24 9.
∴ k= =6
4 0-20 20-40 40-60 60-80
Age of patients (in years)
Hence, the value of k is 6.
7. We have, mode = 36 Number of patients 35( f0 ) 315( f1 ) 120( f2 ) 50
Clearly, it lies in the class interval 30-40.
Here, maximum frequency is 315 and the class
Class Frequency corresponding to this frequency is 20-40. So, the modal
0-10 8 class is 20-40.
Then, l = 20, f1 = 315, f0 = 35, f2 = 120 and h = 20
10-20 10
 f1 − f0 
20-30 f ( f0 ) Now, Mode = l +   ×h
 2 f1 − f0 − f2 
30-40 16 ( f1 ) 315 − 35
 
= 20 +   × 20
40-50 12 ( f2 )  2 × 315 − 35 − 120
50-60 6  280 
= 20 +   × 20
60-70 7  630 − 155
280
So, 30-40 is the modal class. = 20 + × 20
475
Here, l = 30, f1 =16, f0 = f, f2 =12, h = 10 = 20 + 1179
. = 3179.
f1 − f0 Hence, average age of maximum number of patients is 31.79.
Now, Mode = l + ×h
2 f1 − f0 − f2 10. Given, distribution table is
16 − f
⇒ 36 = 30 + × 10 Classes Frequency
2 × 16 − f − 12
10-15 45
10 (16 − f )
⇒ 6= 15-20 30( f0 )
32 − f − 12
20-25 75( f1 )
⇒ 6( 20 − f ) =10 (16 − f )
25-30 20( f2 )
⇒ 120 − 6f = 160 − 10f
30-35 35
⇒ 4f = 40
35-40 15
⇒ f = 10
CBSE Term II Mathematics X (Basic) 161

The highest frequency in the given data is 75 and the 20


= 35 + × 10 = 35 + 2 = 37
corresponding class is 20-25, which is a modal class. 100
Here, l = 20, f1 = 75, f0 = 30, f2 = 20 and h = 5 ∴ Mode is 37.
f1 − f0 13. Here, maximum frequency is 25 and the class corresponding
∴Mode = l + ×h
2 f1 − f0 − f2 to this frequency is 400 − 600.
75 − 30 45 × 5 So, the modal class is 400 − 600.
= 20 + × 5 = 20 +
2 × 75 − 30 − 20 150 − 50 Expenditure (in `) Number of Employees
225
= 20 + = 20 + 2 .25 = 22 . 25 200-400 21 ( f0 )
100
400-600 25 ( f1 )
11. Given, distribution table is
600-800 19 ( f2 )
Classes Frequency
800-1000 23
25-30 20
1000-1200 12
30-35 36 ( f0 )
35-40 53 ( f1 ) Here, l = 400, f0 = 21, f1 = 25, f2 = 19 and h = 200
 f1 − f0 
40-45 40 ( f2 ) Now, Mode = l +   ×h
 2 f1 − f0 − f2 
45-50 28
 25 − 21 
50-55 14 = 400 +   × 200
 2( 25) − 21 − 19
In the given table, the highest frequency is 53 and 4 4
corresponding class of this frequency is 35-40. = 400 + × 200 = 400 + × 200
50 − 40 10
Thus, 35-40 is a modal class.
= 400 + 4 × 20 = 400 + 80 = 480
Here, l = 35, f1 = 53, f0 = 36, f2 = 40 and h = 5
f1 − f0 14. Let us make the following table for the given data.
∴ Mode = l + ×h
2 f1 − f0 − f2 Class Frequency
xi fi xi
53 − 36 Interval fi
= 35 + ×5
2 × 53 − 36 − 40 5-15 2 10 20
17 × 5 85
= 35 + = 35 + 15-25 4 20 80
106 − 76 30
= 35 + 2 . 83 25-35 3 30 60
= 37 . 83 (approx) 35-45 1 40 40
Hence, mode of given data is 37.83. Σfi = 10 Σfi xi = 200
Total
12.
Age ( fi ) Number of patients Here, Σfi = 10 and Σfi xi = 200
Σfi xi
5-15 60 On putting the values of Σfi xi and Σfi in the formula x = ,
Σfi
15-25 110
we get
25-35 210 f0 200
x= = 20
35-45 230 f1 10
Hence, the mean for the given distribution is 20.
45-55 150 f2
15. Here, we observe that, 17 students have scored marks below
55-65 50 20 i.e. it lies between class interval 0-20 and 22 students
have scored marks below 40, so 22 − 17 = 5 students lies in
f1 − f0 the class interval 20-40 continuting in the same manner, we
∴ Mode = l + ×h
2 f1 − f0 − f2 get the complete frequency distribution table for given data.
Modal class = Interval with highest frequency = 35-45
Marks Number of students
Where, l = lower limit of modal class = 35
0-20 17
h = class interval = 15 − 5 = 10
f1 = frequency of modal class = 230 20-40 22 − 17 = 5
f0 = frequency of the class before modal class = 210 40-60 29 − 22 = 7
f2 = frequency of the class after modal class = 150 60-80 37 − 29 = 8
230 − 210
Mode = 35 + × 10 80-100 50 − 37 = 13
2 × 230 − 210 − 150
162 CBSE Term II Mathematics X (Basic)

16. Here, we observe that 10 students have scored marks below 19. The cumulative frequency table for given distribution is
10 i.e. it lies between class interval 0-10. Similarly, 50
students have scored marks below 20. So, 50 − 10 = 40 Class Frequency (fi ) Cumulative Frequency
students lies in the interval10 - 20 and so on. The table of a (cf)
frequency distribution for the given data is 500-600 36 36
600-700 32 36 + 32 = 68 (cf )
Class interval Number of students (fi )
700-800 32 (f) 68 + 32 = 100
0-10 10
800-900 20 100 + 20 = 120
10-20 50 − 10 = 40
900-1000 30 120 + 30 = 150
20-30 130 − 50 = 80
N 150
30-40 270 − 130 = 140 Here, = = 75, which lies in the cumulative frequency
2 2
40-50 440 − 270 = 170 100, whose corresponding class is 700-800. Thus, modal class
50-60 570 − 440 = 130 is 700-800.
Here, l = 700, cf = 68, f = 32 and h = 100
60-70 670 − 570 = 100 N
− cf
70-80 740 − 670 = 70
Q Median = l + 2 ×h
80-90 780 − 740 = 40 f
75 − 68
90-100 800 − 780 = 20 = 700 + × 100
32
700
17. The cumulative distribution (less than type) table is shown = 700 +
below 32
= 700 + 21.88
Weight (in kg) Cumulative frequency (cf) = 721.88
Less than 45 4 Hence, median of the given distribution is 721.88.
20.
Less than 50 4+ 4=8
Less than 55 8 + 13 = 21 Size of agricultural Number of Cumulative
holdings (in hec) families ( fi ) frequency (cf)
Less than 60 21 + 5 = 26
0-5 10 10
Less than 65 26 + 6 = 32
5-10 15 25
Less than 70 32 + 5 = 37
10-15 30 55 (cf )
Less than 75 37 + 2 = 39
15-20 80 (f) 135
Less than 80 39 + 1 = 40
20-25 40 175
18. Here, we observe that, all 34 students have scored marks 25-30 20 195
more than or equal to 0. Since, 32 students have scored
marks more than or equal to 10. So, 34 − 32 = 2 students lies 30-35 5 200
in the interval 0-10 and so on.
I. Here, N = 200
Now, we construct the frequency distribution table.
N 200
Now, = = 100, which lies in the interval 15-20.
Class interval Number of candidates (fi ) 2 2
0-10 34 − 32 = 2 Here, l = 15 , h = 5, f = 80 and cf = 55
N 
10-20 32 − 30 = 2  − cf 
∴ Median = l +  2  ×h
20-30 30 − 27 = 3  f 
 
30-40 27 − 23 = 4
100 − 55
23 − 17 = 6 = 15 +   ×5
40-50  80 
50-60 17 − 11 = 6  45
= 15 +  
60-70 11 − 6 = 5  16 

70-80 6− 4 = 2 = 15 + 2.81 = 17. 81 hec


80-90 4 II. In a given table 80 is the highest frequency.
So, the modal class is 15-20.
CBSE Term II Mathematics X (Basic) 163

21. Given, median = 137 units 24. The given distribution table is
and mean = 137.05 units. Class Frequency
We know that, Mode = 3(Median) − 2(Mean)
0-20 10
= 3 (137 ) − 2(137.05)
= 411 − 274.10 20-40 8
= 136.90 40-60 12 ( f0 )
Hence, the value of mode is 136.90 units. 60-80 16 ( f1 )
22. We first find the class mark xi , of each class and then 80-100 4 ( f2 )
proceed as follows
The highest frequency in the given distribution table is 16,
Number of Deviations whose corresponding class is 60-80. Thus, 60-80 is the modal
Weight Class
wrestlers marks ( x ) di = xi − a, fi di class of the given distribution.
(in kg)
( fi ) i
a = 125 Here, l = 60, f1 = 16, f0 = 12, f2 = 4 and h = 20
−20 −80 f1 − f0
100-110 4 105 ∴ Mode = l + ×h
2 f1 − f0 − f2
110-120 14 115 −10 −140
16 − 12
120-130 21 a = 125 0 0 = 60 + × 20
2 × 16 − 12 − 4
130-140 8 135 10 80 4 × 20 80
= 60 + = 60 +
140-150 3 145 20 60 32 − 16 16
N = Σ fi = 50 Σ fi di = − 80 = 60 + 5 = 65
Hence, mode of the given distribution is 65.
∴Assumed mean (a ) = 125,
25. (i) We observe that the number of patients which take
Class width ( h) = 10 and total observation ( N ) = 50
medical treatment in a hospital on a particular day less
By assumed mean method, than 10 is 0. Similarly, less than 20 include the number of
Σ fi di patients which take medical treatment from 0-10 as well
Mean ( x ) = a +
Σ fi as the number of patients which take medical treatment
( − 80) from 10-20.
= 125 + So, the total number of patients less than 20 is
50
0 + 60 = 60, we say that the cumulative frequency of the
= 125 − 16
. = 123.4 kg
class 10-20 is 60. Similarly, for other classes, which is
23. Here, given mode is 54, which lies between 45-60. shown below the table.
Therefore, the modal class is 45-60.
(ii) Also, we observe that all 300 patients which take medical
Class-interval Frequency treatment more than or equal to 10. Since, there are 60
0-15 3 patients which take medical treatment in the interval
10-20, this means that there are 300 − 60 = 240 patients
15-30 5 which take medical treatment more than or equal to 20.
30-45 f ( f0 ) Continuing in the same manner, which is shown below
45-60 16 ( f1 ) the table.

60-75 12 ( f2 ) (i) Less than type (ii) More than type


75-90 7 Number of Number of
Age (in years) Age (in years)
students students
Here, l = 45, f1 = 16, f0 = f , f2 = 12 and h = 15
Less than 10 0 More than or 300
 f1 − f0  equal to 10
∴ Mode = l +   ×h
 2 f1 − f0 − f2  Less than 20 60 More than or 240
 16 − f  equal to 20
∴ 54 = 45 +   × 15
 2 × 16 − f − 12  Less than 30 102 More than or 198
16 − f equal to 30
⇒ 9= × 15
20 − f Less than 40 157 More than or 143
equal to 40
⇒ 9 ( 20 − f ) = 15(16 − f )
Less than 50 227 More than or 73
⇒ 180 − 9f = 240 − 15f
equal to 50
⇒ 6f = 240 − 180 = 60
Less than 60 280 More than or 20
⇒ f = 10 equal to 60
Hence, required value of f is 10. Less than 70 300
164 CBSE Term II Mathematics X (Basic)

26. 28. Here, the given data is in ascending order of xi .


Cumulative frequency table for the given data is
Frequency Cumulative Cumulative
Height (in cm) ( fi ) frequency frequency x fi cf
(given) (cf ) i
1 8 8
150-155 12 a 12
2 10 18
155-160 b 25 12 + b
3 11 29
160-165 10 c 22 + b 4 16 45
165-170 d 43 22 + b + d 5 20 65
170-175 e 48 22 + b + d + e 6 25 90
175-180 2 f 24 + b + d + e 7 15 105
Total 50 8 9 114
9 6 120
On comparing last two tables, we get
a = 12 Here, n = 120 (even)
∴ 12 + b = 25 1  n  n  
∴Median = Value of   th +  + 1 th observations
⇒ b = 25 − 12 = 13 2   2 2  
22 + b = c
1 120 120  
⇒ c = 22 + 13 = 35 = Value of   th +  + 1 th observations
2   2   2  
22 + b + d = 43
⇒ 22 + 13 + d = 43 1
= [Value of 60th observation + Value of 61th observation]
⇒ d = 43 − 35 = 8 2
22 + b + d + e = 48 Both 60th and 61th observations lie in the cumulative
frequency 65 and its corresponding value of xi is 5.
⇒ 22 + 13 + 8 + e = 48
1
⇒ e = 48 − 43 = 5 ∴ Median = ( 5 + 5) = 5
2
and 24 + b + d + e = f
29. First we construct a cumulative frequency table.
⇒ 24 + 13 + 8 + 5 = f
∴ f = 50 Weekly income Number of families Cumulative frequency
27. First we construct the cumulative frequency table (in `) ( fi ) (cf )
Speed Number of Cumulative 0-1000 250 250 (cf )
(in km/h) players ( fi ) frequency (cf ) 1000-2000 190 ( f ) 250 + 190 = 440
85-100 11 11 (cf ) 2000-3000 100 440 + 100 = 540
100-115 9 (f) 11 + 9 = 20
3000-4000 40 540 + 40 = 580
115-130 8 20 + 8 = 28
4000-5000 15 580 + 15 = 595
130-145 5 28 + 5 = 33
5000-6000 5 595 + 5 = 600
It is given that, N = 33
N 33 It is given that, N = 600
∴ = = 16. 5
2 2 N 600
∴ = = 300
So, the median class is 100-115. 2 2
Here, l = 100, f = 9, cf = 11 and h = 15 Since, cumulative frequency 440 lies in the interval
N  1000 - 2000.
 − cf Here, l = 1000, f = 190, cf = 250 and h = 1000
2 
∴ Median = l + ×h
f N 
 − cf 
(16. 5 − 11) Median = l +  ×h
2
= 100 + × 15 Q
9 f
5. 5 × 15 ( 300 − 250)
= 100 + = 1000 + × 1000
9 190
82. 5 50 5000
= 100 + = 1000 + × 1000 = 1000 +
9 190 19
= 100 + 917
. = 10917. = 1000 + 263.15 = 1263.15
Hence, the median bowling speed is 109.17 km/h. Hence, the median income is ` 1263.15.
CBSE Term II Mathematics X (Basic) 165

30. To calculate the median height, we need to convert the given N


Here, N = 100 ⇒ = 50
data in the continuous grouped frequency distribution. 2
Given, distribution is of less than type and 140, 145, 150, …, Given, median = 32, which belongs to the class 30-40.
165 gives the upper limits of the corresponding class So, the median class is 30-40.
intervals. So, the classes should be below 140, 140-145, Then, l = 30, h = 10, f = 30 and cf = 35 + f1
145-150, …, 160-165.
N 
Clearly, the frequency of class interval below 140 is 4, since − cf
2 
there are 4 boys with height less than 140. For the frequency Q Median = l +  ×h
 f 
of class interval140-145 subtract the number of boys having
height less than 140 from the number of boys having height  
less than 145.  50 − 35 − f1 
∴ 32 = 30 +   × 10
Thus, the frequency of class interval140 -145 is11 − 4 = 7.  30 
Similarly, we can calculate the frequencies of other class 15 − f1
intervals and get the following table ⇒ 32 − 30 =
3
Cumulative ⇒ 2 × 3 = 15 − f1
Class interval Frequency ( fi )
frequency (cf ) ⇒ f1 = 15 − 6 = 9
Below 140 4 4 On putting the value of f1 in Eq. (i), we get
140-145 11 − 4 = 7 11 (cf ) f2 = 25 − 9 = 16
145-150 29 − 11 = 18 ( f ) 29 Hence, the missing frequencies are f1 = 9 and f2 = 16 .
150-155 40 − 29 = 11 40 32. First, we construct a cumulative frequency table
155-160 46 − 40 = 6 46 Salary (in ` Number of Cumulative frequency
160-165 51 − 46 = 5 51 thousand) persons ( fi ) (cf )
5-10 49 ( f0 ) 49 (cf )
Here, N = 51
N 51 10-15 133 ( f1 ) 133 + 49 = 182
∴ = = 25.5
2 2 15-20 63 ( f2 ) 182 + 63 = 245
Since, the cumulative frequency just greater than 25.5 is 29 20-25 15 245 + 15 = 260
and the corresponding class interval is 145-150.
25-30 6 260 + 6 = 266
∴ Median class = 145-150
Now, l = 145, f = 18, cf = 11 and h = 5 30-35 7 266 + 7 = 273
N  35-40 4 273 + 4 = 277
− cf
2   25. 5 − 11 
∴ Median = l +   × h = 145 +  ×5 40-45 2 277 + 2 = 279
 f   18 
  45-50 1 279 + 1 = 280
72.5 N = 280
= 145 + = 145 + 4.03 = 149.03
18
N 280
Hence, the required median height is 149.03 cm. ∴ = = 140
2 2
31. Given, median = 32 and N = Σf = 100
(i) Here, median class is 10-15, because 140 lies in it.
Let f1 and f2 be the frequencies of the class interval 10-20
Then, f = 133, cf = 49 and h = 5
and 40-50, respectively.
N 
Since, sum of frequencies = 100  − cf
2 
∴ 10 + f1 + 25 + 30 + f2 + 10 = 100 ∴ Median = l + ×h
f
⇒ f1 + f2 = 100 − 75 ⇒ f1 + f2 = 25
(140 − 49)
⇒ f2 = 25 − f1 …(i) = 10 + ×5
Now, the cumulative frequency table for given distribution is 133
91 × 5
= 10 +
Class interval Frequency ( fi ) Cumulative frequency 133
455
0-10 10 10 = 10 + = 10 + 3.421
10-20 f1 10 + f1 133
= `13.421 (in thousand)
20-30 25 35 + f1 (cf )
= 13.421 × 1000
30-40 30 ( f ) 65 + f1
= `13421
40-50 f2 65 + f1 + f2 (ii) Here, the highest frequency is 133, which lies in the
50-60 10 75 + f1 + f2 interval 10-15, called modal class.
Total N = f1 + f2 + 75 Then, l = 10, h = 5, f1 = 133, f0 = 49 and f2 = 63.
166 CBSE Term II Mathematics X (Basic)

 f1 − f0  1720
= = 43
∴ Mode = l +   ×h
 2 f1 − f0 − f2  40
∴ Mean time is 43s.
 133 − 49 
= 10 +  ×5 (ii)
 2 × 133 − 49 − 63 
Time Number of Cumulative
84 × 5
= 10 + (in seconds) students ( fi ) frequency (cf )
266 − 112
0-20 8 8
84 × 5
= 10 + = 10 + 2.727
154 20-40 10 8 + 10 = 18
= `12.727 (in thousand) 40-60 13 18 + 13 = 31
= 12.727 × 1000
60-80 6 31 + 6 = 37
= `12727
Hence, the median and modal salary are ` 13421 and 80-100 3 37 + 3 = 40
` 12727, respectively. Σfi = 40
33. (i)
Modal class is a class having highest frequency.
Time Number of Class mark fi xi So, 40-60 is modal class
(in seconds) students ( fi ) ( xi ) To find median class, we find cumulative frequency
8 × 10 = 80 N 40
0-20 8 10 = = 20
2 2
20-40 10 30 10 × 30 = 300 ∴ 40-60 has cumulative frequency greater than 20.
40-60 13 50 13 × 50 = 650 Thus, 40-60 is the median class.
60-80 6 70 6 × 70 = 420 ∴Sum of lower limits of median class and modal class
= 40 + 40 = 80.
80-100 3 90 3 × 90 = 270 (iii) Students finished the race within 1 min
Σfi = 40 Σfi xi = 1720 = Students between 0-20 + Students between 20-40
+ Students between 40-60
Σfi xi = 8 + 10 + 13 = 31
Mean ( x ) =
Σfi
Chapter Test
Multiple Choice Questions (ii) The modal value of the given data is
(a) 150 (b) 150.91
1. A survey conducted by a group of students is (c) 145.6 (d) 140.9
given as
(iii) The median value of the given data is
Family Size 1-3 3-5 5-7 7-9 9-11 (a) 140 (b) 146.67 (c) 130 (d) 136.6
Number of families 7 8 2 2 1 (iv) Assumed mean method is useful in
determining the
The mean of the data is (a) Mean (b) Median
(a) 6.8 (b) 4.2 (c) Mode (d) All of these
(c) 5.4 (d) None of these (v) The manufacturer can claim that the
2. The relationship among mean, median and mileage for his car is
mode for a distribution is (a) 144 km/charge (b) 155 km/charge
(a) Mode = Median − 2 mean (c) 165 km/charge (d) 175 km/charge
(b) Mode = 3 Median − 2 mean Short Answer Type Questions
(c) Mode = 2 Median − 3 mean
(d) Mode = Median − mean 5. Find the median of the first ten prime numbers.
3. For the following distribution 6. An aircraft has 120 passenger seats. The
number of seats occupied during 100 flights is
Class 0-5 5 -10 10 -15 15 - 20 20 - 25
given in the following table.
Frequency 10 15 12 20 9
Number of seats 100-104 104-108 108-112 112-116 116-120
The sum of lower limits of the median class and Frequency 15 20 32 18 15
modal class is
(a) 15 (b) 25 (c) 30 (d) 35 Determine the mean number of seats occupied
Case Based MCQs over the flights.

4. A Tesla car manufacturing industry wants to 7. The following distribution gives the daily
declare the mileage of their electric cars. For income of 50 workers of a factory:
this, they recorded the mileage (km/charge) of
Daily income (in `) 100-120 120-140 140-160 160-180 180-200
100 cars of the same model. Details of which
are given in the following table. Number of workers 12 14 8 6 10
Mileage 100-120 120-140 140-160 160-180 Write the above distribution as ‘less than type’
(km/charge)
cumulative frequency distribution. [CBSE 2015]
Number of Cars 14 24 36 26
8. Find the mode of the following frequency
distribution. [CBSE 2019]

Class 0-10 10-20 20-30 30-40 40-50 50-60 60-70

Frequency 8 10 10 16 12 6 7

Long Answer Type Questions

Based on the above information, answer the 9. Find the mean of the following frequency
following questions. distribution using assumed mean method.
(i) The average mileage is Class 2-8 8-14 14-20 20-26 26-32
(a) 140 km/charge
Frequency 6 3 12 11 8
(b) 150 km/charge
(c) 130 km/charge
(d) 144.8 km/charge

Answers
For Detailed Solutions
1. (b) 2. (b) 3. (b) 4. (i) (d) (ii) (b) (iii) (b) (iv) (a) (v) (a)
Scan the code
5. 12 6. 109.92 8. 36 9. 18.8
168 CBSE Term II Mathematics X (Basic)
Mathem atics(Basic)
Class 10th ( Term II)

*
Practice Paper 1
(Solved)
Instructions Time : 2 Hr
Max. Marks : 40
1. The question paper contains three sections A, B and C.
2. Section A has 5 questions with 3 internal choices.
3. Section B has 4 questions with 3 internal choices.
4. Section C has 1 Case Based MCQs comprises of 5 MCQs.
5. There is no negative marking.
* As exact Blue-print and Pattern for CBSE Term II exams is not released yet. So the pattern of this
paper is designed by the author on the basis of trend of past CBSE Papers. Students are advised
not to consider the pattern of this paper as official, it is just for practice purpose.

Section A (3 Marks Each)


This section consists of 5 questions of Short Answer Type.
1. The mode of the following series is 36. Find the missing ( x ) frequency in it.
Class interval 0-10 10-20 20-30 30-40 40-50 50-60 60-70

Frequency 8 10 x 16 12 6 7

Or The 8th term of an AP is 17 and its 14th term is 29. Find its common difference.
2. If two towers of heights x m and y m subtend angles of 45° and 60°, respectively at the centre of a line
joining their feet, then find the ratio of ( x + y): y.
Or Write the expression a n − a k for the AP a , a + d , a + 2d , …
Hence, find the common difference of the AP for which 25th term is 10 more than the 23rd term.
3. The length of common chord of two intersecting circles is 30 cm. If the diameters of these two circles are
50 cm and 34 cm, then calculate the distance between their centres.
Or Given, a line segment AB. Divide it in the ratio m : n by construction, where both m and n are positive
integers and let m = 4 and n = 3.
4. From a solid cube of side 7 cm, a conical cavity of height 7 cm and radius 3 cm is hollowed out. Find the
volume of the remaining solid.
5. Find the value of k for which the quadratic equation (3k + 1)x 2 + 2(k + 1)x + 1 = 0, has equal roots . Also find
these roots.
CBSE Term II Mathematics X (Basic) 169

Section B (5 Marks Each)


This section consists of 4 questions of Long Answer Type.
6. Construct a tangent to a circle of radius 4 cm from a point on the concentric circle of radius 6 cm and measure
its length. Also, verify the measurement by actual calculation.
Or A decorative block as shown in figure is made of two solids, a cube and a hemisphere.
4.2 cm

cm
5
5 cm

5 cm

The base of the block is the cube with edge of 5 cm and the hemisphere attached on the top has a diameter of
4.2 cm. If the block is to be painted, then find the total area to be painted. [take, π = 22 / 7]
7. From the point, 36 m above the surface of a lake, the angle of elevation of a bird is observed to be 30° and angle
of depression of its image in the water of the lake is observed to be 60°. Find the actual height of the bird above
the surface of the lake.
Or Prove that the intercept of a tangent between two parallel tangents to a circle subtends a right angle at the
centre.
x − 1 2x + 1 5 1
8. Find the solution of the equation + = , x ≠ − , 1 by factorisation method.
2x + 1 x − 1 2 2
Or Find the median for the following frequency distribution.

Height (in cm) Frequency


160-162 15
163-165 117
166-168 136
169-171 118
172-174 14

9. A girl of height 120 cm is walking away from the base of a lamp-post at a speed of 1.2 m/s. If the lamp is 4.8 m
above the ground, then find the length of her shadow after 6 s.

Section C (1 Mark Each)


This section consists of 1 Case Based comprises of 5 MCQs.
Σf i x i
10. In a mathematic class, a teacher explain the concept for determine the mean by defining the formula x = .
Σf i
170 CBSE Term II Mathematics X (Basic)

Further, a teacher give one example for explaining the above (ii) Find the cumulative frequency value in the
concepts. interval (40-55).
The marks obtained by 30 students of class X of a certain (a) 5 (b) 12
school in a mathematics paper consisting of 100 marks are (c) 2 (d) 18
presented in table below (iii) Through cumulative frequency table, which
central measurement can be determined.
Class interval 10-25 25-40 40-55 55-70 70-85 85-100
(a) mean (b) mode
Number of 2 3 7 6 6 6 (c) median (d) None of these
Students (iv) Find the lower limit of the median class.
(i) Find the average marks obtained by the students. (a) 55 (b) 40
(a) 61 (c) 70 (d) 25
(b) 62 (e) Find the upper limit of modal class.
(c) 63 (a) 40 (b) 55
(d) 64 (c) 70 (d) 25

Solutions
1. Since, the mode of the given series is 36, which lies in the Let E be the point (centre) on the line AC.
class 30-40. i.e. AE = EC = a m
So, the modal class is 30-40. In right angled ∆BAE,
Then, l = 30, f1 = 16, f0 = x, f2 = 12 and h = 10 AB  perpendicular 
mode = 36 tan 45° = Q tanθ =
Also, AE  base 
 f1 − f0  x
Q Mode = l + h ×  ⇒ =1 [Q tan 45°= 1]
 2 f1 − f0 − f2  a
 16 − x  ⇒ x =a …(i)
∴ 36 = 30 + 10 × 
 2 × 16 − x − 12  Again, in right angled ∆DCE,
10 (16 − x ) DC
⇒ 36 = 30 + tan60° =
( 20 − x ) CE
y
10 (16 − x ) 6 10(16 − x ) ⇒ 3= [Q tan 60° = 3]
⇒ 36 − 30 = ⇒ = a
( 20 − x ) 1 ( 20 − x )
⇒ y = 3a …(i)
⇒ 10 (16 − x ) = 6 ( 20 − x ) ⇒160 − 10 x = 120 − 6x
∴ ( x + y ): y = (a + 3a ):a 3
⇒ − 10x + 6x = 120 − 160 ⇒ − 4x = − 40
− 40 = (1 + 3 ): 3
∴ x= = 10
−4 Hence, the requred ratio ( x + y ): y is (1 + 3 ): 3.
Hence, the missing frequency is 10. Or
Or Given, first term = a and common difference = d
Given, T8 = 17 , T14 = 29
∴ a n = a + ( n − 1) d …(i)
Let be a first term a
Tn = a + ( n − 1)d and a k = a + ( k − 1) d …(ii)
17 = 9 + ( 8 − 1)d ⇒17 = a + 7d …(i) On subtracting Eq. (ii) from Eq. (i), we get
Similarly, 29 = a + 13d …(ii) an − ak = [a + ( n − 1) d] − [a + ( k − 1) d]
Subtract Eq. (i) from Eq. (ii), ⇒ a n − a k = a + ( n − 1) d − a − ( k − 1) d
a + 13d − a − 7d = 29 − 17 = d = 2 ⇒ a n − a k = ( n − 1 − k + 1) d
2. Let AB = x m be the height of a tower and CD = y m be the ⇒ an − ak = ( n − k) d
height of other tower and ∠AEB = 45° and ∠CED = 60°. Now, an − ak = ( n − k) d
D
B ⇒ a 25 − a 23 = ( 25 − 23) d [put n = 25and k = 23]
y ⇒ 10 = 2d [Qa 25 − a 23 = 10, given]
x ⇒ d=5
45° 60°
Hence, the common difference is 5.
A a E a C
CBSE Term II Mathematics X (Basic) 171

3. Let, PQ be the length of the common chord of two (iv) Now, through the point A 4 ( m = 4), draw a line parallel
intersecting circles. to A 7B by making an angle equal to ∠AA 7 B at A 4
intersecting AB at a point C.
∴ PQ = 30cm [given]
Then, AC : BC = 4 : 3
Diameters of two circles are 50 cm and 34 cm.
Alternative Method
Join AB.
(i) Draw any ray AX making an acute angle with AB.
P (ii) Draw a ray BY parallel to AX by making
∠ABY = ∠BAX.
A B
(iii) Locate the points A 1 , A 2 , A 3 , A 4 (for m = 4) on
O
AX and similarly B 1, B 2 , B3 (for n = 3) on BY, such that
Q A A 1 = A 1A 2 = A 2A 3 = A 3A 4
= BB1 = B1B2 = B2B3
Since, AB bisects the common chord PQ perpendicularly. X
1 1 A4
∴ OP = OQ = × PQ = × 30 = 15 cm
2 2 A3
1 A2
Radius, AP = × 50 = 25 cm A1
2
C
1 A B
and radius, PB = × 34 = 17 cm
2
B1
In right angled ∆AOP, B2
OA = ( AP )2 − (OP )2 B3
[by using Pythagoras theorem]
Y
= ( 25)2 − (15)2
(iv) Join A 4B3 . Let it intersects AB at point C.
= 625 − 225
AC 4
= 400 = 20 cm Then, =
BC 3
In right angled ∆POB, 4. Given, side of a solid cube, a = 7 cm
OB = (PB)2 − (OP )2 Height of conical cavity, i.e. cone, h = 7 cm
Radius of conical cavity, r = 3 cm
= (17 )2 − (15)2
[by using Pythagoras theorem]
= 289 − 225
7 cm
= 64 = 8 cm
∴ Distance between centres,
AB = OA + OB = 20 cm + 8 cm = 28 cm 3 cm
Or Now, volume of cube = a = (7 )3 = 343 cm3
3

Given A line segment AB, m = 4 and n = 3. 1


and volume of conical cavity = π × r2 × h
Steps of construction 3
1 22
(i) Draw any ray AX making an acute angle with AB. = × × 3 × 3 × 7 = 66 cm3
(ii) Locate 7 (i.e. m + n) points A1 , A 2 , ...,A 7 on AX, 3 7
According to the question,
such that AA 1 = A 1 A 2 = ... = A 6 A 7 .
Volume of remaining solid
(iii) Join BA 7.
= Volume of cube − Volume of conical cavity
X
A7 = 343 − 66 = 277 cm3
A6
A5 Hence, the required volume of solid is 277 cm3 .
A4 5. Given quadratic equation is
A3 ( 3k + 1) x 2 + 2 ( k + 1) x + 1 = 0
A2
A1 On comparing with ax 2 + bx + c = 0, we get
a = 3k + 1, b = 2 ( k + 1), c = 1
A
C B
172 CBSE Term II Mathematics X (Basic)

Since, the roots are equal, so b 2 − 4ac = 0 ⇒ PM = 20 = 4.47 ≈ 4.5


∴ [2 ( k + 1)] − 4( 3k + 1) (1) = 0
2
Hence, the length of tangent is 4.5 cm.
⇒ 4( k + 1)2 − 4( 3k + 1) = 0 Or
Given, edge of cube = 5 cm
⇒ 4( k2 + 2 k + 1) − 4( 3k + 1) = 0
and diameter of hemisphere = 4. 2 cm
⇒ 4( k2 + 2 k + 1 − 3k − 1) = 0 4.2
radius of hemisphere = cm = 2.1 cm
⇒ k2 + 2 k + 1 − 3k − 1 = 0 2
⇒ k2 + 2 k + 1 − 3k − 1 = 0 [Q 4 ≠ 0] Clearly, total surface area of the cube
⇒ k − k=0
2 = 6 (Edge) 2

⇒ k( k − 1) = 0 = 6 × 5 × 5 = 150 cm 2
⇒ k = 0 or k = 1 Now, area to be painted on the cube
We know that, if roots are equal, then roots will be the form of = Total surface area of cube
−b −b − Base area of hemisphere
, .
2a 2a 22 4.2 4.2
=150 − πr 2 = 150 − × ×
−b −b 7 2 2
Thus, roots are , .
2a 2a = 150 − 13.86 = 136.14 cm 2

− ( k + 1) − ( k + 1)
∴ Equal roots are , Area to be painted on the hemisphere
( 3k + 1) ( 3k + 1)
= Curved surface area of hemisphere
− (0 + 1 ) − (0 + 1 ) 22 4.2 4.2
When k = 0, equal roots are and = 2 πr 2 = 2 × × × = 27.72 cm2
0+1 0+1 7 2 2
i.e. − 1 and − 1. ∴ Total area to be painted
− (1 + 1) − (1 + 1) = Area to be painted on the cube
When k = 1, equal roots are and
3+1 3+1 + Area to be painted on the hemisphere
1 1 = 136.14 + 27.72 = 163.86 cm2
i.e. − and − .
2 2 7. Let QR be the surface of the lake and P be point above the
6. Given, two concentric circles of radii 4 cm and 6 cm with surface such that PQ = 36 m. Let B represents the bird and B′
common centre O. be its image in the lake.
Here, we have to draw two tangents to inner circle C1 from a B
point of outer circle C 2 .
x
Steps of construction 30°
(i) Draw two concentric circles C1 and C 2 with common P S
36 m 60° 36 m
centre O and radii 4 cm and 6 cm, respectively. Q
(ii) Take any point P on outer circle C 2 and join OP. R
(iii) Now, bisect OP. Let M′ be the mid-point of OP.
(iv) Taking M′ as centre and OM′ as radius, draw a dotted (36+x) m
circle which cuts the inner circle C1 at two points M
and P′.
(v) Join PM and PP′. Thus, PM and PP′ are required B′
tangents.
∴ ∠BPS = 30° and ∠B′ PS = 60°
M Also, SR = PQ = 36 m
Let BS = x
M′
P O
C1 C2
⇒ B′ R = BR = ( 36 + x ) m
∴ B′ S = SR + BR
P′
= 36 + 36 + x = (72 + x ) m
In right angled ∆PSB,
On measuring PM and PP′, we get BS
= tan 30°
PM = PP′ = 4.5 cm PS
Calculation In right angled ∆OMP, ∠ PMO = 90° x 1  1 
∴ = Q tan 30° =
PS 3  3 
∴ (PM)2 = (OP )2 − (OM)2 [by Pythagoras theorem]
⇒ PS = 3x …(i)
⇒ (PM) = (6) − ( 4) = 36 − 16 = 20
2 2 2
CBSE Term II Mathematics X (Basic) 173

Again, in right angled ∆B′ SP , [from Eqs. (i) and (ii)]


B′ S 72 + x 1
= (180°) = 90°
= tan60° ⇒ = 3
PS PS 2
[Q B’S = (72+x) m and tan 60° = 3 ] [Q QR is a straight line, therefore
72 + x ∠1 + ∠ 2 + ∠ 3 + ∠ 4 = 180°]
⇒ PS = …(ii)
3 Hence proved.
From Eqs. (i) and (ii), we get x −1 2 x +1 5
8. We have, + =
72 + x 2x + 1 x −1 2
3x =
3 x −1
Let y = , then given equation becomes
⇒ 3 ⋅ 3x = 72 + x 2 x +1
⇒ 3x − x = 72 ⇒ 2 x = 72 1 5 y2 + 1 5
y+ = ⇒ = ⇒ 2 y 2 − 5y + 2 = 0
72 y 2 y 2
⇒ x= ⇒ x = 36
2 This is a quadratic equation.
∴ Height of bird above surface of the lake, By using factorisation method,
BR = BS + SR = 36 + 36 = 72 m 2 y 2 − 4y − y + 2 = 0
Or ⇒ 2 y( y − 2 ) − 1 ( y − 2 ) = 0
Let, AB and CD are two tangents to a circle and AB || CD. ⇒ ( 2 y − 1)( y − 2 ) = 0
Tangent BD subtends ∠BOD at the centre. ⇒ 2 y − 1 = 0 or y − 2 = 0
1
To prove ∠BOD = 90° ⇒ y = or y = 2
2
Construction Join OP, OQ and OR. x −1
Put y = , we get
A Q 2 x +1
B
x −1 1 x −1
= or =2
2 x +1 2 2 x +1
2
⇒ 2 x − 2 = 2 x + 1 or x − 1 = 4x + 2
O 1 ⇒ −2 = 1, which is not true.
4
3 Consider, x − 1 = 4x + 2
P
⇒ 3x = − 3 ⇒ x = −1
C R D Or
The given series is in inclusive form. Converting it to
Proof Here, OP ⊥ BD exclusive form and preparing the cumulative frequency table
is given below
[Q a tangent at any point of a circle is perpendicular to the
radius through the point of contact] Cumulative
Class interval Frequency (fi )
In right angled ∆OQB and ∆ OPB, frequency
BQ = BP 159.5-162.5 15 15
[Q the lengths of tangents drawn from an 162.5-165.5 117 132
external point are equal] 165.5-168.5 136 268
OQ = OP [radii] 168.5-171.5 118 386
OB = OB [common] 171.5-174.5 14 400
∴ ∆OQB ≅ ∆ OPB [by SSS congruency] Total N = Σ f i = 400
Then, ∠1 = ∠ 2 [by CPCT] …(i)
Here, N = 400
Similarly, in right angled ∆OPD and ∆ORD, N 400
∠ 3 = ∠4 …(ii) Now, = = 200
2 2
1
∴ ∠BOD = ∠1 + ∠ 3 = ( 2 ∠1 + 2 ∠ 3)
2
1 The cumulative frequency just greater than 200 is 268 and the
= ( ∠1 + ∠1 + ∠ 3 + ∠ 3) corresponding class is 165.5-168.5.
2
1 Thus, the median class is 165.5-168.5.
= ( ∠1 + ∠ 2 + ∠ 3 + ∠ 4)
2 ∴ l = 165.5, h = 3 and f = 136 and C = 132
174 CBSE Term II Mathematics X (Basic)

 N  10. (i) (b) Let us make the following table for the given data.
− C. f
 2 
∴ Median = l + h ×  (1/2)
 f  Class marks Cumulative
Class fi xi
  Frequency (xi ) frequency
Interval
 ( 200 − 132 )  (cf )
= 165.5 + 3 × 
 136  10 + 25
10-25 2 = 17.5 35.0 2
3 × 68 2
= 165.5 + = 165.5 + 1.5 = 167
136 25 + 40
25-40 3 = 32.5 97.5 5
Hence, the median height is 167 cm. 2
9. Let AB be the lamp-post, CD be the girl and D be the 40 + 55
position of girl after 6 s. 40-55 7 = 47.5 332.5 12
2
Again, let DE = x m be the length of shadow of the girl. 55 + 70
55-70 6 = 62.5 375.0 18
A 2
70 + 85
70-85 6 = 77.5 465.0 24
2
4.8 m
85 + 100
C
85-100 6 = 92.5 555.0 30
2
B D x
E Σfi xi
Total Σfi = 30
= 18600
.
Given, CD = 120 cm = 1.2 m, AB = 4.8 m
and speed of the girl = 1.2 m/s
Here, Σfi = 30 and Σfi xi = 18600.
∴ Distance of the girl from lamp-post after 6 s.
Σfi xi
BD = 1.2 × 6 = 7.2 m [Q distance = speed × time] Q Average, x =
Σfi
In ∆ABE and ∆CDE,
18600 .
∠B = ∠D [each 90°] = = 62
30
∠E = ∠E [common angle]
Hence, average marks obtained by student is 62.
∴ ∆ABE ~ ∆CDE [by AA similarity criterion]
BE AB (ii) (b) The cumulative frequency value in the interval
⇒ = …(i) 40-55 is 12.
DE CD
(iii) (c) Through cumulative frequency table, median can be
On substituting all the values in Eq. (i), we get determined.
7.2 + x 4.8 N 30
= [Q BE = BD + DE = 7.2 + x] (iv) (a) Here, = = 15, which lies in the cumulative
x 1.2 2 2
7.2 + x frequency 18, whose corresponding frequency is 55-70.
⇒ =4
x Hence, lower limit of the median class is 55.
⇒ 7.2 + x = 4x (v) (b) In the given data, the highest frequency is 7, whose
7.2 corresponding interval is 40-55.
⇒ 3x = 7.2 ⇒ x = = 2.4 m
3 Hence, upper limit of the modal class is 55.
Hence, the length of her shadow after 6 s is 2.4 m.
CBSE Term II Mathematics X (Basic) 175
Mathem atics(Basic)
Class 10th ( Term II)

*
Practice Paper 2
(Unsolved)
Instructions Time : 2 Hr
Max. Marks : 40
1. The question paper contains three sections A, B and C.
2. Section A has 5 questions with 3 internal choices.
3. Section B has 4 questions with 3 internal choices.
4. Section C has 1 Case Based MCQs comprises of 5 MCQs.
5. There is no negative marking.
* As exact Blue-print and Pattern for CBSE Term II exams is not released yet. So the pattern of this
paper is designed by the author on the basis of trend of past CBSE Papers. Students are advised
not to consider the pattern of this paper as official, it is just for practice purpose.

Section A (3 Marks Each)


This section consists of 5 questions of Short Answer Type.
x − 3 x + 3 48
1. Find the solution of the equation − = , x ≠ 3, x ≠ −3.
x+3 x−3 7
2. Write the expression a n − a k for the AP a , a + d , a + 2d , …
Hence, find the common difference of the AP for which 25th term is 10 more than the 23rd term.
Or A statue 1.6 m tall stands on the top of a pedestal. From a point on the ground, the angle of elevation of the
top of the statue is 60° and from the same point the angle of elevation of the top of the pedestal is 45°. Find
the height of the pedestal.
3. In the given figure, PQ and QR are tangents to the circle centre O, at P and R, respectively.
Find the value of x.
R

20°

S O 50° Q


P

Or Draw a circle with the help of circular solid ring. Construct a pair of tangents from a point P to the circle.
4. For grouped data, if Σf i = 20, Σf i x i = 2p + 20 and mean of distribution is 12, then find the value of p.
176 CBSE Term II Mathematics X (Basic)

5. A vessel is in the form of a hollow hemisphere mounted by a hollow cylinder. The diameter of the hemisphere is 14
cm and the total height of the vessel is 13 cm. Find the inner surface area of the vessel.
Or Two pillars of equal height are on either sides of a road, which is 100 m wide. The angles of the top of the pillars
are 60° and 30° at a point on the road between the pillars. Find the position of the point between the pillars.
Also, find the height of each pillar.

Section B (5 Marks Each)


This section consists of 4 questions of Long Answer Type.
6. A solid toy is in the form of a hemisphere surmounted by a right circular cone. Height of the cone is 4 cm and
the diameter of the base is 8 cm. If a right circular cylinder circumscribes the solid. Find how much more space
it will cover?
Or The angle of elevation of the top of a tower from certain point is 30°. If the observer moves 20 m towards the
tower, the angle of elevation of the top increases by 15°. Find the height of the tower.
7. Construct a tangent to a circle of radius 1.8 cm from a point on the concentric circle of radius 2.8 cm and
measure its length. Also, verify the measurement by actual calculation.
Or From an external point P, two tangents PA and PB are drawn to the circle with centre O. Prove that OP is the
perpendicular bisector of AB.
8. Solve the following quadratic equation by factorisation method.
1 1 1 1
= + + , a + b ≠ 0.
a+b+x a b x
Or The angles of a triangle are in AP. If the greatest angle equals to the sum of the other two, then find the angles.
Also, conclude that find these angles are multiple of which angle.
9. The median of the distribution given below is 14.4. Find the values of x and y, if the total frequency is 20.
Class interval 0-6 6-12 12-18 18-24 24-30

Frequency 4 x 5 y 1

Section C (1 Mark Each)


This section consists of 1 Case Based comprises of 5 MCQs.
10. Suppose, there are two windows in a house. A window of the house is at a height of 1.5 m above the ground and
the other window is 3 m vertically above the lower window.
C

h – 4.5 m

B 30°
(Shyam) D

3m 3m

(RAM) A E

1.5 m 1.5 m

P Q
CBSE Term II Mathematics X (Basic) 177

Anil and Sanjeev are sitting in the two windows. At an instant, the angles of elevation of a balloon from these
windows are observed as 45° and 30°, respectively.
(i) Find the height of the balloon from the ground.
(a) 6.8 m (b) 8.6 m
(c) 9.4 m (d) 9.6 m
(ii) Among Anil and Sanjeev, who is more closer to the balloon?
(a) Sanjeev (b) Anil
(c) cannot say (d) None of these
(iii) If the balloon is moving towards the building, then will both the angles of elevation remain same?
(a) cannot say (b) Yes
(c) No (d) None of these
(iv) If the height of any tower is double and the distance between the observer and foot of the tower is also doubled,
then the angle of elevation
(a) remain same (b) become double
(c) become triple (d) None of these
(v) Suppose a tower and a pole is standing on the ground. And the angle of elevation from bottom of pole is θ1 and
elevation from top of pole to the top of tower is θ 2 .

θ1
B

θ1
A

Choose the correct option.


(a) θ1 > θ 2 (b) θ1 = θ 2
(c) θ1 < θ 2 (d) None of these

Answers
9
1. −4, 2. 5 or 0.8( 3 + 1) m 3. 45° 4. 110 5. 572 cm 2 or 25 m, 43.3 m
4
128
6. π cm 3 or10( 3 + 1) m 7. 2.14 cm 8. x = −a or x = −b or ` 30°, 60° and 90° angles are the multiple of 30°
3
9. x = 4 and y = 6 10. (i) (b) (ii) (a) (iii) (c) (iv) (a) (v) (a)
178 CBSE Term II Mathematics X (Basic)
Mathem atics(Basic)
Class 10th ( Term II)

*
Practice Paper 3
(Unsolved)
Instructions Time : 2 Hr
Max. Marks : 40
1. The question paper contains three sections A, B and C.
2. Section A has 5 questions with 3 internal choices.
3. Section B has 4 questions with 3 internal choices.
4. Section C has 1 Case Based MCQs comprises of 5 MCQs.
5. There is no negative marking.
* As exact Blue-print and Pattern for CBSE Term II exams is not released yet. So the pattern of this
paper is designed by the author on the basis of trend of past CBSE Papers. Students are advised
not to consider the pattern of this paper as official, it is just for practice purpose.

Section A (3 Marks Each)


This section consists of 5 questions of Short Answer Type.
1. Find the roots of the quadratic equation 9 x 2 − 9 (a + b )x + (2a 2 + 5ab + 2b 2 ) = 0.
1 2
2. Which term of the progression 19, 18 , 17 , ... is the first negative term?
5 5
Or From the top of a 10 m high building, the angle of elevation of the top of a tower is 60° and the angle of
depression of its foot is 45°. Determine the height of the tower.
3. In the given figure, AB is the diameter of a circle with centre O and QC is a tangent to the circle at C. If
∠CAB = 30°, then find ∠CQA and ∠CBA.

30°
A B Q
O

Or A copper wire 4 mm in diameter is evenly wound about a cylinder whose length is 24 cm and diameter 20
cm so as to cover the whole surface. Find the length and weight of the wire assuming the specific density to
be 8.88 gm/cm 3 .
4. Draw a circle of radius 6 cm. Take a point P on it. Without using the centre of the circle, draw a tangent to
the circle at point P.
CBSE Term II Mathematics X (Basic) 179

5. The mean of the following frequency table is 50 but the frequencies f 1 and f 2 in class intervals 20-40 and 60-80
are missing. Find the missing frequencies.
Class interval 0-20 20-40 40-60 60-80 80-100 Total
Frequency 17 f1 32 f2 19 120

Or Find the volume area of the largest right circular cone that can be cut out of a cube whose edge is 10 cm.

Section B (5 Marks Each)


This section consists of 4 questions of Long Answer Type.
6. A cone of maximum size is cut-out from a cube of edge 14 cm. Find the surface area of the remaining solid left
out after the cone is cut-out.
Or The angle of elevation of a jet plane from a point A on the ground is 60°. After a flight of 15 sec, the angle of
elevation changes to 30°. If the jet plane is flying at a constant height of 1500 3 m, find the speed of the jet
plane.
7. If α and β are the zeroes of the quadratic polynomial f (x ) = 3x 2 − 4 x + 1, find a quadratic polynomial whose
α2 β2
zeroes are and .
β α
Or If m times the mth term of an AP is equal to n times its nth term, then show that ( m + n) th term of the AP is
zero.
8. Construct a tangent to a circle of radius 1.8 cm from a point on the concentric circle of radius 2.8 cm and
measure its length. Also, verify the measurement by actual calculation.
9. In the given figure, PT is a tangent and PAB is a secant. If PT = 6 cm and AB = 5 cm, then find the length of PA.

O
P
A B

Or Find the mean, mode and median of the following data.


Class 0-10 10-20 20-30 30-40 40-50 50-60 60-70

Frequency 3 4 7 15 10 7 4

Section C (1 Mark Each)


This section consists of 1 Case Based comprises of 5 MCQs.
10. In one corner of the drawing room, a flower basket is kept inside the glass, lies on the table. The basket is
designed in such a way that every one pleases to see it.
180 CBSE Term II Mathematics X (Basic)

The shape of flower basket is hemisphere with radius 60 cm and upper shape is conical with height 120 cm from the bottom
surface.
P
F E
120 cm

180 cm

O
D
60 cm

60 cm

O′
A B

(i) Find the capacity of the glass.


14.256 3 12.256 3 142.56 3 14.256 3
(a) m (b) m (c) m (d) m
7 7 7 5
(ii) Find the volume of the cone.
(a) 0.54 m 3 (b) 0.45 m 3 (c) 0.25 m 3 (d) 0.52 m 3
(iii) Find the curve surface area of hemisphere.
(a) 0.201 m 2 (b) 0.104 m 2 (c) 0.102 m 2 (d) 0.401 m 2
(iv) The volume of two combined figure is equal to the sum of
(a) two individual volumes (b) two individual curve surface area
(c) volumes and curve surface area (d) None of these
(v) If the cost of painting the glass outside is ` 1.20 per m 2 , find the total cost of painting the CSA of the glass.
(a) ` 55 (b) ` 55.02 (c) ` 57 (d) ` 57.02

Answers
(2a + b ) (a + 2b )
1. , 2. 25th term or 10( 3 + )1 m 3. ∠CQA = 30° and ∠CBA = 60° or 4.21 kg
3 3
5. f1 = 28 and f 2 = 24 or 261.9 cm 2 6. 1022 + 154 5 cm or 720 km/h 2

 2 28 1
7. k x − x +  where k is any non-zero real number 8. 2.14 cm
 9 3
9. 4 cm or (i) 37.4, (ii) 36.15, (iii) 37.3 10. (i) (a) (ii) (b) (iii) (c) (iv) (a) (v) (d)

You might also like